Apunte Usach - Calculo Avanzado

March 21, 2018 | Author: Macarena Catalán González | Category: Fourier Series, Integral, Euclidean Vector, Geometry, Mathematical Structures


Comments



Description

Universidad de Santiago de Chile.Facultad de Ciencia Departamento de Matem´atica y C. C. C´alculo Avanzado Miguel Martinez - Carlos Silva - Emilio Villalobos Derechos de Autor Autor: c Universidad de Santiago de Chile Se autoriza la reproducci´on parcial o total de esta obra, con fines acad´emicos, por cualquier forma, medio o procedimiento, siempre y cuando se incluya la cita bibliogr´afica del documento. Agradecimientos Este texto fue financiado en el marco de los proyectos concursables de innovaci´on docente que promueve anualmente la Universidad de Santiago de Chile a trav´es de la Vicerrector´ıa Acad´emica por intermedio de la Direcci´on de Docencia. El centro motor que motiv´ o a los autores a emprender tan significativo desaf´ıo fue su compromiso con el proceso de ense˜ nanza aprendizaje que se lleva semestre a semestre en la Universidad de Santiago, con los estudiantes de ingenier´ıa quienes tienen el imperativo de mejorar sus aprendizajes y elevar sus est´ andares de competencias con la finalidad de que puedan asumir con propiedad el desaf´ıo de sus asignaturas profesionales y de especialidad con un mayor empoderamiento en el contexto de: teor´ıa, pr´ actica y aplicaciones a problemas en las ´areas de sus distintas especialidades. En general, cada cap´ıtulo comienza con una presentaci´ on de definiciones, principios y teoremas, junto con material ilustrativo. Los problemas resueltos sirven para ilustrar la teor´ıa y suministrar herramientas de an´alisis de los principios b´ asicos tan importantes en el aprendizaje activo de los estudiantes. El gran n´ umero de problemas resueltos y aplicaciones sirven para encauzar el aprendizaje del material, as´ı como las autoevaluaciones propuestas al fin de cada unidad. Hemos escogido un enfoque y nivel de profundidad de acuerdo con lo que se espera del curso de C´alculo Avanzado, asignatura que se imparte durante el tercer semestre del Plan Com´ un de la Carrera de Ingenier´ıa Civil de la Facultad de Ingenier´ıa de Universidad de Santiago de Chile. El objetivo del primera parte de este texto es presentar los conceptos b´ asicos y las aplicaciones de las series de Fourier, y las funciones integrales, como asimismo, ilustrar su utilizaci´on en la resoluci´ on de problemas de ecua- ciones en derivadas parciales y aplicaciones en el campo de la f´ısica e inge- nier´ıa. En la segunda parte se abordan los temas de funciones vectoriales y c´ alcu- lo diferencial de funciones de dos o m´as variables y sus aplicaciones, incluyen- do aplicaciones e interpretaciones geom´etricas y f´ısicas que contribuyan a la comprensi´ on de los estudiantes. Unido a lo anterior, en la tercera parte se incluyen los temas de integraci´ on m´ ultiple, integral de l´ınea , superficie y los teoremas de Green, Gauss y Stokes por sus m´ ultiples aplicaciones en los campos de la f´ısica y ciencias de la ingeniera Finalmente, queremos aprovechar la ocasi´on para expresar nuestro es- pecial agradecimiento hacia nuestros colegas de la Coordinaci´ on de C´ alculo Avanzado que con sus cr´ıticas constructivas y opiniones han ayudado a el i enriquecimiento del material que conforma este texto. Deseamos tambi´en agradecer la participaci´on directa e indirecta de nuestros estudiantes con los cuales pusimos a prueba el material que se estaba generando incluy´endolos en la p´ agina web de la asignatura de C´ alculo Avanzado. Agradecemos tambi´en muy especialmente la colaboraci´on del profesor Omar Ramos por la confecci´on de diagramas, figuras e im´ agenes de fun- ciones bi y tridimensionales que ilustran conceptos y problemas. Tambi´en se encarg´ o de la versi´ on Latex de los distintos archivos que conforman el manuscrito del texto. No obstante lo anterior, la responsabilidad por los eventuales errores o inexactitudes que se puedan encontrar en el texto corresponde a los autores, quienes estar´ an atentos para recibir cualquier comentario o sugerencia que permita mejorar su contenido en las siguientes direcciones: [email protected], [email protected],[email protected]. Los Autores: Miguel Mart´ınez Concha Carlos Silva Cornejo Emilio Villalobos Mar´ın ii Prefacio El material presentado en el texto contiene los temas tratados en el curso de C´ alculo Avanzado, asignatura semestral para las carreras de Ingenier´ıa de la Universidad de Santiago de Chile, correspondiente al ´ area de Ciencias B´ asicas, tiene por prerrequisitos las asignaturas de C´alculo I y C´ alculo II de primer a˜ no. Proporciona los conceptos, habilidades y t´ecnicas que permiten adquirir las competencias matem´aticas alineadas con el perfil de competen- cias profesionales, necesarias para cursar con ´exito las asignaturas de ciencias b´ asicas de la ingenier´ıa e ingenier´ıa aplicada. Los temas tratados por el texto y en el orden de aparici´ on son los siguientes: Series e Integrales de Fourier, que forma parte de este temario porque por razones de tiempo no se incluye en el Cap´ıtulo de Series de primer a˜ no, este tema resulta necesario en la formaci´ on b´asica de alumnos de ingenier´ıa sobre todo cuando necesiten re- solver ecuaciones diferenciales parciales usando el m´etodo de separaci´on de variables o bien en aplicaciones en el campo de la ingenier´ıa. Este tema bien podr´ıa formar parte de un texto de ecuaciones diferenciales. El tema de fun- ciones vectoriales de una variable real trata la importancia de la derivada de este tipo de funciones, interpretaci´on geom´etrica y anal´ıtica, y su aplicaci´ on a problemas de movimiento, comportamiento de curvas, especialmente en lo que tiene que ver con caracter´ısticas geom´etricas de ellas. Las funciones es- calares son tratadas en detalle, se analiza el concepto de l´ımite y continuidad considerando funciones de dos variables, generalizando en aquellos casos que lo amerita, se ve el tema de la diferenciaci´ on con todas sus potencialidades que garantizan la derivaci´on tanto la derivaci´on parcial, como la derivaci´ on direccional y derivaci´on impl´ıcita, este tema termina con m´aximos y m´ıni- mos y problemas aplicados de optimizaci´on. Este ´ ultimo cap´ıtulo tratan los temas de integraci´ on, integrales dobles y triples en coordenadas cartesianas y generalizando con cambios de coordenadas, integral de l´ınea para funciones escalares y vectoriales, propiedades de los campos gradientes y el teorema de Green; integral de superficie para funciones escalares y vectoriales finalizan- do con el estudio de los teoremas de Gauss y Stokes. Los temas tratados de acuerdo con los objetivos generales los podemos describir como sigue: Series e integrales de Fourier i) Analizar los conceptos asociados a la definici´ on de la Serie de Fourier, sus propiedades y procedimientos de c´ alculo, y aplicarlos a la resoluci´ on de problemas de ingenier´ıa. ii) Formular el concepto de Integral de Fourier, sus propiedades y m´etodos de calculo y aplicar esta informaci´on en la soluci´on de problemas de ingenier´ıa. iii Funciones vectoriales i) Analizar el concepto de diferenciaci´ on de funciones vectoriales, sus propiedades, procedimientos para realizar c´ alculos y aplicarlos a la resoluci´on de problemas de Ciencia e Ingenier´ıa ii) Utilizar los conceptos de vector tangente, normal , binormal, curvatura y torsi´on e identidades de Frenet sus propiedades y procedimientos de c´ alculos para emplearlos en la resoluci´ on de problemas. Diferenciaci´ on parcial i) Definir los conceptos de l´ımite, continuidad y describir las caracter´ısticas gr´ aficas de las funciones de varias variables en IR2 en IR. ii) Analizar criterios para reconocer y evaluar la diferenciabilidad de una funci´ on escalar de varias variables, usar su propiedades, m´etodos de c´ alculos para su aplicaci´on a la resoluci´ on de problemas de Ingenier´ıa. iii) Generalizar el concepto de diferenciaci´on para funciones compuestas e impl´ıcitas de varias variables, sus propiedades, m´etodos de c´ alculos y su aplicaci´on a la resoluci´ on de problemas de Ingenier´ıa iv) Aplicar diferentes m´etodos para determinar m´ aximo y m´ınimos de una funci´ on de varias variables y utilizarlos en la resoluci´ on de problemas de optimizaci´on Integraci´ on i) Analizar el concepto de integral doble sus propiedades y procedimientos de c´alculo , y su aplicaci´ on a problemas de f´ısica e ingenier´ıa ii) Analizar el concepto de integral triple sus propiedades y procedimientos de c´alculo, y su aplicaci´ on en problemas de f´ısica e ingenier´ıa. iii) Analizar los conceptos de integral de trayectoria e integral de l´ınea, y utilizar sus propiedades en la resoluci´ on de problemas matem´ aticos, de f´ısica e ingenier´ıa iv) Analizar los conceptos de integral de superficie, y utilizar sus propiedades en la resoluci´ on de problemas matem´ aticos, de f´ısica e ingenier´ıa Estructura de cada unidad Cada unidad en su desarrollo te´orico y fundamentaci´ on matem´ atica enfatiza los conceptos, los teoremas que avalan los procedimientos y las t´ecnicas de resoluci´ on de problemas. iv Unido a lo anterior, en cada tema hay una unidad de problemas resuel- tos, otra de problemas propuestos, algunos con soluciones y una unidad de aplicaciones a los temas de ingenier´ıa. Finalmente, se incluye tam- bi´en un instrumento de autoevaluaci´ on que consiste en un test con problemas de desarrollo que mide el nivel de las competencias cogniti- vas alcanzado por el estudiante. Este material pretende ser una fuente de motivaci´ on que haga que los estudiantes perseveren en sus estudios y puedan vencer las dificultades de aprendizaje, y alcanzar niveles que le permitan r´ apidamente conectarse con los temas en que est´a inmerso un problema y estructurar un a respuesta al problema usando diversas herramientas matem´aticas, esto le dar´ a desde luego un plus durante toda su vida profesional. v ´ Indice general 1. Serie de Fourier 1 1.1. Introducci´ on . . . . . . . . . . . . . . . . . . . . . . . . 1 1.2. Propiedades Generales . . . . . . . . . . . . . . . . . . 2 1.2.1. Lema Elemental . . . . . . . . . . . . . . . . . . 2 1.3. La serie de Fourier de una funci´on . . . . . . . . . . . . 4 1.3.1. Coeficientes de Fourier . . . . . . . . . . . . . . 6 1.3.2. Atributos de la funci´on . . . . . . . . . . . . . . 7 1.3.3. Convergencia de las series de Fourier . . . . . . 9 1.3.4. La integral de funciones pares e impares . . . . 15 1.3.5. Teorema de las funciones pares y de las impares 16 1.4. Desarrollos llamados de medio rango . . . . . . . . . . 17 1.4.1. Extensi´on impar: . . . . . . . . . . . . . . . . . 17 1.4.2. Extensi´on par . . . . . . . . . . . . . . . . . . . 19 1.5. Diferenciacion e Integraci´ on de la series de Fourier . . . 21 1.5.1. Derivaci´ on . . . . . . . . . . . . . . . . . . . . . 21 1.5.2. Integraci´on . . . . . . . . . . . . . . . . . . . . 23 1.5.3. Identidad de Parseval . . . . . . . . . . . . . . . 25 1.6. Integral de Fourier . . . . . . . . . . . . . . . . . . . . 27 1.6.1. Criterio de convergencia de la integral de Fourier . . . . . . . . . . . . . . . . . . . . . . . 29 1.6.2. Integrales de Fourier de cosenos y senos . . . . . 31 1.7. Aplicaciones de Series de Fourier . . . . . . . . . . . . 38 vi 1.7.1. Onda cuadrada alta frecuencia . . . . . . . . . 38 1.7.2. Rectificador de onda completa. . . . . . . . . . 39 1.7.3. Ecuaci´ on de calor unidimensional . . . . . . . . 40 1.7.4. Ecuaci´on de calor: barra aislada . . . . . . . . 41 1.7.5. Ecuaci´ on de Onda . . . . . . . . . . . . . . . . 42 1.7.6. Deflexi´ on de una viga . . . . . . . . . . . . . . 44 1.8. Problemas Propuestos . . . . . . . . . . . . . . . . . . 45 1.9. Ejercicios Resueltos . . . . . . . . . . . . . . . . . . . . 50 1.9.1. Serie de Fourier . . . . . . . . . . . . . . . . . . 50 1.9.2. Integral de Fourier . . . . . . . . . . . . . . . . 63 1.10. Ejercicios propuestos . . . . . . . . . . . . . . . . . . . 70 1.10.1. Respuestas . . . . . . . . . . . . . . . . . . . . . 74 1.11. Auto evaluaciones . . . . . . . . . . . . . . . . . . . . 77 2. Funciones Vectoriales de una variable real 92 2.1. Introducci´ on . . . . . . . . . . . . . . . . . . . . . . . . 92 2.2. Funciones Vectoriales . . . . . . . . . . . . . . . . . . 93 2.3. L´ımite de una funci´on vectorial. . . . . . . . . . . . . . 95 2.3.1. Teorema del l´ımite . . . . . . . . . . . . . . . . 95 2.3.2. Operaciones con funciones vectoriales . . . . . 98 2.3.3. Teoremas del algebra de l´ımites . . . . . . . . . 98 2.3.4. Teorema: producto de funci´ on escalar por vec- torial . . . . . . . . . . . . . . . . . . . . . . . 99 2.4. Continuidad . . . . . . . . . . . . . . . . . . . . . . . 99 2.5. La Derivada . . . . . . . . . . . . . . . . . . . . . . . 100 2.6. Regularidad de una curva . . . . . . . . . . . . . . . . 102 2.6.1. Camino regular . . . . . . . . . . . . . . . . . . 103 2.6.2. Propiedades de la Derivada . . . . . . . . . . . 103 2.7. Parametrizaci´ on . . . . . . . . . . . . . . . . . . . . . 104 2.7.1. Ejemplos de reparametrizaciones . . . . . . . . 106 2.8. Longitud de Arco . . . . . . . . . . . . . . . . . . . . . 107 vii 2.8.1. La Longitud de Arco como Par´ ametro . . . . . 109 2.8.2. Parametrizaci´ on por Longitud de Arco . . . . . 111 2.9. Trayectorias y curvas . . . . . . . . . . . . . . . . . . . 113 2.10. Vectores Unitarios . . . . . . . . . . . . . . . . . . . . 115 2.10.1. Vector Tangente unitario . . . . . . . . . . . . 115 2.10.2. Vector Normal . . . . . . . . . . . . . . . . . . 116 2.10.3. Vector Binormal . . . . . . . . . . . . . . . . . 116 2.11. Curvatura . . . . . . . . . . . . . . . . . . . . . . . . . 118 2.11.1. C´ alculo de curvatura usando par´ ametro t cualquiera en R 3 . . . . . . . . . . . . . . . . . . . . . . . 120 2.12. Planos por un punto de la curva . . . . . . . . . . . . . 123 2.12.1. Plano Osculador . . . . . . . . . . . . . . . . . 124 2.12.2. Plano Normal . . . . . . . . . . . . . . . . . . . 124 2.12.3. Plano Rectificante . . . . . . . . . . . . . . . . 124 2.12.4. Recta Tangente . . . . . . . . . . . . . . . . . . 125 2.12.5. Recta Normal . . . . . . . . . . . . . . . . . . . 125 2.12.6. Recta Binormal . . . . . . . . . . . . . . . . . . 125 2.13. Torsi´ on . . . . . . . . . . . . . . . . . . . . . . . . . . . 127 2.13.1. C´ alculo de la torsi´ on usando par´ametro t cualquiera (en R 3 ) . . . . . . . . . . . . . . . . . . . . . . 128 2.14. Formulas de Frenet . . . . . . . . . . . . . . . . . . . . 130 2.15. Aplicaciones de Funciones Vectoriales y Curvas . . . . 131 2.15.1. Problemas . . . . . . . . . . . . . . . . . . . . . 132 2.16. Ejercicios resueltos . . . . . . . . . . . . . . . . . . . . 140 2.17. Ejercicios propuestos . . . . . . . . . . . . . . . . . . . 169 2.17.1. Respuestas . . . . . . . . . . . . . . . . . . . . . 173 2.18. Auto Evaluaciones . . . . . . . . . . . . . . . . . . . . 175 viii 3. Funciones de varias variables 189 3.1. Introducci´ on . . . . . . . . . . . . . . . . . . . . . . . . 189 3.2. Funciones Escalares de Variable Vectorial . . . . . . . . 193 3.2.1. Conceptos Topol´ogicos . . . . . . . . . . . . . . 193 3.2.2. Aspectos Geom´etrico de las Funciones Escalares 197 3.2.3. Gr´afica de una Funci´ on . . . . . . . . . . . . . . 197 3.2.4. Curvas y Superficies de Nivel . . . . . . . . . . 198 3.2.5. L´ımite . . . . . . . . . . . . . . . . . . . . . . . 199 3.2.6. Continuidad . . . . . . . . . . . . . . . . . . . . 205 3.2.7. Derivadas Parciales . . . . . . . . . . . . . . . . 207 3.3. Diferenciabilidad en dos variables . . . . . . . . . . . . 209 3.3.1. Derivada Direccional . . . . . . . . . . . . . . . 211 3.3.2. Plano tangente y recta normal . . . . . . . . . . 215 3.3.3. Funci´ on Compuesta. La Regla de la Cadena. . . 218 3.3.4. Funci´ on Impl´ıcita . . . . . . . . . . . . . . . . . 222 3.3.5. Jacobiano . . . . . . . . . . . . . . . . . . . . . 226 3.3.6. M´aximos y M´ınimos . . . . . . . . . . . . . . . 228 3.3.7. Extremos Restringidos . . . . . . . . . . . . . . 233 3.4. Problemas Resueltos . . . . . . . . . . . . . . . . . . . 245 3.4.1. Continuidad y diferenciabilidad . . . . . . . . . 245 3.4.2. Regla de la cadena . . . . . . . . . . . . . . . . 248 3.4.3. Derivaci´ on Impl´ıcita . . . . . . . . . . . . . . . 251 3.4.4. Plano Tangente a una Superficie . . . . . . . . . 256 3.4.5. Derivadas Direccionales . . . . . . . . . . . . . . 258 3.4.6. Valores extremos . . . . . . . . . . . . . . . . . 260 3.4.7. Multimplicadores de Lagrange para extremos re- stringidos . . . . . . . . . . . . . . . . . . . . . 264 3.4.8. Aplicaci´on al c´alculo de errores . . . . . . . . . 275 3.5. Ejercicios Propuestos . . . . . . . . . . . . . . . . . . 276 3.5.1. L´ımites . . . . . . . . . . . . . . . . . . . . . . . 276 3.5.2. Diferenciabilidad, continuidad . . . . . . . . . . 277 ix 3.5.3. Derivadas parciales . . . . . . . . . . . . . . . . 278 3.5.4. Derivadas Direccionales . . . . . . . . . . . . . . 280 3.5.5. Puntos cr´ıticos m´aximos y m´ınimos . . . . . . . 284 3.6. Aplicaciones Derivada Direccional . . . . . . . . . . . . 286 3.7. Aplicaciones de M´aximos y M´ınimos . . . . . . . . . . 289 3.7.1. Aplicaci´on al campo de la mec´ anica . . . . . . . 289 3.7.2. Aplicaciones a la geometr´ıa . . . . . . . . . . . 292 3.7.3. Aplicaci´on al campo de la econom´ıa . . . . . . 297 3.7.4. Problemas Propuestos de Aplicaciones . . . . . 302 3.8. Auto evaluaciones . . . . . . . . . . . . . . . . . . . . . 304 4. Integraci´on Multiple 315 4.1. Integrales dobles y triples . . . . . . . . . . . . . . . . 315 4.1.1. Integrales Dobles . . . . . . . . . . . . . . . . . 315 4.1.2. Integrales sobre conjuntos acotados de R 2 . . . 320 4.1.3. Teorema de Fubini . . . . . . . . . . . . . . . . 322 4.1.4. ´ Areas y Volumenes . . . . . . . . . . . . . . . . 327 4.1.5. Cambio de variable . . . . . . . . . . . . . . . . 329 4.2. Aplicaciones de la integral doble . . . . . . . . . . . . 333 4.2.1. Masa de una regi´on plana de densidad variable. 333 4.2.2. Momentos y centroide de una regi´on plana . . . 334 4.3. Integrales triples . . . . . . . . . . . . . . . . . . . . . 337 4.3.1. Ideas preliminares . . . . . . . . . . . . . . . . . 337 4.3.2. Teorema de Fubini . . . . . . . . . . . . . . . . 337 4.3.3. Teorema de la integral triple (Para dominios m´as generales) . . . . . . . . . . . . . . . . . . . . . 338 4.3.4. Cambio de variable para integrales triples . . . 342 4.3.5. Formula del cambio de variable . . . . . . . . . 345 4.3.6. Masa, Momentos, y Centroide de una Regi´ on del Espacio . . . . . . . . . . . . . . . . . . . . . . 348 4.4. Ejercicios resueltos integrales triples y dobles . . . . . . 350 x 4.4.1. C´alculo de integrales dobles en coordenadas rect´ angu- lares cartesianas . . . . . . . . . . . . . . . . . . 350 4.4.2. Cambios de orden de Integraci´on . . . . . . . . 361 4.4.3. Cambios de variables: Coordenadas polares . . . 363 4.4.4. Cambios de variables. Coordenadas curvil´ıneas . 367 4.4.5. C´alculo de integrales triples en coordenadas rect´ angu- lares cartesianas . . . . . . . . . . . . . . . . . . 373 4.4.6. Coordenadas esf´ericas . . . . . . . . . . . . . . 377 4.4.7. Coordenadas Cil´ındricas . . . . . . . . . . . . . 380 4.5. Ejercicios propuestos integrales dobles y triples . . . . . 389 4.5.1. Integrales dobles . . . . . . . . . . . . . . . . . 389 4.5.2. C´alculo de Integrales dobles usando transforma- ci´ on de coordenadas . . . . . . . . . . . . . . . 392 4.5.3. Integrales triples . . . . . . . . . . . . . . . . . 394 4.5.4. Integrales triples iteradas . . . . . . . . . . . . . 394 4.5.5. Integrales triples en coordenadas rect´angulares cartesianas. . . . . . . . . . . . . . . . . . . . . 396 4.5.6. Calcular las integrales dadas usando las coorde- nadas adecuadas: . . . . . . . . . . . . . . . . . 397 4.5.7. Resolver las integrales usando coordenadas esf´eri- cas: . . . . . . . . . . . . . . . . . . . . . . . . . 399 4.6. Aplicaciones integrales dobles y triples . . . . . . . . . 401 4.6.1. Volumenes de cuerpos en el espacio . . . . . . . 401 4.6.2. ´ Area de figuras planas. . . . . . . . . . . . . . . 404 4.6.3. Momentos y centros de masa para placas planas delgadas . . . . . . . . . . . . . . . . . . . . . . 406 4.6.4. Centroide de figuras geom´etricas . . . . . . . . . 407 4.6.5. Momentos y Centros de masa de un s´olido . . . 412 4.6.6. Masa de un s´ olido . . . . . . . . . . . . . . . . . 413 4.6.7. Determinaci´on del centroide dee un s´ olido . . . 424 4.7. Autoevaluaci´on Integrales dobles y triples . . . . . . . 426 xi 5. Integral de Linea 436 5.1. Campos vectoriales . . . . . . . . . . . . . . . . . . . . 441 5.2. Cambio de parametrizaci´ on . . . . . . . . . . . . . . . 446 5.2.1. Reparametrizaci´on . . . . . . . . . . . . . . . . 447 5.3. Independencia de trayectoria . . . . . . . . . . . . . . . 449 5.4. Campos Conservativos . . . . . . . . . . . . . . . . . . 451 5.4.1. Campo gradiente . . . . . . . . . . . . . . . . . 451 5.4.2. Teorema de Green . . . . . . . . . . . . . . . . 456 5.5. Aplicaciones de la integral de trayectoria . . . . . . . . 461 5.5.1. ´ Area de una pared . . . . . . . . . . . . . . . . 465 5.6. Aplicaciones de la integral de l´ınea . . . . . . . . . . . 467 5.7. Problemas Resueltos . . . . . . . . . . . . . . . . . . . 477 5.7.1. Campo conservativo . . . . . . . . . . . . . . . 481 5.7.2. Teorema de Green . . . . . . . . . . . . . . . . 485 5.8. Problemas propuestos . . . . . . . . . . . . . . . . . . . 492 5.8.1. Integral de trayectoria . . . . . . . . . . . . . . 492 5.8.2. Integral de l´ınea . . . . . . . . . . . . . . . . . . 493 5.8.3. Campos conservativos . . . . . . . . . . . . . . 494 5.8.4. Teorema de Green . . . . . . . . . . . . . . . . 495 5.9. Autoevaluaciones . . . . . . . . . . . . . . . . . . . . . 496 6. Integrales de superficie 504 6.1. Superficie orientada . . . . . . . . . . . . . . . . . . . . 507 6.1.1. Integral de flujo. . . . . . . . . . . . . . . . . . 508 6.1.2. Superficies Parametrizadas. . . . . . . . . . . . 510 6.1.3. Vector normal a S : . . . . . . . . . . . . . . . . 510 6.1.4. ´ Area de una superficie parametrizada . . . . . . 513 6.1.5. Integral de una funci´ on escalar sobre una super- ficie. . . . . . . . . . . . . . . . . . . . . . . . . 516 6.1.6. Integral de Superficie de campos vectoriales . . 517 6.1.7. Aplicaci´on al campo de la f´ısica: . . . . . . . . . 519 xii 6.2. Teoremas de Gauss y de Stokes . . . . . . . . . . . . . 519 6.2.1. Divergencia . . . . . . . . . . . . . . . . . . . . 519 6.2.2. Teorema de la divergencia de Gauss. . . . . . . 520 6.2.3. Teorema de Stokes. . . . . . . . . . . . . . . . . 524 6.3. Problemas Resueltos . . . . . . . . . . . . . . . . . . . 525 6.3.1. Integrales de superficie . . . . . . . . . . . . . . 529 6.3.2. Integral de Flujo de un campo vectorial . . . . . 532 6.3.3. Teorema de la divergencia de Gauss . . . . . . . 537 6.3.4. Teorema de Stokes . . . . . . . . . . . . . . . . 543 6.4. Ejercicios Propuestos . . . . . . . . . . . . . . . . . . . 550 6.4.1. ´ Area de una superficie . . . . . . . . . . . . . . 550 6.4.2. Integrales de funciones escalares sobre superficie 553 6.4.3. Integral de Flujo . . . . . . . . . . . . . . . . . 555 6.4.4. Teorema de la divergencia de Gauss . . . . . . . 557 6.4.5. Teorema de Stokes . . . . . . . . . . . . . . . . 561 6.5. Aplicaciones . . . . . . . . . . . . . . . . . . . . . . . . 564 6.5.1. Aplicaciones Integral de Flujo . . . . . . . . . . 564 6.5.2. Aplicaci´on del teorema de Gauss . . . . . . . . 568 6.5.3. Aplicaci´ on teorema de Stokes . . . . . . . . . . 573 6.5.4. Aplicacion teorema de Green . . . . . . . . . . 576 6.5.5. Aplicaciones al electromagnetismo . . . . . . . 580 6.6. Auto evaluaciones . . . . . . . . . . . . . . . . . . . . . 587 xiii Cap´ıtulo 1 Serie de Fourier En el presente cap´ıtulo se estudiar´an los conceptos b´asicos , m´etodos de c´alculo de los coeficientes y condiciones de convergencia para repre- sentar funciones mediante series e integrales de Fourier . 1.1. Introducci´ on Las funciones peri´ odicas se presentan frecuentemente en una gran var- iedad de problemas de f´ısica e ingenier´ıa, tales como propagaci´ on de ondas en un medio, conducci´ on del calor a lo largo de una varilla , resonancia nuclear magn´etica ,en consecuencia, abordar la soluci´ on de tales problemas, requiere del estudio de la serie de Fourier. La serie de Fourier es la representaci´ on de una funci´on en t´erminos de una serie trigonom´etrica infinita cuyas bases son las funciones seno y coseno. Algunas de las ventajas de ´esta representaci´on sobre otras representaciones, tales como, las series de Taylor, son: a) primero, se puede representar funciones peri´odicas en t´erminos de las bases seno y coseno que tienen diferentes frecuencias; b) segundo, se puede representar funciones discontinuas en un punto o seccionalmente continuas en un n´ umero finito de puntos; c) tercero, permite encontrar la respuesta de un sistema que es pertur- bado por una funci´ on peri´ odica, en t´erminos de una frecuencia funda- mental y cada una de las frecuencias arm´ onicas. 1 1.2. Propiedades Generales Para problemas con condiciones de frontera peri´ odicas en el intervalo −L ≤ x ≤ L, nos preguntamos si es posible expresar una funci´on como una combinaci´ on lineal de funciones seno y coseno de frecuencias cada vez mayores, como la siguiente serie infinita (conocida como serie de Fourier de f(x)): f(x) = a 0 + ∞ n=1 (a n cos _ nπx L _ + b n sin _ nπx L _ ) (1,1,1) Obviando la igualdad, vale preguntarse ¿converge esta serie infinita?,¿qu´e condi- ciones debe cumplir f para que se d´e la convergencia?,¿cu´ando converge a f(x)? Estas preguntas no tienen una respuesta sencilla. Sin embargo, las series de Fourier normalmente funcionan bastante bien. Supongamos que f admite desarrollo en serie de Fourier, ¿c´ omo se ob- tienen los coeficientes a 0 , a n y b n en t´erminos de f(x) ?. Para responder esta ´ ultima pregunta necesitaremos del siguiente lema. 1.2.1. Lema Elemental Lema 1.2.1. i) Si m y n son n´ umeros enteros no negativos distintos, entonces: _ L −L cos _ nπx L _ cos _ mπx L _ dx = _ L −L sin _ nπx L _ sin _ mπx L _ dx = 0 (1.2.1) ii) Para cualquier par de enteros no negativos m y n,entonces: _ L −L cos _ nπx L _ sin _ mπx L _ dx = 0 (1.2.2) iii)Para cualquier entero positivo n, entonces: _ L −L cos 2 _ nπx L _ dx = _ L −L sin 2 _ nπx L _ dx = L (1.2.3) 2 Demostraci´ on: Se prueba integrando directamente: usando la identidad cos αcos β = cos(α −β) + cos(α + β) 2 i) _ L −L cos _ nπx L _ cos _ mπx L _ dx = 1 2 _ L −L cos _ (n −m)πx L _ dx+ 1 2 _ L −L cos _ (n + m)πx L _ dx = 1 2 L (n −m) π sin _ (n −m) πx L _¸ ¸ ¸ ¸ L −L + 1 2 L (n + m) π sin _ (n + m) πx L _¸ ¸ ¸ ¸ L −L = 0 Adem´ as, si m = 0 y n ,= 0 es facilmente verificable que la integral es cero. En forma similar se prueba que _ L −L sin _ nπx L _ sin _ mπx L _ dx = 0 ii) Usando la identidad trigonom´etrica sin αcos β = sin(α −β) + sin(α + β) 2 _ L −L cos _ nπx L _ sin _ mπx L _ dx = 1 2 L _ −L sin _ (n −m) πx L _ dx + 1 2 L _ −L sin _ (n + m) πx L _ dx = − 1 2 L (n −m) π cos _ (n −m) πx L _ [ L −L − 1 2 L (n + m) π sin _ (n + m) πx L _ [ L −L = 0 A estas f´ormulas integrales se les llama relaciones de ortogonalidad y diremos que en tal caso el conjunto de las funciones _ cos _ nπx L _ , sin _ mπx L __ ∀ n = 0, 1, 2, ..., y ∀ m = 1, 2, ..., son ortogonales en [−L, L] iii) La demostraci´ on queda como ejercicio para el lector, se prueba 3 integrando directamente.En s´ıntesis, se puede puntualizar que: 1 L _ L −L cos _ nπx L _ cos _ mπx L _ dx = _ 0, si m ,= n 1, si m = n _ = δ m,n 1 L _ L −L sin _ nπx L _ sin _ mπx L _ dx = _ 0, si m ,= n 1, si m = n _ = δ m,n donde δ m,n se define como el delta de Kroneker. 1 L _ L −L cos _ nπx L _ sin _ mπx L _ dx = 0 ∀m, n _ L −L cos _ nπx L _ dx = 0 ∀m, n ; y _ L −L sin _ mπx L _ dx = 0 ∀m, n 1.3. La serie de Fourier de una funci´on Se debe distinguir entre f(x) y su serie de Fourier en el intervalo −L ≤ x ≤ L: Serie de Fourier de f(x) a 0 + ∞ n=1 _ a n cos _ nπx L _ + b n sin _ nπx L __ La serie trigonom´etrica puede incluso no converger y si converge, puede que no lo haga a f(x). Partiendo del supuesto que la serie converge podr´ıamos determinar los coeficientes de Fourier a 0 , a n y b n , usando las relaciones de ortogonalidad. Sea f(x) definida en el intervalo −L ≤ x ≤ L: f(x) = a 0 + ∞ n=1 _ a n cos _ nπx L _ + b n sin _ nπx L __ (1.3.4) Integrando la identidad ( 1.3.4) se tiene: _ L −L f(x)dx = _ L −L a 0 dx+ ∞ n=1 _ a n _ L L cos _ nπx L _ dx + b n _ L −L sin _ nπx L _ dx _ 4 Como todas las integrales de la derecha valen cero, excepto la primera, se deduce de aqu´ı el valor de a 0 , suponiendo que la L _ −L f(x)dx existe, as´ı. a 0 = 1 2L L _ −L f(x)dx Para el c´ alculo de a n multiplicamos la identidad ( 1.3.4) por cos _ mπx L _ e integramos la serie t´ermino a t´ermino, queda _ L −L f(x) cos _ mπx L _ dx = a 0 _ L −L cos _ mπx L _ dx+ ∞ n=1 _ a n _ L L cos _ nπx L _ cos _ mπx L _ dx + b n _ L −L sin _ nπx L _ cos _ mπx L _ dx _ = = 0 + ∞ n=1 a n Lδ n,m + 0 = La m Por lo tanto, al evaluar δ n,m , queda un s´ olo t´ermino: _ L −L f(x) cos _ mπx L _ dx = a m L, as´ı el valor de a m es a m = 1 L _ L −L f(x) cos _ mπx L _ dx, ∀ m ≥ 1 Cambiando el ´ındice libre m por n , en ambos lados de la ecuaci´on, queda a n = 1 L _ L −L f(x) cos _ nπx L _ dx, ∀ n ≥ 1 Ahora, multiplicando ( 1.3.4) por sin _ mπx L _ e integrando de manera similar y por el lema se tiene b n = 1 L _ L −L f(x) sin _ nπx L _ dx, ∀ n ≥ 1 Hemos determinado los coeficientes a 0 , a n y b n ,claro que, bajo muchos supuestos. Estos c´ alculos sugieren la siguiente definici´ on. 5 1.3.1. Coeficientes de Fourier Definici´ on 1.- i) Sea f una funci´on Riemann integrable en [−L, L], las constantes a 0 = 1 2L L _ −L f(x)dx a n = 1 L _ L −L f(x) cos _ nπx L _ dx para n = 1, 2, 3, ... b n = 1 L _ L −L f(x) sin _ nπx L _ dx para n = 1, 2, 3, ... _ ¸ ¸ ¸ ¸ ¸ ¸ ¸ ¸ ¸ _ ¸ ¸ ¸ ¸ ¸ ¸ ¸ ¸ ¸ _ 2,1,1 se denominan los coeficientes de Fourier de f en [−L, L]. ii) La serie: f(x) ∼ a 0 + ∞ n=1 _ a n cos _ nπx L _ + b n sin _ nπx L __ es la serie de Fourier de f en el intervalo [−L, L] , cuando los coefi- cientes est´an dados por (2,1,1). Para no hablar de convergencia todav´ıa, escribimos el signo”∼”que significa que a la derecha se tiene la serie de Fourier de f en −L ≤ x ≤ L. Observese que, la serie de Fourier de f, se puede interpretar como una generalizaci´ on de una combinaci´on lineal en una base ortogonal seno, coseno, que es aplicada a una funci´ on en lugar de un vector est´andar en R n . El siguiente ejemplo ilustra como dada una funci´ on peri´ odica f(x), de per´ıodo 2π, se calculan los coeficientes de Fourier y expresa la serie trigonom´etrica de Fourier correspondiente. Ejemplo 1: Determinar la serie de Fourier de f(x) = x si x ∈ [−π, π] 6 Soluci´on: La gr´afica de la funci´ on es: Los coeficientes de Fourier de f en [−π, π] , son: a 0 = 1 2π π _ −π xdx = 0 a n = 1 π π _ −π x cos (nx) dx = _ 1 n 2 π cos(nx) + x nπ sin(nx) ¸ π −π = 0 b n = 1 π π _ −π x sin(nx)dx = _ 1 n 2 π sin(nx) − x nπ cos(nx) ¸ π −π ∴ b n = 2 n (−1) n+1 ∀ n ≥ 1 Por tanto, la serie de Fourier de f en[−π, π] es: ∞ n=1 2 n (−1) n+1 sin(nx) 1.3.2. Atributos de la funci´ on Lo anteriormente expuesto es v´alido para cierto tipo de funciones, nos referimos a las funciones f(x) que son seccionalmente continuas. Definici´ on 2.- Sea f(x) definida en [a, b]. Entonces f es seccional- mente continua en [a, b] si: a) f es continua en [a, b] ,excepto quiz´as en un n´ umero finito de puntos. b) Ambos l´ım x→a + f(x) y l´ım x→b − f(x) existen y son finitos. c) f no es continua en x 0 , x 0 ∈ (a, b) y los l´ımites l´ım x→x + 0 f(x) y l´ım x→x − 0 f(x) existen y son finitos . 7 Definici´ on 3.- f(x) es seccionalmente suave en [a, b] si f y f son seccionalmente continuas en [a, b] . Ejemplo 2: Muestre que f(x) = x 1 3 no es seccionalmente suave en ning´ un intervalo cerrado que contenga en su interior al cero. Soluci´on: En efecto, se tiene que f (x) = 1 3 x −2 3 =⇒ l´ım x→0 f (x) = l´ım x→0 1 3 x −2 3 = ∞, no existe. Por tanto, la funci´on no es seccionalmente suave. Observaci´ on: Las funciones seno y coseno, que aparecen como bases en la serie de Fourier, tienen per´ıodos diferentes los que son iguales a 2L n para n ≥ 1. Por otra parte, un m´ ultiplo entero del per´ıodo de una funci´ on per´ıodica es tambi´en un per´ıodo , podemos afirmar entonces, que 2L es el per´ıodo com´ un para las funciones seno y coseno del desarrollo de la serie. Por lo anterior, la serie de Fourier no s´ olo representa a f en el intervalo −L ≤ x ≤ L , sino que, proporciona una extensi´on per´ıodica de f en todos los reales. 8 Ejemplo 3: Encontrar el per´ıodo de la funci´ on f(x) = 100 cos 2 x. Soluci´on: Utilizando la identidad trigonom´etrica cos 2 θ = 1 2 (1+cos 2θ) se tiene f(x) = 100 cos 2 x = 100 1 2 (1 + cos 2x) luego queda f(x) = 50 + 50 cos 2x como el per´ıodo de cos 2x es π y una funci´on constante tiene cualquier per´ıodo, entonces f(x) es de per´ıodo π. 1.3.3. Convergencia de las series de Fourier A continuaci´on vamos a establecer las condiciones de suficiencia que debe cumplir una funci´on f(x) para que pueda ser representada por medio de una serie de Fourier. Teorema 1.3.1. Si f(x) es seccionalmente suave en el intervalo [−L, L], entonces la serie de Fourier de f(x) converge. i) A la ex- tensi´on per´ıodica de f(x), en los puntos que la extensi´on per´ıodica sea continua. ii) Al promedio de los l´ımites laterales 1 2 (f(x + ) + f(x − )) en los puntos donde la extensi´on per´ıodica tenga una discontinuidad de salto. En el siguiente ejemplo, se eval´ ua si la serie de Fourier resultante de una funci´on f(x) en un punto x 0 dado converge o no en ese punto 9 Ejemplo 4: Sea f(x) = _ 0 si −3 ≤ x ≤ 0 x si 0 ≤ x ≤ 3 .Construir la serie de Fourier y analizar la convergencia en todo R Soluci´on: Representemos la gr´afica de la funci´ on Los coeficientes de la serie de Fourier de f(x),son: a 0 = 1 6 3 _ −3 f(x)dx = 1 6 3 _ 0 xdx = 3 4 a n = 1 3 3 _ −3 f(x) cos _ nπx 3 _ dx = 1 3 _ 3 0 x cos _ nπx 3 _ dx = 1 3 ¸ ¸ ¸ ¸ ¸ 9 cos _ nπx 3 _ n 2 π 2 + 3x sin _ nπx 3 _ nπ ¸ ¸ ¸ ¸ ¸ 3 0 = 3 n 2 π 2 (cos(nπ) −1) = 3 n 2 π 2 ((−1) n −1) b n = 1 3 3 _ −3 f(x) sin _ nπx 3 _ dx = 1 3 _ 3 0 x sin _ nπx 3 _ dx = 1 3 ¸ ¸ ¸ ¸ ¸ 9 sin _ nπx 3 _ n 2 π 2 + 3x cos _ nπx 3 _ nπ ¸ ¸ ¸ ¸ ¸ 3 0 = − 3 nπ cos(nπ) = − 3 nπ (−1) n Por consiguiente, la serie de Fourier la podemos escribir 3 4 + ∞ n=1 _ 3 n 2 π 2 ((−1) n −1) cos _ nπx 3 _ − 3 nπ (−1) n sin _ nπx 3 _ _ 10 Tenemos que f es continua en [−3, 3] ,por lo tanto su extensi´on per´ıod- ica es seccionalmente continua en R , con discontinuidad de salto en los puntos x = 3 ±6n, n ∈ Z Por lo tanto, de acuerdo al teorema la serie converge a f E (x) = _ f(x) si x ,= 3 ±6n 3 2 si x = 3 ±6n n ∈ Z entonces f E (x) = 3 4 + 3 π 2 ∞ n=1 _ 1 n 2 ((−1) n −1) cos _ nπx 3 _ − π n (−1) n sin _ nπx 3 _ _ los coeficientes ((−1) n −1) son nulos, si n es n´ umero par e iguales a −2, si n es n´ umero impar. Entonces f(x) = 3 4 − 6 π 2 ∞ n=1 _ 1 (2n −1) 2 cos _ (2n −1)πx 3 _ + π 6n (−1) n sin _ nπx 3 _ _ Al evaluar la convergencia en x 0 = 3, punto de discontinuidad de la funci´ on, se obtiene 3 2 = 3 4 − 6 π 2 ∞ n=1 _ 1 (2n −1) 2 (−1) 2n−1 _ =⇒ ∞ n=1 1 (2n −1) 2 = π 2 8 Obs´ervese que a partir de la convergencia de la serie de Fourier en un punto se puede inferir la convergencia de la suma de t´erminos de la serie resultante. Definici´ on 4.- Una suma parcial de la serie de Fourier es una suma de la forma: S n = a 0 + N n=1 a n cos _ nπ L x _ + b n sin _ nπ L x _ Observaci´on. Al truncar la serie infinita se obtiene un polinomio de grado n. 11 Ejemplo 5 Sea f(x) = x + π, x ∈ [−π, π] . Determine la serie de Fourier y obtener la gr´ afica de sumas parciales S 1 (x), S 3 (x), S 1 0(x). Soluci´ on : La gr´afica de la funci´ on es Los coeficientes de Fourier de f en [−π, π] a 0 = 1 2π π _ −π (x + π)dx = 1 2π _ x 2 2 + πx _ π −π = 1 2π _ 2π 2 _ = π ∴ a 0 = π a n = 1 π π _ −π (x + π) cos (nx) dx = 1 π _ 1 n 2 cos(nx) + x n sin(nx) _ π −π ∴ a n = 0 b n = 1 π π _ −π (x + π) sin(nx)dx = 1 π _ 1 n 2 sen(nx) − x n cos s(nx) _ π −π ∴ b n = 2 n (−1) n+1 As´ı la serie de Fourier de f (x) es π + 2 ∞ n=1 (−1) n+1 n sin(nx) = π + 2 _ sin x − sin 2x 2 + sin3x 3 −.. _ Para visualizar la convergencia de est´a serie gr´afiquemos algunas de sus sumas parciales S n (x) = π + 2 n k=1 (−1) n k sin(kx) 12 Obtengamos S 1 : S 1 (x) = π + 2 1 k=1 (−1) n k sin(kx) 13 Obtengamos S 3 S 3 (x) = π + 2 3 k=1 (−1) n k sin(kx) Finalmente Obtengamos S 10 S 10 = π + 2 10 k=1 (−1) n k sin(kx) A partir de este ejemplo, podemos inferir que para las series de Fourier las gr´ aficas de las sumas parciales son curvas aproximadas de la gr´ afica de la funci´ on per´ıodica representada por la serie. Se puede visualizar adem´ as, que en la medida que es mayor el n´ umero de t´erminos de las sumas parciales estas convergen de mejor forma a la gr´ afica de la funci´ on f. 14 1.3.4. La integral de funciones pares e impares Lema 1.3.1. (de funciones pares e impares) Sea f una funci´on in- tegrable en [−L, L] . a) Si f una funci´on par en [−L, L], entonces L _ −L f(x)dx = 2 L _ 0 f(x)dx. b) Si f es funci´on impar en [−L, L], entonces L _ −L f(x)dx = 0. Demostraci´ on a) f funci´ on par, entonces f(−x) = f(x) ∀x ∈ R. Considerando que f es par y el cambio de variable t = −x se tiene 0 _ −L f(x)dx = 0 _ −L f(−x)dx = L _ 0 f(t)dt = L _ 0 f(x)dx entonces L _ −L f(x)dx = 0 _ −L f(x)dx + L _ 0 f(x)dx = 2 L _ 0 f(x)dx b) f funci´ on impar, entonces f(−x) = −f(x) ∀x ∈ R. Usando este hecho y el cambio de variable t = −x se tiene 0 _ −L f(x)dx = 0 _ −L −f(−x)dx = 0 _ L f(t)dt = L − _ 0 f(x)dx entonces L _ −L f(x)dx = 0 _ −L f(x)dx + L _ 0 f(x)dx = L _ 0 f(x)dx − L _ 0 f(x)dx = 0 lo que demuestra el lema. A continuaci´on, vamos a determinar los coeficientes y la serie de Fourier coseno (o seno) seg´ un corresponda, dada una funci´ on f par (o impar) de periodo 2L. 15 1.3.5. Teorema de las funciones pares y de las im- pares Teorema 1.3.2. Sea f una funci´on integrable en [−L, L], a) Si f es par, la serie de Fourier de f en [−L, L] es a 0 + ∞ n=1 a n cos _ nπx L _ con coeficientes a 0 = 1 L L _ 0 f(x)dx y a n = 2 L L _ 0 f(x) cos _ nπx L _ dx, se denomina serie de cosenos. b) Si f es impar, la serie de Fourier de f en [−L, L] es ∞ n=1 b n sin _ nπx L _ con coeficiente b n = 2 L _ L 0 f(x) sin _ nπx L _ dx, se denomina serie de senos. Demostraci´ on: Se deja al lector, debe aplicar el Lema 1.3.1 en el c´alculo de los coeficientes de Fourier. Ejemplo 6: Calcule la serie de Fourier de f(x) = 1 − [x[ en −2 ≤ x ≤ 2. Soluci´on: A partir de la gr´ afica de la funci´on podemos inferir que la funci´ on es par. Es decir f(−x) = 1 − [−x[ = 1 − [x[ = f(x) ∀x ∈ R, luego se tiene que f es par. Los coeficientes del desarrollo de Fourier, son: 16 a 0 = 1 2 2 _ 0 (1 −x)dx = 1 2 _ x − x 2 2 _ ¸ ¸ ¸ ¸ ¸ ¸ 2 0 = 0 a n = 2 2 _ 2 0 (1 −x) cos _ nπx 2 _ dx = _ 2 0 cos _ nπx 2 _ dx − _ 2 0 x cos _ nπx 2 _ dx = 0 − _ 4 cos _ nπx 2 _ n 2 π 2 + 2x sin _ nπx 2 _ nπ _ 2 0 por consiguiente a n = _ 0 si n es par 8 (2n−1) 2 π 2 si n es impar As´ı la serie de Fourier de f(x) = 1 −[x[ es: 8 π 2 ∞ n=1 1 (2n −1) 2 cos _ (2n −1)πx 2 _ En muchos problemas se tiene la posibilidad de trabajar con series de senos o series de cosenos. Por ejemplo , al resolver ecuaciones diferen- ciales parciales de segundo orden aplicando el m´etodo de separaci´on de variables. 1.4. Desarrollos llamados de medio rango Sea una funci´on f seccionalmente continua que est´ a definida s´ olo en el semi-intervalo [0, L], queremos obtener el desarrollo de f en serie de Fourier ∀x ∈ [0, L] . Una forma de hacer lo anterior es extender f al intervalo [−L, L] y por supuesto , puede ser hecho de muchas maneras, sin embargo, dos extensiones son las m´as convenientes e importantes. Construir una extensi´on impar lo que origina una serie de senos o construir un extensi´on par lo que determina una serie de cosenos. Estas se denominan desarrollos de medio rango. 1.4.1. Extensi´on impar: Supongamos que conocemos f(x) solamente para 0 ≤ x ≤ L, entonces podemos extenderla como una funci´on impar, obteniendo otra funci´ on 17 denotada f i (x) definida por: f i (x) = _ f(x), 0 ≤ x ≤ L −f(−x), −L ≤ x ≤ 0 como se muestra en la figura adjunta. Si f(x) es seccionalmente suave en 0 ≤ x ≤ L, entonces f i (x) es tambi´en seccionamente suave y se puede aplicar el teorema de convergencia de series de Fourier. La serie de Fourier de f i (x) es f i (x) = ∞ n=1 b n sin _ nπx L _ , −L ≤ x ≤ L Como estamos interesados solamente en lo que ocurre entre 0 ≤ x ≤ L. En esa regi´ on f(x) es id´entica a f i (x) y la serie de Fourier es f(x) = ∞ n=1 b n sin _ nπx L _ , 0 ≤ x ≤ L con coeficiente b n = 2 L _ L 0 f(x) sin _ nπx L _ Ejemplo 7. Sea la funci´on f(x) = x en el interior 0 ≤ x ≤ L. Obtener el desarrollo de medio rango considerando una extensi´on im- par. 18 Soluci´on. Consideremos la extensi´ on impar de f(x) en 0 ≤ x ≤ L, la gr´ afica de f muestra que la serie de fourier de senos converge a f(x) en 0 ≤ x ≤ L. Sin embargo, en x = L hay una discontinuidad de salto, luego la serie converge a cero aunque f(L) ,= 0. b n = 2 L _ L 0 f(x) sin _ nπx L _ dx = 2 L _ L 0 x sin _ nπx L _ dx = 2L nπ (−1) n+1 Por lo tanto, la serie resultante es: x = 2L π ∞ n=1 (−1) n+1 n sin _ nπx L _ , 0 ≤ x ≤ L 1.4.2. Extensi´on par Supongamos ahora que conocemos f(x) solamente para 0 ≤ x ≤ L , entonces la extendemos como funci´on par, obteniendo otra funci´ on denotada f p (x) definida por: f p (x) = _ f(x), 0 ≤ x ≤ L f(−x), −L ≤ x ≤ 0 como muestra la figura adjunta: Si f(x) es seccionalmente continua en 0 ≤ x ≤ L, entonces su extensi´ on par f p (x) lo ser´ a tambi´en por lo que se puede aplicar el teorema de convergencia de series de Fourier. En el intervalo 0 ≤ x ≤ L, la funci´ on f(x) es id´entica a su extensi´ on par. La serie que se obtiene se denomina serie de Fourier de cosenos de f(x). a 0 + ∞ n=1 a n cos _ nπx L _ , 0 ≤ x ≤ L, con coeficientes 19 a 0 = 1 L _ L 0 f(x)dx y a n = 2 L _ L 0 f(x) cos _ nπx L _ dx Ejemplo 8: Construir la serie de Fourier de Cosenos de f(x) = x en 0 ≤ x ≤ L. Soluci´on: Por las caracter´ısticas de la extensi´ on en lo que concierne a la continuidad de la funci´ on tenemos: x = a 0 + ∞ n=1 a n cos _ nπx L _ , 0 ≤ x ≤ L a 0 = 1 L _ L 0 f(x)dx = 1 L _ L 0 xdx = 1 L x 2 2 ¸ ¸ ¸ ¸ L 0 = L 2 a n = 2 L _ L 0 f(x) cos _ nπx L _ dx = 2 L _ L 0 f(x) cos _ nπx L _ dx a n = _ 0 si n par. − 4L n 2 π 2 si n impar. 20 Finalmente, la serie de Fourier coseno de f(x) = x en 0 ≤ x ≤ L es: L 2 − 4L π 2 ∞ n=1 1 (2n −1) 2 cos _ (2n −1)πx L _ 1.5. Diferenciacion e Integraci´ on de la se- ries de Fourier 1.5.1. Derivaci´ on Las series infinitas, a´ un las convergentes no siempre se pueden derivar t´ermino a t´ermino. Un caso ilustrativo, es el de la funci´ on f(x) = x definida para −π ≤ x ≤ π, cuya serie de Fourier es ∞ n=1 2(−1) n+1 n sin(nx) que converge para −π < x < π, es decir x = ∞ n=1 2(−1) n+1 n sin(nx), x ∈ ]−π, π[ Si diferenciamos, esta serie t´ermino a t´ermino tenemos: ∞ n=1 2(−1) n+1 cos(nx) la cual es una serie que no converge en ] − π, π[ , ya que si a n = 2(−1) n+1 cos(nx) para cada x ∈] −π, π[, l´ım n→∞ a n no existe, como no ocurre que a n −→0 ,concluimos que ∞ n=1 2(−1) n+1 cos(nx) no converge para cada x ∈] −π, π[. Por otro lado, f (x) = 1 ∀x ∈] −π, π[. Esto muestra en este caso que la derivada t´ermino a t´ermino de la serie, no converge a la derivada de la funci´on que representa. La dificultad se nos presenta cada vez que la serie de Fourier de f(x) tiene una discontinuidad de salto, la derivaci´ on t´ermino a t´ermino no est´ a justificada en estos casos. Sin embargo, podemos aqu´ı considerar el siguiente teorema que precisa las condiciones para permitir la derivaci´ on t´ermino a t´ermino. 21 Teorema 1.5.1. Sea f una funci´on continua en [−L, L] con f(−L) = f(L), si f es seccionalmente suave en [−L, L] donde f (x) existe se tiene. f (x) = ∞ n=1 nπ L _ −a n sin _ nπx L _ + b n cos _ nπx L __ Demostraci´ on.- Se deja al lector, se sugiere escribir la serie de Fourier de f (x), con- siderando que esta serie converge a f (x) siempre que f (x) exista. Use integraci´ on por partes para relacionar los coeficientes de f (x) con los correspondientes de f(x). Ejemplo 9. Dada la funci´ on f(x) = x 2 en −π ≤ x ≤ π , verifique si la derivada de esta serie existe. Soluci´on Claramente se satisface las hip´ otesis de la proposici´on an- terior. La serie de Fourier de la funci´ on f(x) en [−π, π] es: (Ver Problema 2 en problemas resueltos) f(x) = π 2 3 + 4 ∞ n=1 (−1) n n 2 cos(nx) Como f (x) = 2x es continua, y existe f (x) = 2 en todo el intervalo, entonces para −π < x < π f (x) = 2x = 4 ∞ n=1 (−1) n+1 n sin(nx) Note que este resultado concuerda con lo establecido en el ejemplo 1 del inciso 2.1. 22 1.5.2. Integraci´ on La precauci´ on que se tiene para la derivaci´on t´ermino a t´ermino de la serie de Fourier no se requiere para el caso de la integraci´ on . Teorema 1.5.2. Sea f una funci´on seccionalmente suave en [−L, L] con serie de Fourier f(x) = a 0 + ∞ n=1 _ a n cos _ nπx L _ + b n sin _ nπx L __ Entonces para cada x ∈ [−L, L] . _ x −L f(t)dt = a 0 (x+L)+ L π ∞ n=1 1 n _ a n sin _ nπx L _ −b n _ cos _ nπx L _ −(−1) n __ Demostraci´ on; Sea F(x) = x _ −L f(t)dt −a 0 x ∀x ∈ [−L, L] , as´ı definida F es continua en [−L, L] , adem´ as F(−L) = −L _ −L f(t)dt−a 0 (−L) = a 0 L y F(L) = L _ −L f(t)dt−a 0 L = 2a 0 L−a 0 L = a 0 L Por lo cual F(−L) = F(L), asimismo F (x) = f(x) −a 0 ∀x ∈ [−L, L] donde f es continua. Entonces podemos asegurar que F es seccional- mente continua en [−L, L] y por el teorema de convergencia tenemos que F(x) = A 0 + ∞ n=1 _ A n cos _ nπx L _ + B n sin _ nπx L __ (1.5.5) donde para n ≥ 1. 23 A n = 1 L _ L −L F(t) cos _ nπt L _ dt integrando por partes = 1 L F(t) L nπ sin _ nπt L _¸ ¸ ¸ ¸ L L − L nπ _ L −L F (t) sin _ nπt L _ dt = 0 − L nπ _ L −L (f(t) −a 0 ) sin _ nπt L _ dt = − L nπ _ L −L f(t) sin _ nπt L _ dt + L nπ a 0 _ L −L sin _ nπt L _ dt A n = − L nπ b n donde b n es el coeficiente correspondiente de la serie de Fourier de f en [−L, L]. De manera analoga se tiene que: B n = 1 L _ L −L F(t) sin _ nπt L _ dt = L nπ a n donde a n es tambi´en el correspondiente coeficiente de la serie de Fourier de f en [−L, L]. Por lo tanto, reemplazando en 1.5.5 F(x) = A 0 + L π ∞ n=1 1 n _ −b n cos _ nπx L _ + a n sin _ nπx L __ , x ∈ [−L, L] para A 0 tenemos: F(L) = a 0 L = A 0 + ∞ n=1 A n cos(nπ) =⇒ A 0 = a 0 L − ∞ n=1 A n cos(nπ) finalmente A 0 = a 0 L + L π ∞ n=1 1 n b n cos(nπ) 24 ahora sustituyendo A 0 se tiene F(x) = a 0 L+ L π 1 n ∞ n=1 b n cos(nπ)+ L π ∞ n=1 1 n _ −b n cos _ nπx L _ + a n sin _ nπx L __ y reemplazando en la igualdad inicial obtenemos lo que afirma el teo- rema. _ x −L f(t)dt = a 0 (x+L)+ L π ∞ n=1 1 n _ a n sin _ nπx L _ −b n _ cos _ nπx L _ −(−1) n __ 1.5.3. Identidad de Parseval Sea f una funci´ on seccionalmente continua en [−L, L] y tal que f es tambi´en seccionalmente continua en [−L, L]. Si f(x) = a 0 + ∞ n=1 _ a n cos _ nπx L _ −b n sin _ nπx L __ es la serie de Fourier de f, entonces 1 L L _ −L [f(x)] 2 dx = 2 (a 0 ) 2 + ∞ n=1 _ (a n ) 2 + (b n ) 2 ¸ que se conoce como identidad de Parseval Prueba: La serie de Fourier de f converge a f(x) para cada x del intervalo [−L, L]. f(x) = a 0 + ∞ n=1 _ a n cos _ nπx L _ −b n sin _ nπx L __ Multiplicando por f(x) se tiene f(x) 2 = a 0 f(x) + ∞ n=1 _ a n f(x) cos _ nπx L _ −b n f(x) sin _ nπx L __ 25 podemos integrar t´ermino a t´ermino. _ L −L [f(x)] 2 dx = a 0 _ L −L f(x)dx+ ∞ n=1 _ a n _ L −L f(x) cos _ nπx L _ −b n _ L −L f(x) sin _ nπx L _ _ de aqu´ı recordando lo que son los coeficientes de una serie de Fourier se tiene. L _ −L [f(x)] 2 dx = 2 (a 0 ) 2 L + L ∞ n=1 [a n a n + b n b n ] =⇒ 1 L L _ −L [f(x)] 2 dx = 2 (a 0 ) 2 + ∞ n=1 _ (a n ) 2 + (b n ) 2 ¸ Obs´ervese que la identidad de Parseval, permite inferir la suma de una serie infinita, dada una funci´ on f que tiene una representaci´on de Fourier para cada x del intervalo [−L, L]. Ejemplo 10. Sea f(x) = _ x −π < x < π 0 x = −π, π , per´ıodica de per´ıodo 2π. Pruebe que ∞ n=1 1 n 2 = π 2 6 . Figura 1.1: gr´ afica funci´on per´ıodo 2π Soluci´on: Como f(x) en es una funci´ on impar se tiene que : a n = 0 para n = 0, 1, 2, ...y 26 b n = 1 π π _ −π x sin (nπ) dx = 2 π π _ 0 x sin (nπ) dx = − _ 2x cos(nx) nπ _ π 0 =⇒ b n = _ 2 n n = 1, 3, 5, ... − 2 n n = 2, 4, 6, ... Por tanto f(x) ∼ 2 ∞ n=1 (−1) n+1 sin(nx) n = 2 _ sin x 1 − sin 2x 2 + sin 3x 3 ... _ Aplicando la identidad de Parseval 1 π π _ −π x 2 dx = 4 _ 1 1 2 + 1 2 2 + 1 3 2 + 1 4 2 + ... _ =⇒ ∞ n=1 1 n 2 = 1 4π π _ −π x 2 dx = 1 4π _ x 3 3 _ π −π = π 2 6 ∞ n=1 1 n 2 = π 2 6 1.6. Integral de Fourier Las series de Fourier nos proporcionan una herramienta poderosa para representar funciones per´ıodicas. Luego, es conveniente generalizar este m´etodo para incluir funciones no per´ıodicas. A continuaci´on en esta secci´ on vamos a representar una funci´on f no per´ıodica por medio de la integral de Fourier Definici´ on.- Si f(x) definida en (−∞, ∞) es seccionalmente continua en cada intervalo finito y tiene derivadas por la derecha e izquierda en 27 todo punto y tal que ∞ _ −∞ [f(x)[ dx converge, entonces la integral de Fourier de f se define como: ∞ _ 0 [A(w) cos wx + B(w) sin wx] dw donde: A(w) = 1 π _ ∞ −∞ f(t) cos wtdt B(w) = 1 π _ ∞ −∞ f(t) sin wtdt A(w) y B(w) se llaman los coeficientes de la integral de Fourier de f(x). Ejemplo 11. Encontrar la representaci´ on por medio de la integral de Fourier de la funci´on: f(x) = _ 1 , [x[ < 1 0 , [x[ > 1 Soluci´on: Primeramente, determinemos la gr´afica de la funci´ on Ahora, calculemos los coeficientes de la Integral de Fourier A(w) = ∞ 1 π _ −∞ f(u) cos wudu = 1 _ −1 cos wudu = _ sin wu w _ [ 1 −1 = 2 1 π sin w w B(w) = ∞ 1 π _ −∞ f(u) sin wudu = 1 _ −1 sin wudu = 0 28 Por lo tanto, la integral de Fourier de f(x) es: 1 π ∞ _ 0 2 w sin wcos wxdw 1.6.1. Criterio de convergencia de la integral de Fourier Si f(x) es seccionalmente continua en [−L, L] ∀ L > 0 y tal que ∞ _ −∞ [f(t)[ dt existe, entonces la integral de Fourier converge a 1 2 [f(x + ) + f(x − )] (Promedio de los l´ımites izquierdo y derecho de f(x)), ∀ x donde f (x + ) y f (x − ) existen. Ejemplo 12. Estudie la convergencia de la Integral de Fourier del ejemplo 11 Soluci´on Sea f(x) definida en ejemplo 11, debido a que f(x) es sec- cionalmente suave, la integral de Fourier de f(x) converge a 1 2 [f(x + ) + f(x − )] ∀ x. De acuerdo con el criterio de convergencia se tiene: 2 π ∞ _ 0 sin w w cos wxdw = _ _ _ 1 si −1 < x < 1 1 2 si x = ±1 0 si [x[ > 1 En particular, una situaci´ on interesante se da cuando x = 0. 2 π ∞ _ 0 sin w w cos 0dw = 1 =⇒ ∞ _ 0 sin w w dw = π 2 Aunque integrales de este tipo no pueden expresarse en t´erminos de funciones elementales, suelen presentarse en matem´ aticas aplicadas con tal frecuencia , que han recibido un nombre especial y se encuentran 29 tabuladas. En particular sabemos que: l´ım w→0 sin w w = 1 y que ∞ _ 0 sin w w dw = π 2 En el caso de la integral de Fourier, la gr´ afica de la funci´on f se obtiene mediante aproximaciones sucesivas sustituyendo el l´ımite superior de la integral ∞ por los n´ umeros x. De aqu´ı que la integral z _ 0 sin w w cos wxdw es una aproximaci´ on de la integral encontrada anteriormente, y por lo tanto de f (x) . Supongamos que s´ olo consideramos las frecuencias w < w 0 .En este caso, nos da como aproximaci´ on de f (x) f (x) ≈ 2 π w 0 _ 0 cos wx sin w w dw 30 Ahora bien, cos wx sin w = sin (wx + w) −sin(wx −w) 2 y, por consiguiente, podemos escribir la ´ ultima integral en la forma f (x) ≈ 1 π w 0 _ 0 sin w(x + 1) w dw − 1 π w 0 _ 0 sin w(x −1) w dw Consideremos el cambio de variable u = w(x ±1) =⇒du = wdx para la primera y la segunda de estas integrales. Entonces tenemos f (x) ≈ 1 π w 0 (x+1) _ 0 sin u u du − 1 π w 0 (x−1) _ 0 sin u u du f (x) ≈ 1 π Si [w 0 (x + 1)] − 1 π Si [w 0 (x −1)] En t´erminos f´ısicos, estas curvas describen la salida de un filtro ideal de pasa baja, que elimina todas las frecuencias superiores w 0 cuando la se˜ nal de entrada es un impulso aislado rectangular. 1.6.2. Integrales de Fourier de cosenos y senos Sea f(x) una funci´ on definida en [0, ∞), podemos extender esta funci´ on a una funci´ on par o impar en (−∞, ∞) y calcular la integral de Fourier de esta ´ ultima, que resulta ser de coseno y seno respectivamente, lo cual es completamente an´ aloga a los desarrollos en cosenos y senos de una funci´on definida en un intervalo [0, L] para el caso de las series de Fourier. Definici´ on: Sea f definida en [0, ∞) y sea ∞ _ 0 [f(u)[ du convergente, la integral de Fourier en cosenos de f es ∞ _ 0 A(w) cos(wx)dw donde el coeficiente es: 31 A(w) = 2 π ∞ _ 0 f(u) cos(wu)du A su vez, la integral de Fourier en senos de f es ∞ _ 0 B(w) sin(wx)dw donde el coeficiente es: B(w) = 2 π ∞ _ 0 f(u) sin(wu)du En cuanto a la convergencia de la integral de Fourier, en este caso, si f es seccionalmente suave en todo el intervalo [0, ∞], entonces esta integral converge a 1 2 [f(x + ) + f(x − )] en (0, ∞). Ejemplo 13: Encontrar la integral de Fourier de f(x) = _ x 2 si 0 ≤ x ≤ 10 0 si x > 10 , si: a) se considera una extension par de f(x) b) se considera una extension impar de f(x); y luego c) en ambos casos, determinar las convergencias de estas integrales . Soluci´on: Consideremos la gr´ afica de la funci´ on a) Para obtener la integral de Fourier de cosenos, extendemos f como una funci´on par f P definida en toda la recta real, luego: 32 A(w) = 2 π ∞ _ 0 f(u) cos(wu)du = 2 π 10 _ 0 u 2 cos(wu)du = 2 π _ _ u 2 w sin(wu)[ 10 0 − 2 w 10 _ 0 usin(wu)du _ _ = 2 π _ u 2 w sin(wu) − 2 w _ 1 w 2 sin(wu) − u w cos(wu) __ 10 0 = 1 π _ 200 w − 4 w 3 _ sin 10w + 40 πw 2 cos 10w Por tanto, la integral de Fourier de cosenos es: 1 π ∞ _ 0 __ 200 w − 4 w 3 _ sin 10w + 40 w 2 cos 10w _ cos wxdw Al aplicar el criterio de convergencia obtenemos: 1 π ∞ _ 0 __ 200 w − 4 w 3 _ sin 10w + 40 w 2 cos 10w _ cos wxdw = _ ¸ ¸ _ ¸ ¸ _ x 2 si 0 < x < 10 0 si x > 10 0 si x = 0 50 si x = 10 33 b) Para obtener la integral de Fourier de senos, extendemos f como una funci´on impar f I definida en toda la recta real. B(w) = 1 π ∞ _ −∞ f(t) sin wtdt = 2 π 10 _ 0 u 2 sin wudu = 2 π _ _ _ − u 2 w cos wu _ 10 0 + 2 w 10 _ 0 ucos wudu _ _ = 2 π _ − u 2 w cos wu + 2 w _ 1 w 2 cos wu + u w sin wu __ 10 0 = 2 π _ − 10 2 w cos 10w + 2 w 3 cos 10w + 20 w 2 sin 10w − 2 w 3 _ entonces la integral de Fourier de senos es: 1 π ∞ _ 0 __ − 200 w + 4 w 3 _ cos 10w + 40 w 2 sin 10w − 4 w 3 _ sin wxdw Finalmente, al aplicar el criterio de convergencia obtenemos: 1 π ∞ _ 0 __ − 200 w + 4 w 3 _ cos 10w + 40 w 2 sin 10w − 4 w 3 _ sin wxdw = _ ¸ ¸ _ ¸ ¸ _ x 2 si 0 < x < 10 0 si x > 10 0 si x = 0 50 si x = 10 Ejemplo 14: Encontrar la integral de Fourier de f (x) = e −ax si x > 0 y a es una constante tal que a > 0, considerando una extensi´on a) par de f. b) impar de f. c) en ambos casos, determinar las convergencias de estas integrales. Soluci´on Extensi´ on Par 34 Extensi´ on impar a) Puesto que f es par , es decir f (x) = f (−x) ∀x ∈ R se tiene f (x) = ∞ _ 0 A(w) cos(wx)dw donde el coeficiente es: A(w) = 2 π ∞ _ 0 e −au cos(wu)du Integrando por partes se tiene ∞ _ 0 e −au cos(wu)du = − a a 2 + w 2 l´ım R→∞ _ e −au _ − w a senwu + cos wu __ R 0 = a a 2 + w 2 35 Por consiguiente, A(w) = 2 π a a 2 + w 2 Sustituyendo esta expresi´on se obtiene: 2a π ∞ _ 0 cos(wx) a 2 + w 2 dw para x > 0, a > 0. Finalmente, como la funci´ on es continua ∀x > 0, la integral converge a f (x) , entonces f (x) = e −ax = 2a π ∞ _ 0 cos(wx) a 2 + w 2 dw =⇒ ∞ _ 0 cos(wx) a 2 + w 2 dw = πe −ax 2a b) Puesto que f es impar , es decir f (x) = −f (−x) ∀x ∈ R se tiene f (x) = ∞ _ 0 B(w) sin(wx)dw donde el coeficiente es: B(w) = 2 π ∞ _ 0 e −au sin(wu)du Integrando por partes se tiene ∞ _ 0 e −au sin(wu)du = 1 a 2 + w 2 l´ım R→∞ _ e −au (asenwu −wcos wu) ¸ R 0 = w a 2 + w 2 36 Por consiguiente, B(w) = 2 π w a 2 + w 2 Sustituyendo esta expresi´on se obtiene: f (x) = e −ax = 2 π ∞ _ 0 wsin(wx) a 2 + w 2 dw para x > 0, a > 0. Estos ejemplos ilustran como puede aplicarse la representaci´ on de la integral de Fourier para evaluar integrales. 37 1.7. Aplicaciones de Series de Fourier Para dar una visi´on del uso de las series e integrales de Fourier, se for- mular´ an, analizar´ an y resolver´ an problemas de sistemas f´ısicos sujetos a perturbaciones peri´ odicas y no peri´odicas. 1.7.1. Onda cuadrada alta frecuencia Una aplicaci´ on simple de la Serie de Fourier la podemos encontrar en el an´ alisis de circuitos electr´ onicos que son dise˜ nados para manejar pulsos variables agudos, tales como, una onda cuadrada o un ”diente de sierra”. Supongamos que una onda cuadrada est´a definida por la funci´ on: f(x) = _ 0, −π < x < 0 h, 0 < x < π Encuentre la serie de Fourier que representa esta se˜ nal. Soluci´on Los coeficientes de Fourier son: a 0 = 1 2π _ π 0 hdt = h 2 a n = 1 π _ π 0 hcos ntdt = 0, n ≥ 1 b n = 1 π _ π 0 hsin ntdt = h nπ (1 −cos nπ) b n = _ 2h nπ , n impar =⇒ b n = 2h (2n−1)π 0 ; n par As´ı la serie resultante es: f(x) = ∞ n=1 2h (2n −1)π sin (2n −1) x = h 2 + _ sin x 1 + sin 3x 3 + sin 5x 5 + ... _ 38 Es importante decir que el primer t´ermino representa el promedio de f(x) sobre el intervalo [−π, π] y que todos los t´erminos en base coseno se anulan. Adem´ as f (x) − h 2 es una funci´ on impar, luego ,tenemos una serie de fourier s´olo con base seno. Por otra parte, los coficientes b n decrecen inversamente proporcional con n. Fisicamente esto signifi- ca que la onda cuadrada debe contener muchos componentes de alta frecuencia. Si el aparato electr´onico no deja pasar estos componentes, la onda cuadrada resultante emerge m´ as o menos redondeada. 1.7.2. Rectificador de onda completa. Consideremos ahora la salida de un rectificador de onda completa, que produce corriente continua pulsante como muestra la figura. El rectifi- cador se puede modelar como un dispositivo que se alimenta con una onda senoidal ,que deja pasar los los pulsos positivos, e invierte los pulsos negativos. Esto produce: f(x) = _ sin ωx, 0 < ωx < π −sin ωx, −π < ωx < 0 Encuentre la serie de Fourier que respresenta esta se˜ nal Soluci´ on Puesto que f (x) es una funci´ on par, es decir f (x) = f (−x), la serie de fourier ser´ a cosenoidal a 0 = 1 2π __ 0 −π −sin ωtd(ωt) + _ π 0 sin ωtd(ωt) _ = 2 2π _ π 0 sin ωtd(ωt) = 2 π a n = 2 π _ π 0 sin ωt cos nωtd(ωt), n ≥ 1 a n = _ −2 π 2 n 2 −1 , n par =⇒ a n = − 1 π 4 4n 2 −1 0, n impar b n = 0, ∀ n 39 Por lo tanto, la serie resultante es: f(x) = 2 π − 4 π ∞ n=1 1 (4n 2 −1) cos (2nωx) La frecuencia de oscilacion m´ as baja es 2ω.Las componentes de alta frecuencia decaen inversamente con n 2 , lo que muestra que el rectifi- cador de onda completa hace un buen trabajo para producir un modelo aproximado de la corriente continua. 1.7.3. Ecuaci´on de calor unidimensional El flujo unidimensional de calor en un cuerpo material homog´eneo est´ a modelado por la ecuaci´ on c 2 ∂ 2 u(x,t) ∂x 2 = ∂u(x,t) ∂t donde u(x, t) es la temperatura del cuerpo y c 2 = 2 la constante de difusi´on del calor. Si se considera que 0 < x < 3 y t > 0, y que las temperaturas en la fronteras son u(0, t) = u(3, t) = 0, lim x→0 u(x, t) < ∞ , entonces la soluci´ on general de este problema esta dado por: u(x, t) = ∞ n=1 C n e −2n 2 π 2 t sin nπx 3 , 0 < x < 3 y t > 0 Encontrar la temperatura de la barra , si la temperatura inicial es u(x, 0) = 25 o C , 0 < x < 3 . Soluci´ on: Evaluemos la soluci´on general para t = 0, lo que produce: u(x, 0) = 25 = ∞ n=1 C n sin nπx 3 , 0 < x < 3 Se obtiene una serie de Fourier seno. As´ı, para determinar la constante C n se debe construir una extensi´on impar f (x) = _ 25 0 < x < 3 −25 −3 < x < 0 . 40 Podemos encontrar entonces: C n = 2 L _ L 0 f (x) sin nπx L dx = 2 3 _ 3 0 25 sin nπx 3 dx C n = 50 3 _ − 3 nπ cos nπx 3 _ 3 0 = 50 (1 −cos nπ) nπ De modo, que la temperatura en la barra queda u(x, t) = ∞ n=1 50 (1 −cos nπ) nπ e −2n 2 π 2 t sin nπx 3 , 0 < x < 3 y t > 0 Este problema ilustra la importancia de la serie de Fourier para re- solver problemas de aplicaci´ on modelados por ecuaciones diferenciales parciales de segundo orden. 1.7.4. Ecuaci´on de calor: barra aislada El flujo unidimensional de calor en un cuerpo material homog´eneo est´ a modelado por la ecuaci´ on c 2 ∂ 2 u(x,t) ∂x 2 = ∂u(x,t) ∂t donde u(x, t) es la temperatura del cuerpo y c 2 la constante de difusi´ on del calor. En el caso de una barra aislada, que se prolonga hacia el infinito en ambos sentidos, la soluci´ on general est´ a dada por u(x, t) = _ ∞ 0 (A(w) cos(wx)+B(w) sin(wx) ) e −c 2 w 2 t dw. Si se aplica la condicion inicial u(x, 0) = f (x) , −∞< x < ∞ ,donde f(x) es la tem- peratura inicial, se obtiene que u(x, 0) = f (x) = _ ∞ 0 (A(w) cos(wx) + B(w) sin(wx) ) dw es una integral de Fourier con coeficientes A(w) = 1 π _ ∞ −∞ f (v) cos(wv) dv y B(w) = 1 π _ ∞ −∞ f (v) sin(wv) dv Determine la integral de Fourier, si la funci´on temperatura inicial es f(x) = e −x 2 /2 ; −∞< x < ∞, y la soluci´ on general de est´ a ecuaci´ on. Soluci´on: Como f es una funci´on par se tiene I p = _ ∞ 0 A(w) cos(wx) dw, con , y B(w) = 0 luego 41 A(w) = 2 π _ ∞ 0 e −v 2 /2 cos(wv) dv =⇒ A (w) = − 2 π _ ∞ 0 ve −v 2 /2 sin(wv) dv Integrando por partes se tiene A (w) = − 2 π _ −e −v 2 /2 sin(wv) + w _ ∞ 0 e −v 2 /2 sin(wv) dv _ ∞ 0 Evaluando la integral y resolviendo EDO(1) A (w) = − 2 π _ 0 + w( π 2 A(w) _ ∞ 0 =⇒ A (w) = −wA(w) A(w) = Ce −w 2 /2 , C constante Luego la integral de Fourier es: e −x 2 /2 = C _ π 0 e −w 2 /2 cos(wx) dw Por tanto, la soluci´ on general queda: u(x, t) = C _ ∞ 0 (e −w 2 /2 cos(wx)) e −c 2 w 2 t dw Este problema ilustra la importancia de la Integral de Fourier para resolver problemas de aplicaci´ on modelados por ecuaciones de difusion del calor. 1.7.5. Ecuaci´on de Onda Una onda unidimensional que se desplaza en una cuerda el´ astica ho- mog´enea, est´ a modelado por la ecuaci´ on c 2 ∂ 2 u(x,t) ∂x 2 = ∂ 2 u(x,t) ∂t 2 donde u(x, t) es el desplazamiento de la cuerda desde el eje x en el tiempo t y c 2 la constante la rapidez de la onda en el medio. Si los extremos de la cuerda est´an fijos en x = 0, x = L , t > 0, es decir que las condiciones de frontera son u(0, t) = u(L, t) = 0 , entonces la soluci´on general de este problema est´a dado por: u(x, t) = ∞ n=1 (A n cos nπct L + B n sin nπct L ) sin nπx L , 0 < x < L y t > 0 42 Considere que la forma inicial de la cuerda est´a dado por f (x) , es decir u(x, 0) = f (x) , y que la velocidad inicial de la cuerda es cero, es decir ∂u (x, t) ∂t = 0. Encontrar el desplazamiento u(x, t) de la cuerda en un tiempo posterior. Soluci´ on. Determinemos las constantes A n y B n de la soluci´on general aplicando las condiciones iniciales. Para satisfacer la condici´on ∂u (x, t) ∂t = 0 , ser´ a necesario derivar la soluci´ on general, entonces u t (x, t) = ∞ n=1 nπc L (−A n sin nπct L + B n cos nπct L ) sin nπx L u t (x, t) = ∞ n=1 nπc L B n sin nπx L ⇐⇒ B n = 0 ∀n De manera que la soluci´on general se reduce a u(x, t) = ∞ n=1 A n cos nπct L sin nπx L , 0 < x < L y t > 0 Ahora, apliquemos la condici´ on u(x, 0) = f (x) , para determinar la constante A n . Esto da como resultado u(x, 0) = f (x) = ∞ n=1 A n sen nπx L , 0 < x < L y t > 0 que corresponde a una serie de Fourier senoidal. As´ı, es necesario con- siderar una extension impar de la funci´ on dada f i (x) = _ f (x) si 0 < x < L −f (−x) si −L < x < 0 , de este modo el coeficiente queda A n = 2 L _ L 0 f (x) sin nπx L dx El resultado final es u(x, t) = ∞ n=1 _ 2 L _ L 0 f (x) sin nπx L dx _ cos nπt L sin nπx L , 0 < x < L y t > 0 43 1.7.6. Deflexi´on de una viga Una viga de longitud L , esta soportada desde sus extremos como mues- tra la figura adjunta . Sobre la viga act´ ua una carga uniformemente dis- tribuida q por unidad de longitud y su deflexi´ on est´ a dada por y (x) . Si se escoge la direcci´on del eje y apuntando hacia abajo, como indica la figura, se sabe que la funci´on y (x) satisface la ecuaci´on: d 4 y dx 4 = 1 EI q (x) donde q (x) es la carga por unidad de longitud en el punto x, I es el momento de inercia y E el m´odulo de elasticidad de la viga. Si en nuestro caso estas tres cantidades son constantes encuente la deflexi´ on y (x) de la viga. Soluci´on. Puesto que la funci´on y (x) debe se nula en los extremos x = 0 y x = L, la podemos representar mediante una serie de Fourier de senos. y (x) = ∞ n=1 b n sin nπx L , x ∈ [0, L] Si suponemos que y (x) es una funci´ on continua , con derivadas contin- uas hasta el cuarto orden en [0, L] , entonces y (4) (x) = ∞ n=1 _ nπx L _ 4 b n sin nπx L , x ∈ [0, L] A su vez la carga distrribuida por unidad de longitud q (x) = q, tambi´en puede ser desarrollada en serie de Fourier de senos q = ∞ n=1 q n sin nπx L , x ∈ [0, L] 44 de donde q n = 1 L _ L 0 q sin _ nπx L _ dx = _ 4 nπ n = impar 0 n = par Sustituyendo ambas series de Fourier en la ecuaci´ on diferencial ∞ n=1 _ nπx L _ 2 b n sin nπx L = ∞ n=1 q n sin nπx L , x ∈ [0, L] Comparando los coeficientes de ambas serie queda _ nπx L _ 2 b n = _ 4 nπ n = impar 0 n = par =⇒ b n _ 4qL 4 EIπ 5 1 n 5 n = impar 0 n = par Por tanto la deflexi´on queda determinada por y (x) = 4qL 4 EIπ 5 ∞ n=1 1 (2n −1) 5 sin (2n −1)πx L , x ∈ [0, L] 1.8. Problemas Propuestos Rectificador media onda La funci´ on adjunta sirve para modelar la salida de un rectificador de media onda: f(x) = _ sin ωx, 0 ≤ ωx ≤ π 0, −π ≤ ωx ≤ 0 a) Represente graficamente la se˜ nal de salida si ´esta se extiende peri- odicamente con periodo 2π. b) Determine la serie de Fourier que la representa. Soluci´on: f(x) = 1 π + 1 2 sin ωx − 2 π ∞ n=1 1 (4n 2 −1) cos (2nωx) Onda triangular Una onda tri´ angular se representa por la funci´on: f(x) = _ −x, −π < x < 0 x, 0 < x < h 45 a) Represente graficamente la funci´ on. b) Represente f(x) mediante una serie de Fourier. c) Estudie la convergencia de la serie en x = −π, x = 0, y x = π d) Muestre que: ∞ n=1 1 (2n −1) 2 = π 2 8 Soluci´on: b) f(x) = π 2 − 4 π ∞ n=1 cos (2n −1) x (2n −1) 2 Conducci´on del calor. Consideremos una varilla delgada, aislada, situada a lo largo del eje x, desde x = 0 hasta x = a,y supongamos que la conducci´ on de calor desde la varilla hacia el exterior se da solamente por los extremos de ella, los cuales se mantienen a temperatura cero. En f´ısica se muestra que si en tiempo t = 0 la temperatura u a lo largo de la varilla es igual a u(x, 0) = b n sin nx, donde b n = cte y n ∈ Z + , entonces para el tiempo t > 0 la temperatura es igual a u(x, t) = b n (sin nx) e −κn 2 t , donde κ > 0 es una constante positiva. Asimismo, hay un principo de superposici´ on que nos permite a˜ nadir los efectos de diferentes distribuciones iniciales de temperatura. Por lo tanto, si la temperatura inicial es: u(x, 0) = f (x) = ∞ n=1 b n sin nx entonces en tiempo t > 0, se tiene: u(x, t) = ∞ n=1 b n (sin nx) e −κn 2 t para 0 ≤ x ≤ a De acuerdo con todo esto, hallar la temperatura para t > 0 para las siguientes temperaturas iniciales dadas. a) u(x, 0) = f (x) = 3 sin x+5 sin 2x. ¿Que tipo de extensi´ on de f(x)se requiere en este caso? b) u(x, 0) = f (x) = e x sin x.¿Que tipo de extensi´ on de f(x)se requiere en este caso? 46 Soluciones. a) u(x, t) = f (x) = 3 sin xe −κt + 5 sin 2xe −4κt b) u(x, t) = 4 π ∞ n=1 n n 2 + 4 _ (−1) n−1 e π −1 ¸ sin nx e −κn 2 t Valor de la ra´ız media cuadr´atica Las series de fourier se constituyen en una herramienta poderosa en el an´ alisis del comportamiento de los sistemas f´ısicos sujetos a pertuba- ciones peri´odicas f(t). El valor de la ra´ız media cuadr´ atica ´ o RMC de una funci´on f(t), sobre un intervalo (a, b) ,se define como: ¸f(t)¸ = ¸ _ b a f 2 (t) dt b −a a) Sea f (t) una funci´on definida x ∈ [a, b] , con un per´ıodo funda- mental T = b − a. Pruebe que aplicando la identidad de Parseval el valor RMC se reduce a la formula: ¸f(t)¸ = ¸ ¸ ¸ _ a 2 0 + 1 2 ∞ n=1 [a 2 n + b 2 n ] b) Determine RMC de f(t) = E sin ωt, con E y ω constantes positi- vas. Soluci´ on: 47 b) El per´ıodo fundamental de la funci´ on f(t) = E sin ωt, es 2π ω . Entonces el valor RMC de f(t) es: ¸f(t)¸ = ¸ 1 (2π/ω) _ 2π ω 0 E 2 sin 2 (ωt) dt = E √ 2 Cuerda vibrante. Extremos fijos Un cuerda vibra libremente con ambos estremos fijos en x = 0 y x = L. a) Si su movimiento esta descrito por la ecuaci´on de onda: ∂ 2 u(x, t) ∂t 2 = v 2 ∂ 2 u(x, t) ∂x 2 con las condiciones iniciales: u(x, t) = f (x) y ∂u (x, 0) ∂t = g(x) Suponga que la soluci´on de esta ecuaci´ on es una serie de Fourier de la forma: u(x, t) = ∞ b n n=1 (t) sin( nπx L ) sustituya esta soluci´on en la ecuacion anterior y determine los coefi- cientes b (t) . b) Considere la presencia de un medio resistivo que amortigua las vi- braciones de acuerdo con la ecuaci´on ∂ 2 u(x, t) ∂t 2 = v 2 ∂ 2 u(x, t) ∂x 2 −k ∂u (x, t) ∂t Suponga que rige la soluci´on anterior con las mismas condiciones ini- ciales y nuevamente determine el coeficiente b (t) , suponiendo que el amortiguamiento es peque˜ no, es decir _ nπυ L _ 2 − _ k 2 _ 2 > 0 c) Repita los calculos pero suponiendo que el amortiguamiento es grande es decir _ nπυ L _ 2 − _ k 2 _ 2 < 0. Soluciones: 48 a) b n (t) = A n cos( nπνt L ) + B n sin( nπυt L ) A n = 2 L _ L 0 f (x) sin _ nπx L _ dx, B n = 2 nπυ _ L 0 g (x) sin _ nπx L _ dx b) b n (t) = e − k 2 t (A n cos(ω n t) + B n sin(ω n t)) A n = 2 L _ L 0 f (x) sin _ nπx L _ dx, B n = 2 Lω n _ L 0 g (x) sin _ nπx L _ dx + k 2ω n A n , ω 2 n = _ nπυ L _ 2 − _ k 2 _ 2 > 0 c) b n (t) = e − k 2 t (A n cosh(σ n t) + B n sinh(σ n t)) A n = 2 L _ L 0 f (x) sin _ nπx L _ dx, B n = 2 Lσ n _ L 0 g (x) sin _ nπx L _ dx + k 2σ n A n donde, σ 2 n = _ nπυ L _ 2 − _ k 2 _ 2 < 0 Distribuci´on de temperatura en un disco En una placa circular de radio ρ = 1, cuyas secciones superior e inferior est´ an aisladas, se mantiene la mitad de su periferia superior a una temperatura constante T 1 y la otra mitad a una temperatura constante T 2 .Encontrar la temperatura de la placa en condiciones estacionarias. a) La ecuaci´ on de difusi´ on del calor, en coordenadas polares (ρ, θ) ,en condiciones estacionarias esta dada por ∂ 2 φ ∂ρ 2 + 1 ρ ∂φ ∂ρ + 1 ρ 2 ∂ 2 φ ∂θ 2 = 0,donde φ(ρ, θ) es la funcion temperatura. Suponga que φ(ρ, θ) ,se puede sepa- rar como φ(ρ, θ) = M (ρ) N (θ)y pruebe que la ecuaci´ on se transforma en ρ 2 M M + ρ M M = − N N . b) A partir del resultado anterior , haga cada lado de la ecuaci´on igual a λ 2 y encuentre las EDO(2) N” (θ) + λ 2 N (θ) = 0 49 ρ 2 M” (ρ) + ρM (ρ) + M (ρ) = 0 b) Pruebe que N (θ) = A 1 cos λθ + A 2 sin λθ y M (ρ) = B 1 ρ λ + B 2 ρ −λ son soluciones de las correspondientes ecuaciones anteriores. c) Pruebe que la soluci´on general es φ(ρ, θ) = M (ρ) N (θ) = ∞ n=1 T 1 + T 2 2 (T −T) (1 −cos nπ) nπ ρ n sennθ 1.9. Ejercicios Resueltos Mediante la inclusi´on de ejercicios resueltos se espera que los estudiantes tengan oportunidad de movilizar sus capaci- dades para buscar, analizar, procesar, representar y comu- nicar diferentes tipos de informaci´on, decodificando y tra- duciendo la informaci´ on contenida en las funciones, gr´ aficos, series de Fourier, integrales de Fourier y sus propiedades. 1.9.1. Serie de Fourier Problema 1 Sea f (x) peri´ odica de per´ıodo 2 dada por f(x) = _ 1 2 −x, 0 ≤ x ≤ 1 x − 3 2 , 1 ≤ x ≤ 2 a) determinar su serie de Fourier b) estudie la convergencia de la serie en x 0 = −π Soluci´on La serie de Fourier en este intervalo es a 0 + ∞ n=1 (a n cos (nx) + b n sin n(nx)), x ∈ [0, 2] con coeficientes 50 a 0 = 1 2 2 _ 0 f (x) dx = 1 2 1 _ 0 ( 1 2 −x)dx + 1 2 2 _ 1 (x − 3 2 )dx a 0 = 1 2 _ 1 2 x − x 2 2 _ 1 0 + 1 2 _ x 2 2 − 3 2 x _ 2 1 = 0 a n = 2 _ 0 f (x) cos nxdx a n = 1 _ 0 _ 1 2 −x _ cos nxdx + 2 _ 1 (x − 3 2 ) cos nxdx Integrando por partes, se tiene a n = _ 1 2nπ sin nπx _ 1 0 − _ cos nπx (nπ) 2 + x sin nπx nπ _ 1 0 + + _ cos nπx (nπ) 2 + x sin nπx nπ _ 2 1 − _ 3 2nπ sin nπx _ 2 0 ∴ a n = 2 (1 −(−1) n ) (nπ) 2 b n = 2 _ 0 f (x) sin nxdx b n = 1 _ 0 _ 1 2 −x _ sin nxdx + 2 _ 1 (x − 3 2 ) sin nxdx Integrando por partes, se tiene b n = _ − 1 2nπ cos nπx _ 1 0 − _ sin nπx (nπ) 2 − x cos nπx nπ _ 1 0 + + _ sin nπx (nπ) 2 − x cos nπx nπ _ 2 1 + _ 3 2nπ cos nπx _ 2 1 = 0 La serie de Fourier de f en [0, 2] es 2 ∞ n=1 (1 −(−1) n ) (nπ) 2 cos nx = 4 π 2 ∞ n=1 1 (2n −1) 2 cos ((2n −1) x) b) Como f es continua ,entonces la serie en x = −π converge a f (−π) = f (4 −π) = 1 2 −(4 −π) = π − 7 2 51 Problema 2 a) Desarrollar en serie de Fourier la funci´ on peri´ odica de per´ıodo 2π, definida por: f(x) = x 2 , −π ≤ x ≤ π b) A partir del resultado obtenido calcular la suma de: ∞ n=1 1 n 2 c) Determine la convergencia de la serie ∞ n=1 1 n 4 Soluci´on: a) La funci´ on f es par por lo cual obtendremos una serie de cosenos, que tiene la forma: a 0 + ∞ n=1 a n cos (nx) a 0 = 1 π π _ 0 f(x)dx = 1 π π _ 0 x 2 dx = 1 π _ x 3 3 _ π 0 = π 2 3 a n = 2 π π _ 0 f(x) cos(nx)dx = 2 π π _ 0 x 2 cos(nx)dx a n = _ x 2 sin(nx) n + 2xcos(nx) n 2 _ π 0 = 4 cos(nπ) n 2 = 4(−1) n n 2 Luego, la serie de Fourier de f en [−π, π]es: π 2 3 + 4 ∞ n=1 (−1) n n 2 cos (nx) Como la funci´ on es continua en R ,tenemos: x 2 = π 2 3 + 4 ∞ n=1 (−1) n n 2 cos (nx) , ∀ x ∈ R 52 b) La serie num´erica se puede obtener poniendo x = π y f(π) = π 2 , π 2 = π 2 3 −4 _ − 1 1 2 − 1 2 2 − 1 3 2 −... _ de donde ∞ n=1 1 n 2 = 1 4 _ π − π 2 3 _ = π 2 6 c) Como la funci´ on f es seccionalmente suave para −π ≤ x ≤ π y f (−π) = f (π) se cumplen las condiciones de suficiencia de la identidad de Parseval, entonces: 1 π π _ −π _ x 2 ¸ 2 dx = 2 _ π 2 3 _ 2 + ∞ n=1 _ 4 (−1) n n 2 _ 2 =⇒ 1 π _ x 5 5 _ π −π = 2 9 π 4 + ∞ n=1 16 n 4 =⇒ ∞ n=1 1 n 2 = π 4 90 Problema 3 Sea f(x) = [x[ + 1, −1 ≤ x ≤ 1, la funci´ on peri´odica de per´ıodo 2, determinar: a) Su serie de Fourier b) La convergencia de la serie: ∞ n=1 1 (2n −1) 2 c) La convergencia de la serie ∞ n=1 1 (2n −1) 4 Soluci´on a) f(x) = [x[ + 1 es funci´ on par, con semiper´ıodo L = 1, entonces tenemos una serie coseno, que tiene la forma: S (x) = a 0 + ∞ n=1 a n cos (nπx) 53 Con coeficientes a 0 = 1 2 1 _ 0 f(x)dx = 1 2 1 _ 0 (x + 1) dx = 3 2 a n = 2 1 _ 0 f(x) cos(nπx)dx = 2 1 _ 0 (x + 1) cos(nx)dx a n = 2 _ sin(nπx) nπ _ 1 0 + 2 _ xsin(nπx) nπ + 2 cos(nπx) (nπ) 2 _ 1 0 a n = _ 2 cos(nπx) (nπ) 2 _ 1 0 = 2((−1) n −1) (nπ) 2 a n = _ 0; si n par − 4 (nπ) 2 ; si n impar Por consiguiente, la serie de Fourier de f en [−1, 1] es: S (x) = 3 2 − 4 π 2 ∞ n=1 1 (2n −1) 2 cos ((2n −1) πx) b) Como la funci´ on es continua en R ,considerando el valor x = 0,se obtiene por el teorema de la convergencia puntual: 3 2 + 4 π 2 ∞ n=1 1 (2n −1) 2 = f (0) = 1 ∞ n=1 1 (2n −1) 2 = π 2 8 c) Como la funci´on f es seccionalmente suave para −1 ≤ x ≤ 1 y f (−1) = f (1) se cumplen las condiciones de suficiencia de la identidad de Parseval, entonces: 2 1 _ 0 [x + 1] 2 dx = 2 _ 3 2 _ 2 + ∞ n=1 _ 4 (2n −1) 2 _ 2 =⇒ 2 _ (x + 1) 3 3 _ 1 0 = 9 2 + ∞ n=1 16 π 2 (2n −1) 4 =⇒ ∞ n=1 1 (2n −1) 2 = π 4 96 54 Problema 4 a) Para f(x) = e −[x] , 0 ≤ x ≤ 2 ,obtener su serie de Fourier en cosenos, de per´ıodo 4. b) Del resultado determinar la convergencia de: ∞ n=1 (−1) n−1 2n −1 Soluci´on a) Evaluando la funci´on parte entera tenemos f(x) = _ _ _ 1 si 0 ≤ x < 1 e −1 si 1 ≤ x < 2 e −2 si x = 2 Con extensi´on par f p (x) de f(x) se obtiene la serie: a 0 + ∞ n=1 a n cos nπx 2 a 0 = 1 2 _ 1 _ 0 1dx + 2 _ 1 e −1 dx _ = 1 2 [1 + e −1 ] a n = _ 1 _ 0 cos nπx 2 dx + 2 _ 1 e −1 cos nπx 2 dx _ = sin nπx 2 nπ 2 [ 1 0 + e −1 sin nπx 2 nπ 2 [ 2 1 = 2 sin nπ 2 nπ + 2e −1 sin nπ−sin nπ 2 nπ = 2 sin nπ 2 nπ [1 −e −1 ] Finalmente, la serie es: 1 + e −1 2 + 2(1 −e −1 ) ∞ n=1 sin nπ 2 nπ cos nπx 2 b) Convergencia de x 0 = 2 punto de discontinuidad con l´ımites laterales e −1 se tiene convergencia: e −1 = 1 + e −1 2 + 2(1 −e −1 ) ∞ n=1 sin nπ 2 nπ cos nπ 55 e −1 −1 2 = 2(1 −e −1 ) ∞ n=1 sin nπ 2 nπ cos nπ ∞ n=1 (−1) n−1 2n −1 = π 4 Problema 5 Sea f (x) = x 2 −[x] , para x ∈ [0, 2] . a) Obtener la serie de Fourier coseno de f (x) . b) Obtener a qu´e valores converge la serie para cada x ∈ [0, 2] . Soluci´ on a) Si se eval´ ua la funci´ on parte entera de x tenemos [x] = 0, ∀x ∈ (0, 1) y [x] = 1, ∀x ∈ (1, 2) . Entonces la funci´on queda f (x) = _ x 2 , 0 ≤ x < 1 x 2 −1, 1 ≤ x < 2 Consideremos ahora una extensi´on par de la funci´ on f, entonces la serie coseno de f (x) es S (x) = a 0 + ∞ n=1 a n cos _ nπx 2 _ con coeficientes a 0 = 1 2 2 _ 0 f (x) dx = 1 2 1 _ 0 x 2 dx + 1 2 2 _ 1 (x 2 −1)dx a 0 = 1 2 _ x 3 3 _ 1 0 + 1 2 _ x 3 3 −x _ 2 1 = 5 6 a n = 2 _ 0 f (x) cos nπx 2 dx a n = 1 _ 0 x 2 cos nπx 2 dx + 2 _ 1 (x 2 −1) cos nπx 2 dx Integrando por partes, se tiene a n = _ 8x (nπ) 2 cos nπx 2 + _ 2 nπ x 2 − 16 (nπ) 3 _ sin nπx 2 _ 1 0 56 + _ 8x (nπ) 2 cos nπx 2 + _ 2 nπ x 2 − 16 (nπ) 3 _ sin nπx 2 _ 2 1 − _ 2 nπ sin nπx 2 _ 1 0 ∴ a n = _ 16 (nπ) 2 cos nπ + 2 nπ sin nπ _ Sustituyendo estos resultados, se obtiene la serie de Fourier S (x) = 5 6 + ∞ n=1 _ 16 (nπ) 2 cos nπ + 2 nπ sin nπ _ cos nπx 2 Tenemos que f es seccionamente continua en [0, 2] , por lo tanto su ex- tensi´ on per´ıodica es seccionalmente continua en R , con discontinuidad de salto en los puntos x = 1 y x = 2 Por lo tanto, de acuerdo con el teorema de convergencia, la serie con- verge a S(x) = _ ¸ ¸ _ ¸ ¸ _ f(x) si 0 ≤ x < 1 1 2 si x = 1 f (x) si 1 ≤ x < 2 3 2 si x = 2 Problema 6 Utilice la serie de Fourier para demostrar la identidad trigonom´etrica sin 3 (x) = 3 4 sin(x) − 1 4 sin(3x) Soluci´on Se calcula la serie de Fourier de f(x) = sin 3 (x) en [−π, π] . Como f (x) es impar la serie ser´a: ∞ n=1 b n sin nπ con coeficientes: b n = 2 π π _ 0 sin 3 (x) sin(nx)dx 57 En primer lugar, calculemos la integral para n ,= 1 π _ 0 sin 3 x sin nxdx = _ −sin 3 x cos nx n _ [ π 0 + 3 n π _ 0 sin 2 x cos x cos nxdx Usando la identidad trigom´etrica: cos x cos nx = cos(n −1)x − cos(n + 1)x 2 La ´ ultima integral se puede expesar como = 3 2n π _ 0 sin 2 x [cos(n −1)x −cos(n + 1)x] dx (1) En segundo lugar, calculemos el valor del coeficiente b 1 para n = 1 en (1) b 1 = − 1 π 3 2 π _ 0 sin 2 x cos 2xdx = − 3 4π π _ 0 (1 −cos 2x) cos 2xdx = 3 4π π _ 0 1 −cos 4x 2 dx b 1 = 2 3 4π π 2 = 3 4 En tercer lugar, para n > 1 en (1) b n = 3 2n _ _ sin 2 x _ sin(n + 1)x n + 1 + sin(n −1)x n −1 _ [ π 0 − π _ 0 _ sin(n + 1)x n + 1 + sin(n −1)x n −1 _ sin 2xdx _ _ b n = − 3 2n π _ 0 _ sin(n + 1)x n + 1 + sin(n −1)x n −1 _ sin 2xdx Usando la identidad trigonom´etrica b n = − 3 2n 1 n + 1 1 2 π _ 0 (cos(n + 1)x −cos(n + 3)xπ) dx − 3 2n 1 n −1 1 2 π _ 0 (cos(n −3)x −cos(n + 1)x)dx = 0, ∀ n ,= 3 Para n = 3 el c´alculo directo, produce: b 3 = − 3 2 3 2 π 2 2 π = − 1 4 Por tanto, la serie de Fourier de f en [−π, π]es: 3 4 sin(x) − 1 4 sin(3x) 58 Problema 7 Sea f(x) = x(sin x), para −π ≤ x ≤ π, entonces: a) Determine la serie de esta funci´ on. b) Pruebe la convergencia de la serie: ∞ n=1 (−1) n n 2 −1 = 1 4 c) Pruebe que esta serie se puede diferenciar t´ermino a t´ermino y utilice este hecho para obtener el desarrollo de Fourier de sin (x) + x cos (x) . Soluci´on a) La funci´ on f(x) es par, es decir f(x) = f(−x) ∀ x ∈ (−π, π), entonces: b n = 0 a 0 = 1 π π _ 0 f(x)dx = 1 π π _ 0 x sin xdx = ∴ a 0 = 1 π _ _ [x (−cos x)] π 0 + π _ 0 cos xdx _ _ = 1 a n = 2 π π _ 0 f(x) cos(nx)dx = 2 π π _ 0 x sin x cos(nx)dx Para n ,= 1 a n = 1 π π _ 0 x [sin ((n + 1) x) −sin ((n −1) x)] dx Integrando por partes, queda a n = 1 π _ x _ − cos ((n + 1) x) (n + 1) + cos ((n −1) x) (n −1) __ π 0 − − 1 π _ − sin ((n + 1) x) (n + 1) 2 + sin ((n −1) x) (n −1) 2 _ π 0 Evaluando los l´ımites de la integral produce 59 ∴ a n = 2 (−1) n+1 n 2 −1 Para n = 1 a 1 = 2 π π _ 0 x sin x cos xdx = 1 π π _ 0 x sin(2x)dx = − 1 2 Por tanto, la serie de Fourier de f para x ∈ [−π, π]es: f (x) = 1 − 1 2 cos x + 2 ∞ n=2 (−1) n+1 n 2 −1 cos (nx) b) En x = 0 hay un punto de continuidad de la funci´ on, entonces la serie converge a f(0) f(0) = 0 = 1 − 1 2 cos 0 + 2 ∞ n=2 (−1) n+1 n 2 −1 cos (0) Finalmente ∞ n=2 (−1) n+1 n 2 −1 = 1 4 c) Sea f(x) = x(sin x), para −π ≤ x ≤ π. i) Como f (x) = x sin x ,es producto de funciones continuas, es continua en [−π, π] . ii) f (x) = sin x + x cos x ,es producto y suma de funciones continuas, es continua en [−π, π] . iii) Existe f (x) = 2 cos x −x sin x, y tambi´en es continua en [−π, π] . Adem´ as f (−π) = (−π) (sin (−π)) = (−π) (−sin (π)) = π sin π = f (π) Por tanto, se satisfacen las hip´otesis del teorema de diferenciaci´ on de la serie de Fourier, entonces para −π < x < π 60 f (x) = 1 − 1 2 cos x + 2 ∞ n=2 (−1) n+1 n 2 −1 cos (nx) =⇒ f (x) = sin x + x cos x = 1 2 sin x + 2 ∞ n=2 (−1) n n 2 −1 sin (nx) Problema 8 a) Desarrollar en serie de Fourier la funci´on per´ıodica de per´ıodo 2π. Representar graficamente y estudiar la convergencia de la serie en R. f(x) = _ 0, si −π ≤ x ≤ 0 x, si 0 < x ≤ π b) A partir del resultado anterior obtenga la suma de la serie: ∞ n=1 1 (2n −1) 2 c) Pruebe que esta serie se puede integrar t´ermino a t´ermino y obtener un desarrollo en serie trigonom´etrica para _ x −π f (u) du en [−π, π] . Soluci´on a) Calculemos los coeficientes de Fourier. a 0 = 1 2π π _ −π f(x)dx = 1 2π _ _ 0 _ −π f(x)dx + π _ 0 f(x)dx _ _ = 1 2π π _ 0 xdx ∴ a 0 = 1 2π _ x 2 2 _ π 0 = π 4 a n = 1 π π _ −π f(x) cos(nx)dx = 1 π π _ 0 x cos(nx)dx Usando el m´etodo de integraci´on por partes se tiene: a n = 1 π _ x cos(nx) n + cos(nx) n 2 _ π 0 = 1 π _ 0 −0 + (−1) n n 2 − 1 n 2 _ a n = (−1) n −1 n 2 π = _ 0 si n par − 2 n 2 π si n impar 61 As´ı: a 2n = 0 ∀ n a 2n−1 = − 2 (2n −1) 2 π ∀ n. b n = 1 π π _ −π f(x) sin(nx)dx = 1 π π _ 0 x sin(nx)dx = 1 π _ −x cos(nx) n + sin(nx) n 2 _ π 0 = − cos(nπ) n luego estos coeficientes son: ∴ b n = (−1) n+1 n Por lo tanto, la serie de Fourier de f para x ∈ [−π, π] ,es: π 4 + ∞ n=1 _ − (−1) n−1 n 2 π cos nx + (−1) n+1 n sin(nx) _ Esta serie converge a: i) f(x) = 0 para −π < x ≤ 0, puesto que, son puntos de continuidad de f. ii) f (x) = x para 0 < x < π, son puntos de continuidad de f. iii) f (π+) + f (π−) 2 = π 2 en los puntos de discontinuidad del tipo x = π + 2nπ con n ∈ Z. b) Aplicando el criterio de convergencia en x = 0, f (0) = 0 se tiene 0 = π 4 − 2 π _ 1 1 2 + 1 3 2 + 1 5 2 + ... _ de donde π 4 = 2 π _ 1 1 2 + 1 3 2 + 1 5 2 + ... _ y de aqu´ı ∞ n=1 1 (2n −1) 2 = π 2 8 62 c) Como f(x) = _ 0, si −π ≤ x ≤ 0 x, si 0 < x ≤ π es una funci´on seccionalmente continua en [−π, π] , con serie de Fourier π 4 + ∞ n=1 _ − 2 π (2n −1) 2 cos ((2n −1) x) + (−1) n+1 n sin(nx) _ se satisface las hip´otesis del teorema de integraci´ on de series de Fourier, luego puede integrase t´ermino a t´ermino. Entonces, para cualquier x ∈ [−π, π] , se tiene: Primero, _ x −π f (u) du = _ 0, si −π ≤ x ≤ 0 x 2 2 si 0 < x ≤ π en [−π, π] Segundo, integrando la serie produce _ x −π _ π 4 + ∞ n=1 _ − 2 cos ((2n −1) u) π (2n −1) 2 + (−1) n+1 sin(nu) n __ du = _ π 4 u + ∞ n=1 _ − 2 sin ((2n −1) u) π (2n −1) 3 − (−1) n+1 cos(nu) n 2 __ x −π = π 4 x + π 2 + ∞ n=1 _ − 2 sin (2n −1) x π (2n −1) 3 + (−1) n+1 cos(nx) n 2 − 1 n 2 _ Por tanto, la funci´ on f queda representada por la serie anteriormente obtenida. 1.9.2. Integral de Fourier Problema 9 a) Halle la representaci´ on de la integral de Fourier de la funci´on f(x) = _ x, [x[ < π 0, [x[ ≥ π b) De esta representacion deducir que: _ ∞ 0 sin(wπ) w 2 sin(wx)dx = π _ ∞ 0 cos(wπ) w sin(wx)dx 63 Soluci´on a) Como f es una funci´ on impar, entonces f (x) = ∞ _ 0 B(w) sin(wu)dw con coeficiente B(w) = 2 π π _ 0 usin(wu)du = 2 π _ −u cos(wu) w + sin(wu) w 2 _ π 0 B(w) = 2 π _ −π cos(wπ) w + sin(wπ) w 2 _ Por consiguiente f (x) = 2 π ∞ _ 0 _ −π cos(wπ) w + sin(wπ) w 2 _ sin(wx)dw Es la integral de Fourier de f para [x[ , = 0 b) En particular cuando [x[ > π, se tiene f (x) = 0. Entonces la integral converge a cero 0 = 2 π ∞ _ 0 _ −π cos(wπ) w + sin(wπ) w 2 _ sin(wx)dw Por tanto, ∞ _ 0 sin(wπ) w 2 sin(wx)dw = π ∞ _ 0 cos(wπ) w sin(wx)dw 64 Problema 10 Halle la representaci´on de la integral de Fourier de la funci´on f(x) = xe −|x| si x ∈ (−∞, ∞) y estudie su convergencia en R. Soluci´on Se tiene que f(x) es una funci´ on impar. Examinemos, si se cumplen las condiciones de existencia de integral de Fourier. En primer lugar ∞ _ −∞ ¸ ¸ xe −|x| ¸ ¸ dx = 2 ∞ _ 0 xe −x dx = 2 _ _ −xe −x [ ∞ 0 + ∞ _ 0 e −x dx _ _ = 2 1 = 2 la integral es convergente Adem´ as, f es continua y diferenciable ∀x. Los coeficientes de Fourier de f son: A(w) = 0 ya que f es una funci´ on impar B(w) = ∞ _ −∞ ue −|u| sin(wu)du = 4w (1 + w 2 ) 2 Entonces, para todo x la integral de Fourier converge a: xe −x = 4 π ∞ _ 0 w (1 + w 2 ) 2 sin(wx)dw 65 Problema 11 Sea f la funci´on pulso rectangular unitario de per´ıodo 2 definida por f (x) = _ 1 2δ si −δ < x < δ 0 si −1 ≤ x < δ ´ o δ < x ≤ 1 a) Representar graficamente f (x) b) Obtener la serie de Fourier de f (x) . c) Si a n (δ) es el coeficiente n-´esimo de la serie anterior, calcular los l´ımites: l´ım n→∞ ( l´ım δ→0 + (a n (δ)) , l´ım δ→0 + ( l´ım n→∞ (a n (δ))) Soluci´on b) Como f es una funci´ on par de per´ıodo 2 ,entonces : a 0 = 1 _ 0 f (x) dx = δ _ 0 1 2δ dx = 1 2 a n = 2 1 _ 0 f (x) cos(nπx)dx = 2 δ _ 0 1 2δ cos (nπx) dx = 1 δ sen(nπδ) nπ = a n (δ) donde en este caso definimos a n (δ) = 1 δ sin(nπδ) nπ b n = 0 ∀n Luego, se tiene que: f (x) ∼ 1 2 + 1 δ ∞ n=1 sen(nπδ) nπ cos (nπx) , x ∈ [−1, 1] c) En primer lugar calculemos: l´ım δ→0 + ( l´ım n→∞ (a n (δ))) = l´ım δ→0 + ( l´ım n→∞ 1 δ sen(nπδ) nπ n→∞ ) = l´ım δ→0 + (0) = 0 En segundo lugar l´ım n→∞ ( l´ım δ→0 + (a k (δ)) = l´ım n→∞ ( l´ım δ→0 + _ 1 δ sen(nπδ) nπ _ ) = l´ım n→∞ (1) = 1 66 Problema 12 Dada la funci´ on f(x) = xe −x con x 0, a) Verifique que considerando las extensiones par e impar de la funci´ on f: _ ∞ 0 _ 1 −w 2 (1 + w 2 ) 2 _ cos wx dw = _ ∞ 0 _ 2w (1 + w 2 ) 2 _ senwx dw b) Estudiar la convergencia de la IF para deducir que: _ ∞ 0 _ 1 (1 + w 2 ) 2 _ dw = _ ∞ 0 _ w 2 (1 + w 2 ) 2 _ dw Soluci´on Consideremos para f(x) = xe x con x 0 su extensi´ on par f p (x) = _ xe −x si x 0 −xe x si x < 0 =⇒ f p (x) ∼ 1 π ∞ _ 0 A(w) cos wxdw con A(w) = 2 ∞ _ 0 xe −x cos wx dx Ahora, consideremos la extensi´ on impar de f f i (x) = _ xe −x si x 0 xe x si x < 0 =⇒ f i (x) ∼ 1 π ∞ _ 0 B(w) sin wxdw con B(w) = 2 ∞ _ 0 xe −x senwx dx Podemos calcular los coeficientes A(w) y B(w) integrando por partes: 67 A(w) = 2 ∞ _ 0 xe −x cos wx dx = 2 _ ∞ 0 e −x (x cos wx)dx =⇒ A(w) = 2 _ xe −x (−cos wx + wsenwx) (1 + w 2 ) − e −x ((1 −w 2 ) cos wx −2wsenwx) (1 + w 2 ) 2 _ ∞ 0 A(w) = 2 _ 1 −w 2 (1 + w 2 ) 2 _ B(w) = 2 ∞ _ 0 xe −x sin wx dx = 2 _ ∞ 0 e −x (x sin wx)dx =⇒ B(w) = 2 _ xe −x (−sin wx −wcos wx) (1 + w 2 ) − e −x ((1 −w 2 ) sin wx + 2wcos wx) (1 + w 2 ) 2 _ ∞ 0 B(w) = 2 _ 2w (1 + w 2 ) 2 _ Construyendo las respectivas integrales de Fourier y aplicando el teo- rema de la convergencia , puesto que f es una funci´on seccionalmente suave ∀x 0 ,se tiene que : xe −x = 2 π ∞ _ 0 _ 1 −w 2 (1 + w 2 ) 2 _ cos wxdw xe −x = 2 π ∞ _ 0 _ 2w (1 + w 2 ) 2 _ senwxdw Por lo tanto, las extensiones son iguales: ∞ _ 0 _ 1 −w 2 (1 + w 2 ) 2 _ cos wx dw = ∞ _ 0 _ 2w (1 + w 2 ) 2 _ sin wx dw b) En x = 0 se tiene un punto en que estas extensiones son continuas, luego ambas integrales convergen a f(0) = 0 ∞ _ 0 1 −w 2 (1 + w 2 ) 2 dw = 0 =⇒ ∞ _ 0 1 (1 + w 2 ) 2 dw = ∞ _ 0 w 2 (1 + w 2 ) 2 dw 68 Problema 13 Si f (x) es una funci´ on par ,con integral de Fourier f (x) = 1 π ∞ _ 0 A(w) cos(wx)dw, demuestre que: a) xf (x) = 1 π ∞ _ 0 A ∗ (w) cos(wx)dw, donde A ∗ (w) = − dA(w) dw b) x 2 f (x) = 1 π ∞ _ 0 A ∗ (w) cos(wx)dw, donde A ∗ (w) = − d 2 A(w) dw 2 Soluci´on a) Se tiene que xf (x) = 1 π ∞ _ 0 A ∗ (w) sin(wx)dw, es una funci´ on impar, entonces A ∗ (w) = 2 ∞ _ 0 v f (v) sin(wv)dv (1). Como f (x) = 1 π ∞ _ 0 A(w) cos(wx)dw con A(w) = 2 ∞ _ 0 f (v) cos(wv)dv. Entonces, derivando el coeficiente queda dA(w) dw = −2 ∞ _ 0 vf (v) sin(wv)dv (2) Por lo tanto, comparando (1) y (2) se tiene dA(w) dw = −A ∗ (w) b) Como x 2 f (x) = 1 π ∞ _ 0 A ∗ (w) cos(wx)dw, es una funci´ on par, entonces A ∗ (w) = 2 π ∞ _ 0 v 2 f (v) cos(wv)dv (1) Como, f (x) = 1 π ∞ _ 0 A(w) cos(wx)dw con A(w) = 2 ∞ _ 0 f (v) cos(wv)dv. Por consiguiente dA(w) dw = −2 ∞ _ 0 vf (v) sin(wv)dv =⇒ 69 d 2 A(w) dw 2 = −2 ∞ _ 0 v 2 f (v) cos(wv)dv (2) Por lo tanto, comparando (1) y (2)se tiene d 2 A(w) dw 2 = −A ∗ (w) . 1.10. Ejercicios propuestos 1.- Sea f una funci´on de per´ıodo π dada por f(x) = _ sin 2x si 0 ≤ x ≤ π/2 0 si π/2 ≤ x ≤ π a) Obtener la serie de Fourier de f(x). b) Deducir la convergencia de la serie: ∞ n=1 1 4n 2 −1 2.- Sea f una funci´on dada por f(x) = _ x (π −x) si 0 < x < π x (π + x) si −π < x < 0 a) Represente graficamente la funci´ on f usando Maple b) Obtener la serie de Fourier de f(x). c) Deducir la convergencia de la serie: ∞ n=1 (−1) (n−1) (2n−1) 3 3.- Un pulso tri´ angular sim´etrico de altura y ancho ajustables es descrito por: f(x) = _ a _ 1 − x b _ si 0 ≤ x ≤ b 0 si b ≤ x ≤ π a) Muestre que los coefientes de Fourier son: a 0 = ab 2π , a n = 2ab π (1−cos nb) (nb) 2 b) Tome a = 1 y b = π 2 calcule y represente las cinco primeras sumas parciales. 70 4. Sea f una funci´on dada por f(x) = 1 +[x[ x ∈ [−1, 1] a) Obtener la serie de Fourier de f(x). b) Deducir la convergencia de la serie: ∞ n=1 1 (2n−1) 2 5. Encontrar la serie de coseno de Fourier de la funci´ on per´ıodica de per´ıodo 4, dada por f (x) = e −[x] 0 ≤ x ≤ 2. b) Deducir la convergencia de la serie: ∞ n=1 1 2n−1 c) Usando la identidad de Parseval deducir la convergencia de la serie: ∞ n=1 1 (2n−1) 2 6. Sea f (x) = x sin x, −π ≤ x ≤ π a) Obtener la serie de Fourier de f. b) Pruebe que esta serie se puede diferenciar t´ermino a t´ermino. c) Use el resultado anterior para obtener el desarrollo de Fourier. de sin x + x cos x, −π ≤ x ≤ π. b) Deducir la convergencia de la serie ∞ n=1 1 (2n−1) 6 7. Sea f(x) = _ 0 si −π ≤ x ≤ 0 x si 0 < x ≤ π a) Obtener la serie de Fourier de f. b) Pruebe que esta serie se puede integrar t´ermino a t´ermino. c) Use los resultados anteriores para obtener el desarrollo en serie trigonom´etrica para _ x −π f (u) du 8. a) Establecer que si f(x) = x, −π < x < π entonces x = 2 ∞ n=1 (−1) n+1 n sin nx b) Con la identidad de Parseval deducir la convergencia ∞ n=1 1 n 2 = π 2 6 c) Muestre que la integraci´ on de la serie de Fourier de f(x) = x, −π < x < π 71 conduce a. ∞ n=1 (−1) n+1 n 2 = π 2 12 d) Sea f(x) una funci´on continua definida en −π < x < π con serie de Fourier a 0 + ∞ n=1 (a n cos nx +b n sin nx). Si g(x) = f(x −π) pruebe que la serie de Fourier de g(x) es a 0 + ∞ n=1 ((−1) n a n cos nx + (−1) n b n sin nx) e) Aplicando los resultados de a) y d), obtener la serie de Fourier de per´ıodo 2π de la funci´ on definida por g(x) = x −π , 0 < x < π. 9. Sea f(x) una funci´ on seccionalmente cont´ınua, impar de per´ıodo 2π, con serie de Fourier ∞ n=1 b n sin(nx) a) Verificar que g(x) = x _ 0 f(t)dt, x ∈ R es funci´on par de per´ıodo 2π b) Deducir que ∞ n=1 b n n (1 −cos(nx)) es la serie de Fourier de g(x) y que ∞ n=1 b n n = 1 π π _ 0 ( x _ 0 f(t)dt) 10. Sea f (x) , x ∈ R funci´ on impar con integral de Fourier I i = 1 π π _ 0 B(w) sin wx dw. Pruebe que la integral de Fourier de g (x) = f (x) sin x es: I p = π _ 0 A(w) cos(wx) dw, con A(w) = _ 1 π [B(w + 1) −B(w −1)] w 1 1 π [B(w + 1) + B(w −1)] 0 ≤ w < 1 72 11. Sea f(x) = _ 1 −x 2 si [x[ ≤ 1 0 si [x[ > 1 , obtener la integral de Fourier y estudie su convergencia en x 0 = 0. 12. a) Obtener la integral de Fourier de f(x) = _ cos x si [x[ ≤ π 0 si [x[ > π b) Estudiar la convergencia de la IF en x 0 = 0 , x 1 = π. 13. Establecer la igualdad 2 π ∞ _ 0 w 1 + w 2 sin wπdw = e −x si x > 0 y de esto deducir el valor al cual converge ∞ _ 0 w 2 (1+w 2 ) 2 dw. 14. Obtener la Integral de Fourier de f (x) = e −|x| , x ∈ R. Del resultado, deducir el valor de ∞ _ 0 cos(wx) (1+w 2 ) dw. 15. Aplicando la la representaci´ on de la integral de Fourier demostrar que: a) ∞ _ 0 cos(πw/2) cos wx 1 −w 2 dw = _ π 2 cos x si [x[ < π 2 0 si [x[ > π 2 b) ∞ _ 0 1 −cos πw w sin(wx)dw = _ π 2 si 0 < x < π 0 si x > π 16. Si f (x)es una funci´ on par con integral de Fourier f (x) = ∞ _ 0 A(w) cos(wx)dw, demuestre que: f (ax) = 1 πa ∞ _ 0 A _ w a _ cos(wx)dw, a > 0 17. Pruebe que la integral de Fourier de f puede escribirse como l´ım w→∞ 1 π ∞ _ −∞ f (t) sin(w(t −x)) t −x dt 73 1.10.1. Respuestas 1) a) f (x) ∼ 1 π + 1 2 sin 2x − 2 π ∞ n=1 1 4n 2 −1 cos 4nx b) Estudie la convergencia en f _ π 2 _ ∞ n=1 1 4n 2 −1 = 1 2 2) a) f(x) = _ x (π −x) si 0 < x < π x (π + x) si −π < x < 0 b) f (x) ∼ 8 π ∞ n=1 1 (2n −1) 3 sin(2n −1)x c) Como f(x) es discontinua en x = π la serie converge a f(π + ) + f(π − ) 2 luego ∞ n=1 (−1) (n−1) (2n −1) 2 = π 3 32 3) Si a = 1 y b = π 2 el gr´afico de la funci´ on es 4) a) 74 f (x) ∼ 3 2 − 4 π 2 ∞ n=1 1 (2n −1) 2 cos(2n −1)πx b) Como f(x) es continua ∀x la serie converge a f(0), luego ∞ n=1 1 (2n −1) 2 = π 2 8 c) Aplicando la identidad de Parseval, se tiene: ∞ n=1 1 (2n −1) 4 = π 4 96 5. a) f (x) ∼ 1 + e −1 2 + 2 π _ 1 −e −1 _ ∞ n=1 sin( nπ 2 ) n cos( nπ 2 x) b) Como f(x) es discontinua en x 0 = 2 converge a los l´ımites laterales en ese punto, entonces se tiene la convergencia ∞ n=1 1 2n −1 = π 4 6. a) x sin x = π − 1 2 π cos x + 2π ∞ n=1 (−1) n+1 n 2 −1 cos (nx) , -π ≤ x ≤ π b) f y f son seccionalmente suave en −π ≤ x ≤ π y f (−π) = f (π) , luego se satisfacen las condiciones del teorema de diferenciaci´ on. c)Tenemos que x cos x + senx = 1 2 π sin x + 2π ∞ n=1 (−1) n n 2 −1 nsin (nx) , −π ≤ x ≤ π 7. a) 75 f (x) ∼ 1 4 π+ ∞ n=1 _ (−1) n −1 πn 2 cos (nx) + (−1) n+1 n sin (nx) _ , −π ≤ x ≤ π b) f es seccionalmente continua en −π ≤ x ≤ π ,entonces se satisfacen las condiciones del teorema de integracion. Luego, su serie puede integrarse t´ermino a t´ermino c) Tenemos que _ x −π f (u) du = _ 0 −π ≤ x ≤ 0 x 2 2 0 < x ≤ π Esta funci´on esta representada por la serie obtenida al integrar la serie de Fourier anterior 1 4 πx + 1 4 π 2 + ∞ n=1 _ (−1) n −1 πn 2 sin (nx) n + (−1) n+1 n (−cos nx + (−1) n ) n _ 8)e) x −π = −2 ∞ n=1 sin(nx) n 11)a) 1 π _ ∞ 0 _ 4 sin w w 3 − 4 cos w w 2 _ cos wx dw b) La funci´ on es continua en x 0 = 0 luego la IF converge a f(0) _ ∞ 0 _ 4 sin w w 3 − 4 cos w w 2 _ dw = π 12) a) Comof(x) es una funci´ on par se tiene que: A(w) = 2 π π _ 0 cos v cos(wv) dv = 2 π wsin wπ 1 −w 2 B(w) = 0 76 Por lo tanto, IF = 2 π π _ 0 wsin wπ 1 −w 2 cos wxdw. b) En x 0 = 0 hay un punto de continuidad de f (x), entonces 2 π π _ 0 wsin wπ 1 −w 2 dw = f (0) = 1 y x 1 = π es un punto de discontinuidad de f (x), entonces: 2 π π _ 0 wsin 2wπ 1 −w 2 dw = f (π + ) + f (π − ) 2 = 0 + 1 2 13) Considere una extensi´on impar de f(x), entonces A(w) = 0 y B(w) = 2w 1+w 2 La Integral de Fourier de f(x)es 2 π ∞ _ 0 w 1+w 2 sin wx dw. Usando la identidad de Parseval ∞ _ 0 w 2 (1+w 2 ) 2 dw = π 4 14) Como f es par tiene Integral de Fourier f(x) = 2 π ∞ _ 0 cos(wx) (1+w 2 ) dw. Al estudiar la continuidad en x 0 = 1 se obtiene la convergencia ∞ _ 0 cos(w) (1+w 2 ) dw = π 2e 1.11. Auto evaluaciones En el aprendizaje de C´ alculo Avanzado como parte de la matem´atica se requiere el dominio de dos tipos b´asicos de conocimientos: a) el conocimiento conceptual y b) el conocimiento procedimental. El primero est´ a vinculado al razonamiento y reflexi´on, se caracteriza por ser un conocimiento te´orico, producido por la actividad cognitiva, 77 que permite establecer relaciones entre sus componentes; su car´ acter es declarativo y se asocia con el saber qu´e y el saber por qu´e. El segundo est´a vinculado a la acci´ on y se relaciona con m´etodos y las estrategias para representar conceptos y transformar estas representa- ciones; usando habilidades y destrezas para analizar comprensivamente, elaborar, comparar, resolver algoritmos, evaluar y argumentar. De este modo el conocimiento procedimental permite depurar y refinar la con- strucci´ on del conocimiento conceptual, como asimismo, usar de manera eficaz y flexible, en un contexto aplicado, los conceptos, proposiciones, teor´ıas y modelos matem´ aticos. En consecuencia, est´ a asociado con el saber hacer. Evaluaci´on formativa Al t´ermino de cada unidad , se incluyen una serie de evaluaciones for- mativas que deben realizar los propios alumnos en situaciones de auto- evaluaci´on de acuerdo a la din´ amica de trabajo individual, por equipos o grupal si se utiliza en el trabajo en el aula o en trabajo personal realizado en casa. Por consiguiente, esta modalidad de evaluaci´ on tiene por funci´ on gener- ar evidencias de los resultados de aprendizaje logrados por los propios estudiantes para retroalimentar su avances de aprendizaje. Asimismo, debe proporcionar informaci´ on a los estudiantes sobre sus aciertos, difi- cultades, falencias y omisiones en el proceso de aprendizaje del C´ alcu- lo Avanzado de modo de introducir cambios y ajustes en los m´eto- dos de estudios. De este modo, las autoevaluaciones abarcan tanto los conocimientos conceptuales como procedimentalales luego son una in- stancia para que los estudiantes perfeccionen su capacidad para: re- conocer los conceptos, principios, reglas y propiedades que se sustenta el C´alculo Avanzado; identificar y aplicar m´etodos matem´aticos en la resoluci´ on de problemas; analizar y evaluar informaci´ on matem´atica proveniente de otras ciencias ; y analizar y evaluar las soluciones de un problema para fundamentar su pertinencia. Estrategia de aplicaci´ on Como se ha se˜ nalado el objetivo fundamental de estos instrumentos de autoevaluaci´ on es que el alumno verifique, constate y descubra las falencias que a´ un persisten en el ambito cognitivo y procedimental de 78 su aprendizaje del tema objeto del estudio, ya sea que este sea el total o una parcialidad de un tema. Se pretende que con esta constataci´ on el alumno se motive para persistir en su estudio, si a´ un no ha logrado el nivel necesario que asegure su ´exito en las evaluaciones regulares que se har´ an del tema. Para que este ejercicio tenga el ´exito propuesto, se debe trabajar el tiempo estipulado, y con las condiciones medioambientales necesarias de privacidad y silencio , que aseguren la efectividad de la actividad, o que reproduzcan condiciones requeridas de una prueba o examen.. Series e Integrales de Fourier Autoevaluaci´ on N o 1 El estudiante: 1) Representar´ a una funci´ on f(x) definida en el intervalo [0, L] por una serie de Fourier coseno o seno, aplicando extensiones pares o impares seg´ un sea el caso, y evaluar si la serie converge o no en un punto x 0 dado del dominio. 2) Obtendr´a la suma de ciertas series infinitas empleando el teorema de la convergencia de las series de Fourier. 3) Verificar´a si una serie de Fourier se puede diferenciar t´ermino a t´ermi- no y aplicar el teorema de la diferenciaci´ on para obtener la derivada de la serie de fourier 4) Representar´ a una funci´on f (x) no per´ıodica por una Integral de Fourier y emplear el teorema de la convergencia para probar si la Inte- gral Fourier converge o no en un punto x 0 del dominio. 5) Probar´a el teorema de Parseval para una funci´on f que se representa por una integral de Fourier y deducir´ a la convergencia de ciertas sumas de series. 6) Representar´a una funci´ on peri´ odica f(x) de per´ıodo 2L por una serie de Fourier, y aplicar´ a el teorema de la convergencia para determinar si la serie converge o no en un punto x 0 del dominio. Tiempo : 1 hora 20 minutos Problema 1 79 a) Desarrollar en serie de Fourier de cosenos la funci´on f (x) = sin _ x 2 _ definida en el intervalo [0, π]. b) Deducir la convergencia de la serie ∞ n=1 1 4n 2 −1 = 1 2 b) Pruebe que la serie de f se puede diferenciar t´ermino a t´ermino y utilice este hecho para obtener el desarrollo de Fourier de cos _ x 2 _ . Problema 2 a) Probar que si f (x) = _ ∞ 0 (A(w) cos (wx)+B(w) sin (wx))dw, con co- eficientes A(w) = 1 π _ ∞ −∞ f (u) cos (wu) du y B(w) = 1 π _ ∞ −∞ f (u) sin (wu) du. Entonces 1 π _ ∞ −∞ [f (x)] 2 dx = _ ∞ 0 (A 2 (w) + B 2 (w))dw b) Si f (x) = _ π, [x[ ≤ a 0, [x[ > a ,pruebe que _ ∞ 0 sin 2 wa w 2 dw = aπ 2 Problema 3 Sea f (x) = _ 0 si −5 < x < 0 3 si 0 < x < 5 tal que f (x + 10) = f (x) a) Obtener la serie de Fourier de f (x) b) Defina f (x) en los puntos −5, 0 y 5 de manera que la serie de Fourier obtenida anteriormente converja a f (x) ∀x con [x[ ≤ 5. Pauta de Correcci´ on Problema 1 Para obtener el desarrollo en serie de Fourier de cosenos de la funci´ on dada, construimos una extensi´ on par de la funci´on f tal que f p (x) = _ _ _ sin _ x 2 _ , 0 ≤ x ≤ π −sin _ x 2 _ , −π ≤ x < 0 Como esta funci´ on es par, de per´ıodo 2π ,tenemos que 80 b n = 0 a 0 = 1 π π _ 0 f(x)dx = 1 π π _ 0 sin _ x 2 _ dx = 1 π _ −2 cos _ x 2 _¸ π 0 a 0 = 2 π _ 1 −cos π 2 ¸ = 2 π a n = 2 π π _ 0 f(x) cos(nx)dx = 2 π π _ 0 sin x 2 cos(nx)dx a n = 1 π π _ 0 _ sin _ 1 2 + n _ x −sin _ 1 2 −n _ x ¸ dx a n = 1 π _ − cos _ 1 2 + n _ x _ 1 2 + n _ − cos _ 1 2 + n _ x _ 1 2 + n _ _ π 0 a n = 1 π _ 2 1 + 2n + 2 1 −2n _ = 4 π (1 −4n 2 ) Por lo tanto, la serie de f p en [0, π] ,es: 2 π + 1 π ∞ n=1 4 (1 −4n 2 ) cos (nx) La funci´ on f p seccionalmente continua, de per´ıodo 2π, aplicando el teorema de convergencia de Fourier , tenemos que: f (x) = 2 π + 1 π ∞ n=1 4 (1 −4n 2 ) cos (nx) , ∀x ∈ (0, 2π) En particular para x = 0, 2 π + 1 π ∞ n=1 4 (1 −4n 2 ) = 0 ∞ n=1 4 (4n 2 −1) = 1 2 c) Sea f(x) = sin _ x 2 _ , para 0 ≤ x ≤ π. i) Como f (x) = sin _ x 2 _ , es continua en (0, π) . 81 ii) f (x) = 1 2 cos _ x 2 _ , es continua en (0, π) . iii) Existe f (x) = − 1 4 sin _ 1 2 _ , y tambi´en es continua en (0, π) . Adem´ as f (0) = (sin (0)) = (sin (π)) = sin 2π = f (π) Luego, se satisfacen las hip´ otesis del teorema de diferenciaci´on de la serie de Fourier, entonces para −π < x < π,derivando la serie de Fourier de f , obtenemos: sin _ x 2 _ = 2 π + 1 π ∞ n=1 4 (1 −4n 2 ) cos (nx) 1 2 cos _ x 2 _ = 0 − 1 π ∞ n=1 4n (1 −4n 2 ) sin (nx) cos _ x 2 _ = 1 π ∞ n=1 8n (4n 2 −1) sin (nx) Problema 2 a) Mutiplicando la integral de Fourier por f y realizando el producto interno en (−∞, ∞) , se tiene _ ∞ −∞ [f (x)] 2 dx = _ ∞ 0 _ ∞ −∞ [(A(w) f (x) cos (wx) + B(w) f (x) sin (wx))dw]dx _ ∞ −∞ [f (x)] 2 dx = _ ∞ 0 (A(w) _ ∞ −∞ f (x) cos (wx) dx + B(w) _ ∞ −∞ f (x) sin (wx) dx)dw _ ∞ −∞ [f (x)] 2 dx = π _ ∞ 0 (A 2 (w) + B 2 (w))dw =⇒ 1 π _ ∞ −∞ [f (x)] 2 dx = _ ∞ 0 (A 2 (w) + B 2 (w))dw b) Como f (x) = _ π, [x[ ≤ a 0, [x[ > a , es funci´ on par, entonces la integral de Fourier de f es _ ∞ 0 A(w) cos (wx) dw 82 con coeficiente A(w) = 2 π _ ∞ 0 f (u) cos (wu) du = 2 π _ a 0 π cos (wu) du = 2 _ sin wu w _ a 0 = 2 sin wa w ∴ A 2 (w) = 4 sin 2 wa w 2 Por otra parte: 1 π _ ∞ −∞ [f (x)] 2 dx = 1 π _ a −a π 2 dx = 2aπ Como la funci´ on f es par, se tiene 1 π _ ∞ −∞ [f (x)] 2 dx = _ ∞ 0 A 2 (w) dw Entonces, al reemplazar los t´erminos 2aπ = 4 _ ∞ 0 sin 2 wa w 2 dw ∴ _ ∞ 0 sin 2 wa w 2 dw = aπ 2 Problema 3 La serie de Fourier buscada es de la forma: f (x) ∼ a 0 + ∞ n=1 (a n cos nπx 5 + b n sin nπx 5 ) con coeficientes a 0 = 1 10 _ 5 0 3dx = _ 3x 10 _ 5 0 = 3 2 a n = 1 5 _ _ 0 −5 0 cos nπx 5 dx + _ 5 0 3 cos nπx 5 dx _ a n = 3 5 _ 5 nπ sin nπx 5 _ 5 0 = 0 b n = 1 5 _ _ 0 −5 0 sin nπx 5 dx + _ 5 0 3 sin nπx 5 dx _ 83 b n = 3 5 _ − 5 nπ cos nπx 5 _ 5 0 b n = 3 nπ [1 −cos nπ] Sustituyendo los coeficientes en la serie obtenemos 3 2 + 3 π ∞ n=1 [1 −cos nπ] n sin( nπx 5 ) b) Como f (x) es una funci´ on seccionalmente continua entonces en los puntos de discotinuidad converge a f (x + o ) + f (x − o ) 2 . Luego en los pun- tos −5, 0 y 5 converge a 3 2 . Por tanto la funci´ on debe asumir estos valores en los puntos menciona- dos para que se cumpla la covergencia propuesta. Autoevaluaci´ on N o 2 Tiempo : 2 horas Problema 1 Dada la funci´ on f (x) = x −[2x] , 0 < x < 1 a) Obtener la serie de Fourier de cosenos. b) Establecer la convergencia de la serie en x 0 = 0 y x 1 = 1 2 . Problema 2 Deducir la igualdad de la integral de Fourier x = 2 π _ _ − π cos πw w + sin πw w 2 _ sin wxdw para 0 < x < π. Problema 3 Sea f (x) continua por tramos, impar de per´ıodo 2π con serie de Fourier ∞ n=1 b n sin (nx) . a) Verificar que g (x) = _ x 0 f (t) dt , x ∈ R es funci´ on par de per´ıodo 2π. 84 b) Deducir ∞ n=1 b n n − ∞ n=1 b n n cos (nx) es la serie de Fourier de g (x) y que ∞ n=1 b n n = 1 π _ π 0 __ x 0 f (t) dt _ dx Pauta de Correcci´ on Problema 1 a) Consideremos una extensi´on par de f (x) de per´ıodo P = 2, es decir f (x + 2) = f (x) Como f (x) = x − [2x] , 0 < x < 1, se tiene que 0 < x < 1 2 , [2x] = 0 y 1 2 < x < 1, [2x] = 1. As´ı f (x) = _ ¸ _ ¸ _ x si 0 < x < 1 2 x −1 si 1 2 < x < 1 , luego los coeficientes de Fourier son a 0 = _ 1/2 0 xdx + _ 1 1/2 (x −1) dx = _ x 2 2 _ 1/2 0 + _ (x −1) 2 2 _ 1 1/2 = 0 a n = 2 _ 1/2 0 x cos nπxdx + 2 _ 1 1/2 (x −1) cos nπxdx a n = 2 _ x sin nπx nπ + cos nπx (nπ) 2 _ 1/2 0 +2 _ x sin nπx nπ + cos nπx (nπ) 2 _ 1 1/2 −2 _ sin nπx nπ _ 1 1/2 a n = 2 (nπ) 2 [cos nπ −1] + 2 sin _ nπ 2 _ nπ a n = 2 (nπ) 2 [(−1) n −1] + 2 (−1) n−1 (2n −1)π b n = 0 De este modo la serie de Fourier queda ∞ n=1 _ 2 (nπ) 2 [(−1) n −1] + 2 (−1) n−1 (2n −1)π _ cos nπx b) En x 0 = 0 hay un punto de continuidad de f, entonces la serie converge a 85 f (0) = 0 = ∞ n=1 _ 2 (nπ) 2 [(−1) n −1] + 2 (−1) n−1 (2n −1)π _ Para x 1 = 1 2 se tiene un punto de discontinuidad de f , entonces la serie converge a f _ 1 2 + _ + f _ 1 2 _ 2 = 1 2 + _ − 1 2 _ 2 = 0. Problema 2 Consideremos la funci´on f (x) = _ x si −π < [x[ < π 0 si [x[ > π , la cual es impar, entonces la integral de Fourier de f es _ ∞ 0 B(w) sin (wx) dx, con coeficiente: B(w) = 2 π _ ∞ 0 f (x) sin (wx) dx = 2 π _ π 0 x sin (wx) dx B(w) = 2 π _ −x cos wx w + sin (wx) w 2 _ π 0 B(w) = 2 π _ −π cos wπ w + sin (wπ) w 2 _ Por tanto la integral de Fourier , queda 2 π _ ∞ 0 _ −π cos wπ w + sin (wπ) w 2 _ sin (wx) dx Como f es continua en (0, π) , la integral de Fourier converge a f (x) = x = 2 π _ ∞ 0 _ −π cos wπ w + sin (wπ) w 2 _ sin wxdw Problema 3 a) En primer lugar, verifiquemos que g (x) es una funci´on par. En efecto sea g (−x) = _ −x 0 f (t) dt = _ x 0 f (−u) (−du), si cambiamos la variable u = −t Pero la funci´ on del integrando f (u) es una impar es decir f (−u) = −f (u) ∀u entonces g (−x) = _ x 0 f (u) (du) = g (x) 86 En segundo lugar, estudiemos el per´ıodo de la funci´on g (x + 2π) = _ x+2π 0 f (t) dt = _ x 0 f (t) dt + _ x+2π x f (t) dt = _ x 0 f (t) dt + _ π −π f (t) dt = _ x 0 f (t) dt = g (x) Por tanto g (x) tiene per´ıodo 2π b) Integrando la serie de Fourier de f , se tiene g (x) = _ x 0 f (t) dt = _ x 0 ∞ n=1 b n sin (nx) dt = _ − ∞ n=1 b n cos (nx) n _ x 0 = ∞ n=1 b n n − ∞ n=1 b n n cos (nx) Como g (x) es par su serie de Fourier es de la forma g(x) = a 0 + ∞ n=1 a n cos nx donde −π ≤ x ≤ π y coeficientes a 0 = 1 2π _ π −π g (x) dx = 1 π _ π 0 g (x) dx = 1 π _ π 0 __ x 0 f (t) dt _ dx = ∞ n=1 b n n Autovaluaci´on N o 3 Tiempo : 1 hora 20 minutos 87 Problema 1 Sea f (x) = cos bx, −π ≤ x ≤ π, con f (x + 2π) = f (x) , b constante no entera. a) Obtener la serie de Fourier de f. b) A partir de la convergencia de la serie, deducir la igualdad π senbπ = 2b _ 1 2b 2 − 1 b 2 −1 2 + 1 b 2 −2 2 ∓... _ Problema 2 Sea la funci´ on f (x) = _ _ _ _ 1 − [x[ a _ si [x[ < a 0 si [x[ > a a) Obtener la integral de Fourier de f. b) Deducir que 1 −x = 2 π _ ∞ 0 1 −cos w w 2 cos wxdw, si 0 ≤ x ≤ 1; y _ ∞ 0 1 −cos w w 2 cos wx dw = 0, si x > 1. Problema 3 a) Utilice la integral de Fourier para obtener la convergencia _ ∞ 0 sin w w dw = π 2 . b) Probar que la funci´on definida por g (x) = _ 1 x 0 sin w x w dw, x > 0 g (0) = π 2 es continua en cero. Pauta de Autocorrecci´ on Problema 1 a) Como f (x) = cos bx, en −π ≤ x ≤ π es funci´ on par, entonces la serie de Fourier de f es cosenoidal, es decir f (x) ∼ a 0 + ∞ n=1 a n cos nx con coeficientes 88 a 0 = 1 π _ π 0 cos bxdx = _ 1 bπ sin bπ _ π 0 = sin bπ bπ a n = 2 π _ π 0 cos bx cos nxdx = 2 π _ sin (b −n) x 2(b −n) + sin (b + n) x 2 (b + n) _ π 0 a n = 2 π _ sin (b −n) π 2(b −n) + sin (b + n) π 2 (b + n) _ a n = 2b (−1) n sin bπ π(b 2 −n 2 ) Por tanto la serie de fourier queda f (x) ∼ sin bπ bπ + 2b sin bπ π ∞ n=1 (−1) n (b 2 −n 2 ) cos nx b) En x 0 = 0 hay un punto de continuidad de f entonces por el teorema de convergencia de la serie, esta converge a f (0) = 1 = sin bπ bπ + 2b sin bπ π ∞ n=1 (−1) n (b 2 −n 2 ) Entonces π sin bπ = 1 b + 2b ∞ n=1 (−1) n (b 2 −n 2 ) ⇐⇒ π sin bπ = 2b _ 1 2b 2 + ∞ n=1 (−1) n (b 2 −n 2 ) _ ⇐⇒ π sin bπ = 2b _ 1 2b 2 − 1 b 2 −1 2 + 1 b 2 −2 2 ∓... _ Problema 2 a) Como: f (−x) = _ 1 − [−x[ a _ = _ 1 − [x[ a _ = f (x) ∀x ∈ [−a, a] la funci´on f es par, entonces la integral de Fourier de f es _ ∞ 0 A(w) cos (wx) dx, con coeficiente: A(w) = 2 π _ _ a 0 _ 1 − x a _ cos (wx) dx + _ ∞ a 0 cos (wx) dx _ A(w) = 2 π _ a 0 _ 1 − x a _ cos (wx) dx A(w) = 2 π _ 1 −cos aw aw 2 _ 89 Por tanto la integral de Fourier queda 2 π _ ∞ 0 _ 1 −cos aw aw 2 _ cos (wx) dx, b) Como la funci´ on f es continua para ∀x ∈ [0, 1],aplicando el teorema de convergencia con a = 1, se deduce que la integral de Fourier converge a f (x) = 1 −x = 2 π _ ∞ 0 1 −cos w w 2 cos wxdw, y del mismo modo f (x) = 0 = 2 π _ ∞ 0 1 −cos w w 2 cos wxdw, ∀x > 1 Problema 3 a)Sea la funci´ on f (x) = _ 1 si [x[ ≤ 1 0 si [x[ > 1 , como la funci´ on f es par, entonces la integral de Fourier de f es _ ∞ 0 A(w) cos (wx) dx, con coeficiente: A(w) = 2 π _ _ 1 0 1 cos (wx) dx + _ ∞ 1 0 cos (wx) dx _ A(w) = 2 π _ 1 0 cos (wx) dx A(w) = 2 π _ sin wx w _ 1 0 = 2 π sin w w Por tanto, la integral de Fourier queda f (x) = 2 π _ ∞ 0 _ sin w w _ cos (wx) dx Como la funci´ on es continua en x = 0,entonces la integral converge a f (0) = 1 = 2 π _ ∞ 0 sin w w dx =⇒ _ ∞ 0 sin w w dx = π 2 b) Sea la integral g (x) = _ 1 x 0 sin w x w dw, x > 0 g (0) = π 2 90 aplicando el cambio de variables u = w x =⇒ xdu = dw, se tiene g (x) = _ 1 x 2 0 sin u xu xdu = _ 1 x 2 0 sin u u du Calculemos ahora el l´ımite de ´esta funci´on cuando x →0 lim x→0 g (x) = l´ım x→0 _ 1 x 2 0 sin u u du = _ ∞ 0 sin u u du = π 2 = g (0) Lo que implica que g(x) es continua en cero. 91 Cap´ıtulo 2 Funciones Vectoriales de una variable real 2.1. Introducci´ on La recta de R 3 que pasa por el punto − → P 0 = (x 0 , y 0 , z 0 ) y es paralela a un vector − → a = (a 1, a 2, a 3 ) se define como el conjunto _ − → P 0 + t − → a [ t ∈ R _ .En esta definici´on de recta a cada n´ umero real t corresponde el punto − → P 0 + t − → a de R 3 es decir a cada valor t de R le asocia el punto (x 0 + ta 1, y 0 + ta 2, z 0 + ta 3 ) de R 3 . Tal correspondencia o asociaci´on genera lo que llamaremos una funci´on vectorial de una variable real que en este caso es de R en R 3 . Si denotamos por − → f a tal funci´on entonces su regla de correspondencia es − → f (t) = (x 0 + ta 1, y 0 + ta 2, z 0 + ta 3 ) El dominio de − → f es el conjunto de todos los n´ umeros reales y el rango de − → f es la recta que pasa por el punto − → P o y es paralela al vector − → a . Este es un ejemplo del tipo de funciones que estudiaremos en este m´ odulo; para tales funciones consideraremos los conceptos de l´ımite, continuidad, derivada e integral. Desde el punto de vista conceptual no hallaremos ideas nuevas y en la mayor parte de los casos las t´ecnicas usadas son las mismas desarrolladas en el c´ alculo de funciones real de una variable real. 92 2.2. Funciones Vectoriales Definici´ on 2.2.1. Una funci´on vectorial de una variable real es una funci´on cuyo dominio es un conjunto de n´ umeros reales y el rango es un conjunto de vectores o puntos de R n Notaci´ on − → f : D ⊆ R →R n tal que ∀ t ∈ D, − → f (t) = (f 1 (t), f 2 (t), . . . , f n (t)), donde f k : D ⊆ R → R para cada k = 1, 2, .., n es una funci´on real de variable real. Cada f k es la k-´esima componente del vector − → f (t). Si la funci´on − → f describe el movimiento de una part´ıcula, el vector − → f (t) = (f 1 (t), f 2 (t), . . . , f n (t)) se˜ nala la posici´ on en el instante t, es decir en estos casos t representa la variable tiempo. Ejemplo 1 Sea − → f : I ⊆ R → R 3 tal que − → f (t) = (cos t, sin t, t), I = [0, 2π] Hacer un esquema del rango de −→ f Soluci´ on: Pongamos − → f (t) = (x(t), y(t), z(t)) donde x = cos t, y = sin t, z = t. En este caso para cualquier valor de t se cumple x 2 + y 2 = 1 que es la proyecci´ on en el plano XY de cualquier punto − → f (t) de la curva que est´ a sobre el manto de un cilindro de radio unitario x 2 + y 2 = 1, y z = t se˜ nala la distancia de − → f (t) al plano XY. El rango de − → f es entonces una curva que partiendo de (1, 0, 0) describe un arco completo de una helicoidal en el manto del cilindro x 2 + y 2 = 1 de R 3 . 93 Ejemplo 2 Sea − → f : I ⊆ R →R 3 tal que − → f (t) = (t, t, t), describa el rango de − → f . Soluci´on: Las imagenes − → f(t) = (t, t, t) las podemos escribir vectorialmente de la forma − → f (t) = (0, 0, 0) + t(1, 1, 1) lo que nos permite reconocer que se trata de una recta que pasa por el origen (0, 0, 0) en la direcci´ on del vector − → v = (1, 1, 1). Ejemplo 3 Sea − → f : I ⊆ R → R 3 tal que − → f = (t, t, 2t 2 ), I = [−3, 3] describa el rango de − → f. Soluci´on: Ponemos − → f (t) = t(1, 1, 0) + t 2 (0, 0, 2), de esta expresi´ on se puede afir- mar que − → f (t) es la suma de un vector a lo largo de la recta y = x en el plano XY y un vector perpendicular al plano XY. Quiere decir entonces que el rango de − → f se encuentra en el plano que contiene los vectores (1, 1, 0), (0, 0, 2) perpendicular al plano XY. Si se considera en un punto (t, t, 0) en el plano XY y u distancia al origen, u = √ t 2 + t 2 = √ 2t, resulta que z = 2t 2 = u 2 . Por lo tanto, el rango de − → f es una porci´ on de la par´ abola z = u 2 que esta en el plano y = x perpendicular al plano XY y que contiene al eje z. 94 2.3. L´ımite de una funci´ on vectorial. Previamente aclaremos o recordemos algunos conceptos en cuanto a la m´etrica que usaremos. Definici´ on 2.3.1. (distancia) Si − → a y − → b son una par de elementos (puntos) de R n dados por − → a = (a 1 , a 2 , a 3 , . . . , a n ), − → b = (b 1 , b 2 , b 3 , . . . , b n ) la distancia desde − → a hasta − → b es _ _ _ − → b − − → a _ _ _ se define por _ _ _ − → b − − → a _ _ _ = _ n i=1 (b i −a i ) 2 _1 2 que en R 2 y R 3 viene a corresponder a lo que definimos como distancias entre dos puntos. Definici´ on 2.3.2. Se dice que el vector − → l = (l 1 , l 2 , l 3 , . . . , l n ) es el l´ımite de la funci´on vectorial − → f : I ⊆ R →R n en t 0 ∈ I , si para cada > 0 existe un n´ umero δ > 0 tal que siempre que t esta en el dominio de − → f y 0 < [t −t 0 [ < δ entonces _ _ _ − → f (t) − − → l _ _ _ < 2.3.1. Teorema del l´ımite Teorema 2.3.1. Sea − → f : I ⊆ R →R n funci´on vectorial. Entonces 95 l´ım t→t 0 − → f (t) = − → l = (l 1 , l 2 , l 3 , . . . , l n ) ⇐⇒ l´ım t→t 0 f k (t) = l k , k = 1, 2, ..., n donde − → f (t) = (f 1 (t), f 2 (t), . . . , f n (t)) Demostraci´ on.- i) (⇐) Suponemos que l´ım t→t 0 f k (t) = l k , k = 1, 2, ..., n Sea > 0 dado _ _ _ − → f (t) − − → l _ _ _ = |(f 1 (t) −l 1 , f 2 (t) −l 2 , ..., f n (t) −l n )| = |(f 1 (t) −l 1 , 0, 0, ..., 0) + (0, f 2 (t) −l 2 , 0, ..., 0) + (0, 0, 0, ..., f n (t) −l n )| ≤ |(f 1 (t) −l, 0, ..., 0)| +|(0, f 2 (t) −l 2 , 0, ..,0)| + ... +|(0, ..., 0, f n (t) −l n )| ≤ [f 1 (t) −l[ +[f 2 (t) −l 2 [ + ... +[f n (t) −l n [ Si l´ım t→t 0 f k (t) = l k =⇒Si n > 0 existe δ k > 0 tal que 0 < [t −t 0 [ < δ k =⇒[f k (t) −l k [ < n , k = 1, 2, ..., n tomando δ = m´ın ¦δ k ¦ , k = 1, 2, ..., n se tiene que 0 < [t −t 0 [ < δ =⇒ [f k (t) −l k [ < n para todo k = 1, 2, .., n. De la desigualdad anterior, mayorando por n . _ _ _ − → f (t) − − → l _ _ _ < n + ... + n = n i=1 n = n n = ∀ t ∈ (t 0 −δ, t 0 + δ), lo que prueba esta parte del teorema. ii) (⇒) Suponemos que l´ım t→t 0 − → f (t) = − → l Sea > 0 dado l´ım t→t 0 − → f (t) = − → l = (l 1 , l 2 , l 3 , . . . , l n ) y − → f (t) = (f 1 (t), f 2 (t), . . . , f n (t)) implica que existe δ > 0 tal que 96 _ _ _ − → f (t) − − → l _ _ _ < todo t en el dominio de − → f tal que 0 < [t −t 0 [ < δ pero [f k (t) −l k [ < |f(t) −l| todo k, entonces [f k (t) −l k [ < todo t en el dominio de −→ f tal que 0 < [t −t 0 [ < δ. Por lo tanto l´ım t→t 0 f k (t) = l k todo k = 1, 2, ..., n. Se ha establecido entonces que: lim t→t o − → f (t) = − → l = (l 1 , l 2 , l 3 , ...., l n ) ⇔ lim t→t o f k (t) = l k , k = 1, 2, 3, ...., n. Ejemplo 4 Si − → f (t) = (cos t, sin t) , calcule l´ım t→ π 2 − → f (t) Soluci´on: l´ım t→ π 2 − → f (t) = _ l´ım t→ π 2 cos t, l´ım t→ π 2 sin t _ = (0, 1) Ejemplo 5 La trayectoria de una part´ıcula en el espacio R 3 est´ a dada por la funci´ on vectorial − → c (t) = (cos 2t, sin t, 2t π ). Calcule: lim t→ π 2 − → c (t), lim t→π − → c (t) y lim t→− π 2 − → c (t) . Soluci´on.- l´ım t→ π 2 (cos 2t, sin t, 2t π ) = (l´ım t→ π 2 cos 2t, l´ım t→ π 2 sin t, l´ım t→ π 2 2t π ) =(−1, 1, 1) l´ım t→π (cos 2t, sin t, 2t π ) = (l´ım t→π cos 2t, l´ım t→π sin t, l´ım t→π 2t π ) = (1, 0, 2) l´ım t→− π 2 (cos 2t, sin t, 2t π ) = ( l´ım t→− π 2 cos 2t, l´ım t→− π 2 sin t, l´ım t→− π 2 2t π ) = (−1, −1, −1) 97 2.3.2. Operaciones con funciones vectoriales Definici´ on 2.3.3. Sean − → f , − → g : I ⊆ R → R n funciones vectoriales, entonces, para cada t ∈ I se define − → f + − → g , − → f − − → g , − → f − → g , − → f − → g de la forma siguiente: a) ( − → f + − → g )(t) = − → f (t) + − → g (t) = ((f 1 + g 1 )(t), (f 2 + g 2 )(t), ..., (f n + g n )(t)) b) ( − → f − − → g )(t) = − → f (t) − − → g (t) = ((f 1 −g 1 )(t), (f 2 −g 2 )(t), ..., (f n −g n )(t)) c) ( − → f − → g )(t) = − → f (t) − → g (t) = f 1 (t)g 1 (t) + f 2 (t)g 2 (t) + ... + f n (t)g n (t) = n i=1 f i (t) g i (t) d) ( − → f − → g )(t) = − → f (t) − → g (t) = ((f 2 g 3 )(t) −(f 3 g 2 )(t)), (f 3 g 1 )(t) −(f 1 g 3 )(t), (f 1 g 2 )(t) −(f 2 g 1 )(t)). en este caso para n = 3. Definici´on 2.3.4. Producto por escalar Si γ : I →R y − → f : I →R n es funci´on vectorial, definimos: γ − → f : I →R n tal que (γ − → f )(t) = γ(t) − → f (t) = (γ(t)f 1 (t), γ(t)f 2 (t), ..., γ(t)f n (t)) Estas definiciones nos llevan al siguiente teorema 2.3.3. Teoremas del algebra de l´ımites Teorema 2.3.2. Sean − → f , − → g : I ⊆ R → R n funciones vectoriales, t o ∈ I , si l´ım t→t o − → f (t) = − → a y l´ım t→t o − → g (t) = − → b ; − → a , − → b ∈ R n ,entonces: a) l´ım t→t o _ − → f + − → g _ (t) = lim t→t o − → f (t) + l´ım t→t o − → g (t) = − → a + − → b b) l´ım t→t o _ − → f − − → g _ (t) = l´ım t→t o − → f (t) − l´ım t→t o − → g (t) = − → a − − → b c) l´ım t→t o _ − → f − → g _ (t) = l´ım t→t o − → f (t) l´ım t→t o − → g (t) = − → a − → b d) l´ım t→t o _ − → f − → g _ (t) = l´ım t→t o − → f (t) l´ım t→t o − → g (t) = − → a − → b , con n = 3 en este caso. 98 Demostraci´ on: Es consecuencia directa de aplicaci´ on del Teorema del L´ımite y la definici´ on de las operaciones. Se deja como ejercicio al lector. 2.3.4. Teorema: producto de funci´ on escalar por vectorial Teorema 2.3.3. Sean γ : I ⊆ R → R y − → f : I ⊆ R → R n , t o ∈ I tal que: l´ım t→t o γ(t) = α ; y l´ım t→t o − → f (t) = − → a , α ∈ R y − → a ∈ R n , entonces: lim t→t o _ γ − → f _ (t) = lim t→t o γ(t)lim t→t o − → f (t) = α − → a Demostraci´ on: Se deja como ejercicio al lector. 2.4. Continuidad Definici´ on 2.4.1. Sea − → f : I → R n y t o ∈ I , diremos que − → f es con- tinua en t o si para cada ε > 0,existe δ > 0 tal que ∀ t ∈ I : [t −t o [ < δ,entonces _ _ _ − → f (t) − − → f (t o ) _ _ _ < ε Note que: si − → f = (f 1 , f 2 , f 3 , ..., f n ) tal que − → f : I → R n y f i : I → R, i = 1, 2, 3, ..., n. Entonces − → f es continua en t o , ssi, f i es cont´ınua en t o para todo i = 1, 2, 3, ..., n. En efecto, vea que [f i (t) −f i (t o )[ ≤ _ n i=1 ((f i (t) −f i (t o )) 2 _1 2 =⇒ si − → f es continua, entonces f i es continua. Reciprocamente co- mo _ _ _ − → f (t) − − → f (t o ) _ _ _ ≤ n i=1 [f i (t) −f i (t o [ podemos inferir que si f i es continua, i = 1, 2, 3, ..., n,entonces − → f es continua. Por lo tanto, decimos que una funci´ on vectorial es continua ,si y s´ olo si, lo son cada una de sus funciones componentes 99 Observaci´ on: Los teoremas de continuidad que obviaremos en este caso y que son una r´eplica de los teoremas de l´ımites se pueden probar f´ acilmente. 2.5. La Derivada Definici´ on 2.5.1. Sea − → f : I ⊆ R →R n funci´on vectorial y t o ∈ I. Se define la derivada de − → f en t o , denotada d −→ f dt (t 0 ) o − → f´(t o ) por el l´ımite: − → f´(t o ) = l´ım h→0 −→ f (t o +h)− −→ f (t o ) h , cuando este l´ımite existe. Geom´etricamente − → c (t o ) es un vector tangente a la curva C descrita por la trayectoria − → c (t) en el punto − → c (t 0 ) orientado. De la definici´ on del l´ımite se deduce que: Teorema 2.5.1. Sea − → f : I ⊆ R →R n , t o ∈ I y − → f = (f 1 , f 2 , f 3 , ..., f n ) entonces − → f es derivable en t o si y solo si cada f i es derivable en t o . Demostraci´on − → f ‘(t o ) = l´ım h→0 − → f (t o + h) − − → f (t o ) h 100 = lim h→0 1 h [f 1 (t o + h) −f 1 (t o ), f 2 (t o + h) −f 2 (t o ), ..., f n (t o + h) −f n (t o )] = _ lim h→0 f 1 (t o +h)−f 1 (t o ) h , l´ım h→0 f 2 (t o +h)−f 2 (t o ) h , ..., l´ım h→0 f n (t o +h)−f n (t o ) h _ = _ f 1 (t o ), f 2 (t o ), ..., f n (t o ) ¸ Por lo tanto − → f (t o ) existe si y solo si f k (t o ) existe para k = 1, 2, 3, ..., n Definici´ on Si suponemos que − → c (t) describe una trayectoria seguida por una part´ıcula podemos definir al vector − → c´(t) como el vector velocidad en la trayectoria − → c en el punto − → c (t). Asimismo , definiremos la rapidez en ese punto como ¸ ¸ − → c ´ (t) ¸ ¸ Ejemplo 6 Sea − → c (t) = (r cos(t), r sin(t)), trayectoria cuyo camino corresponde a una circunferencia de radio r. Muestre que − → c (t) y − → c (t) son ortogonales Muestre que − → c (t) y − → c (t) son ortogonales Soluci´on. Tenemos − → c (t) es derivable lo que implica − → c (t) = (−r sin t, r cos t) Por consiguiente − → c (t) − → c (t) = (r cos(t), r sin(t)) (−r sin t, r cos t) = r (sin t) r cos (t) −r (cos t) r sin (t) = 0 ∴ − → c (t) y − → c (t) son ortogonales ∀ t. 101 2.6. Regularidad de una curva Una diferencia significativa de la derivada de funciones vectoriales re- specto de la suavidad o regularidad de la curva en el aspecto geom´etrico es que en este caso la derivabilidad no detecta picos en la curva. Para describir este hecho consideramos el siguiente ejemplo: Ejemplo 7 Sea − → f : R → R 2 definida por − → f (t) = (t 3 , t 2 [t[). Su gr´ afico como muestra la figura es similar al gr´ afico de y = [x[ en R 2 y sabemos que la funci´on f (x) = [x[ no es derivable en x = 0. Muestre que la funci´ on dada es derivable para t = 0. Soluci´on − → f (t) = (t 3 , t 2 [t[) ⇒x(t) = t 3 , y(t) = t 2 [t[ x(t) = t 3 ⇒x/(t) = 3t 2 , ∀ t, y(t) = t 2 [t[ ⇒y/(t) = 3t 2 , t > 0−3t 2 , t < 0 Adem´ as si t = 0 ⇒ y/(0) = lim h→0 y(h) −y(0) h = lim h→0 h 2 [h[ h = lim h→0 h[h[ = 0 ∴ − → f´(0) = (0, 0) Lo que prueba que esta funci´on es derivable en todo R y que hay puntos en los cuales la derivada es cero, esto geometricamente significa que la curva no es suave en ese punto, cambia rapidamente de direcci´on, presenta un peak. Con el objeto de advertir este comportamiento geom´etrico en una curva y su relaci´ on con la derivada demos la siguiente definici´on. 102 2.6.1. Camino regular Definici´ on 2.6.1. Sea − → f : I ⊆ R →R n una funci´on de clase C 1 (I).Se dice que − → f (t) describe un camino regular si − → f (t) ,= − → 0 ∀ t ∈ I. Consideremos el siguiente ejemplo para aclarar a´ un m´ as este concepto Ejemplo 8 Sea la curva C es descrita por − → c (t) = (cos 3 t, sin 3 t), 0 ≤ t ≤ 2π, que corresponde a un Astroide Muestre que no es una curva regular en 0 ≤ t ≤ 2π, Soluci´on Esta trayectoria regular no es regular, pues en t = 0, π 2 , π, y 3π 2 la derivada se anula como podemos verificar en el siguiente c´ alculo. − → c´(t) = (3 cos 2 t(−sin t); 3 sin 2 t cos 3 t) = (−3 cos 2 t sin t; 3 sin 2 t cos 3 t) − → c´(0) = (0, 0) ; − → c´( π 2 ) = (0, 0) , − → c´(π) = (0, 0) , y − → c´( 3π 2 ) = (0, 0) 2.6.2. Propiedades de la Derivada Teorema 2.6.1. Sean − → f , − → g : I ⊆ R → R n funciones derivables y α : I ⊆ R→R una funci´on derivable, entonces: i) (k − → f ) (t) = k − → f´(t) 103 ii) ( − → f + − → g ) (t) = − → f (t) + − → g (t) iii) ( − → f − − → g ) (t) = − → f (t) − − → g (t) iv) (α − → f ) (t) = α (t) − → f (t) + α(t) − → f (t) v) ( − → f − → g ) (t) = − → f´(t) − → g (t)+ − → f (t) − → g´(t) Producto Punto o Producto interno vi) ( − → f ◦ α)´(t) = − → f´(α(t))α´(t) vii) ( − → f − → g ) (t) = − → f´(t) − → g (t) + − → f (t) − → g (t) para n = 3 Demostraci´ on: i), ii), iii) se dejan al lector (α − → f ) (t) = (α(t) − → f (t))´= g ((αf 1 )´(t), (αf 2 )´(t), (αf 3 )´(t), ..., (αf n )´(t)) = (α (t)f 1 (t) + α(t)f´ 1 (t), α (t)f 2 (t) + α(t)f 2 (t), ..., α (t)f n (t) + α(t)f n (t)) = (α (t)f 1 (t), α (t)f 2 (t), ..., α (t)f n (t) + α(t)f 1 (t), α(t)f 2 (t), ..., α(t)f n (t)) = α (t)(f 1 (t), f 2 (t), ..., f n (t)) + α(t)(f 1 (t), f 2 (t), ..., f n (t)) = α (t) − → f (t) + α(t) − → f (t) v) ( − → f − → g ) (t) = _ − → f (t) − → g (t) _ = d dt _ n k=1 f i (t)g i (t) _ = n k=1 d dt (f k (t)g k (t)) = n k=1 [f k (t)g k (t) + f k (t)g k (t)] = n k=1 f k (t)g k (t) + n k=1 f k (t)g k (t) = − → f (t) − → g (t) + − → f (t) − → g (t) v), vi) se dejan al lector. 2.7. Parametrizaci´ on Si − → c (t) define una trayectoria donde t es el par´ ametro, podemos mod- ificar la expresi´ on que define − → c (t) por − → c (s) de tal modo de tener el mismo conjunto de imagenes, esto lo llamaremos reparametrizaci´on. 104 Definici´ on 2.7.1. Sean − → f : I ⊆ R →R n tal que − → f = (f 1 , f 2 , f 3 , ..., f n ) define un camino regular en el espacio R n y ϕ : I 1 → I una fun- ci´on de clase C 1 (I) biyectiva y tal que ϕ (s) ,= 0, ∀ s ∈ I 1 , entonces − → f o : I 1 →R n tal que − → f o (s) = ( − → f ◦ ϕ)(s) se llama reparametrizaci´ on de la trayectoria) − → f . Observaci´ on: De esta definici´ on se tiene que: 1) − → f o (s) = ( − → f ◦ ϕ)(s) ⇒ − → f (s) = − → f´(ϕ(s)) ϕ´(s) si ϕ(s) = t ⇒ − → f o (s) = − → f´(t) ϕ´(s) = ϕ´(s) − → f´(t) ∴ − → f o (s) = ϕ´(s) − → f (t) como ϕ(s) es un escalar, esto significa que − → f o (s) es ϕ´(s) veces la velocidad que llevar´ıa una part´ıcula parametrizada por − → f (t). 2) ϕ (s) ,= 0 ⇒ϕ´(s) > 0, ∀s ∈ I 1 v ϕ´(s) < 0, ∀s ∈ I, de la expresi´ on − → f o (s) = ϕ´(s) − → f (t) se puede inferir que: a.- Si ϕ´(s) > 0 , ∀ s ∈ I 1 ⇒ − → f o (s) conserva la orientaci´ on de − → f (t) b.- Si ϕ´(s) < 0 , ∀ s ∈ I 1 ⇒ − → f o (s) invierte la orientaci´on de − → f Si − → f o (s) es una reparametrizaci´ on de − → f (t) y del hecho que − → f o (s) = ϕ´(s) − → f´ (t) en cada punto − → f o (s) = − → f (t) si ϕ(s) = t, se deduce el 105 siguiente teorema. Teorema 2.7.1. Sea − → f : I ⊆ R → R 2 ( o R 3 ) un camino regular y − → f o = − → f ◦ ϕ : J ⊆ R →R 2 ( o R 3 ) una reparametrizaci´on de ´el ( donde ϕ tiene las condiciones pedidas en la definici´on). Entonces la recta tangente a la curva C (traza de − → f ) en − → f (t o ) con t 0 ∈ I , es la misma que la recta tangente a C en − → f o (s 0 ) si t 0 = ϕ(s 0 ). Demostraci´ on.- t 0 = ϕ(s 0 ) ⇒ − → P 0 = − → f (t o ) = − → f (ϕ(s 0 ) ) = − → f o (s 0 ) adem´as − → f o ´(s) = ϕ´(s) − → f´(s) implica que los vectores − → f (t o ) y − → f o (s 0 ) son paralelos,entonces las rectas tangentes a la curva en − → f (t o ) coinciden. 2.7.1. Ejemplos de reparametrizaciones Sea − → f : [a, b] →R 3 ,una trayectoria regular . Entonces: 1) La trayectoria − → f o : [a, b] → R 3 tal que t −→ − → f (a + b −t) es la reparametrizacion de − → f que corresponde a la aplicacion ϕ : [a, b] −→ [a, b] dada por t −→ a + b − t, llamamos a − → f o trayectoria opuesta a − → f . 2) La trayectoria − → g : [0, 1] →R 3 tal que t −→ − → f (a + (b −a)t) es una reparametrizacion de − → f que corresponde a la aplicaci´on ϕ : [0, 1] −→ [a, b] dada por t −→a + (b −a)t, y que conserva la trayectoria de − → f . Ejemplo 9 Sea − → f : [−5, 10] →R 3 definida por t −→ − → f (t) = (t, t 2 , t 3 ) . Reparametrizar como trayectoria opuesta a − → f . Soluci´ on Aplicando el apartado 1) tenemos: − → f o : [−5, 10] → R 3 tal que t −→ − → f (5 −t) = ((5 −t), (5 −t) 2 , (5 −t) 3 ) es la reparametrizaci´on opuesta a − → f que corresponde a la aplicaci´ on ϕ : [−5, 10] −→[−5, 10] dada por t −→5 −t. Ejercicio Sea − → f : [a, b] →R 3 , una trayectoria regular y k una contante positiva . Sea la aplicaci´on ϕ : _ 0, b−a k ¸ −→ [a, b] dada por ϕ(t) −→ kt + a. 106 Muestre que la trayectoria reparametizada − → f ◦ ϕ : _ 0, b−a k ¸ → R 3 mantiene la trayectoria de − → f pero la recorre k veces m´as r´ apido. 2.8. Longitud de Arco Sea C una curva descrita por − → f (t) = (f 1 (t), f 2 (t), f 3 (t), ..., f n (t)), defini- da en un intervalo I = [a, b] en R.Sea P una partici´ on de [a, b] y L p la longitud de la poligonal originada por P. L p = n i=1 _ _ _ − → f (t i ) − − → f (t i−1 ) _ _ _ Para cada partici´on P se tiene una correspondiente L p . Definici´ on 2.8.1. La curva C descrita por − → f (t) = (f 1 (t), f 2 (t), f 3 (t), ..., f n (t)) definida en [a, b] se dice que es rectificable si ¦L p /P es partici´on de [a, b]¦ tiene una cota superior. Si C es rectificable entonces la longitud de C es el supremo del conjunto de los L p , es decir = sup ¦L p /P es partici´on de [a, b]¦ Definici´ on 2.8.2. Sea − → f : I ⊆ R → R n una trayectoria regular de clase C 1 . La longitud de − → f entre t = a, t = b denotada por ( − → f ) o simplemente se define por: = _ b a _ _ _ − → f´(t) _ _ _ dt 107 Ejemplo 10. Si − → c : [0, 2π] →R 2 tal que − → c (t) = (r cos t, r sin t). ¿ Es una trayectoria regular? ¿Cu´al es la longitud de la Curva asociada? Soluci´on. Como − → c´(t) = (−r sin t, r cos t) ,= (0, 0) ∀t ∈ [0, 2π] la trayectoria es regular, luego | − → c´(t)| = _ (−r cos t) 2 + (r sin t) 2 ) = r ∴ = _ 2π 0 | − → c´(t)| dt = _ 2π 0 rdt = 2πr Ejemplo 11. ¿Cu´ al es la longitud del astroide dado por la ecuaci´ on: − → c (t) = (a cos 3 t, a sin 3 t) con t ∈ [0, 2π]? Soluci´on: Verifiquemos primeramente si el astroide es una curva regular: − → c´(t) = (−3a cos 2 t sin t, 3a cos t sin 2 t) Podemos inferir que el astroide no es una curva regular, pues − → c´(t) = (−3a cos 2 t sin t, 3a cos t sin 2 t) = (0, 0) para t = 0, π 2 , π, 3π 2 , 2π pero dado que es sim´etrica respecto de ambos ejes podemos calcular su longitud en el segmento del dominio ¸ 0, π 2 _ y luego multiplicamos por 108 cuatro. 4 = π 2 _ 0 | − → c´(t)| dt = π 2 _ 0 _ 9a 2 cos 4 t sin 2 t + 9a 2 sin 4 t cos 2 tdt = 3a π 2 _ 0 sin t cos tdt = _ 3a sin 2 t 2 _ π 2 0 = 3a 2 =⇒ ∴ = 6a 2.8.1. La Longitud de Arco como Par´ametro Teorema 2.8.1. Sea − → f : I ⊆ R → R n funci´on de clase C 1 tal que − → f´(t) ,= − → 0 ∀ t ∈ I . Entonces la longitud s definida por s(t) = t _ 0 _ _ _ − → f (u) _ _ _ du, con t ∈ I , de una curva puede introducirse como un par´ametro de la curva y : _ _ _ d −→ f ds _ _ _ = 1 Demostraci´ on: s(t) = t _ 0 _ _ _ − → f (u) _ _ _ du ⇒s´(t) = ds dt = _ _ _ − → f (t) _ _ _ ≥ 0 Claramente s = s(t) es mon´ otona, estrictamente creciente y cont´ınua, por lo tanto s(t) tiene funci´on inversa, que denominaremos por t(s) tal que t´(s) existe y es positiva para todo s con 0 ≤ s ≤ l( − → f ). − → f (t(s)) = (f 1 (t(s)), f 2 (t(s)), ..., f n (t(s)) Derivando usando la regla de la cadena tenemos: 109 d − → f ds = (f 1 ´(t(s)) t´(s), f 2 ´(t(s)) t´(s), ..., f n ´(t(s)) t´(s)) = d − → f dt dt ds = d −→ f dt ds dt De donde se obtiene que: _ _ _ _ _ d − → f ds _ _ _ _ _ = _ _ _ − → f´(t) _ _ _ |s´(t)| =⇒ _ _ _ − → f´(t) _ _ _ s´(t) = 1 Observaci´ on: 1. Del teorema anterior se desprende, la rapidez con que recorre − → f es constante e igual a 1 si est´a param´etrizada con el par´ ametro longitud de arco. 2. Si − → r (s) = (x(s), y(s), z(s)) describe una curva de R 3 , y s es par´ ametro longitud de arco, entonces ´ T = d −→ r ds es vector tangente unitario, ya que: _ _ _ ´ T _ _ _ = _ _ d r ds _ _ = 1 Propiedades 2.8.1. La longitud de un arco es independiente de la parametrizaci´on Demostraci´ on: Sean − → f : [a, b] → R n un camino regular y − → f ∗ : [c, d] → R n una reparametrizaci´ on de − → f ; por lo cual existe ϕ de [c, d] en [a, b] de C 1 que es sobreyectiva y ϕ´(s) ,= 0 ∀ s ∈ [c, d] Entonces: 110 s _ − → f o _ = d _ c _ _ _ − → f ∗ ´(u) _ _ _ du = d _ c _ _ _ − → f´(ϕ(u))ϕ´(u) _ _ _ du = d _ c _ _ _ − → f´(ϕ(u)) _ _ _ ϕ´(u)du si ϕ´(u) > 0 t = ϕ(u) ⇒s _ − → f o _ = b _ a _ _ _ − → f´(t) _ _ _ dt = s( − → f ) ∴ s _ − → f o _ = s( − → f ) De la misma forma si ϕ´(u) < 0, la parametrizaci´ on invierte el sentido y ϕ(c) = b, ϕ(d) = a ∴ s _ − → f ∗ _ = d _ c _ _ _ − → f´(ϕ(u))ϕ´(u) _ _ _ du = d _ c _ _ _ − → f´(ϕ(u)) _ _ _ (−ϕ´(u)) du = − d _ c _ _ _ − → f´(ϕ(u)) _ _ _ ϕ´(u)du = d _ c _ _ _ − → f´(t)) _ _ _ dt =⇒ s _ − → f o _ = s( − → f ) 2.8.2. Parametrizaci´ on por Longitud de Arco Definici´ on 2.8.3. Sea − → f : I ⊆ R →R n , funci´on de clase C 1 y tal que − → f´(t) ,= 0 en I = [a, b] , se define s por s = t _ a _ _ _ − → f (u) _ _ _ du = ϕ(t) De la definici´ on de s se pueden hacer las siguientes precisiones: 1. s = ϕ(t) = t _ a _ _ _ − → f´(u) _ _ _ du ⇒ ds dt = ϕ´(t) = _ _ _ − → f´(t) _ _ _ 111 2. Si t ∈ (a, b] ⇒ ds dt > 0 ⇒ s = ϕ(t) es funci´ on estrictamente creciente. 3. Por (2) s = ϕ(t) es biyectiva y por lo tanto invertible, sea t = ϕ −1 (s). Por tanto, la parametrizaci´on de − → f en t´erminos s es: − → f (s) = − → f (ϕ −1 (s)) Ejemplo 12 Considere la trayectoria: − → r (t) = (a cos t, asent) ,y reparametrice en funci´ on de la longitud de arco s. Soluci´on La ecuaci´ on cartesiana de la curva C es x 2 +y 2 = a 2 ,que corresponde a una circunsferencia centrada en el origen de radio a. Verifiquemos si es una curva regular, su derivada es: − → r´(t) = (−a sin t, a cos t) ,= (0, 0) ∀t ⇒ | − → r´(t)| = _ a 2 sin 2 t + a 2 cos 2 t ∴ | − → r´(t)| = a Determinemos su longitud de arco, s = t _ 0 | − → r´(u)| du = t _ 0 adu = [au] t 0 = at ∴ s = at o t = s a Por lo tanto, − → r (s) = (a cos s a , a sin s a ) es una parametrizaci´ on por lon- gitud de arco de la circunferencia de radio a. Observaci´ on: No siempre es sencillo parametrizar por longitud de arco, a modo de ejemplo veamos el siguiente caso. Ejemplo 13 112 Considere la curva C curva descrita por la trayectoria − → r (t) = (t, t 2 +1), t ∈ [0, 3] , y parametrice en funci´ on de la longitud de arco s. Soluci´on. La curva corresponde a una parab´ ola que tiene por ecuaci´ on cartesiana: y = x 2 + 1, x ∈ [0, 3] Determinemos la longitud de la curva: − → r´(t) = (1, 2t) ⇒|r´(t)| = √ 1 + 4t 2 s = t _ 0 √ 1 + 4u 2 du = _ ln ¸ ¸ √ 1 + 4u 2 + 2u ¸ ¸ 4 + u √ 1 + 4u 2 2 _ t 0 s = 1 4 ln ¸ ¸ ¸ √ 1 + 4t 2 + 2t ¸ ¸ ¸ + t 2 √ 1 + 4t 2 En este caso no es posible obtener t = ϕ −1 (s) es decir despejar t en funci´ on de s a partir de s = ϕ(t) = 1 4 ln ¸ ¸ ¸ √ 1 + 4t 2 + 2t ¸ ¸ ¸ + t 2 √ 1 + 4t 2 Luego no es posible hacer explicita la funci´on t = g −1 (s). 2.9. Trayectorias y curvas La aplicaci´ on − → c : [a, b] −→ R 3 (o R 2 ) continua , definida de un inter- valo I al espacio R 3 o al plano R 2 , la llamaremos trayectoria. La imagen C en R 3 (o R 2 ) de la trayectoria corresponde a lo que es una curva. Definici´ on 2.9.1. Se llama curva de la trayectoria − → c : I ⊂ R → R 3 (R 2 ) dada por − → c (t) = (x(t), y(t), z(t)) al conjunto de imagenes de − → c ;es decir C = _ − → c (t) ∈ R 3 / t ∈ I _ = _ (x(t), y(t), z(t)) ∈ R 3 : t ∈ I _ 113 Ejemplo 1: Sea − → c : R → R 3 una funci´ on definida por − → c (t) = (x 0, y 0, z 0 )+t(v 1, v 2, v 3 ) es una recta L en el espacio que pasa por (x 0, y 0, z 0 ) y tiene la direcci´ on − → v = (v 1, v 2, v 3 ). A partir de esta funci´on se deducen las ecuaciones: x(t) = x 0 + tv 1, y(t) = y 0 + tv 2, z(t) = z 0 + tv 3 , que se conocen como ecuaciones param´etricas de una recta en el espa- cio.Luego, la recta es la imagen de la trayectoria: − → c (t) = (x 0, y 0, z 0 ) + t(v 1, v 2, v 3 ), t ∈ R Generalmente, usamos t como variable independiente y − → c (t) se˜ nala la posici´on de una part´ıcula en el espacio y t en este caso es la variable tiempo. Ejemplo 2: Sea − → c : [0, 2π] → R 2 una funci´ on definida por − → c (t) = (cos t, sin t) ,¿cu´ al es la curva asociada a esta trayectoria? Soluci´on A partir de las funciones param´etricas tenemos , x (t) = cos t y (t) = sin t que cumplen con (x (t)) 2 + (y (t)) 2 = (cos t) 2 + (sin t) 2 = 1 ∀t ∈ [0, 2π] Por lo tanto, es una trayectoria cuya imagen corresponde a una cir- cunsferencia centrada en el origen y radio unitario. Observe que − → c (t) = (cos 3t, sin 3t) con t ∈ _ 0, 2π 3 ¸ describe la misma curva. En este caso tenemos que trayectorias parametrizadas en forma diferente describen una misma curva. La variable t se designa usual- mente con el nombre de par´ametro. De ahora en adelante diremos que − → c (t) = (x(t), y(t), z(t)) es una representaci´ on param´etrica de la curva C, donde t es el par´ ametro. Definici´ on 2.9.2. Sea − → c (t):[a, b] →R 3 continua, una trayectoria que describe la curva C.diremos que C en una curva cerrada si y solo si − → c (a) = − → c (b). 114 Una curva cerrada simple, o curva de Jordan es una curva cerrada que tiene la propiedad :si − → c (t 1 ) = − → c (t 2 ) =⇒ (t 1 = t 2 ) ∨ (t 1 = a y t 2 = b).Como vemos esto ocurrre si la funci´on − → c es inyectiva en I es decir ∀t 1 , t 2 ∈ I, t 1 ,= t 2 =⇒ − → c (t 1 ) ,= − → c (t 2 ) Ejercicios 3 A partir de la representaci´on param´etrica dada, describa las curvas y encuentre las ecuaciones cartesianas de las mismas, si ello es posible: 1) − → c (t) = (t, −t) t ≥ 0 2) − → c (t) = (4 −t 2 , t) t ∈ [−2, 3] 3) − → c (t) = (cos t, sin t, 2t) 0 ≤ t ≤ ∞ 4) − → c (t) = (a cos t, b sin t, 1) 0 ≤ t ≤ 2π 2.10. Vectores Unitarios 2.10.1. Vector Tangente unitario Sea − → c (t) : [a, b] →R 3 una trayectoria y C la curva descrita por − → c (t), supongamos que − → c tiene derivada de tercer orden y que − → c´(t) ,= − → 0 ∀ t ∈ [a, b] . Definimos el Vector Tangente unitario en un punto − → c (t) de la trayectoria, como sigue: ´ T(t) = − → c´(t) _ _ − → c´(t) _ _ En el caso que el par´ ametro sea el par´ ametro longitud de arco, entonces: ´ T(s) = − → c´(s) Propiedad Sea − → c (s) : [0, L] →R 3 una trayectoria parametrizada por longitud de arco. Pruebe que los vectores ´ T y ´ TT´ son ortogonales. En efecto: Como: ´ T ´ T = _ _ _ ´ T _ _ _ 2 = 1 =⇒ d ds ( ´ T ´ T) = 0 ⇒ ´ T ´ T´+ ´ T´ ´ T = 0 ⇒ 2 ´ T ´ T´= 0 ∴ ´ T ´ T´= 0 ; es decir que ´ T es ortogonal a ´ T´ 115 2.10.2. Vector Normal En todos los puntos donde ´ T´,= 0 definimos : ´ N = ´ T (s) _ _ _ ´ T (s) _ _ _ ,Vector normal principal a la curva C en el punto − → c (s). 2.10.3. Vector Binormal Hay un tercer vector unitario que es perpendicular tanto a ´ T como a ´ N . Se define por el producto cruz de estos vectores, y es denominado vector Binormal, denotado por: ´ B = ´ T ´ N El conjunto de vectores _ ´ T, ´ N, ´ B _ forma un sistema de vectores unitarios, ortogonales entre s´ı, orientados positivamente en este orden en cada punto − → c (s) de la curva, es decir se cumple que: ´ T ´ N = 0, ´ N ´ B = 0, ´ B ´ T = 0 ´ T ´ T = 1, ´ N ´ N = 1, ´ B ´ B = 1 ´ T ´ N = ´ B, ´ N ´ B = ´ T, ´ B ´ T = ´ N En la medida que var´ıa el conjunto de vectores _ ´ T, ´ N, ´ B _ ,este se de- splaza a lo largo de la curva y se llama triedro movil. Ejemplo 4 Considere la h´elice circular definida por: − → c (t) = (3 cos t, 3 sin t, 4t), t > 0 . Hallar los vectores ´ T, ´ N, y ´ B. 116 Soluci´on. Sea − → c ´ (t) = (−3 sin t, 3 cos t, 4) =⇒ _ _ − → c ´ (t) _ _ = 5 ⇒ s = t _ 0 |c(u)| du = 5t ⇒t = s 5 ∴ − → c (s) = (3 cos s 5 , 3 sin s 5 , 4 s 5 ) Por lo tanto: ´ T = − → c (s) = (− 3 5 sin s 5 , 3 5 cos s 5 , 4 5 ) Del resultado anterior podemos inferir que _ _ _ ´ T _ _ _ = 1 Ahora determinemos el vector normal a partir de su definici´on: ´ N = ´ T´(s) _ _ _ ´ T´(s) _ _ _ ´ T´(s) = (− 3 25 cos s 5 , − 3 25 sin s 5 , 0) _ _ _ ´ T´(s) _ _ _ = _ _ − 3 25 cos s 5 _ 2 + _ − 3 25 sin s 5 _ 2 = _ _ 3 25 _ 2 = 3 25 117 ´ N = (− 3 25 cos s 5 , − 3 25 sin s 5 , 0) 3 25 = (−cos s 5 , −sin s 5 , 0) Del resultado anterior podemos inferir que: _ _ _ ´ N _ _ _ = 1 A partir de los vectores ´ T y ´ N se tiene ´ B = ´ T ´ N ⇒ ´ B = ¸ ¸ ¸ ¸ ¸ ¸ ´ i ´ j ´ k − 3 5 sin s 5 3 5 cos s 5 4 5 −cos s 5 −sin s 5 0 ¸ ¸ ¸ ¸ ¸ ¸ = 4 5 sin s 5 i − 4 5 cos s 5 j + 3 5 j y podemos com- probar que: _ _ _ ´ B _ _ _ = 1 De los calculos anteriores podemos resumir que tenemos: En cada punto − → c (s) de la curva ´ T = (− 3 5 sin s 5 , 3 5 cos s 5 , 4 5 ), ´ N = (−cos s 5 , −sin s 5 , 0), ´ B = ( 4 5 sin s 5 , − 4 5 cos s 5 + 3 5 , 0) como t = s 5 podemos reparametrizar la trayectoria en funcion de t, como sigue ´ T(t) = (− 3 5 sin t, 3 5 cos t, 4 5 ), ´ N(t) = (−cos t, −sin t, 0), ´ B(t) = ( 4 5 sin t, − 4 5 cos t + 3 5 , 0) Como ejercicio dejamos que verifique que: ´ T ´ N = ´ B ´ N ´ B = ´ T ´ B ´ T = ´ N 2.11. Curvatura Sea − → f : I →R n funci´ on vectorial dos veces diferenciable, parametriza- da por el par´ametro longitud de arco. Al n´ umero k(s) = _ _ _ − → f ”(s) _ _ _ se le llama curvatura de − → f en el punto − → f (s). Intuitivamente, de la definici´on se infiere que la curvatura es una medida de cuanto se ”dobla¨ una curva, como una medida del alejamiento de la curva de la recta tangente. Ejemplo 5 Calcular la curvatura de la h´elice − → c (t) = (3 cos t, 3 sin t, 4t), t ≥ 0 Soluci´on. 118 Sabemos que al parametrizar en funci´on del arco se tiene − → c (s) = (3 cos s 5 , 3 sin s 5 , 4 s 5 ) k(s) = | − → c ”(s)| = _ _ _ _ (− 3 25 cos s 5 , − 3 25 sin s 5 , 0) _ _ _ _ = 3 25 En este caso, la curvatura es constante y como k(s) = | − → c ”(s)| = _ _ _ ´ T´(s) _ _ _ ,significa que el vector tangente unitario ´ T(s) tiene la misma rapidez de variaci´ on de su direcci´on, en todos los puntos. Ejemplo 6 Mostrar que la curvatura de una recta es cero. Soluci´on Sea P 0 = (x o , y o , z o ) punto de la recta y − → v = (a, b, c) su vector direcci´ on, entonces podemos escribir − → c (t) = (x o + ta, y o + bt, z o + ct) − → c´(t) = (a, b, c) =⇒s = t _ 0 √ a 2 + b 2 + c 2 dt = t √ a 2 + b 2 + c 2 ∴ s = t √ a 2 + b 2 + c 2 o t = s √ a 2 + b 2 + c 2 Reparametrizando en funci´on del arco se tiene − → c (s) = (x o + sa √ a 2 + b 2 + c 2 , y o + sb √ a 2 + b 2 + c 2 , z o + sc √ a 2 + b 2 + c 2 ) Derivando la expresion anterior: − → c (s) = 1 √ a 2 + b 2 + c 2 (a, b, c) ⇒ − → c ”(s) = (0, 0, 0) ∴ k(s) = 0 Por lo tanto, la curvatura en cualquier punto es cero. C´ırculo, circunferencia de curvatura 119 Si − → c (s) es un punto de la curva C y k la curvatura. La circunferencia que es tangente la curva C en el punto − → c (s) de radio R = 1 k se llama circunferencia de curvatura y R radio de curvatura. El centro de esta circunferencia se encuentra en la direcci´ on del vector − → c ”(s). 2.11.1. C´alculo de curvatura usando par´ametro t cualquiera en R 3 Teorema 2.11.1. Sea − → f : I ⊆ R → R 3 funci´on vectorial, al menos dos veces diferenciable ,tal que: − → f´(t) ,= 0 ∀t ∈ I. Entonces: k(t) = _ _ _ − → f´(t) − → f ”(t) _ _ _ _ _ _ − → f´(t) _ _ _ 3 Demostraci´ on: Sea s par´ ametro longitud de arco tal que t = ϕ(s) − → f´(s) = ´ T(s) = − → f ‘(t) _ _ _ − → f ‘(t) _ _ _ ⇒ − → f´(s) = 1 _ _ _ − → f ‘(t) _ _ _ − → f ‘(t) Debemos calcular − → f (s). Usando la regla de la cadena se tiene: 120 − → f ”(s) = d dt _ _ 1 _ _ _ − → f ‘(t) _ _ _ − → f ‘(t) _ _ dt ds con dt ds = 1 ds dt = 1 _ _ _ − → f ‘(t) _ _ _ Por otra parte, como _ _ _ − → f ‘(t) _ _ _ 2 = − → f ‘(t) − → f ‘(t) Se tiene: 2 _ _ _ − → f ‘(t) _ _ _ d dt __ _ _ − → f ‘(t) _ _ _ _ = − → f ‘(t) − → f ”(t) + − → f ”(t) − → f ‘(t) ⇒ d dt __ _ _ − → f ‘(t) _ _ _ _ = − → f ‘(t) − → f ”(t) _ _ _ − → f ‘(t) _ _ _ Volviendo a la derivada de − → f´(s) : − → f ”(s) = − → f ”(t) _ _ _ − → f ‘(t) _ _ _ − − → f ‘(t) d dt __ _ _ − → f ‘(t) _ _ _ _ |f‘(t)| 2 1 ds dt − → f ”(s) = − → f ”(t) _ _ _ − → f ‘(t) _ _ _ − − → f ‘(t) − → f ‘(t) − → f ”(t) _ _ _ − → f ‘(t) _ _ _ _ _ _ − → f ‘(t) _ _ _ 2 1 _ _ _ − → f ‘(t) _ _ _ − → f ”(s) = − → f ”(t) _ _ _ − → f ‘(t) _ _ _ 2 − − → f ‘(t) _ − → f ‘(t) − → f ”(t) _ _ _ _ − → f ‘(t) _ _ _ 4 Como _ _ _ − → f ”(t) _ _ _ 2 = − → f ”(t) − → f ”(t) ⇒ _ _ _ −→ f”(s) _ _ _ 2 = _ _ _ −→ f”(t) _ _ _ − → f ‘(t) _ _ _ 2 − − → f ‘(t) _ − → f ‘(t) − → f ”(t) _ _ _ _ − → f ‘(t) _ _ _ 4 _ _ _ 2 _ _ _ − → f ”(s) _ _ _ 2 = _ _ _ − → f ”(t) _ _ _ 2 _ _ _ − → f ‘(t) _ _ _ 4 −2 _ _ _ − → f ‘(t) _ _ _ 2 _ − → f ‘(t) − → f ”(t) _ 2 + _ _ _ − → f ‘(t) _ _ _ 2 _ − → f ‘(t) − → f ”(t) _ 2 |f‘(t)| 8 121 _ _ _ − → f ”(s) _ _ _ 2 = _ _ _ − → f´(t) _ _ _ 2 _ _ _ − → f ”(t) _ _ _ 2 − _ − → f ‘(t) − → f ”(t) _ 2 _ _ _ − → f ‘(t) _ _ _ 6 _ _ _ − → f ”(s) _ _ _ 2 = _ _ _ − → f ‘(t) − → f ”(t) _ _ _ 2 _ _ _ − → f ‘(t) _ _ _ 6 Por lo tanto: k(t) = _ _ _ − → f ‘(t) − → f ”(t) _ _ _ _ _ _ − → f ‘(t) _ _ _ 3 Caso particular es el de curvas en el plano. Ejemplo 7 Sea − → r (t) = (x(t), y(t)) una trayectoria de R 2 , en este caso k(t) = [x´(t)y”(t) −x”(t)y´(t)[ [(x´(t)) 2 + (y´(t)) 2 ] 3/2 Soluci´on Se deja al alumno, como indicaci´on se sugiere poner − → r (t) ≡ − → f (t) = (x(t), y(t), 0) y aplicar la formula del teorema precedente Ejemplo 8 Calcule la curvatura de la espiral de Arqu´ımides ρ = aθ. Soluci´on.- x = ρ cos θ y = ρsinθ =⇒ r(θ) = (aθ cos θ, aθsenθ) se considera que x(θ) = aθ cos θ y y(θ) = aθsenθ, derivando estas funciones se tiene x´(θ) = a cos θ −aθsenθ y y´(θ) = asenθ + aθ cos θ x´ ´(θ) = −2asenθ −aθ cos θ y y´ ´(θ) = 2a cos θ −aθsenθ 122 Figura 2.1: Espiral de Arqu´ımides efectuando los productos y simplificando se tiene x´(θ) y”(θ) −x”(θ)y´(θ) = 2a 2 + a 2 θ 2 = a 2 (2 + θ 2 ) por otro lado tambi´en (x´(t)) 2 + (y´(t)) 2 = a 2 + a 2 θ 2 = (1 + θ 2 )a 2 reemplazando en la formula se tiene k(θ) = [a 2 (2 + θ 2 )[ [(1 + θ 2 )a 2 ] 3/2 y simplificando k(θ) = 2 + θ 2 a(1 + θ 2 ) 3/2 para cualquier θ positivo. Obs´ervese que l´ım θ→∞ 2 + θ 2 a(1 + θ 2 ) 3/2 = 0, esto significa que la curvatura de la espiral de Arqu´ımides muy lejos del origen tiende a ser casi una recta. 2.12. Planos por un punto de la curva Sea − → c : I ⊆ IR → R 3 un camino regular dos veces diferenciable y −→ P 0 = − → c (t 0 ) (donde t puede ser par´ ametro longitud de arco). Si − → T, − → N y − → B son los vectores tangente, normal y binormal de la curva en − → c (t 0 ) = (x 0 , y 0 , z 0 ).Podemos definir los siguientes planos por −→ P 0 = − → c (t 0 ) 123 2.12.1. Plano Osculador Plano determinado por − → T y −→ N en el −→ P 0 cuya ecuaci´ on es: (x −x 0 , y −y 0 , z −z 0 ) − → B = 0 2.12.2. Plano Normal Plano determinado por −→ N y − → B en el punto −→ P 0 cuya ecuaci´ on es: (x −x 0 , y −y 0 , z −z 0 ) − → T = 0 2.12.3. Plano Rectificante Plano determinado por − → B y − → T en el punto −→ P 0 cuya ecuaci´ on es: (x −x 0 , y −y 0 , z −z 0 ) − → N = 0 Con estos mismos vectores en el punto −→ P 0 se definen las rectas: 124 2.12.4. Recta Tangente Cuya ecuaci´ on vectorial es: − → P (t) = −→ P 0 + t − → T Es decir (x, y, z) = (x 0 , y 0 , z 0 ) + t (T 1 , T 2 , T 2 ) , t ∈ IR 2.12.5. Recta Normal Cuya ecuaci´ on vectorial es: − → P (t) = − → P 0 + t − → N Es decir (x, y, z) = (x 0 , y 0 , z 0 ) + t (N 1 , N 2 , N 2 ) , t ∈ IR 2.12.6. Recta Binormal Cuya ecuaci´ on vectorial es: − → P (t) = − → P 0 + t − → B Es decir (x, y, z) = (x 0 , y 0 , z 0 ) + t (B 1 , B 2 , B 2 ) , t ∈ IR Ejemplo 9 Considere el camino regular definido por − → c (t) = (t 3 , t 2 , t). Obtener: a) las ecuaciones de los planos Osculador, Normal, Rectificante y b) las rectas tangente, normal y binormal a esta curva en el punto (1, 1, 1). Soluci´ on Previo a responder este problema observemos que para determinar los planos 125 y rectas pedidas no es obligatorio trabajar con los vectores unitarios por lo cual ´ util observar que: − → c´(t) es vector paralelo a − → T (t) , es decir: − → c´(t) = α − → T − → c´(t) − → c ”(t) es vector paralelo a − → B(t), es decir : − → c´(t) − → c ”(t) = β − → B ( − → c´(t) − → c ”(t)) − → c´(t) es vector paralelo a − → N (t) es decir : ( − → c´(t) − → c ”(t)) − → c´(t) = γ − → N De este modo, en nuestro problema particular. − → c´(t) = (3t 2 , 2t, 1) y − → c´ ´(t) = (6t, 2, 0) Lo que implica al evualar en t = 1 queda: − → c´(1) = (3, 2, 1) y − → c´ ´(1) = (6, 2, 0) por consiguiente − → c´(1) − → c ”(1) = ¸ ¸ ¸ ¸ ¸ ¸ i j k 3 2 1 6 2 0 ¸ ¸ ¸ ¸ ¸ ¸ = −2i + 6j −6k = −2(i −3j + 3k) ( − → c´(1) − → c ”(1) − → c´(1) = ¸ ¸ ¸ ¸ ¸ ¸ i j k −2 6 −6 3 2 1 ¸ ¸ ¸ ¸ ¸ ¸ = 18i − 16j − 22k = 2(9i − 8j −11k) As´ı estamos en condiciones de calcular en el punto (1, 1, 1): Plano Rectificante (x −1, y −1, z −1) (9, −8, −11) = 0 ⇒ 9x −8y −11z + 10 = 0 ecuaci´on del plano Plano Osculador (x −1, y −1, z −1) (1, −3, 3) = 0 ⇒ x −3y + 2z −1 = 0 ecuaci´on del plano Plano Normal (x −1, y −1, z −1) (3, 2, 1) = 0 126 ⇒ 3 x + 2y + z −6 = 0 ecuaci´ on del plano Recta tangente: x −1 3 = y −1 2 = z −1 1 Recta Normal: x −1 9 = y −1 −8 = z −1 −11 Recta binormal: x −1 1 = y −1 −3 = z −1 3 2.13. Torsi´on Si una part´ıcula se mueve siguiendo un camino C, el plano osculador en un punto P de la curva es un buen referente para observar el giro o torsimiento de la curva, que no es otra cosa que la medida del ale- jamiento de la curva del plano osculador en una vecindad del punto P. El comportamiento de la derivada del vector Binormal ´ B(s) respecto del par´ametro longitud de arco da la raz´on de cambio del vector − → B respecto del plano osculador Por otro lado se puede obsevar que: ´ B(s) ´ B(s) = 1 =⇒ ´ B´(s) ´ B(s)+ ´ B(s) ´ B´(s) = 0 =⇒∴ ´ B´(s) ´ B(s) = 0 por lo que afirmamos que ´ B´(s) es perpendicular a ´ B(s) . Adem´ as como ´ B(s) ´ T (s) = 0 =⇒ ´ B(s) ´ T´(s) + ´ B´(s) ´ T (s) = 0 ´ T´(s) = α ´ N (s) por lo que ´ B(s) ´ T´(s) = 0 =⇒∴ d B ds ´ T = 0 por lo que se deduce d B ds es perpendicular a ´ T. Estas dos ´ ultimas conclusiones se˜ nalan que d B ds es un vector perpendicular a ´ B y a ´ T, esto significa entonces que d B ds es paralelo a ´ N ,es decir normal al plano rectificante en el punto P. Este razonamiento nos permite formular la siguiente definici´ on. Sea − → c : I ⊆ IR → R 3 un camino regular tres veces diferenciable parametrizado por longitud de arco y tal que − → f´ ´(s) ,= 0 ∀s ∈ I ( es decir con curvatura no nula). Al n´ umero real τ(s) tal que 127 d ´ B ds = −τ(s) ´ N(s) se llama Torsi´on de la curva en el punto − → c (s). Entonces, se tiene que τ(s) = − d ´ B ds ´ N(s) Observaciones: 1) El signo menos tiene el prop´osito de que si τ(s) > 0,entonces d ´ B ds tiene la direcci´ on de − ´ N(s). As´ı cuando P se mueve sobre la curva en una direcci´ on positiva ´ T, ´ B(s) gira alrededor de ´ T en el mismo sentido que un tirabuz´ on diestro que avanza en la direcci´ on de ´ T, como se muestra en la figura adjunta. 2) La |τ(s)| = _ _ _ d B ds _ _ _es una medida de la rapidez con que la curva se despega del plano osculador 3) La primera consecuencia importante, de lo anterior, es que si ´ B(s) no var´ıa ( ´ B constante) significa que la curva se mantiene en el plano osculador y d ´ B ds = 0. Afirmamos, entonces que: − → c (s) describe una curva plana si y solo si su torsi´on es cero 2.13.1. C´alculo de la torsi´on usando par´ametro t cualquiera (en R 3 ) Teorema 2.13.1. Sea − → f : I ⊆ R → R 3 funci´on vectorial al menos tres veces diferenciable tal que: − → f ´( t) ,= 0 y − → f ( t) ,= 0 ∀t ∈ I. Entonces: 128 τ(t) = _ − → f (t) − → f (t) _ − → f (t) _ _ _ − → f (t) − → f (t) _ _ _ 2 La demostraci´on de esta f´ ormula la dejamos como interesante aunque largo ejercicio para el lector. Ejemplo 10 La curva C resulta de la intersecci´on de las superficies z = 2x 2 y y z = x + y. Verifique usando la f´ormula que τ(1) = 0. Soluci´ on.- Sea z = 2x 2 y y z = x + y =⇒ x + y = 2x 2 y de aqu´ı despejando y se tiene y = x 2x 2 −1 Haciendo x = t se tiene una parametrizaci´ on para la curva intersecci´ on de las superficies: − → c (t) = _ t, t 2t 2 −1 , 2t 3 2t 2 −1 _ Las derivadas son: − → c’(t) = _ 1, −(2t 2 + 1) (2t 2 −1) 2 , 2t 2 (2t 2 −3) (2t 2 −1) 2 _ − → c ”(t) = _ 0, 4t(2t 2 + 3) (2t 2 −1) 3 , 4t(2t 2 + 3) (2t 2 −1) 3 _ − → c ”’(t) = _ 0, −12(4t 4 + 12t 2 + 1) (2t 2 −1) 4 , −12(4t 4 + 12t 2 + 1) (2t 2 −1) 4 _ En P = (1, 1, 2), t = 1 y evaluando las derivadas en t = 1 se tiene: − → c ’(1) =(1, −3, −2) , − → c ”(1) =(0, 20, 20) , − → c ”’(1) =(0, −204, −204) _ − → c ’(t) − → c ”(t) ¸ − → c ”’(t) = ¸ ¸ ¸ ¸ ¸ ¸ 1 −3 −2 0 20 20 0 −204 −204 ¸ ¸ ¸ ¸ ¸ ¸ = 0. =⇒ τ(1) = 0. Comentario.- El resultado no pod´ıa ser otro ya que la curva C est´ a en un plano, el plano z = x + y. 129 2.14. Formulas de Frenet Fueron obtenidas por el matem´atico Frances Jean Fr´ed´eric Frenet en 1847 en su tesis doctoral, hoy se les conoce como las formulas de Frenet. 1) d ´ T ds = k ´ N 2) d ´ N ds =−k ´ T + τ ´ B 3) d ´ B ds = − τ ´ N Demostraci´ on.- 1) Igualdad establecida en la fundamentaci´ on de la definici´ on de la torsi´ on 2) De las respectivas definiciones se tiene: ´ N = d T ds _ _ _ d T ds _ _ _ , ´ T(t) = − → c´(s) y k(s) = _ _ − → c ”(s) _ _ ´ N = d T ds _ _ _ d T ds _ _ _ =⇒ d ´ T ds = _ _ _ d T ds _ _ _ ´ N ´ T(t) = − → c´(s) y k(s) = _ _ − → c ”(s) _ _ =⇒ d ´ T ds = _ _ − → c ”(s) _ _ ´ N = k(s) ´ N ∴ d ´ T ds = k ´ N. 2) A partir de ´ N = ´ B ´ T y diferenciando se tiene: d N ds = d B ds ´ T + ´ B d T ds = (−τ ´ N) ´ T + ´ B (k ´ N) = τ ( ´ N ´ T) + k( ´ B ´ N) = τ ´ B −k ´ T. ∴ d ´ N ds = τ ´ B −k ´ T 130 Finalmente, las f´ormulas de Frenet se pueden resumir en una repre- sentaci´ on matricial, donde la matriz de transformaci´ on es antisim´etri- ca. _ _ _ ´ T (s) ´ N (s) ´ B (s) _ _ _ = _ _ 0 k 0 −k 0 τ 0 −τ 0 _ _ _ _ _ ´ T ´ N ´ B _ _ _ 2.15. Aplicaciones de Funciones Vectori- ales y Curvas Cuando una part´ıcula se mueve en el espacio y describe una curva en tres dimensiones, es conveniente fijar a la part´ıcula un sistema de coor- denadas rectangular, de tal manera que una coordenada sea tangente a la trayectoria, otra sea normal a ella y una tercera que sea ortogonal a las dos primeras. Es decir , las coordenadas tangencial, normal y bi- normal constituyen un sistema de referencia cuya base esta constituida por los vectores _ ´ T, ´ N, ´ B _ . Las ecuaciones de la velocidad y la aceleracion pueden obtenerse como sigue: − → v = − → r = d −→ r ds ds dt = s ´ T − → a = − → v = − → r = s ´ T + s ´ T = s ´ T + (s ) 2 ρ ´ N donde ρ = 1 k Puede concluirse que: 1. la velocidad es tangente a la trayectoria y su magnitud es s/ 131 2. la aceleraci´ on tiene dos componentes. a) Una es la componente tangencial , de magnitud (s ),debida al cambio de magnitud de la velocidad. b) La otra es la componente normal, de magnitud (s ) 2 /ρ, hacia el centro de curvatura. Esta componente se debe al cambio de direcci´ on de la velocidad. Tanto la velocidad como la aceleraci´on siempre est´an sobre el plano osculador. 2.15.1. Problemas Problema 1 Una part´ıcula se mueve a lo largo de la curva: y = 2x−x 2 con una componente horizontal de la velocidad de 4m/s, es decir v x = 4m/s. Hallar las componentes tangencial y normal de la aceleraci´ on en el punto (1, 1), si en el instante t = 0, x(0) = 0 Soluci´ on: La funci´on de posici´on de la part´ıcula es: − → r (t) = (x(t), 2x(t) −x 2 (t)), t ∈ I y derivando obtenemos su velocidad − → v (t) = (x/(t), 2x/(t) −2x(t)x/(t)). Como la componente horizontal de la velocidad es 4m/s entonces x/(t) = 4 , integrando x(t) = 4t+C =⇒ x(0) = 0+C = 0 =⇒ C = 0 Luego − → r (t) = (4t, 8t −16t 2 ) =⇒ − → v (t) = − → r (t) = (4, 8 −32t) =⇒ − → a = − → r (t) = (0, −32) Como − → r (1/4) = (4, 0), entonces − → T (1/4) = − → r (1/4) | − → r (1/4)| = (1, 0) y − → N(1/4) = (0, −1) . Luego, las componentes tangencial y normal de la aceleraci´on se obtienen proyectando el vector aceleraci´on en esas direcciones. a T = − → a (1/4) − → T (1/4) = (0, −32) (1, 0) = 0 y a N = − → a (1/4) − → N(1/4) = (0, −32) (0, −1) = 32 132 Problema 2 a) Una part´ıcula se mueve en el espacio de tal forma que en el in- stante t = t 0 ,el vector velocidad es (1, 1, 1) ,y el vector aceleraci´ on es (−2, 1, 0) .Hallar las componentes tangencial y normal de la aceleraci´on en el instante t = t 0 . b) Si la part´ıcula describe una curva en el espacio de tal manera que los vectores velocidad y aceleraci´on tienen siempre magnitud constante, pruebe que la curvatura es constante en cada punto de la curva. Soluci´on: a) Puesto que , se conoce la velocidad tenemos: − → v (t 0 ) = | − → v (t 0 )| ´ T = (1, 1, 1) =⇒ | − → v (t 0 )| = √ 3 − → a (t 0 ) = a T (t 0 ) ´ T + a N (t 0 ) ´ N =⇒ − → v (t 0 ) − → a (t 0 ) = a T (t 0 ) | − → v (t 0 )| ´ T ´ T + a N (t 0 ) | − → v (t 0 )| ´ T ´ N a T (t 0 ) = − → v (t 0 ) − → a (t 0 ) | − → v (t 0 )| = (1, 1, 1) (−2, 1, 0) √ 3 Por otra parte: − → v (t 0 ) − → a (t 0 ) = a T (t 0 ) | − → v (t 0 )| ´ T ´ T + a N (t 0 ) | − → v (t 0 )| ´ T ´ N a N (t 0 ) = | − → v (t 0 ) − → a (t 0 )| | − → v (t 0 )| = |(1, 1, 1) (−2, 1, 0)| √ 3 = 3 √ 2 2 b) La curvatura esta dada por: k (t) = | − → v (t) − → a (t)| | − → v (t)| 3 Supongamos que | − → v (t)| = c 1 y| − → a (t)| = c 2 Dado que | − → v (t)| = c 1 ,al derivar est´ a expresi´ on , se tiene − → v (t) − → a (t) = 0, entonces − → v (t) es ortogonal con − → a (t). As´ı k (t) = | − → v (t) − → a (t)| | − → v (t)| 3 = | − → v (t)| | − → a (t)| | − → v (t)| 3 = | − → a (t)| | − → v (t)| 2 = c 2 c 2 1 Problema 3 133 La aceleraci´ on de una part´ıcula en funci´ on del tiempo viene dada por − → a (t) = (2t, 3t 2 , 4t 3 ) . Si en el instante t = 0 est´ a en el origen de coor- denadas con velocidad inicial − → v (0) = (1, 0, −1). a) Hallar la velocidad y la posici´ on en cualquier instante de tiempo. b) Hallar el valor de t en el que la part´ıcula pasa por el plano xy. Soluci´on: a) Sea − → r (t) la posici´on de la part´ıcula en el instante t. Entonces − → r (t) = − → v (t) y − → r (t) = − → a (t). Por lo tanto: t _ 0 d − → v = t _ 0 − → a (u)du = t _ 0 _ 2u, 3u 2 , 4u 3 _ du = (u 2 , u 3 , u 4 ) ¸ ¸ t 0 = (t 2 , t 3 , t 4 ) Entonces: − → v (t) − − → v (0) = (t 2 , t 3 , t 4 ) =⇒ − → v (t) = (t 2 + 1, t 3 , t 4 −1) Ahora: t _ 0 d − → r (t) = t _ 0 − → v (u)du = t _ 0 (u 2 + 1, u 3 , u 4 −1) du − → r t) − − → r (0) = ( t 3 3 + t, t 4 4 , t 5 5 −t) =⇒ − → r t) = ( t 3 3 + t, t 4 4 , t 5 5 −t) b) La part´ıcula pasa por el plano xy cuando z = 0 =⇒ t 5 5 − t = 0 =⇒ t = 0 y t = 5. Problema 4 Usando la ecuaci´ on 1 ρ = ¸ ¸ ¸ ¸ d 2 y/dx 2 (1 + (dy/dx) 2 ) 3/2 ¸ ¸ ¸ ¸ . 134 a) Hallar el radio de la curvatura de la curva representada por y = x 2 + 2x en el punto x = 2. b) Hallar los radios de la curvatura de la curva representada por x 2 16 + y 2 9 = 1 en los puntos x = 0 y x = 4 respectivamente. Soluci´on: a) La curva representada por y = x 2 +2x corresponde a una par´abola que se abre hacia arriba cuyo v´ertice est´a en el punto (−1, −1). Usando la ecuaci´ on calculemos el radio de curvatura: y = x 2 + 2x =⇒ dy dx = 2x + 2 =⇒ d 2 y dx 2 = 2 1 ρ = ¸ ¸ ¸ ¸ 2 (1 + 4(x + 1) 2 ) 3/2 ¸ ¸ ¸ ¸ .As´ı, el radio de curvatura en x = 0 es: 1 ρ = ¸ ¸ ¸ ¸ 2 (37) 3/2 ¸ ¸ ¸ ¸ b) La curva representada por x 2 16 + y 2 9 = 1 corresponde a una elipse con semiejes mayor a = 4 y menor b = 3 respectivamente. Derivando la ecuaci´ on y = 3 _ 1 − x 2 16 se tiene dy dx = − 3 16 x 1− x 2 16 =⇒ d 2 y dx 2 = − 3 16 1   1− x 2 16   3/2 135 Por lo tanto , el radio de curvatura queda 1 ρ = 3 16 ¸ ¸ ¸ ¸ ¸ ¸ ¸ 1 (1 − 7x 2 256 ) 3/2 ¸ ¸ ¸ ¸ ¸ ¸ ¸ As´ı, el radio de curvatura en x = 0 es: 1 ρ = 3 16 =⇒ ρ = 16 3 Problema 5 El vector posici´on de una part´ıcula m´ ovil a lo largo de una curva, est´ a dado por: − → r = at ´ i + b cos t ´ j + b sin ´ tk en donde a y b son constante usando la ecuaci´ on: 1 ρ = ¸ ¸ ¸ ¸ d 2− → r ds 2 ¸ ¸ ¸ ¸ donde ρ es el radio de curvatura. Hallar la curvatura de la curva. Soluci´on: Parametricemos , el vector posici´on en funci´ on de la longitud de arco s. 136 s = t _ 0 | − → r (τ)| dτ = t _ 0 |(a, −b sin t, b cos t)| dτ = √ a 2 + b 2 τ ¸ ¸ ¸ t 0 =⇒ t = s √ a 2 + b 2 − → r (s) = _ a s √ a 2 + b 2 , b cos s √ a 2 + b 2 , b sin s √ a 2 + b 2 _ − → r (s) = _ a √ a 2 + b 2 , − b √ a 2 + b 2 sin s √ a 2 + b 2 , b √ a 2 + b 2 cos s √ a 2 + b 2 _ − → r (s) = _ 0, − b a 2 + b 2 sin s √ a 2 + b 2 , b a 2 + b 2 cos s √ a 2 + b 2 _ Por consiguiente, el radio de curvatura es: 1 ρ = ¸ ¸ ¸ ¸ d 2− → r ds 2 ¸ ¸ ¸ ¸ = b a 2 + b 2 =⇒ ρ = a 2 + b 2 b = constante Problema 6 En un cierto instante, las magnitudes de la velocidad y la aceleraci´on de un proyectil son 20 m/s y 80 m/s respectivamente, y el ´ angulo de inclinaci´ on entre ellas es de 30 o .Determinar el radio de curvatura de la trayectoria en ese instante Soluci´on: Sean ´ T y ´ N los vectores unitarios en las direcciones tangencial y normal respectivamente. Como la velocidad es tangente a la trayectoria tenemos que − → v = | − → v | ´ T = 20 (m/s) ´ T Las componentes tangencial y normal de la aceleraci´ on son: 137 a T = a cos θ = 80 cos 30 o = 40 √ 3 _ m/s 2 _ a N = asenθ = 80 sen30 o = 40 _ m/s 2 _ − → a = 40 √ 3 _ m/s 2 _ ´ T + 40 _ m/s 2 _ ´ N El radio de curvatura puede calcularse a partir de: a N = | − → v | 2 ρ =⇒ ρ = | − → v | 2 a N = 400 40 = 10m Problema 7 La aceleraci´ on de una part´ıcula, est´a dada por: − → a = 2t ´ i +3t 2 ´ j +2 ´ k en donde − → a est´ a m/s 2 y t est´ a en s. Si en el instante t = 0, − → v = 0. A partir de la componente normal de la aceleraci´ on hallar el radio de curvatura de la trayectoria de la part´ıcula en el instante t = 1 s. Soluci´ on: En primera instancia determinemos la velocidad de la part´ıcula t _ 0 d − → v = t _ 0 − → a (u)du = t _ 0 _ 2τ ´ i + 3τ 2 ´ j + 2 ´ k _ dτ = t 2 ´ i + t 3 ´ j + 2t ´ k − → v (t) − − → v (0) = t 2 ´ i + t 3 ´ j + 2t ´ k =⇒ − → v (t) = t 2 ´ i + t 3 ´ j + 2t ´ k Ahora evaluemos la velocidad y la aceleraci´ on respectivamente, en el instante t = 1s : − → v (1) = ´ i + ´ j + 2 ´ k − → a (1) = 2 ´ i + 3 ´ j + 2 ´ k As´ı la componente normal de la aceleraci´ on en el instante t = 1 es a N (1) = | − → v (1)| 2 ρ = | − → v (1) − → a (1)| | − → v (1)| =⇒ ρ = | − → v (1)| 3 | − → v (1) − → a (1)| = _√ 6 _ 3 |(−4, 2, 1)| Por lo tanto el radio de curvatura es:ρ = _√ 6 _ 3 |(−4, 2, 1)| = _√ 6 _ 3 √ 21 m Problema 8 138 El vector posici´ on de una part´ıcula se describe por la funci´on vectorial − → r (t) = a cos(wt) ´ i + a sin (wt) ´ j + bt 2 ´ k ,donde a y b son constantes positivas. Expresar la velocidad y la aceleraci´on en sus componentes tangencial y normal. Soluci´on: Calculemos la velocidad y la aceleraci´ on en coordenadas rectangulares. − → v = − → r (t) = −awsen(wt) ´ i + awcos (wt) ´ j + 2bt ´ k − → a = − → r (t) = −aw 2 cos(wt) ´ i −aw 2 sin (wt) ´ j + 2b ´ k A continuaci´ on , podemos calcular la rapidez y la magnitud de la acel- eraci´ on | − → v | = √ − → v − → v = √ a 2 w 2 + 4b 2 t 2 | − → a | = √ − → a − → a = √ a 2 w 4 + 4b 2 Ahora, estamos en condiciones de expresar la velocidad y la aceleraci´ on en sus componentes tangencial y normal. − → v = | − → v | ´ T = √ a 2 w 2 + 4b 2 t 2 ´ T a T (t) = − → v (t) − → a (t) | − → v (t 0 )| = 4b 2 t √ a 2 w 2 + 4b 2 t 2 a N (t) = | − → v (t) − → a (t)| | − → v (t 0 )| = √ a 4 w 6 + 4a 2 b 2 w 4 t 2 + 4a 2 b 2 w 2 √ a 2 w 2 + 4b 2 t 2 Por tanto, la aceleraci´on es: − → a = 4b 2 t √ a 2 w 2 + 4b 2 t 2 ´ T + √ a 4 w 6 + 4a 2 b 2 w 4 t 2 + 4a 2 b 2 w 2 √ a 2 w 2 + 4b 2 t 2 ´ N Problema 9 La posici´ on de una part´ıcula sobre la periferia de una rueda de 8 cm de di´ametro se especifica como s(t) = t 3 − 4t 2 + 8t en donde s es la longitud del arco, medida en cm, a lo largo de la periferia circular, a partir de un origen conveniente, y t es el tiempo, en segundos. Hallar la magnitud de la aceleraci´ on | − → a |en el instante en que la mag- nitud de la aceleraci´on tangencial es de 4cm/s 2 . Soluci´on: A partir de la ecuaci´on dada s(t) = t 3 −4t 2 + 8t, obtenemos s (t) = | − → v | = 3t 2 −8t + 8 139 s (t) = a N (t) = 6t −8 El radio de curvatura ρ en este caso es el radio de la rueda. As´ı que: ρ = 8 2 = 4cm. La aceleraci´on es − → a (t) = s (t) ´ T + (s (t)) 2 ρ ´ N = (6t −8) ´ T + (3t 2 −8t + 8) 2 4 ´ N En el instante que a T = 4, 6t − 8 = 4, de donde t = 2s. Luego, en el instante t = 2s. − → a (2) = 4 ´ T + 4 ´ N Por lo tanto, la magnitud de la aceleraci´ on es: | − → a | = √ − → a − → a = 4 √ 2cm/s 2 2.16. Ejercicios resueltos Ejercicio 1 Un par de trayectorias de [0, ∞) en R 3 se definen por − → c (t) = (cos t, sin t, bt) y − → r (t) = (1, 0, t). Responda las siguientes preguntas: a) ¿Se intersectan las curvas generadas por − → c (t) y − → r (t)? b) Si estas trayectorias representan el desplazamiento de un par de part´ıculas. ¿En qu´e puntos ,si los hay, estas part´ıculas se encuentran? Soluci´ on: a) − → c (t) es la ecuaci´ on de la h´elice ascendente sobre el manto del cilindro x 2 + y 2 = 1 y cada vuelta demora 2π unidades de tiempo. Asimismo, − → r (t) = (1, 0, t) es una recta vertical paralela al eje axial del cilindro , que est´ a sobre el manto de x 2 + y 2 = 1 y pasa por (1, 0, 0). Igualando las primeras componentes cost = 1 ,obtenemos que las curvas se intersectan para t = 0, 2π, 4π, . . . 140 b) Igualando las terceras componentes bt = t =⇒Si b = 1,entonces las part´ıculas se encuentran en los puntos (1, 0, 0), (1, 0, 2π), ..., (1, 0, 2nπ) con n ∈ Z + 0 . Ejercicio 2 La curva C es definida a partir de la trayectoria − → c (t) = (2 cos(t), 2 sin(t), t) con 0 ≤ t ≤ 2π . Describa la representaci´ on gr´afica de C y pruebe que si se usa como par´ ametro la longitud de arco s , el vector tangente a la curva es un vector unitario. Soluci´ on: Por la continuidad de las funciones x(t) = 2cos(t), y(t) = 2sin(t) y z(t) = t podemos inferir que C parte del punto − → c (0) = (x(0), y(0), z(0)) = (2, 0, 0) y termina en − → c (2π) = (x(2π), y(2π), z(2π)) = (2, 0, 2π); adem´as que la curva asciende a trav´es del manto del cilindro x 2 + y 2 = 4 porque [x(t)] 2 +[y(t)] 2 = [2 cos(t)] 2 +[2 sin(t)] 2 = 4 como se ilustra en la figura El vector posici´on de esta curva es − → c (t) = (2cos(t), 2sin(t), t). El vector tangente es − → c (t) = (−2 sin(t), 2 cos(t), 1) ¤(a) y la longitud del vector tangente es | − → c (t)| = _ [−2 sin(t)] 2 + [2 cos(t)] 2 + 1 = √ 5 (b) 141 La longitud total de esta curva es Longitud = _ 2π 0 | − → c (t)| dt = _ 2π 0 √ 5dt = 2π √ 5 Definimos s(t) = _ t 0 |c (u)| du para t ∈ [0, 2π] =⇒ s(t) es la longitud de curva C desde (2, 0, 0) hasta (x(t), y(t), z(t)). Claramente s(t) es continua y estrictamente creciente en [0, 2π] la ecuaci´ on s = s(t) puede resolverse para t como una funci´ on de s, es decir t = t(s) (c) En este caso t = s √ 5 as´ı es que − → c (s) = − → c (t(s)) = _ 2 cos _ s √ 5 _ , 2 sin _ s √ 5 _ , s √ 5 _ es vector posici´ on en t´erminos de s, derivando − → c (s) = − → c (t(s)) = _ −2 sin _ s √ 5 _ 1 √ 5 , 2 cos _ s √ 5 _ 1 √ 5 , 1 √ 5 _ = 2 √ 5 _ sin _ s √ 5 _ , cos _ s √ 5 _ , 1 2 _ (2.16.1) Calculando su m´ odulo | − → c (s)| = 2 √ 5 ¸ _ sin _ s √ 5 __ 2 + _ cos _ s √ 5 __ 2 + 1 4 = 2 √ 5 _ 1 + 1 4 = 1 Por lo tanto, − → c (s) es vector unitario. Especificaciones: a) Si − → c (t) describe la trayectoria de una part´ıcula en el espacio, el vector − → c (t) = (−2 sin(t), 2 cos(t), 1) es la velocidad con que se desplaza la part´ıcula por la curva C en el punto − → c (t), en el instante “t”. 142 b)| − → c (t)| = √ 5 es la rapidez con que se desplaza la part´ıcula ∀t, lo que significa que la part´ıcula se mueve con rapidez constante ∀t. c) Asimismo, la longitud del arco es s(t) = _ t 0 | − → c (t)| du = _ t 0 √ 5du = √ 5t s = √ 5t =⇒t = s √ 5 En general y en teor´ıa la ecuaci´on s = s(t) siempre se puede resolver para t en t´erminos de s, es decir tener t = t(s). En la pr´ actica existen casos en los que por razones algebraicas no se puede tener t = t(s) ¿Conoces alg´ un caso? Ejercicio 3 Una part´ıcula se mueve en el espacio con vector posici´ on − → r (t) = t − → A + t 2 − → B + 2 _ 2 3 t _3 2 − → A − → B donde − → A y − → B son dos vectores unitarios fijos que forman ´ angulo de π 3 radianes. Calcular la velocidad de la part´ıcula en el instante t y determinar en cuanto tiempo recorre una distancia de 12 unidades de longitud de arco desde la posici´on en t = 0. Soluci´on: 143 La velocidad es el vector − → v (t) = − → r (t) donde − → r (t) = − → A + 2t − → B + 3 _ 2 3 t _1 2 2 3 − → A − → B Por lo tanto − → v (t) = − → A + 2t − → B + 2 _ 2 3 t _1 2 − → A − → B Para la segunda parte del problema usaremos s(t) = _ t 0 | − → r (t)| du con | − → r (t)| = _ − → r (t) − → r (t) Calculando − → r (t) − → r (t) = = _ − → A + 2t − → B + 2 _ 2 3 t _1 2 − → A − → B _ _ − → A + 2t − → B + 2 _ 2 3 t _1 2 − → A − → B _ = 1 + 4t + 4t 2 (a) por lo tanto − → r (t) − → r (t) = (1 + 2t) 2 =⇒| − → r (t)| = _ (1 + 2t) 2 = 1 + 2t as´ı que s(t) = _ t 0 (1 + 2u) du = _ u + u 2 ¸ t 0 = t + t 2 De acuerdo a lo que se pide en el problema s(t) = 12 =⇒t 2 +t−12 = 0 Las soluciones de esta ecuaci´on son t 1 = 3 y t 2 = −4 por la naturaleza del problema debe ser t ≥ 0 luego desconsideramos t 2 = −4. En conse- cuencia, 3 unidades de tiempo son necesarios 144 Especificaciones: a) _ ¸ ¸ ¸ ¸ ¸ ¸ _ ¸ ¸ ¸ ¸ ¸ ¸ _ _ − → A + 2t − → B + 2 _ 2 3 t _1 2 − → A − → B _ _ − → A + 2t − → B + 2 _ 2 3 t _1 2 − → A − → B _ = − → A − → A + 2t − → B − → A + 2 _ 2 3 t _1 2 − → A − → B − → A + 2t − → A − → B + 4t 2 − → B − → B +2(2t) _ 2 3 t _1 2 − → A − → B − → B + 2 _ 2 3 t _1 2 − → A _ − → A − → B _ +2(2t) _ 2 3 t _1 2 − → B _ − → A − → B _ + 4 _ 2 3 t _ _ − → A − → B _ _ − → A − → B _ Como: − → A − → A = 1, − → B − → B = 1, − → A − → B = cos π 3 = 1 2 − → A − → A − → B = 0, − → B − → A − → B = 0, y _ − → A − → B _ _ − → A − → B _ = _ _ _ − → A − → B _ _ _ 2 = _ √ 3 2 _ 2 = 3 4 Se concluye que: − → r (t) − → r (t) = 1 + t + 0 + t + 4t 2 + 0 + 0 + 0 + 8 3 + 3 4 = 1 + 4t + 4t 2 = (1 + 2t) 2 Ejercicio 4 Sea − → F : I →R n una funci´on velocidad dos veces diferenciable. a) Pruebe que la aceleraci´ on se puede escribir como combinaci´ on lineal de los vectores unitarios ´ T y ´ N, espec´ıficamente se pide establecer que: − → a (t) = _ d 2 s dt 2 _ ´ T + k(t) _ ds dt _ 2 ´ N = a T ´ T + a N ´ N b) Aplicando a) calcule las componentes tangencial y normal del vector aceleraci´ on correspondiente a − → r (t) = (t, cos 2 t, sin 2 t). Soluci´on: a) Sea − → g una reparametrizaci´ on de − → F por longitud de arco entonces − → F (t) = − → g (s) si y s´olo si t = t(s) ⇐⇒s = s(t). Aplicando la regla de la cadena, queda − → F (t) = − → g (s(t)) =⇒ − → F (t) = − → g (s) s (t) = ´ T ds dt 145 derivando nuevamente respecto de t − → F (t) = − → g (s) [s (t)] 2 + − → g (s) s (t) Por definici´ on k = k(s) = | − → g (s)| es la, curvatura de C en g(s). por otro lado ´ N = d T ds _ _ _ d T ds _ _ _ =⇒ − → g (s) = | − → g (s)| ´ N Relacionando las igualdades anteriores − → a (t) = − → F (t) = _ | − → g (s)| ´ N _ _ ds dt _ 2 + ´ T d 2 s dt 2 = d 2 s dt 2 ´ T +| − → g (s)| _ ds dt _ 2 ´ N = d 2 s dt 2 ´ T + k _ ds dt _ 2 ´ N lo que significa que − → a (t) = d 2 s dt 2 ´ T + k _ ds dt _ 2 ´ N b) En la aplicaci´ on de a) en − → r (t) = _ t, cos 2 t, sin 2 t _ tenemos − → r (t) = (1, −sin(2t), sin(2t)) . Usando la expresion anterior, se tiene s (t) = | − → r (t)| = _ 1 + 2 sin 2 (2t) =⇒s (t) = 4 sin(2t) cos(2t) _ 1 + 2 sin 2 (2t) Luego, la componente tangencial a T es a T = s = 2 sin(2t) _ 1 + 2 sin 2 (2t) 146 Adem´ as, − → r (t) = (0, −2 cos(2t), 2 cos(2t)) y haciendo el producto cruz entre − → r (t) y − → r (t) se tiene − → r (t) − → r (t) = −2 cos(2t) ´ j −2 cos(2t) ´ k (e) y | − → r (t) − → r (t)| = _ 4 cos 2 (2t) + 4 cos 2 (2t) = _ 8 cos 2 (2t) = 2 √ 2 [cos(2t)[ Como la curvatura k en funci´on de t es k = | − → r (t) − → r (t)| | − → r (t)| 3 reemplazando k = 2 √ 2 [cos(2t)[ _ 1 + 2 sin 2 (2t) _3 2 de a N = k(t) _ ds dt _ 2 = 2 √ 2 [cos(2t)[ _ 1 + 2 sin 2 (2t) _3 2 _ 1 + 2 sin 2 (2t) _ Por lo tanto, la componente normal de la aceleraci´ on en este caso es a N = 2 √ 2 [cos(2t)[ _ 1 + 2 sin 2 (2t) Ejercicio 5 Sea la trayectoria regular − → r : R →R 3 , definida por: − → r (u) = ( 4au 2 (1 + u 2 ) 2 , 2au(1 −u 2 ) (1 + u 2 ) 2 , a(1 −u 2 ) 1 + u 2 ), a > 0 a) Pruebe que la funci´on ϕ : R → ]−π, π[ tal que t = ϕ(u) = 2 arctan u 147 define la reparametrizaci´on de − → r , − → r ∗ : ]−π, π[ →R tal que ¯ r(t) = (a sin 2 t, a sin(t) cos(t), a cos t) b) Verifique que − → r ∗ (t) est´a contenida en una superficie esf´erica. c) Probar que los planos normales a la curva descrita por ¯ r(t) pasan por el centro de la esfera. Soluci´ on De ϕ(u) = 2 arctan u se tiene que: ϕ (u) = 2 1+u 2 > 0 ∀u ∈ R, por lo tanto, es estrictamente creciente en R y como ϕ es continua, ϕ es biyectiva, y por tanto, invertible. Adem´ as ϕ −1 (t) = tan( t 2 ) ϕ −1 : ]−π, π[ →R es de clase C 1 en ]−π, π[ . Por lo anterior, se puede definir ¯ r(t) : − → r ∗ (t) = ( − → r ◦ ϕ −1 )(t) = − → r (ϕ −1 (t)) = − → r (tan( t 2 )) =⇒ − → r ∗ (t) = ( 4a tan 2 ( t 2 ) (1 + tan 2 ( t 2 )) 2 , 2a tan( t 2 )(1 −tan 2 ( t 2 ) (1 + tan 2 ( t 2 )) 2 , a(1 −tan 2 ( t 2 )) (1 + tan 2 ( t 2 )) Sustituyendo − → r ∗ (t) = ( 4a( 1−cos t 1+cos t ) ( 2 1+cos t ) 2 , 2a _ 1−cos t 1+cos t ( 2 cos t 1+cos t ) ( 2 1+cos t ) 2 , a( 2 cos t 1+cos t ) ( 2 1+cos t ) ) = (a(1 −cos 2 t), a cos t √ 1 −cos 2 t, a cos t) Por lo tanto, la reparametrizaci´ on la podemos definir por: − → r ∗ (t) = (a sin 2 t, a sin(t) cos(t), a cos t) 148 b) Consideremos ahora x(t) = a sin 2 t, y(t) = a sin t cos t, z(t) = a cos t =⇒ x 2 + y 2 + z 2 = a 2 sin 4 t + a 2 sin 2 t cos 2 t + a 2 cos 2 t = a 2 sin 2 t(sin 2 t + cos 2 t) + a 2 cos 2 t = a 2 (sin 2 t + cos 2 t) = a 2 ∴ la curva est´a en la esfera x 2 + y 2 + z 2 = a 2 c) Se − → r ∗ (t 0 ) un punto cualquiera de la curva. La ecuaci´on del plano normal es: π N : ((x, y, z) − ¯ r(t 0 )) ¯ r (t 0 ) = 0 y (0, 0, 0) ∈ π N si s´olo si − → r ∗ (t 0 ) − → r ∗ (t 0 ) = 0 − → r ∗ (t 0 ) − → r ∗ (t 0 ) = (a sin 2 t 0 , a sin(t 0 ) cos(t 0 ), a cos t 0 )(2a sin t 0 cos t 0 , a(cos 2 t 0 −sin 2 t 0 ), −a sin t 0 ) = 2a 2 sin 3 t 0 cos t 0 + a 2 (sin t 0 cos 3 t 0 −sin 3 t 0 cos t 0 ) −a 2 sin t 0 cos t 0 = a 2 sin 3 t 0 cos t 0 + a 2 sin t 0 cos 3 t 0 −a 2 sin t 0 cos t 0 = a 2 sin t 0 cos t 0 −a 2 sin t 0 cos t 0 = 0 ∴ (0, 0, 0) ∈ π N Ejercicio 6 Sea − → r : R + →R 3 , definida por: − → r = (t 2 , 2 3 t 3 , t),la trayectoria regular que describe una part´ıcula que se mueve a lo largo de una curva C. Para el instante t = 1, determine: 149 a) Su velocidad ,rapidez y aceleraci´on. b) Los versores tangente, normal y binormal a trayectoria en ese in- stante c) La curvatura y torsi´ on de la curva en ese punto. d) Las componentes tangencial y normal de la aceleraci´ on en el punto. e) La hod´ ografa del movimiento o trayectoria que se determina en el espacio de velocidad. Soluci´on: a) A partir de la definici´ on tenemos que la velocidad ,rapidez y acel- eraci´ on en funci´on del tiempo est´ an dadas por: − → v (t) = − → r (t) = (2t, 2t 2 , 1) =⇒ − → v (1) = − → r (1) = (2, 2, 1) [ − → v (t)[ = √ − → r − → r = _ (4t 2 + 4t 4 + 1) =⇒ [ − → v (1)[ = 3 − → a (t) = − → r (t) = (2, 4t, 0) =⇒ − → a (1) = (2, 4, 0) b) Los versores se pueden calcular usando las identidades ´ T (t) = −→ r (t) [ −→ r (t) [ =⇒ ´ T (1) = −→ r (1) [ −→ r (1) [ = (2,2,1) 3 ´ B(t) = −→ r (t)× −→ r (t) [ −→ r (t)× −→ r (t) [ =⇒ ´ B(1) = −→ r (1)× −→ r (1) [ −→ r (1)× −→ r (1) [ = (−4,2,4) 6 ´ N (t) = ´ B(t) ´ T (t) =⇒ ´ N (1) = ´ B(1) ´ T (1) = (−1,2.−2) 3 c) La curvatura y torsi´ on en el punto se pueden determinar usando las identidades: κ(t) = −→ r (t)× −→ r (t) [ −→ r (t) [ 3 =⇒ κ(1) = −→ r (1)× −→ r (1) [ −→ r (1) [ 3 = 6 3 3 = 2 9 > 0 τ (t) = −→ r (t)× −→ r (t) · −→ r (t) [ −→ r (t)× −→ r (t) [ 2 =⇒ τ (1) = −→ r (1)× −→ r (1) · −→ r (1) [ −→ r (1)× −→ r (1) [ 2 = 8 36 = 2 9 d) Derivando la velocidad tenemos − → a (t) = − → r (t) = [ − → v (t)[ ´ T (t) + κ(t) [ − → v (t)[ 2 ´ N (t) , entonces las componentes tangencial y normal de la aceleraci´ on son: 150 a T = [ − → v (t)[ = 4t + 8t3) √ (4t 2 +4t 4 +1) a N = κ(t) [ − → v (t)[ 2 =⇒ a T = [ − → v (1)[ = 4 + 8 √ (4+4+1) = 12 3 = 4 a N = κ(1) [ − → v (1)[ 2 = 2 9 (3) 2 = 2 e) A partir del vector velocidad − → v (t) = − → r (t) = (2t, 2t 2 , 1) podemos inferir sus componentes x (t) = 2t y (t) = 2t 2 z (t) = 1 _ _ _ =⇒ y (x) = x 2 2 z = 1 Por lo tanto, la hod´ ografa es una parab´ola en el plano z = 1 del espacio de velocidades. Ejercicio 7 Sea − → r : R + →R 3 , definida por: − → r = (a cos t, asent, β(t)), trayectoria regular que describe una part´ıcula que se mueve a lo largo de una curva C. ¿Cu´al debe ser la funci´on β(t) para que la trayectoria est´e contenida en un plano para todo t?. Soluci´on: La trayectoria de la part´ıcula es plana si y solo s´ı la torsi´ on es nula para todo t. Es decir: τ (t) = −→ r (t)× −→ r (t) · −→ r (t) [ −→ r (t)× −→ r (t) [ 2 = 0 ∀t =⇒ − → r (t) − → r (t) − → r (t) = 0 ∀t, luego derivemos y calculemos el producto mixto − → r = (−asent, a cos t, β (t)) − → r = (−a cos t, −asent, β (t)) − → r = (asent, −a cos t, β (t)) _ _ _ =⇒ − → r (t) − → r (t) − → r (t) = a 2 (β (t) + β (t)) = 0 =⇒ (β (t) + β (t)) = 0 La ecuaci´on diferencial hom´ ogenea de tercer orden , tiene soluci´ on de la forma β(t) = e λt ; entonces β (t) = λe λt =⇒ β (t) = λ 2 e λt =⇒ β (t) = λ 3 e λt 151 reemplazando t´erminos en la ecuaci´on anterior: (λ 3 +λ)e λt = 0, e λt ,= 0 ∀t conduce a la ecuaci´ on caracter´ıstica (λ 3 + λ) = 0, luego β(t) = C 1 + C 2 cos t + C 3 sent, donde C 1 , C 2 , C 3 son constantes reales. Por lo tanto, si la trayectoria est´ a dada por − → r (t) = (a cos t, asent, C 1 + C 2 cos t + C 3 sent) la part´ıcula siempre se mover´a sobre un mismo plano. Ejercicio 8 a) Sea − → r : I ⊆ R →R 3 una trayectoria regular de modo que − → r (t) ,= 0 ∀t. supongamos que hay un t 0 ∈ I para el que la distancia del origen al punto − → r (t 0 ) ∈ R 3 alcanza un valor m´ınimo. Pruebe que en ese punto − → r (t 0 ) es perpendicular a − → r (t 0 ) . b) Usando lo anterior , hallar los puntos en que la recta dada por − → r (t) = (t + 1, 3t −2, 2t −1) est´a m´ as cerca del origen. Soluci´on: a) En efecto consideremos la funci´ on distancia d(t)= [ − → r (t)[de un punto cualquiera al origen y supongamos que hay un t 0 ∈ I, tal que se alcanza un m´ınimo, entonces se tiene que d (t 0 ) = [ − → r (t 0 )[ = − → r (t 0 ) − → r (t 0 ) [ − → r (t 0 )[ = 0, de donde se tiene que − → r (t 0 ) − → r (t 0 ) = 0 , por lo tanto, − → r (t 0 ) y − → r (t 0 ) son ortogonales en ese punto. b) Basta encontrar un t 0 ∈ I para el que la distancia del punto − → r (t 0 ) ∈ R 3 alcanza su valor m´ınimo que satisface − → r (t 0 ) − → r (t 0 ) = 0 − → r (t 0 ) − → r (t 0 ) = (t 0 + 1, 3t 0 −2, 2t 0 −1) (1, 3, 2) = 0 =⇒ (t 0 + 1) + 3(3t 0 −2) + 2(2t 0 −1) = 0 =⇒ t 0 = 1 2 152 Por lo tanto, el punto buscado es − → r _ 1 2 _ = _ 3 2 , − 1 2 , 0 _ . Ejercicio 9 Se llama evoluta de una curva − → r : I ⊆ R + → R 2 a la curva que describen los centros de curvatura de − → r . Determine la evoluta de la curva − → r (t) = (t, t 2 ). Soluci´ on: La curva que describe el centro de curvatura est´a dada por la ecuaci´ on: − → c (t) = − → r (t)+ 1 κ(t) ´ N (t) , determinemos la curvatura y el versor normal. − → r (t) = (1, 2t, 0) =⇒ − → r (t) = (0, 2, 0) =⇒ − → r (t) − → r (t) = (0, 0, 2) =⇒ − → r (t) − → r (t) − → r (t) = (−4t, 2, 0) Adem´ as: ¸ ¸ − → r (t) − → r (t) ¸ ¸ = 2 y ¸ ¸ − → r (t) − → r (t) − → r (t) ¸ ¸ = 2 (1 + 4t 2 ) 1/2 κ(t) = −→ r (t)× −→ r (t) [ −→ r (t) [ 3 = 2 (1 +4t 2 ) 3/2 ´ N (t) = −→ r (t)× −→ r (t)× −→ r (t) [ −→ r (t)× −→ r (t)× −→ r (t) [ = (−4t,2) 2(1 +4t 2 ) 1/2 , reemplazando t´erminos en la ecuaci´ on tenemos − → c (t) = (t, t 2 )+ ( 1 +4t 2 ) 3/2 2 (−4t,2) 2(1 +4t 2 ) 1/2 = _ −4t 3 , 3t 2 + 1 2 _ ,que corresponde a la ecuaci´on param´etrica de la evoluta. 1.10 Ejercicio 10 Sea − → r : I ⊆ R →R 3 una trayectoria regular, dada por − → r = (x(t), y(t), z(t)), t ∈ I. Pruebe que la ecuaci´ on del plano osculador en − → r (t 0 ), t 0 ∈ I,es: ¸ ¸ ¸ ¸ ¸ ¸ x −x (t 0 ) y −y (t 0 ) z −z (t 0 ) x (t 0 ) y (t 0 ) z (t 0 ) x (t 0 ) y (t 0 ) z (t 0 ) ¸ ¸ ¸ ¸ ¸ ¸ = 0 Soluci´ on: La ecuaci´on del plano osculador es : ( − → f − − → r (t 0 )) − → B(t 0 ) = 0, donde − → B(t 0 ) = − → r (t 0 ) − → r (t 0 ) = ¸ ¸ ¸ ¸ ¸ ¸ ´ i ´ j ´ k x (t 0 ) y (t 0 ) z (t 0 ) x (t 0 ) y (t 0 ) z (t 0 ) ¸ ¸ ¸ ¸ ¸ ¸ 153 − → B(t 0 ) = (y (t 0 ) z (t 0 ) − y (t 0 ) z (t 0 ) , x (t 0 ) z (t 0 ) − x (t 0 ) z (t 0 ) , x (t 0 ) y (t 0 ) −x (t 0 ) y (t 0 )) pongamos: x 0 = x (t 0 ) , y 0 = y (t 0 ) y z 0 = z (t 0 ) ( − → f − − → r (t 0 )) − → B(t 0 ) = 0 =⇒ (x −x 0 , y −y 0 , z −z 0 ) (y 0 z 0 −y 0 z 0 , x 0 z 0 −x 0 z 0 , x 0 y 0 −x 0 y 0 ) = 0 desarrollando (y 0 z 0 −y 0 z 0 )(x−x 0 ) + (x 0 z 0 −x 0 z 0 )(y −y 0 ) +(x 0 y 0 −x 0 y 0 )(z −z 0 ) = 0 esto ´ ultimo se puede escribir ¸ ¸ ¸ ¸ ¸ ¸ x −x (t 0 ) y −y (t 0 ) z −z (t 0 ) x (t 0 ) y (t 0 ) z (t 0 ) x (t 0 ) y (t 0 ) z (t 0 ) ¸ ¸ ¸ ¸ ¸ ¸ = 0 lo que prueba la hip´otesis. Ejercicio 11 Un automovilista se desplaza por una carretera recta. En el instante t = 0 llega a una rotonda la que recorre con un trayectoria − → f(t) = (acost, asent, bt(2 −t)), t ∈ [0, 2] En el instante t = 2 sale de la rotonda y vuelve a continuar por una carretera recta. a) C´ alcule la curvatura m´ axima de la rotonda para t ∈ [0, 2]¿en qu´e pun- to ocurre? b) Determine la torsi´on de la rotonda para t ∈ [0, 2] .Haga un gr´afico de la torsi´ on en funci´ on del tiempo. 154 Soluci´on: a) Calculemos la curvatura de la trayectoria mediante la identidad: κ(t) = −→ f (t)× −→ f (t) [ −→ f (t) [ 3 − → f (t) = (−asent, a cos t, 2b(1−t)) =⇒ ¸ ¸ ¸ − → f (t) ¸ ¸ ¸ = _ _ a 2 + 4b 2 (1 −t) 2 _ − → f (t) = (−acost, −asent, −2b) =⇒ − → f (t) − → f (t) = (−2ab cos t + 2ab(1 − t)sent, −2absent − 2ab(1 − t) cos t, a 2 ) ¸ ¸ ¸ − → f (t) − → f (t) ¸ ¸ ¸ = a _ a 2 + 4b 2 (1 + (1 −t) 2 ,reemplazando en la identi- dad, tenemos κ(t) = a _ a 2 + 4b 2 (1 + (1 −t) 2 (a 2 + 4b 2 (1 −t) 2 ) 3/2 Sea Ψ(t) = a 2 + 4b 2 (1 − t) 2 =⇒ Ψ (t) = −8b 2 (1 − t) = 0 =⇒ En t = 1 hay un punto cr´ıtico de Ψ(t) Como Ψ (1) = 8b 2 > 0, en t = 1 hay un m´ınimo de Ψ(t) y un m´ aximo de κ(t) pues son inversamente proporcionales Luego, κ(t) = √ a 2 +4b 2 a 2 y se alcanza en el punto − → f(1) = (a cos 1, asen1, b). b) Calculemos ahora la torsi´on; τ (t) = −→ f (t)× −→ f (t) · −→ f (t) [ −→ f (t)× −→ f (t) [ 2 ∀t − → f (t) = (asent, −a cos t, 0) =⇒ − → f (t) − → f (t) − → f (t) = 2a 2 b(1 −t) τ (t) = 2a 2 b(1−t) a 2 [a 2 +4b 2 (1+(1−t) 2 ] Se tiene que τ (0) = 2b [a 2 +8b 2 ] , τ (1) = 0, τ (2) = − 2b [a 2 +8b 2 ] τ (t) = 0 =⇒ 4b 2 (1 −t) 2 = a 2 + 4b 2 =⇒ (1 −t) 2 = a 2 +4b 2 4b 2 (1 − t) 2 = 1 + a 2 4b 2 > 1 =⇒ (1 − t) 2 > 1 lo cual es imposible porque (1 −t) ≤ 1 para 0 ≤ t ≤ 2. Por lo tanto, τ no tiene punto cr´ıtico en[0, 2] 155 El gr´afico es del tipo Ejercicio 12 a) Demuestre que la curva descrita por − → r (t) = (t cos t, t sin t, t) se encuentra sobre la superficie de un cono. Dibuje la curva. b) Si una part´ıcula parte del origen siguiendo la trayectoria anterior- mente descrita, determine en qu´e punto intersecta la esfera: x 2 + y 2 + z 2 = 2. c) Calcule la longitud de la curva desde el origen al punto de itersecci´ on. Soluci´ on: a) A partir de las ecuaciones param´etricas tenemos: x (t) = t cos t y (t) = t sin t z (t) = t _ _ _ =⇒ x 2 + y 2 = t 2 (cos 2 t + sin 2 t) =⇒ x 2 + y 2 = t 2 ∴ z 2 = x 2 + y 2 corresponde a la ecuaci´ on de un cono, cuyo sector superior ,se dibuja en el gr´ afico adjunto ∀t ≥ 0 b) Calculemos para que valor del par´ametro t la part´ıcula intersecta la esfera, sustityendo las ecuaciones param´etricas en la esfera x 2 + y 2 + z 2 = 2t 2 = 2 =⇒ t = 1.Luego, la posici´ on del punto de impacto es − → r (1) = (cos 1, sin 1, 1) c) La longitud de la curva es: l = _ 1 0 | − → r (t)| du con | − → r (t)| = _ − → r (t) − → r (t) 156 con − → r (t) = (cos t −t sin t, sin t + t cos t, 0) | − → r (t)| = _ − → r (t) − → r (t) = √ 2 + t 2 =⇒ l = _ 1 0 √ 2 + t 2 dt = _ ln _ √ 2 + x 2 + x _ + x √ 2 + x 2 2 _ 1 0 l = ln( √ 3 + 2) 2 + √ 3 2 Ejercicio 13 Un proyectil es lanzado desde el nivel del suelo con una velocidad inicial de 100 m/seg con un ´ angulo de elevaci´ on de 30 o .Determine: a) la funci´ on vectorial y las ecuaciones param´etricas de la trayectoria del proyectil. b) la altura m´ axima alcanzada. c) el alcance del proyectil d) la velocidad y rapidez en el impacto contra el suelo. e) la curvatura en el punto de impacto. Soluci´on: a) Inicialmente tenemos t = 0 , − → r 0 = (0, 0) ,y − → v 0 = (100 cos 30, 100 sin 30) Integrando: − → a (t) = − → r ” (t) = (0, −10) =⇒ − → v (t) = − → r ´ (t) = (100 cos 30, −10t + 100 sin 30) Integrando por segunda vez se obtiene: − → r (t) = ((100 cos 30) t, −5t 2 + (100 sin 30)t) 157 Evaluando las funciones trigonom´etricas queda: − → r (t) = (50 √ 3 t, −5t 2 + 50t) Por lo tanto, las ecuaciones param´etricas de la trayectoria del proyectil son x(t) = 50 √ 3 t, y(t) = −5t 2 + 50t b) Determinemos el tiempo que demora en llegar a la altura m´axima dy dt = 0 =⇒ −10t + 50 = 0 =⇒ t = 5s Asi la altura m´ axima h alcanzada por el proyectil es h = y(5) = −125 + 250 = 125m c) El alcance m´ aximo se logra cuando y(t) = 0, es decir si: −5t 2 +50t = 0 t(−5t + 50) = 0 ⇒t = 0, t = 10. Entonces el alcance es: x (10) = 500 √ 3 d) La velocidad del proyectil en el impacto es: − → v (10) = (50 √ 3, −50) e) La curvatura en el punto de impacto es:K (10) = −→ r (10)× −→ r (10) | −→ r (10) | 3 = 5 √ 3 10 4 Ejercicio 14 Sea C una curva determinada por la intersecci´ on de los cilindros: x 2 = 1 −y, z 2 = y a) Parametrizar C de forma − → r (t) = (x (t) , y (t) , z (t)), t ∈ I. Indicaci´ on: x 2 + z 2 = 1 b) Obtener ´ T, ´ N, ´ B, K y τ en P = (0, 1, 1) Soluci´ on: a) Se puede parametrizar como − → r (t) = (cos t, sin 2 t, sin t), t ∈ [0, 2π] , Calculemos el valor del par´ ametro para − → r (t 1 ) = (cos t 1 , sin 2 t 1 , sin t 1 ) = (0, 1, 1) =⇒ t 1 = π 2 . As´ı, − → r _ π 2 _ = (0, 1, 1). b) − → r (t) = (−sin t, sin t cos t, cos t) =⇒ − → r _ π 2 _ = (−1, 0, 0) =⇒ 158 ´ T _ π 2 _ = (−1, 0, 0) − → r (t) = (−cos t, 2 cos 2t, −sin t); − → r _ π 2 _ = (0, −2, −1) − → r _ π 2 _ − → r _ π 2 _ = (0, −1, 2) =⇒ _ _ − → r _ π 2 _ − → r _ π 2 __ _ = √ 5 =⇒ ´ B _ π 2 _ = _ 0, − 1 √ 5 , 2 √ 5 _ y ´ N _ π 2 _ = _ 0, − 2 √ 5 , − 1 √ 5 _ Derivando por tercera vez tenemos: − → r (t) = (sin t, −4 sin 2t, −cos t) =⇒ − → r _ π 2 _ = (1, 0, 0) ´ N _ π 2 _ = − → r _ π 2 _ − → r _ π 2 _ − → r _ π 2 _ _ _ − → r _ π 2 _ − → r _ π 2 _ − → r _ π 2 __ _ = _ 0, − 2 √ 5 , − 1 √ 5 _ Por otra parte K _ π 2 _ = −→ r ( π 2 ) × −→ r ( π 2 ) | −→ r ( π 2 )| 3 = √ 5 y τ _ π 2 _ = −→ r ( π 2 ) × −→ r ( π 2 ) · −→ r ( π 2 ) | −→ r ( π 2 ) × −→ r ( π 2 )| 2 = 0 Ejercicio 15 Dada la ecuaci´on param´etrica de la trayectoria − → r (t) = (2t 3 − 3t 2 , t − 2arctan(t)), encontrar todos los valores de t para los cuales la curva (i) Tiene tangente horizontal. (ii) Tiene tangente vertical. (iii) No es regular. Soluci´on: El vector tangente a la curva − → r (t) = (2t 3 −3t 2 , t −2arctan(t)) es − → r (t) = (6t 2 −6t, t 2 −1 t 2 + 1 ) cuya pendiente es m(t) = y (t) x (t) =⇒ m(t) = y (t) x (t) = t + 1 6t (t 2 + 1) (i) Para que la tangente sea horizontal, ´esta tiene que existir, es decir − → r (t) ,= − → 0 , y adem´ as m(t) = 0. Por tanto, t = −1. ii) Para que la tangente sea vertical, ´esta tiene que existir, es decir − → r (t) ,= − → 0 , y adem´ as m(t) = ∞. Por tanto,t = 0. (iii) Para que la curva sea no regular − → r (t) = − → 0 =⇒ x (t) = y (t) = 0, es decir, t = 1. Ejercicio 16 159 Se llama evoluta de una curva parametrizada regular − → r (t) ,con curvatu- ra no nula, al lugar geom´etrico de los centros de curvatura. Denotamos la evoluta de − → r (t) por − → c (t). (i) Encontrar una parametrizaci´on de − → c (t). (ii) Hallar la evoluta de la par´ abola − → r (t) = _ t, t 2 2 _ . (iii) Hallar la evoluta de la h´elice − → r (s) = _ √ 2 2 cos(s), √ 2 2 sen(s), √ 2 2 s _ , s ∈ IR y comprobar que es regular y que s es su par´ ametro arco. Soluci´on: (i) Si − → r (t) es punto de la curva y − → c (t) su correspondiente centro de curvatura para t entonces la condici´ on que define el lugar geom´etrico se tiene la relaci´on − → c (t) − − → r (t) = 1 k(t) − → N (t) de donde se deduce que la ecuaci´ on de la evoluta es − → c (t) = − → r (t) + 1 k(t) − → N (t) (ii) Calculamos en primer lugar el vector normal y la curvatura de la par´ abola. − → r (t) = _ t, t 2 2 _ =⇒ − → r (t) = (1, t) =⇒ [ − → r (t)[ = √ 1 + t 2 =⇒ − → N (t) = (−t, 1) √ 1 + t 2 Por otra parte − → r ” (t) = (0, 1) =⇒ k(t) = [ − → r (t) − → r ” (t)[ [ − → r (t)[ 3 = 1 _√ 1 + t 2 _ 3 Por lo tanto − → c (t) = _ t, t 2 2 _ + _√ 1 + t 2 _ 3 _√ 1 + t 2 _ (−t, 1) = _ −t 3 , 1 + 3 2 t 2 _ iii) Calculamos el vector tangente a la curva a partir de su definici´on − → r (s) = _ √ 2 2 cos(s), √ 2 2 sen(s), √ 2 2 s _ =⇒ − → r (s) = _ − √ 2 2 sen(s), √ 2 2 cos(s), √ 2 2 _ 160 =⇒ [ − → r (s)[ = 1 Por tanto ´ T (s) = _ − √ 2 2 sen(s), √ 2 2 cos(s), √ 2 2 _ Por otra parte, ´ T (s) = − → r (s) = _ − √ 2 2 cos(s), − √ 2 2 sen(s), 0 _ =⇒ [ − → r (s)[ = √ 2 2 de donde ´ N (s) = − → r (s) [ − → r (s)[ = (−cos(s), −sen(s), 0) Adem´ as k(s) = [ − → r (s)[ = √ 2 2 Finalmente − → c (s) = − → r (s) + 1 k(s) − → N (s) = _ − √ 2 2 cos(s), − √ 2 2 sen(s), √ 2 2 s _ , s ∈ IR lo que implica − → c (s) = _ √ 2 2 sen(s), − √ 2 2 cos(s), √ 2 2 s _ , y por tanto que [ − → c (s)[ = 1 para cada s ∈ IR y en definitiva que − → c es regular y est´ a parametrizada por arco s. Ejercicio 17 Encontrar una parametrizaci´ on de las siguientes curvas. (i) y = x 2 + 3x (ii) x 2 + y 2 + 2y = 0 (iii) x 2 − y 2 4 = 1 (iv) x 2 + 3y 2 = 1 Calcular en cada caso los vectores tangente unitario ´ T, normal ´ N , la curvatura κ(t) y la torsi´ on τ (t) Soluci´on: (i) Como se trata de una par´ abola podemos parametrizarla tomando x = t =⇒ y = t 2 + 3t As´ı − → r (t) = (t, t 2 + 3t), y calculamos el vector tangente, − → r (t) = (1, 2t + 3) =⇒ | − → r (t)| = √ 4t 2 + 12t + 10 161 y ´ T (t) = − → r (t) | − → r (t)| = (1, 2t + 3) √ 4t 2 + 12t + 10 Para calcular el vector normal en IR 2 tenemos en cuenta que es un vector ortogonal al tangente y unitario. ´ N (t) = (−2t −3, 1) √ 4t 2 + 12t + 10 La curvatura de una curva plana es k(t) = [ − → r (t) − → r ” (t)[ [ − → r (t)[ 3 , con- siderando que IR 2 es subespacio de IR 3 En nuestro caso − → r (t) = (1, 2t + 3, 0) =⇒ − → r (t) = (0, 2, 0) Por tanto, k(t) = 2 _√ 4t 2 + 12t + 10 ¸ 3 , la curvatura de la par´ abola var´ıa en funci´ on de t. Finalmente, la torsi´on de la curva plana es τ(t) = − → r (t) − → r ” (t) − → r (t) [ − → r (t)[ 3 = 0, pues − → r (t) = (0, 0, 0) lo que significa que la curva siempre est´ a en el mismo plano. (ii) La ecuaci´on dada representa una circunferencia de centro el (0, −1) y radio r = 1, ya que x 2 + y 2 + 2y = x 2 + (y + 1) 2 −1. Por tanto, una parametrizaci´on es − → r (t) = (cos(t), −1 + sen(t)) Calculamos el vector tangente. − → r (t) = (−sen(t), cos(t)) =⇒ | − → r (t)| = 1 =⇒ ´ T (t) = − → r (t) | − → r (t)| = (−sen(t), cos(t)) Para calcular el vector normal tenemos en cuenta que es un vector ortogonal al tangente y unitario. ´ N (t) = (−cos(t), −sen(t)) 162 La curvatura de una curva plana es k(t) = [ − → r (t) − → r ” (t)[ [ − → r (t)[ 3 , con- siderando que IR 2 es subespacio de IR 3 En nuestro caso − → r (t) = (−sen(t), cos(t), 0) =⇒ − → r (t) = (−cos(t), −sent(t), 0) =⇒ [ − → r (t) − → r ” (t)[ = 1 Por tanto, k(t) = 1 la curvatura de una circunsferencia es constante Finalmente, la torsi´on de la curva plana es τ(t) = − → r (t) − → r ” (t) − → r (t) [ − → r (t)[ 3 = 0,pues − → r (t) − → r ” (t) − → r (t) = (0, 0, 0). (iii) La ecuaci´ on dada representa una hip´erbola de semieje a = 1 y b = 2. Por tanto, una parametrizaci´on es − → r (t) = (cosh(t); 2senh(t)) Calculamos el vector tangente. − → r (t) = (senh(t), 2 cos h(t)) =⇒ | − → r (t)| = _ senh 2 (t) + 4 cos h 2 (t) y ´ T (t) = − → r (t) | − → r (t)| = (senh(t), 2 cos h(t)) _ senh 2 (t) + 4 cos h 2 (t) Para calcular el vector normal tenemos en cuenta que es un vector ortogonal al tangente y unitario. ´ N (t) = (−2 cos h(t), senh(t)) _ senh 2 (t) + 4 cos h 2 (t) La curvatura de una curva plana es k(t) = [ − → r (t) − → r ” (t)[ [ − → r (t)[ 3 , con- siderando que IR 2 es subespacio de IR 3 En nuestro caso : − → r (t) = (senh(t), 2 cos h(t), 0) =⇒ − → r (t) = (cosh(t); 2senh(t), 0) =⇒ [ − → r (t) − → r ” (t)[ = 2 Por tanto k(t) = 2 _ _ senh 2 (t) + 4 cos h 2 (t) _ 3 , la curvatura de la hip´erbo- la 163 var´ıa en funci´ on de t. Finalmente, la torsi´on de la curva plana es τ(t) = − → r (t) − → r ” (t) − → r (t) [ − → r (t)[ 3 = 0, pues − → r (t) − → r ” (t) − → r (t) = (0, 0, 0) (iv) La ecuaci´on dada representa una elipse de centro el (0, 0)y semiejes a = 1 y b = 1 √ 3 Entonces, una parametrizaci´on es − → r (t) = (cos(t), 1 √ 3 sen(t)) Calculamos el vector tangente. − → r (t) = (−sen(t), 1 √ 3 cos(t)) =⇒ | − → r (t)| = √ 3 3 _ 2sen 2 (t) + 1 =⇒ ´ T (t) = − → r (t) | − → r (t)| = √ 3 _ 2sen 2 (t) + 1 (−sen(t), 1 √ 3 cos(t)) Para calcular el vector normal tenemos en cuenta que es un vector ortogonal al tangente y unitario. ´ N (t) = √ 3 _ 2sen 2 (t) + 1 (− 1 √ 3 cos(t), −sen(t)) La curvatura de una curva plana es k(t) = [ − → r (t) − → r ” (t)[ [ − → r (t)[ 3 , con- siderando que IR 2 es subespacio de IR 3 . En nuestro caso − → r (t) = (−sen(t), 1 √ 3 cos(t), 0) =⇒ − → r (t) = (−cos(t), − 1 √ 3 sent(t), 0) =⇒ [ − → r (t) − → r ” (t)[ = 1 √ 3 Por tanto, k(t) = 3 _ _ 2sen 2 (t) + 1 _ 3 la curvatura de la elipse var´ıa en funci´ on de t. 164 Finalmente, la torsi´on de la curva plana es τ(t) = − → r (t) − → r ” (t) − → r (t) [ − → r (t)[ 3 = 0 pues − → r (t) − → r ” (t) − → r (t) = (0, 0, 0) Ejercicio 18 Sean − → r : I →IR 3 una curva regular y consideremos − → c la evoluta de − → r . Demostrar que − → c es regular si y s´ olo si la torsi´on y la derivada de la curvatura de − → r no se anulan simult´ aneamente en ning´ un punto y en este caso demostrar que el par´ ametro arco de − → c , s ∗ satisface que (s ∗ (t)) = 1 k 2 (t) _ (k (t)) 2 + (k (t)) 2 (τ (t)) 2 [ − → r (t)[ 2 ∀t ∈ I Soluci´on: La evoluta de est´ a definida como la curva − → c (t) = − → r (t) + 1 k(t) − → N (t) = − → r (t) +ρ (t) − → N (t), donde ρ (t) = 1 k(t) es el radio de curvatura de − → r . Por lo tanto, si s denota el par´ ametro arco de − → r tenemos que s (t) = [ − → r (t)[ − → c (t) = − → r (t) + ρ (t) − → N (t) + ρ (t) − → N (t) = − → r (t) + ρ (t) − → N (t) + ρ (t) − → N (s) s (t) Por Frenet sabemos que: − → N (s) = −k (s) ´ T (s) − τ (s) ´ B(s) =⇒ − → N (s) = −ρ (s) ´ T (s) − τ (s) ´ B(s) − → c (t) = [ − → r (t)[ ´ T (t)+ρ (t) − → N (t)+ρ (t) s (t) _ − ´ T (s) −ρ (s) τ (s) ´ B(s) _ =⇒ − → c (t) = ρ (t) − → N (t) −ρ (t) τ (t) [ − → r (t)[ ´ B(t) . As´ı [ − → c (t)[ = _ (ρ (t)) 2 + ρ 2 (t) τ 2 (t) [ − → r (t)[) 2 puesto que el Triedro de Frenet es ortonormal. Sustituyendo ρ y ρ por su expresi´ on en funci´ on de la curvatura, resulta que [ − → c (t)[ = ¸ (k (t)) 2 (k (t)) 4 + τ 2 (t) k 2 (t) [ − → r (t)[ 2 = 1 k 2 (t) _ (k (t)) 2 + k 2 (t) τ 2 (t) [ − → r (t)[ 2 165 para cada t ∈ I y por tanto, como ρ (t) y [ − → r (t)[ ,= − → 0 c es regular en t ∈ I si y s´ olo si k ´ (t) = τ (t) = 0. La expresi´ on anterior muestra tambi´en que cuando − → c es regular, la derivada de su par´ametro arco coincide con el t´ermino de la derecha de la igual- dad Ejercicio 19 Consideremos la curva C dada por − → r (t) = (e t , e 2t , t), t ∈ R. Hallar su curvatura y su torsi´ on en el punto (1, 1, 0). ¿Es cierto qu´e la curva tiene torsi´on negativa en todos sus puntos? Soluci´on: Calculamos primero las derivadas: − → r (t) = (e t , 2e 2t , 1) ⇒| − → r (t)| = √ 1 + e 2t + 4e 4t − → r = (e t , 4e 2t , 0) − → r − → r = (−4e 2t , e t , 2e 3t ) ⇒ _ _ − → r − → r _ _ = √ e 2t + 16e 4t + 4e 6t − → r = (e t , 8e 2t , 0) Y podemos obtener la curvatura y la torsi´on en cada punto: k(t) = _ _ − → r − → r _ _ | − → r (t)| 3 = √ e 2t + 16e 4t + 4e 6t _√ 1 + e 2t + 4e 4t ¸ 3 τ(t) = − → r − → r − → r | − → r − → r | 2 = 4e 3t _√ e 2t + 16e 4t + 4e 6t ¸ 2 En el punto (1, 1, 0) = (e t 0 , e 2t 0 , t 0 ) =⇒ t = 0 De donde deducimos que la curvatura y la torsi´ on son k(0) = _ _ − → r (0) − → r (0) _ _ | − → r (0)| 3 = √ 21 6 3/2 τ(0) = − → r (0) − → r (0) − → r (0) | − → r (0) − → r (0)| 2 = 4 21 Por ´ ultimo, la funci´ on τ(t) es claramente positiva, ∀ t ∈ IR Ejercicio 20 166 Considere la curva C dada por − → r (t) = (cosh(t), sinh(t), t), t ∈ IR (a) Dibuje aproximadamente su traza. (b) Si se recorre la traza partiendo desde punto (1, 0, 0), tras recorrer una longitud de arco √ 2 sobre la curva, ¿cu´ ales son las coordenadas del punto del espacio en el que nos encontraremos? Soluci´on: Consideremos la proyecci´ on de la curva sobre el plano XY : x 2 − y 2 = cos h 2 (t) − sinh 2 (t) = 1 se trata de una rama de hip´erbola, cuyo eje de simetria es X, toma valores positivos de X y se recorre en el sentido creciente de Y . La tercera componente es simplemente z = t, para valores t > 0 la curva se eleva con respecto al plano z = 0 hacia, mientras que para valores de t < 0, “baja” con respecto el plano z = 0. Determinemos el valor del par´ ametro en el punto de partida (1, 0, 0) = (cosh(t), sinh(t), t) =⇒ t = 0 Empezamos en s(0), la longitud de la curva, desde ese punto, se mide la longitud de arco. Calculemos la rapidez, y luego la longitud del arco 167 − → r (t) = (cosh(t), sinh(t), t), t ∈ IR ⇒ − → r (t) = (senh(t), cos h(t), 1), t ∈ IR =⇒ | − → r (t)| = _ cosh 2 (t) + sinh 2 (t) + 1 = _ 2cosh 2 (t) = √ 2cosh(t) . As´ı que la longitud desde s(0) hasta s(t) = _ t 0 | − → r (u)| du = √ 2 _ t 0 cosh(u)du = √ 2 senh(u)[ t 0 = √ 2senh(t) Ahora s´olo queda determinar el valor de t que hace que la longitud sea exactamente √ 2. Esto es, resolver √ 2 = √ 2sinh(t) ⇒1 = sinh(t) ⇒t = arcsinh(1). De manera que estaremos en el punto de coordenadas − → r (arcsinh(1)) = (cosh(arcsinh(1)), sinh(arcsinh(1)), arcsinh(1)) =⇒ − → r (arcsinh(1)) = ( √ 2, 1, arcsinh(1)) Ejercicio 21 Consideremos la curva C dada por − → r (t) = (t, t 2 , t 3 ) ∈ R. Hallar su curvatura y su torsi´on en el punto (0, 0, 0). ¿En qu´e punto tiene la curva una torsi´on (en valor absoluto) m´ axima? Soluci´on: Calculamos primero las derivadas: k(t) = _ _ − → r (t) − → r (t) _ _ | − → r (t)| 3 = √ 4 + 36t 2 + 36t 4 _√ 1 + 4t 2 + 9t 4 ¸ 3 τ(t) = − → r − → r − → r | − → r − → r | 2 = 12 [4 + 36t 2 + 36t 4 ] 2 De donde deducimos que la curvatura y la torsi´ on son k(0) = _ _ − → r (0) − → r (0) _ _ | − → r (0)| 3 = 2 τ(0) = − → r (0) − → r (0) − → r (0) | − → r (0) − → r (0)| 2 = 3 Como el denominador que aparece en la expresi´ on de τ(t) es siempre una cantidad positiva que toma su valor m´ınimo en t = 0, concluimos que el valor absoluto de la torsi´on alcanza su valor m´ aximo en t = 0. 168 2.17. Ejercicios propuestos 1. Sean f, g : [a, b] →R n funciones derivables, probar que: a) (α f + βg)(t) = α f (t) + βg (t) , α y β constantes b) (α f) (t) = α (t) f (t)+α(t)f (t) si α = α(t) es una funci´ on escalar. c) _ f g _ (t) = f (t) g(t) + f(t) g (t) d) _ f g _ (t) = (f g) (t) + (f g ) (t) 2. Para f(t) = (e 2t , t 2 , e −2t ) calcular: a) f (t) b) f (t) c) _ _ _ f (t) _ _ _ 3. Obtener una funci´ on param´etrica vectorial de la curva resultante al intersectar las superficies: x 2 + y 2 + z 2 = 2 y z = _ 2 (1 + y) 169 4. Parametrizar en la forma r = r(t) = (x(t), y(t), z(t)), t ∈ [a, b] las curvas C definidas por: a) Recta por A = (1, 2, −3) y B = (2, 2, 2) b) Recta intersecci´ on de los planos x + y + z = 0 y y −z = 0 c) Intersecci´ on del cilindro recto x 2 + y 2 = 9 y el plano z = 1 d) El cilindro x 2 −ax +y 2 = 0 intersecci´ on la esfera x 2 +y 2 +z 2 = a 2 5. Calcular la longitud l de la curva descrita por la trayectoria r(t) = (a cos 3 t, a sin 3 t), t ∈ [0, 2π] 6. Una hormiga camina sobre la superficie S : x(u, v) = sin ucos v, y (u, v) = sin usin v, z (u, v) = cos u π 6 ≤ u ≤ π 2 , − π 2 ≤ v ≤ π en el instante t = 0, se mueve de tal forma que en cualquier tiempo t > 0 describe la trayectoria − → r (t) = _ √ 3 2 sin t, 1 2 , √ 3 2 cos t _ . Demuestre que la hormiga se mueve sobre la superficie esferica S y determine cuando y en donde la hormiga abandona el sector descrito de la esfera 170 7. Halle el o los valores de t para los cuales el vector tangente a la curva descrita por r(t) = (2t 2 + 1, 3t − 2), t ∈ R sea paralelo al vector u = (2, −1). 8. Calcule l´ım t→0 ( (x+t) 2 −x 2 t , (x+t) 3 −x 3 t , (x+t) 4 −x 4 t ) 9. Discuta la continuidad de la funci´on f(t) = ¦ (t, sin t t ) si t ,= 0 (0, 1) si t = 0 10. Demuestre que: si _ _ _ − → f (t) _ _ _ es constante, entonces − → f y − → f son or- togonales. 11. Encuentre un vector tangente y la recta tangente a la h´elice c´onica de representaci´ on param´etrica − → f (t) = (t cos t, t sin t, t 2π ) en los puntos (0, 0, 0) y (0, π 2 , 1 4 ) 12. Sea la h´elice descrita por − → f (t) = (cos t, sin t, t) t ∈ [0, π 2 ]. Pruebe que en ning´ un punto de esta curva − → f (t) es paralelo a la cuerda de − → f (0) a − → f ( π 2 ) 13. Una part´ıcula se mueve en el plano a lo largo de la espiral ρ = e θ con una rapidez constante de 5 pies por segundo. 171 a) ¿Cu´ ales son la velocidad y la aceleraci´on de la part´ıcula cuando θ = π 4 ? b) ¿Cu´ anto tarda la part´ıcula en ir desde el punto correspondiente a θ = 0 hasta el punto correspondiente a θ = π? c) Si θ = 0 cuando t = 0, proporcione ecuaciones param´etricas para la trayectoria de la part´ıcula. 14. Determine los puntos en que la recta tangente a la curva de R 3 dada por r(t) = _ 6t + 2t 3 _ ˆı + (6t −2t 3 )ˆ  + 3t 2 ˆ k es paralela al plano x + 3y + 2z = 5 15. Demuestre que el camino descrito por − → f : [−2, 2] → R 2 definida por − → f (t) = (t 2 −1, t 3 +2t 2 −t −2) no es simple. ¿Es cerrado?¿Es cerrado simple? 16. Determine las ecuaciones de la recta tangente y del plano nor- mal a la curva descrita por − → f : I → R 3 definida por f(t) = (e t sint, e t cost, 5t) en el punto P = − → f (0) 17. Sea − → f : [0, 5] →R 3 la trayectoria − → f (t) = (sin t, cos t, t). Obtenga una reparametrizaci´ on − → f c de − → f que conserve su orientaci´ on y que recorra la trayectoria de − → f en la quinta parte del tiempo en el que lo hace − → f . 18. Un objeto ubicado inicialmente en el origen de coordenadas se desplaza de acuerdo al movimiento r(t) = (t 2 , −t cos t, t sin t) a) Determine el punto en que el objeto impacta a la esfera x 2 +y 2 +z 2 = 2 b) Calcular la longitud recorrida por el objeto hasta el instante del impacto. c) Calcular el ´angulo entre la trayectoria y la esfera en el instante del impacto. 172 19. El movimiento de una part´ıcula se realiza en el c´ırculo unitario del plano XY de acuerdo con la f´ ormula (x, y, z) = (cos t 2 , sin t 2 , 0) , t ≥ 0. a) Como funciones de t. ¿Cu´ al es el vector velocidad y la rapidez de la part´ıcula? b) ¿En qu´e punto del c´ırculo se debe liberar la part´ıcula para que golpee un blanco localizado en (2, 0, 0)? c) ¿En que tiempo t se debe liberar? (usar t ≥ 0 m´as pregunta b)) d) ¿Cu´ al es su velocidad y rapidez en el instante en que se libera? e) ¿En qu´e instante se golpea el blanco? 20. Si f : I → IR es una funci´ on diferenciable, demostrar que la gr´ afica de f, es decir la curva y = f(x), es una curva regular y calcular sus vectores tangente , normal, su curvatura, su radio de curvatura, y su centro de curvatura. 21. Sea C una curva de IR 3 dada por la ecuaci´ on − → r (t) = (4 cos(t), 4sen(t) , 4 cos t(t)), t ∈ [0, 2π] a) Verificar que es una elipse. b) Determinar los vectores ´ T, ´ N, ´ B en cualquier punto C c) Calcular la curvatura y torsi´ on de C en todo punto. d) Determinar en qu´e puntos de C la curvtura k es m´ axima y en cu´ ales es m´ınima 2.17.1. Respuestas 2. a) f (t) = 2 (e 2t , t, −e −2t ) b) f (t) = 2(2e 2t , 1, 2e −2t ) c) _ _ _ f (t) _ _ _ = 2 √ e 4t + e −4t + t 2 3. r(t) = (cos t, −1 + sin t, √ 2 sin t), 0 ≤ t ≤ 2π 4. a) r(t) = (1 + t, 2, −3 + 5t) t ∈ R 173 b) r(t) = (−2t, t, t) t ∈ R c) r(t) = (3 cos t, 3 sin t, 1) t ∈ [0, 2π] d) r(t) = ( a 2 + a 2 cos t, a 2 sin t, √ 2a sin t 4 ) t ∈ [0, 2π] 5. 6a, 6 −t = π 2 , (x, y, z) = ( √ 3 2 , 1 2 , 0) 7. t = − 3 2 8. (2x, 3x 2 , 4x 3 ) 9. Es continua 10. − → f − → f = 0 11. _ 1, 0, 1 2π _ , − → r (t) = (t, 0, 1 2π t) (0, π 2 , 1 4 ), − → r (t) = (− π 2 t, π 2 + t, 1 4 + t 1 2π ) 13. a) v = (0, 5), a = (− 25 √ 2 e − π 4 , 0) b) 1 5 √ 2(e π −1) c) − → f (t) = [ 5 √ 2 t + 1](cos ln( 5 √ 2 t + 1), sin ln( 5 √ 2 t + 1)) 14. r(2) = (28, −4, 12) 15. No es simple − → f (1) = (0, 0) = − → f (−1) 16. r(t) = (t, 1 + t, 5t), x + y + 5z = 1 plano normal 17. ¯ f : [0, 1] →R 3 , ¯ f (s) = (sin(5s), cos(5s), 5s) 18. a) (1, −cos1, sin1) b) 1,57 19. a) v = (−2t sin t 2 , 2t cos t 2 , 0); s = 2t b) ( 1 2 , − √ 3 2 , 0) c) _ 5π 3 d) v = 2 _ 5π 3 ( √ 3 2 , 1 2 , 0); s = 2 _ 5π 3 e) 3 √ 3+10π 2 √ 15π 174 20. La gr´ afica de la funci´on es la curva − → r : I →IR 2 determinada por − → r (x) = (x, f(x)) Como − → r (x) = (x, f (x)), resulta que [ − → r (x)[ = _ 1 + (f (x)) 2 > 0 ∀x ∈ I y por tanto la curva es regular ´ T (x) = (1, f (x)) _ 1 + (f (x)) 2 y −→ N (x) = (−f (x) , 1) _ 1 + (f (x)) 2 k(x) = [f” (x)[ ( _ 1 + (f (x)) 2 ) 3 y R(x) = ( _ 1 + (f (x)) 2 ) 3 [f” (x)[ − → c (x) = (x, f(x)) + (1 + (f (x) 2 ) [f” (x)[ (−f (x) , 1) 21. A partir de las ecuaciones param´etricas, se tiene que: x (t) = 4 cos(t) y (t) = 4sen(t) =⇒ x 2 + y 2 = 16 z (t) = 4 cos(t) z = x ´ T (t) = (−4sen(t) , 4 cos (t) , −4sen(t)) _ 16 + 16sen 2 (t) , ´ B(t) = (−16, 0, 16) 16 √ 2 ´ N (t) = ´ B(t) ´ T (t) = (cos (t) , 2sen(t) , cos (t)) √ 2 _ 1 + sen 2 (t) k(t) = 16 √ 2 4 3 _ _ 1 + sen 2 (t) _ 3 , τ(t) = 0 La curvatura es m´ axima si t = 0, π =⇒P 0 = (4, 0, 4) La curvatura es m´ınima si t = π 2 , 3π 2 =⇒P 1 = (0, 4, 0) 2.18. Auto Evaluaciones Autoevaluaci´ on N o 1 El estudiante: 1) Calcular´ a ecuaci´ on cartesiana de la trayectoria, curvatura y torsi´on en un punto de la curva, dada la ecuaci´ on vectorial de una curva regular, 175 dos veces diferenciable, con ecuaciones param´etricas en funci´ on de un par´ ametro cualquiera t, 2) Determinar´ a la ecuaci´ on de la recta tangente y del plano osculador a la curva descrita por la trayectoria − → f : I ⊆ R −→ R 3 en un punto de la curva. 3) Resolver´ a problemas de din´amica de part´ıculas querequiera calcular longitud de una curva, curvatura m´ axima , torsi´on de la curva, ecua- ciones de rectas ,y planos principales en un punto dado de la curva. Tiempo : 2 horas Tiempo : 2 horas Pregunta 1 Sea la h´elice − → r (t) = (a cos t, a sin t, bt) , b ,= 0, determine: a) los valores de a y b si su curvatura es κ(t) = 1 5 y su torsi´on τ (t) = 2 5 . b) La ecuaci´ on del cil´ındro sobre el cu´ al se encuentra la h´elice. Pregunta 2 Para la curva C dada por: − → r (t) = _ t 3 3 , 2t, 2 t _ en P 0 = _ 8 3 , 4, 1 _ Obtener las ecuaciones vectoriales y cartesianadas de: a) Recta tangente. b) Planos osculador. Pregunta 3 En el instante t= 0 , una casa es lanzada al espacio por causa de un tornado siguiendo la trayectoria − → r (t) = _ e at cos t, e at sin t, be at −b _ con a > 0, b > 0, t ≥ 0. a) Calcular la distancia que recorre la casa hasta el instante t = π. b) En el instante t 1 con t 1 ∈ [0, π] , en el cual la trayectoria tiene curvatura m´ axima, un residente que dorm´ıa tranquilo es lanzado 176 hacia el exterior. i) ¿qu´e trayectoria sigue el residente? ii) ¿en qu´e punto se encuentra la casa en t = π? iii) Obtener la ecuaci´ on de plano osculador a la trayectoria de la casa en t = π. Pauta de Autocorrecci´on Pregunta 1 En primer lugar determinemos la curvatura la torsi´ on en funci´on del tiempo − → r (t) = (−a sin t, a cos t, b) ∀t ∈ IR =⇒ | − → r (t)| = √ a 2 + b 2 − → r (t) = (−a cos t, −a sin t, 0) − → r (t) = (−a sin t, a cos t, 0) − → r (t) − → r (t) = (ab sin t, −ab cos t, a 2 ) =⇒ | − → r (t) − → r (t)| = a √ a 2 + b 2 − → r (t) − → r (t) − → r (t) = _ −a 2 b sin 2 t, −a 2 b cos 2 t, 0 _ Entonces, se tiene κ(t) = _ _ − → r (t) − → r (t) _ _ | − → r (t)| 3/2 = a a 2 + b 2 , y τ (t) = _ _ − → r (t) − → r (t) − → r (t) _ _ | − → r (t)| 3/2 = b a 2 + b 2 Luego, a partir de los valores dados de curvatura y torsi´ on, obtenemos a a 2 + b 2 = 1 5 , b a 2 + b 2 = 2 5 =⇒ a = 1, b = 2 Por tanto la ecuaci´on de la h´elice es − → r (t) = (cos t, sin t, 2t) b) A partir de la ecuaciones param´etricas de la h´elice , se tiene x = cos t, y = sin t, z = 2t ≥ 0 =⇒ x 2 + y 2 = 1, z ≥ 0, corresponde a un 177 cilindro de radio r = 1, cuyo eje axial coincide con el eje z. Pregunta 2 a) La ecuaci´ on de la recta tangente al punto P 0 se define por − → f (λ) = − → r (t 0 ) + λ − → r (t 0 ) , con λ ∈ IR siendo − → r (t 0 ) = _ t 3 0 3 , 2t 0 , 2 t 0 _ = _ 8 3 , 4, 1 _ =⇒ 2t 0 = 4 ⇐⇒ t 0 = 2 Calculemos ahora la direcci´ on de la recta tangente − → r (t) = _ t 2 , 2, − 2 t 2 _ =⇒ − → r (t 0 ) = _ 4, 2, − 1 2 _ Sustituyendo t´erminos en la ecuaci´on de la recta tangente, obtenemos (x, y, z) = _ 8 3 , 4, 1 _ + λ _ 4, 2, − 1 2 _ , λ ∈ IR Eliminando el par´ametro λ , tenemos la ecuaci´ on cartesiana x − 8 3 4 = y −4 2 = z −1 − 1 2 b) La ecuaci´ on del plano osculador al punto P 0 se define por _ − → f (t) − − → r (t 0 ) _ − → B (t 0 ) = 0, siendo − → B (t 0 ) = − → r (t 0 ) − → r (t 0 ) el vector normal al plano osculador en el punto P 0 . Como − → r (t) = _ t 2 , 2, − 2 t 2 _ =⇒ − → r (t) = _ 2t, 0, 4 t 3 _ ⇐⇒ − → r (t 0 ) = _ 4, 0, 1 2 _ . De manera que − → B (t 0 ) = − → r (t 0 ) − → r (t 0 ) = ¸ ¸ ¸ ¸ ¸ ¸ i j k 4 2 − 1 2 4 0 1 2 ¸ ¸ ¸ ¸ ¸ ¸ = (1, −4, −8) Por tanto la ecuaci´on de plano, queda 178 _ (x, y, z) − _ 8 3 , 4, 1 __ (1, −4, −8) = 0 3x −12y −24z = −64 Pregunta 3 a) Derivando la ecuaci´on de la trayectoria se tiene − → r (t) = _ ae at cos t −e at sin t, ae at sin t + e at cos t, abe at _ = e at (a cos t −sin t, a sin t + cos t, ab) Calculando la norma de este vector obtenemos | − → r (t)| = e at √ a 2 + 1 + a 2 b 2 Entonces, la longitud de la curva es l( − → r ) = _ π 0 | − → r (t)| dt = √ a 2 + 1 + a 2 b 2 _ e aπ −1 a _ b) La curvatura est´a dada por la expresi´on κ(t) = _ _ − → r (t) − → r (t) _ _ | − → r (t)| 3/2 Derivando por segunda vez, − → r (t) = ae at (a cos t −sin t, a sin t + cos t, ab) +e at (−a sin t −cos t, a cos t −sin t, 0) =⇒ − → r (t) − → r (t) = e 2at (ab(sin t −a cos t), −ab(a sin t + cos t, a 2 + 1) =⇒ _ _ − → r (t) − → r (t) _ _ = e 2at √ a 2 + 1 √ a 2 + 1 + a 2 b 2 Luego, obtenemos κ(t) = √ a 2 + 1 e at (a 2 + 1 + a 2 b 2 ) Por tanto, κ(t) es m´ aximo para t 1 = 0, cuando se tiene el valor m´ınimo del denominador 179 i) El residente sale despedido por la recta tangente a la trayectoria del tornado, en el instante t 1 = 0, luego la ecuaci´on de la recta ser´a − → R (0) = − → r (0) + λ − → r (0) = (1, 0, 0) + λ(a, 1, ab) ii) En el instante t = π la casa se encuentra en el punto − → r (π) = (−e aπ , 0, be aπ −b) iii) El plano osculador a la trayectoria de la casa en t 1 = π es ( − → R − − → r (π)) − → B (π) = 0 donde el vector binormal se determina por − → B (π) = − → r (π) − → r (π) = e 2πa _ a 2 b, ab, a 2 + 1 _ Por tanto, la ecuaci´on que produce es (x −e aπ , y, z −(be aπ −b)) e 2πa _ a 2 b, ab, a 2 + 1 _ = 0 Autoevaluaci´ on N o 2 Tiempo : 2 horas Pregunta 1 Una part´ıcula se desplaza a partir del instante t = 0 siguiendo la trayec- toria: − → r (t) = (t, ln (sec t + tagt) , ln sec t) , 0 ≤ t < π 2 Determinar la velocidad y rapidez en el instante que ha recorido una distancia igual √ 2 2 . Pregunta 2 Dada la curva C definida por las superficies x 2 +y 2 +z 2 = 11, x 2 +y 2 = 2, z > 0, determinar en el punto _ 0, √ 2, 3 _ ; a) la ecuaci´ on del plano osculador b) los versores tangente, normal y binormal 180 Pregunta 3 Para la curva C dada por − → r = − → r (s) ,donde s es el par´ametro longitud de arco, deducir las f´ ormulas: a) − → r (s) − → r (s) = −κ 2 (s) b) − → r (s) − → r (s) − → r (t) = κ 2 τ c) Usar las f´ormulas anteriores para calcular κ, τ de la curva C − → r (s) = _ 4 5 cos s, 1 −sin s, − 3 5 cos s _ d) Identificar la curva C Pauta de Autocorrecci´on Pregunta 1 Derivando la ecuaci´ on de la trayectoria: − → r (t) = (t, ln (sec t + tagt) , ln sec t) , 0 ≤ t < π 2 obtenemos la velocidad − → r (t) = (1, sec t, tagt) , 0 ≤ t < π 2 En consecuencia la rapidez de la part´ıcula, queda − → r (t) = _ 1 + sec 2 t + tag 2 = √ 2 sec t Entonces, la distancia recorrida en funci´on del tiempo, es s (t) = √ 2 _ t 0 sec udu = √ 2 2 ln _ 1 + sin t 1 −sin t _ A continuaci´on determinemos el tiempo t 0 , que corresponde a la dis- tancia 181 √ 2 2 : √ 2 2 ln _ 1 + sin t 0 1 −sin t 0 _ = √ 2 2 ln _ 1 + sin t 0 1 −sin t 0 _ = 1 t 0 = arcsin _ e −1 e + 1 _ ≈ 0, 48 Por lo tanto la velocidad en ese tiempo es − → v (t 0 ) = − → r (t 0 ) = (1, sec (0,48) , tag (0, 48)) = (1; 1, 127; 0, 521) y la rapidez | − → r (t 0 )| = _ 1 + (1, 127) 2 + (0, 521) 2 Pregunta 2 a) Determinemos la ecuaci´ on de la curva C, sustituyendo la segunda ecuaci´ on en la primera, obtenemos 2 + z 2 = 11 =⇒ z 2 = 9, z = ±3. Como z > 0, la curva dada es la circunsferencia x 2 + y 2 = 2, z = 3. La ecuaci´ on param´etrica de la curva es − → r (t) = _√ 2 cos t, √ 2 sin t, 3 _ , y el valor del par´ ametro en el punto es − → r (t 0 ) = _√ 2 cos t 0 , √ 2 sin t 0 , 3 _ = _ 0, √ 2, 3 _ =⇒ √ 2 cos t 0 = 0 =⇒ t 0 = π 2 . Calculemos ahora la ecuaci´on del plano osculador, derivando, obten- emos − → r (t) = _ − √ 2 sin t, √ 2 cos t, 0 _ =⇒ − → r _ π 2 _ = _ − √ 2, 0, 0 _ − → r (t) = _ − √ 2 cos t, − √ 2 sin t, 0 _ =⇒ − → r _ π 2 _ = _ 0, − √ 2, 0 _ El vector normal al plano en el punto dado es − → r _ π 2 _ − → r _ π 2 _ = ¸ ¸ ¸ ¸ ¸ ¸ i j k − √ 2 0 0 0 − √ 2 0 ¸ ¸ ¸ ¸ ¸ ¸ = (0, 0, 2) 182 La ecuaci´on del plano osculador viene dada por _ − → f − − → r _ π 2 _ _ − → N _ π 2 _ = 0 ⇐⇒ _ (x.y.z) − _ 0, √ 2, 3 _¸ (0, 0, 2) =⇒ z = 3 b) El versor tangente, en el punto es ´ t = − → r _ π 2 _ _ _ − → r _ π 2 __ _ = _ 0, − √ 2, 0 _ √ 2 = (0, −1, 0) . El versor binormal viene dado por ´ b = − → r _ π 2 _ − → r _ π 2 _ _ _ − → r _ π 2 _ − → r _ π 2 __ _ = (0, 0, 2) 2 = (0, 0, 1) . Finalmente el versor normal es ´ n = ´ b ´ t = (1, 0, 0) . Pregunta 3 a) A partir de − → r = − → r (s) tenemos que − → r (s) = ´ T (s) derivando por segunda vez y aplicando la identidad de Frenet, queda − → r (s) = ´ T (s) = κ ´ N (s) =⇒ − → r (s) = κ ´ N (s) + κ ´ N (s) Utilizando nuevamente la identidades de Frenet , obtenemos − → r (s) = κ ´ N (s) + κ _ τ ´ B −κ ´ T _ =⇒ − → r (s) = κ ´ N (s) + κτ ´ B −κ 2 ´ T Calculando el producto interno y considerando la ortonormalidad de la base _ ´ T, ´ N, ´ B _ , produce − → r (s) − → r (s) = ´ T (s) _ κ ´ N (s) + κτ ´ B −κ 2 ´ T _ = −κ 2 − → r (s) − → r (s) − → r (s) = ´ T (s) κ ´ N (s) _ κ ´ N (s) + κτ ´ B −κ 2 ´ T _ = κ 2 τ b) A partir de − → r (s) = _ 4 5 cos s, 1 −sin s, − 3 5 cos s _ =⇒ 183 − → r (s) = _ − 4 5 sin s, −cos s, 3 5 sin s _ =⇒ − → r (s) = _ − 4 5 cos s, sin s, − 3 5 cos s _ =⇒ − → r (s) = _ 4 5 sin s, cos s, 3 5 sin s _ Realizando el producto interno − → r (s) − → r (s) = −1 = −κ 2 =⇒ κ = 1 − → r (s) − → r (s) − → r (s) = 0 c) Desde κ = 1 podemos inferir que la curva es una circunferencia y por τ = 0 concluimos, que la curva es plana. d) Eliminando el par´ ametro s en las ecuaciones param´etricas (x, y, z) = _ 4 5 cos s, 1 −sin s, − 3 5 cos s _ Produce x 2 + (y −1) 2 + z 2 = 1, 3x + 4z = 0 Autoevaluaci´ on N o 3 Pregunta 1 Dada la ecuaci´ on param´etrica de la curva − → r (t) = _ t, 1 t + 1, 1 t −t _ , en el punto (1, 2, 0) , determine: a) la curvatura de la curva b) La torsi´ on de la curva Pregunta 2 Sea r = r (θ) la ecuaci´ on de una curva C en coordenadas polares, con a ≤ θ ≤ b ´ angulo polar y P (θ) punto de C. Demuestre que a) La longitud de C desde P(a) hasta P(b) es L = _ b a _ r 2 + (r ) 2 dθ b) La curvatura de C en P (θ) es κ = ¸ ¸ 2(r ) 2 −rr + r 2 ¸ ¸ [r 2 + (r ) 2 ] 3/2 . 184 c) Usando lo anterior,calcular la longitud L y la curvatura κ de la espiral logar´ıtmica r (θ) = e λθ , donde λ es constante. Pregunta 3 Un punto se mueve sobre la par´abola y = x 2 . Si en el instante t = α , P se encuentra en (0, 0) y s (α) = a ,s (α) = b ; donde la funci´on s = s (t) es la longitud de arco de la curva y a , b constantes. Determine la velocidad y la aceleraci´ on de P en el instante t = α. Pauta de Autocorrecci´on Pregunta 1 El valor del par´ ametro en el punto viene dado por − → r (t 0 ) = _ t 0 , 1 t 0 + 1, 1 t 0 −t 0 _ = (1, 2, 0) =⇒ t 0 = 1 Derivando sucesivamente − → r (t) = _ 1, − 1 t 2 , − 1 t 2 −1 _ =⇒ − → r (1) = (1, −1, −2) − → r (t) = _ 0, 2 t 3 , 2 t 3 _ =⇒ − → r (1) = (0, 2, 2) − → r (t) = _ 0, − 6 t 4 , − 6 t 4 _ =⇒ − → r (1) = (0, −6, −6) La curvatura se determina mediante la identidad κ(1) = _ _ − → r (1) − → r (1) _ _ | − → r (1)| 3/2 = |(2, −2, 2)| √ 6 3 = √ 2 6 Mientras que la torsi´ on est´a dada por la identidad τ (1) = _ _ − → r (1) − → r (1) − → r (1) _ _ | − → r (1) − → r (1)| 2 = |(2, −2, 2) (0, −6, −6)| √ 6 3 = 0 Obs´ervese que la torsi´ on es cero en todos los puntos ya que la curva est´ a contenida siempre en el mismo plano. Pregunta 2 185 a) En este caso podemos parametrizar la ecuaci´ on de la curva C de la forma − → r = − → r (θ) = (r (θ) cos θ, r (θ) sin θ) Derivando esta expresi´ on respecto de θ obtenemos − → r = − → r (θ) = (r (θ) cos θ −r (θ) sin θ, r (θ) sin θ + r (θ) cos θ) De modo que _ _ − → r (θ) _ _ = _ − → r (θ) − → r (θ) ¸ 1/2 = _ (r cos θ −r sin θ) 2 + (r sin θ + r cos θ) 2 =⇒ _ _ − → r (θ) _ _ = _ r 2 + (r ) 2 . Entonces la longitud del arco entre θ = a y θ = b para la curva − → r (θ) es L = _ b a _ _ − → r (θ) _ _ dθ = _ b a _ r 2 + (r ) 2 dθ. b) Deduciremos la curvatura usando la f´ ormula κ(θ) = _ _ − → r (θ) − → r (θ) _ _ | − → r (θ)| 3/2 Tenemos que − → r = r (θ) (cos θ, sin θ) + r (θ) (−sin θ, cos θ) Derivando por segunda vez obtenemos − → r = r (θ) (cos θ, sin θ) + 2r (θ) (−sin θ, cos θ) −r (θ) (cos θ, sin θ) Determinemos el producto vectorial considerando R 2 un subespacio vectorial de R 3 − → r − → r = _ r (cos θ, sin θ, 0) + r(−sin θ, cos θ, 0) _ _ r (cos θ, sin θ) + 2r (−sin θ, cos θ) −r(cos θ, sin θ) _ = _ 0, 0, 2(r ) 2 −rr + r 2 _ ya que se satisfacen que (cos θ, sin θ, 0) (cos θ, sin θ, 0) = (0, 0, 0) y (cos θ, sin θ, 0) (−sin θ, cos θ, 0) = (0, 0, 1) De esta forma _ _ − → r − → r _ _ = ¸ ¸ 2(r ) 2 −rr + r 2 ¸ ¸ 186 Por lo tanto κ(θ) = _ _ − → r (θ) − → r (θ) _ _ | − → r (θ)| 3/2 = ¸ ¸ 2(r ) 2 −rr + r 2 ¸ ¸ [r 2 + (r ) 2 ] 3/2 c) Sea r (θ) = e λθ =⇒ r (θ) = λe λθ =⇒ r (θ) = λ 2 e λθ . Aplicando directamente las f´ ormulas obtenidas en a) y b), obtenemos la longitud de la curva y su curvatura L = _ b a _ r 2 + (r ) 2 dθ = _ b a _ e 2λθ + λ 2 e 2λθ dθ = 1 + λ 2 λ _ e λb −e λa _ κ(θ) = ¸ ¸ 2(r ) 2 −rr + r 2 ¸ ¸ [r 2 + (r ) 2 ] 3/2 = (1 + λ 2 ) e 2λθ (1 + λ 2 ) √ 1 + λ 2 e 3λθ = 1 √ 1 + λ 2 e λθ Pregunta 3 Dada la ecuaci´ on cartesiana de la curva y = x 2 Sea − → r (t) = (x (t) , y (t)) la funci´on vectorial que describe el movimiento del punto tal que y (t) = [x (t)] 2 , entonces al derivar esta expresi´ on tenemos y (t) = 2x (t) x (t) =⇒ y (t) = 2((x (t)) 2 + x (t) x (t)). Sabemos que en el instante t = 0, − → r (α) = (x (α) , y (α)) = (0, 0) =⇒ x (α) = 0, y (α) = 0 Al evaluar las derivadas de y produce: y (α) = 0, y (α) = 2 _ x (α) _ 2 Por otra parte , sabemos que: s (α) = _ _ − → r (α) _ _ = _ (x (α)) 2 + (y (α)) 2 = ¸ ¸ x (α) ¸ ¸ = a s (α) = __ _ − → r (t) _ _ _ (α) = _ x (t) x (t) + y (t) y (t) | − → r (t)| _ (α) = s (α) = x (α) x (α) a = b =⇒ x (α) = b 187 Por lo tanto, la velocidad y la aceleraci´ on en el punto P son − → r (α) = _ x (α) , y (α) _ = (a, 0) − → r (α) = _ x (α) , y (α) _ = _ b, 2a 2 _ 188 Cap´ıtulo 3 Funciones de varias variables 3.1. Introducci´ on El concepto de funci´on, ya establecido, en cursos precedentes puede ser extendido a situaciones del tipo − → f : A ⊂ IR m →B ⊂ IR n , m, n ∈ IN en el sentido que son correspondencias que asignan a cada vector − → x ∈ A un ´ unico vector − → y ∈ B anotado − → y = − → f ( − → x ) . Consideradas las componentes de − → x y de − → y la igualdad anterior, tambi´en, se anota de la forma (y 1 , y 2 , . . . , y n ) = − → f (x 1 , x 2 , . . . , x m ); estas funciones se denominan funciones vectoriales de variable vectorial , o bien, n funciones dependientes de m variables independientes. Sus respectivos dominio de definici´ on A y recorrido B se denominan campos vectoriales. En este curso se consider´an funciones del tipo − → f : A ⊂ IR m →B ⊂ IR esto es con m > 1 y n = 1 , que por las caracter´ıstica de los espacios pasan a llamarse, funciones reales de variable vectorial ( de m variables independientes). En la unidad anterior fueron consideradas funciones vectoriales − → f : A ⊂ IR →IR n 189 es decir, con m = 1 y n > 1. Estas funciones se denotan como y = − → f (x 1 , x 2 , . . . , x m ) alternativamente − → y = f ( − → x ) con − → x = (x 1 , x 2 , . . . , x m ) y se estudiar´an sus propiedades en cu´ anto a variaciones, gr´aficas, aplicaciones, etc, a partir de los conceptos de l´ımite, continuidad, y derivada. En particular, para los casos usuales m = 2 y m = 3, se denotan z = f(x, y), (x, y) ∈ A ⊂ IR 2 ; w = f(x, y, z)(x, y, z) ∈ A ⊂ IR 3 , respectivamente. Ejemplos Caso 1) a) La correspondencia z = f(x, y) = xy , A = IR 2 , B = IR , restringi- dos x e y a valores positivos f(x, y) = xy corresponde al ´area del rect´ angulo de lados x e y (comparado con f (r) = πr 2 , ´esta representa el ´ area del c´ırculo de radio r y es funci´ on del tipo f : A ⊂ IR → B ⊂ IR, o sea con m = 1 y n = 1. b) La correspondencia w = f(x, y, z) = xyz , tiene A = IR 3 , B = IR , y con x, y, z positivos representa el volumen V del prisma recto de aristas x, y, z. Caso 2) a) La funci´ on f : A ⊂ IR 2 →B ⊂ IR dada por z = f(x, y) = _ x 2 + y 2 , A = IR 2 , B = IR, representa la distancia del punto P = (x, y) al origen O = (0, 0) de IR 2 . b) La funci´ on f : A ⊂ IR 3 →B ⊂ IR dada por w = f(x, y, z) = _ x 2 + y 2 + z 2 , A = IR 3 , B = IR, representa la distancia del punto P = (x, y, z) al origen O = (0, 0, 0) de IR 3 . Caso 3) La funci´on z = f(x, y) = 1 _ x 2 + y 2 , en que f : A ⊂ IR 2 → B ⊂ IR tiene dominio A = IR 2 ,y recorrido B = IR, representa el potencial 190 electrost´ atico en cada punto P = (x, y) del plano debido a una carga unitaria colocada en el origen O = (0, 0) de IR 2 . Nota: En estos ejemplos de funciones con m = 2, ´o m = 3 se puede considerar los conceptos de l´ıneas de nivel y superficies de nivel como S = ¦(x, y) ∈ A/ f (x, y) = c¦ para c constante dada; y S = ¦(x, y, z) ∈ A/ f (x, y, z) = c¦ para c constante dada. a) En el caso 2) se tiene que S = _ (x, y) ∈ IR 2 / _ x 2 + y 2 = c _ con c ≥ 0 son circunsferencias de radio √ c (´ o el origen si c = 0). b) En el caso 3) S = _ (x, y) ∈ IR 2 −¦(0, 0)¦ / 1 _ x 2 + y 2 = c, c > 0 _ las l´ıneas que verifican x 2 + y 2 = 1 c 2 se llaman l´ıneas equipotenciales alrededor de la carga, describen circunsferencias centradas el (0, 0) de radio 1 c . Unido a lo anterior, en topograf´ıa las curvas determinadas por ecuaci´ on f (x, y) = c se llaman tambi´en contornos de nivel de una superficie. Por ejemplo, la funci´ on z = f(x, y) = _ 25 −x 2 −y 2 que tiene A = _ (x, y) ∈ IR 2 / x 2 + y 2 ≤ 25 _ , B = [0, 5] representa una ”monta˜ na. es f´erica de radio basal 5 y altura 5; con c = 3 se tiene _ 25 −x 2 −y 2 = 3, de lo cual se tiene el contorno de nivel x 2 + y 2 = 16. Caso 4) La presi´on P ejercida por un gas ideal encerrado en un cilindro pist´ on es dada por la funci´ on P (T, V ) = k T V donde k cte, T es la temperatura y V el volumen del cilindro. En este caso las l´ıneas de nivel reciben el nombre de l´ıneas isob´aricas P = c, ´o isotermas cuando T = c con T funci´ on temperatura en cada punto (x, y, z) . Caso 5) 191 a) El ´ area S de la superficie del cuerpo humano es una funci´on del peso w y la altura h dada por S (w, h) = 0, 091w 0,425 h 0,725 (deducci´ on emp´ırica), donde w est´ a en libras y h en pulgadas y S en pie 2 . b) Tambi´en S es dada por S (h, t) = 2ht donde h es la altura en cm y t es la longitud de la circunferencia m´ axima del muslo en cm. Por ejemplo, una ni˜ na de altura h = 156 cm y 50 cm de de circunsferencia m´ axima del muslo tienen una superficie corporal de 15,600 cm 2 . Caso 6) La funci´on f (x, y) = C x a y 1−a con C y a constantes, donde x son las unidades de trabajo e y las unidades de capital representa un modelo de unidades de producci´ on (Modelo de Cobb-Douglas), que se utiliza en econom´ıa y en la evaluaci´on de proyectos. Caso 7) a) La potencia el´ectrica para un voltage E y resistencia R es la funci´on de dos variables P(E, R) = E 2 R . b) La resistencia total al conectar en paralelo dos resistencias R 1 , R 2 es regido por la ecuaci´ on 1 R = 1 R 1 + 1 R 2 , de donde se deduce R = R(R 1 , R 2 ) . Caso 8) a) La aceleraci´on centr´ıpeta de una part´ıcula que se mueve en la cir- cunferencia de radio r siendo v la rapidez es dada por a (r, v) = v 2 r . b) El desplazamiento vertical de una cuerda larga sujeta en el origen, que cae bajo la acci´ on de su propio peso es dada por u(x, t) = _ _ _ − g 2a 2 (2axt −x 2 ) , si 0 ≤ x ≤ at − g 2a 2 t 2 , si x > at, a cte Caso 9) La concentraci´ on molecular C (x, t) de un l´ıquido es dada por C (x, t) = t −1/2 e − x 2 k t y esta funci´ on verifica la ecuaci´ on de difusi´ on 192 k 4 ∂ 2 C ∂x 2 = ∂C ∂t . Caso 10) Cuando una chimenea de h metros de altura emite humo que contiene contaminante ´ oxido n´ıtrico la concentraci´ on C (x, z) a x km de distancia y z metros de altura es dada por C (x, z) = a x 2 _ e b(z−h) 2 x 2 + e − b(z−h) 2 x 2 _ Calcular e interpretar los valores de ∂C ∂x , ∂C ∂t ,en el punto P = (2, 5) para a = 200, b = 0, 02; h = 10 m. Caso 11) Un ejemplo de una funci´ on que depende de 4 variables es la que es- tablece la ley de Poiseuille en la cual la intensidad del flujo de un fluido viscoso (como la sangre) trav´es de un conducto (como una arteria), es Q(R, L, p 1 , p 2 ) = k R 4 L (p 1 −p 2 ) con k cte, R radio de conducto, L su longitud y p 1 , p 2 presiones en los extremos del conducto. 3.2. Funciones Escalares de Variable Vec- torial 3.2.1. Conceptos Topol´ogicos Nuestro espacio universo ser´a R n pensado preferentemente con n = 2 o n = 3. Si − → x ∈ R n entonces cada vector − → x = (x 1 , x 2 , . . . , x n ) esta conformado por una n- upla ordenada de n´ umeros reales. La m´etrica que se usar´a, es la m´etrica usual, es decir si − → x , − → y ∈ R n tal que: − → x = (x 1 , x 2 , . . . , x n ) − → y = (y 1 , y 2 , . . . , y n ) 193 entonces | − → x − − → y | = _ n i=1 (x i −y i ) 2 _1 2 A || se le llama norma euclidea y permite calcular la distancia entre los puntos x e y. Si y = 0 entonces | − → x | = _ n i=1 x 2 i _1 2 Vecindad Sea x 0 ∈ R n , una vecindad de x 0 es: V δ (x 0 ) = ¦x ∈ R n [ |x −x 0 | < δ¦ En R 2 esta vecindad es un disco centrado en P 0 . En R 3 es una esfera centrada en P 0 y de radio δ Caso especial es el de la vecindad despuntada V ∗ δ (x 0 ) = V δ (x 0 ) −¦x 0 ¦ Punto Interior Sea S ⊆ R n , x ∈ S es un punto interior de S si existe δ > 0 tal que: V δ (x) ⊂ S Conjunto Abierto Sea A ⊆ R n , diremos que A es un conjunto abierto si y s´olo si todos sus puntos son interiores. 194 Ejemplo: ¿Cu´ales de los siguientes conjuntos son abiertos? A = ¦(x, y) ∈ R 2 [ x 2 + y 2 ≤ 1¦ No es conjunto abierto A = ¦(x, y) ∈ R 2 [ [x[ < 1, [y[ < 1¦ Es conjunto abierto A = ¦(x, y) ∈ R 2 [ 0 ≤ x < 1¦ No es conjunto abierto Conjunto Cerrado Sea B ⊆ R n , diremos que B es un conjunto cerrado si su complemento R n −B es abierto. Punto Frontera Sea A ⊆ R n , x 0 es un punto frontera de A si y solo si ∀δ > 0 : V δ (x 0 ) ∩ A ,= ∅ y V δ (x 0 ) ∩ (R n −A) ,= ∅ Interior de un Conjunto Sea A ⊆ R n ,el conjunto de todos los puntos interiores de A se llama el interior de A y se denota o A o sea: o A = ¦x [ x es punto interior de A¦ Frontera de un Conjunto Sea A ⊆ R n , la frontera de A es el conjunto de todos los puntos frontera de A y se denota Fr(A). Proposici´on 1 Si S es abierto, no contiene ninguno de sus puntos fronteras. Dem. Directamente de la definici´ on de conjunto abierto. 195 Proposici´on 2 Si S es cerrado, contiene a todos sus puntos fronteras. Segmento Lineal Si P 1 , P 2 ∈ R n , el segmento lineal que une P 1 con P 2 es P = (1−t)P 1 + P 2 , 0 ≤ t ≤ 1. P 1 se llama punto inicial y P 2 punto terminal. Linea Poligonal Una poligonal est´ a formada por un n´ umero finito de segmentos lineales unidos sucesivamente por sus extremos. Punto Aislado Sea A ⊆ R n , x 0 es punto aislado de A si y solo si x 0 ∈ A y ∃δ > 0 : V ∗ δ (x 0 ) ∩ A = ∅ Punto de Acumulaci´ on Sea A ⊆ R n , x ∈ R n , x es punto de acumulaci´on de A si toda vecindad despuntada de x contiene puntos de A, es decir: x es punto de acumulaci´on de A ⇐⇒V ∗ δ (x 0 ) ∩ A ,= ∅ Regi´on Sea R ⊆ R n ,diremos que el conjunto R es una regi´ on si es un conjunto abierto y cualquier par de puntos de ella pueden unirse mediante una l´ınea poligonal, contenida en R. 196 Regi´on Cerrada Es una regi´ on unida con su frontera. Teorema 3.2.1. Sea C ⊆ R n y C es conjunto cerrado, entonces C contiene en todos sus puntos de acumulaci´on. 3.2.2. Aspectos Geom´etrico de las Funciones Es- calares Sea f : D ⊆ R n → R esta funci´ on a cada x ∈ D, x = (x 1 , x 2 , . . . , x n ) le asigna una imagen b ∈ R, b un escalar. b = f(x 1 , x 2 , . . . , x n ) a este tipo de funciones se les llama usualmente campo escalar. Con el objeto de resaltar sus aspectos geom´etricos, especialmente para funciones de dominio en R 2 o R 3 analizaremos conceptos como, gr´afi- cas, curvas de nivel, superficies de nivel, trazos, etc. de estas funciones. Sea f : D ⊆ R n → R si x ∈ D =⇒ x = (x 1 , x 2 , . . . , x n ), D es el dominio de la funci´ on f. f (D) = Im(f) = ¦z ∈ R [ z = f(x 1 , x 2 , . . . , x n )¦ 3.2.3. Gr´afica de una Funci´ on Sea f : D ⊆ R n → R , definimos la gr´ afica de f como el subconjunto de R n+1 formado por todos los puntos (x 1 , x 2 , . . . , x n , f(x 1 , x 2 , . . . , x n )) para cada (x 1 , x 2 , . . . , x n ) ∈ R n . Simb´ olicamente G f = _ (x 1 , x 2 , . . . , x n , f(x 1 , x 2 , . . . , x n )) ∈ R n+1 [ (x 1 , x 2 , . . . , x n ) ∈ D _ As´ı, si n = 2 y f es tal que z = f(x, y) entonces su gr´ afica ser´a: G f = _ (x, y, z) ∈ R 3 [ z = f(x, y) _ 197 3.2.4. Curvas y Superficies de Nivel Sea f : D ⊆ R n →R y C un escalar. Entonces el conjunto de puntos de nivel de valor C ,se define como el conjunto de puntos (x 1 , x 2 , . . . , x n ) ∈ D para los cuales f(x 1 , x 2 , . . . , x n ) = C.Simb´ olicamente, el conjunto de nivel S = ¦(x 1 , x 2 , . . . , x n ) ∈ D [ f(x 1 , x 2 , . . . , x n ) = C¦ Si n = 2 el conjunto ¦(x, y) [ f(x, y) = C¦ es una curva de nivel. Si n = 3 el conjunto ¦(x, y, z) [ f(x, y, z) = C¦ es una superficie de nivel. Ejemplo: 1.- Trazar la curva de nivel de f(x, y) = x −y, C = 0, 1, 2 2.- Trazar la curva de nivel de f(x, y) = 10 −x 2 −4y 2 , C = 0, 1, 2 Es familia de elipses centradas para C < 10, cuyo eje mayor se encuentra sobre el eje x Observaci´ on: Las curvas y las superficies de nivel permiten dar una imagen de la gr´afica de la funci´on. 198 3.2.5. L´ımite Sea f : D ⊆ R n →R una funci´on escalar y x 0 un punto de acumulaci´ on del dominio D. Formularemos la definici´ on de l´ımite de una funci´on escalar de la siguiente forma. Definici´ on 3.2.1. La funci´on f tiene como l´ımite al n´ umero L ∈ R cuando x →x 0 , si para cada > 0, existe δ (, x 0 ) > 0 tal que x ∈ D y 0 < |x −x 0 | < δ entonces [f (x) −L[ < Simb´ olicamente: l´ım x→x 0 f (x) = L ⇐⇒∀ > 0, ∃δ > 0 : x ∈ D y 0 < |x −x 0 | < δ =⇒[f (x) −L[ < En el caso particular n = 2 esta definici´on es l´ım (x,y)→(x 0 ,y 0 ) f(x, y) = L ⇐⇒∀ > 0, ∃δ > 0 : (x, y) ∈ D y 0 < |(x, y) −(x 0 , y 0 )| < δ =⇒ [f(x, y) −L[ < Ejemplo Sea f(x, y) = x 2 + 2xy. Determinar si existe l´ım (x,y)→(3,−1) (x 2 + 2xy) 199 An´alisis: Si (x, y) → (3, −1) significa que x est´ a cercano a 3 e y est´ a cercano a −1 por lo tanto el valor de x 2 + 2xy debe estar pr´ oximo a 3 se espera entonces que si este l´ımite existe debe ocurrir que: l´ım (x,y)→(3,−1) _ x 2 + 2xy _ = 3 Observe que: [x −3[ ≤ _ (x −3) 2 ≤ _ (x −3) 2 + (y + 1) 2 = |(x, y) −(3, −1)| [y + 1[ ≤ _ (y + 1) 2 ≤ _ (x −3) 2 + (y + 1) 2 = |(x, y) −(3, −1)| por lo cual: |(x, y) −(3, −1)| < δ =⇒[x −3[ < δ y [y + 1[ < ¸ ¸ x 2 + 2xy −3 ¸ ¸ < δ por otro lado , se tiene ¸ ¸ x 2 + 2xy −3 ¸ ¸ = ¸ ¸ (x −3) 2 + 2(x −3)(y + 1) + 4(x −3) + 6(y + 1) ¸ ¸ ≤ [x −3[ 2 + 2 [x −3[ [y + 1[ + 4 [x −3[ + 6 [y + 1[ Sin p´erdida de generalidad se puede poner la condici´ on δ < 1 entonces mayorando t´ermino a t´ermino, produce ¸ ¸ x 2 + 2xy −3 ¸ ¸ < [x −3[ + 2 [y + 1[ + 4 [x −3[ + 6 [y + 1[ < δ + 2δ + 4δ + 6δ = 13δ Definiendo δ = min _ 1, ε 13 _ , Todo lo anterior permite afirmar que: |(x, y) −(3, −1)| < δ ⇒ ¸ ¸ x 2 + 2xy −3 ¸ ¸ < 13 ¸ ¸ x 2 + 2xy −3 ¸ ¸ < 13δ = 13 ε 13 = ε 200 Lo cual prueba que l´ım (x,y)→(3,−1) (x 2 + 2xy) = 3 Consideremos otro ejemplo en que usamos tambi´en la definici´ on pero otro procedimiento Ejercicio: Probar que l´ım (x,y)→(0,0) _ x 2 y 2 x 2 + y 2 _ = 0 Soluci´on: Sea > 0 y x 2 ≤ x 2 + y 2 , y 2 ≤ x 2 + y 2 =⇒x 2 y 2 ≤ _ x 2 + y 2 _ 2 =⇒ x 2 y 2 x 2 + y 2 ≤ x 2 + y 2 , (x, y) ,= (0, 0) Sea δ = √ ε, |(x, y) −(0, 0)| = |(x, y)| = _ x 2 + y 2 _ x 2 + y 2 < δ =⇒ _ x 2 + y 2 < √ ε =⇒x 2 + y 2 < ε ∴ ¸ ¸ ¸ ¸ x 2 y 2 x 2 + y 2 ¸ ¸ ¸ ¸ = x 2 y 2 x 2 + y 2 < ε Lo que prueba que l´ım (x,y)→(0,0) _ x 2 y 2 x 2 + y 2 _ = 0 201 Los Teoremas de L´ımites Sean f : D 1 ⊆ R n → R , g : D 2 ⊆ R n → R tales que si a = (a 1 , a 2 , . . . , a n ) es punto de acumulaci´on de D 1 y D 2 y l´ım x→a f(x) = L, l´ım x→a g(x) = M, aqu´ı x = (x 1 , x 2 , . . . , x n ). Entonces: a.) l´ım x→a (f + g) (x) = L + M b.) l´ım x→a (f −g) (x) = L −M c.) l´ım x→a (f g) (x) = L M d.) l´ım x→a _ f g _ (x) = L M , M ,= 0 La demostraci´ on de estas propiedades es id´entica a la correspondiente de las funciones de R en R. Teorema 3.2.2. Sea f : D ⊆ R n →R tal que l´ım x→a f(x) = L, y g : I ⊆ R →R es una funci´on continua en I. Entonces g ◦ f : D ⊆ R n →R y l´ım x→a (g ◦ f) (x) = g _ l´ım x→a f(x) _ = g(L) Ejemplo: Evaluar l´ım (x,y)→(3,2) e x+y 2 Soluci´ on: Sea g(z) = e z continua en todo R, l´ım (x,y)→(3,2) x + y 2 = 7, entonces l´ım (x,y)→(3,2) e x+y 2 = e 7 Trayectorias: Sea (x, y) un punto de R 2 . Una trayectoria por (x, y) es cualquier recta o curva que contenga a (x, y) 202 Regla de las dos trayectorias Una condici´ on necesaria (no sufi- ciente) para que l´ım (x,y)→(x 0 ,y 0 ) f(x, y) exista y sea L, es que si los l´ımites l´ım x→x 0 f(x, ϕ(x)) y l´ım x→x 0 f(x, ψ (x)) existen , para cualquier trayectoria y = ϕ(x) , y = ψ (x) que pase por (x 0 , y 0 ),deben valer L Ejemplo: Sea f(x, y) = x 2 y x 4 +y 2 ¿Existir´ a l´ım (x,y)→(0,0) f(x, y)?. Soluci´on: Dominio de f es R 2 −¦(0, 0)¦ y claramente (0, 0) es punto de acumu- laci´ on del dominio de f. Sea T 1 = ¦(x, y) [ y = ax¦ familia de rectas que pasan por el origen si el l´ımite existe deber´ıa ocurrir que: l´ım (x,y)→(0,0) x 2 y x 4 + y 2 = l´ım x→0 x 2 (αx) x 4 + α 2 x 2 = l´ım x→0 (αx) x 2 + α 2 = 0 Esto se˜ nala que si el l´ımite existe este debe ser cero, seguimos averiguan- do: Sea T 2 = ¦(x, y) [ y = x 2 ¦ par´ abola por el origen. l´ım (x,y)→(0,0) x 2 y x 4 + y 2 = l´ım x→0 x 4 x 4 + x 4 = 1 2 No puede ser, el l´ımite si existe no puede tener dos valores diferentes. Luego, no existe l´ım (x,y)→(0,0) x 2 y x 4 + y 2 L´ımites Iterados Se llaman l´ımites iterados a los siguientes. l´ım x→x 0 _ l´ım y→y 0 f(x, y) _ ; l´ım y→y 0 _ l´ım x→x 0 f(x, y) _ 203 Ejemplo: Sea f(x, y) = x 2 −y 2 x 2 +y 2 , (x, y) ,= (0, 0) .Determine los l´ımites iterados de f. Soluci´on: Tenemos que (0, 0) es punto de acumulaci´ on del dominio de f. Evaluemos, los l´ımites iterados l´ım x→0 _ l´ım y→0 x 2 −y 2 x 2 + y 2 _ = l´ım x→0 x 2 x 2 = 1 l´ım y→0 _ l´ım x→0 x 2 −y 2 x 2 + y 2 _ = l´ım y→0 −y 2 y 2 = −1 Los conceptos de l´ımites y l´ımites iterados se relacionan seg´ un los sigu- ientes teoremas. Teorema 3.2.3. Si l´ım (x,y)→(x 0 ,y 0 ) f(x, y) existe, y si para cada x en una vecindad reducida de x 0 , l´ım y→y 0 f(x, y) existe. Entonces l´ım x→x 0 _ l´ım y→y 0 f(x, y) _ = l´ım (x,y)→(x 0 ,y 0 ) f(x, y) Este teorema nos lleva a formular otro equivalente. Teorema 3.2.4. Si l´ım (x,y)→(x 0 ,y 0 ) f(x, y) existe, y si para cada y en una vecindad reducida de y 0 , l´ım x→x 0 f(x, y) existe. Entonces l´ım y→y 0 _ l´ım x→x 0 f(x, y) _ = l´ım (x,y)→(x 0 ,y 0 ) f(x, y) Ahora, se puede combinar ambos teoremas, lo que produce: Teorema 3.2.5. Si l´ım (x,y)→(x 0 ,y 0 ) f(x, y) existe, y si para cada x en una vecindad reducida de x 0 , l´ım y→y 0 f(x, y) existe, y si para cada y de una vecindad reducida de y 0 , l´ım x→x 0 f(x, y) existe. Entonces l´ım y→y 0 _ l´ım x→x 0 f(x, y) _ = l´ım x→x 0 _ l´ım y→y 0 f(x, y) _ = l´ım (x,y)→(x 0 ,y 0 ) f(x, y) En el ejemplo anterior podemos concluir que no existe l´ım (x,y)→(0,0) x 2 −y 2 x 2 +y 2 ,pues de existir su l´ımites iterados deben ser iguales. 204 3.2.6. Continuidad Una funci´ on f es continua en a ( a es punto de acumulaci´ on de D ⊆ R n ) si para cada ε > 0 existe δ > 0 tal que ∀x ∈ D : |x −a| < δ =⇒[f(x) −f(a)[ < ε donde x = (x 1 , x 2 , ..., x n ), a = (a 1 , a 2 , ..., a n ). O lo que es lo mismo: Se tiene que f es continua en un punto a si: i) f(a) existe, ii)l´ım x→a f(x) existe y ; iii)l´ım x→a f(x) = f(a). Es decir, para que una funci´on de varias variables sea continua en un punto debe estar definida all´ı, debe tener l´ımite en ´el y el valor de la funci´ on en el punto debe ser igual al valor del l´ımite en ese punto. En R 2 podemos enunciar esta propiedad de la siguiente forma. Una funci´ on f en continua en un punto interior (x 0 , y 0 ) de una regi´ on R si f (x 0 , y 0 ) est´a definida y l´ım (x,y)→(x 0 ,y 0 ) f(x, y) = f (x 0 , y 0 ) f ser´ a continua en la regi´on R si es continua en cada punto de R. Las funciones que no son continuas se dicen que son discontinuas. Teoremas de Continuidad Son similares a los teoremas para funciones de una variable. Esto sig- nifica que, si una funci´on es combinaci´ on de otras funciones y estas funciones a su vez son continuas entonces la funci´ on es continua excep- to en aquellos puntos en los que no est´a definida. Ejemplo. Sea la funci´ on f(x, y) = _ _ _ x 2 y 2 x 2 + y 2 si (x, y) ,= (0, 0) 0 si (x, y) = (0, 0) ,estudie la continuidad de f en R 2 . Soluci´ on. 205 Claramente si (x, y) ,= (0, 0), f es un cuociente de funciones con- tinuas por lo que tambi´en es continua. Adem´ as, la funci´ on es continua en (0, 0) pues se demostr´o en 1.4.2, que l´ım (x,y)→(0,0) x 2 y 2 x 2 + y 2 = 0 = f(0, 0), Por lo tanto, esta funci´ on es continua en todo R 2 . Continuando con este mismo ejemplo, podemos usar coordenadas polares para mostrar de que ´este ´ ultimo l´ımite vale cero. Decir que (x, y) → (0, 0) , en coordenadas polares es equivalente a que r → 0 (independientemente del valor de θ). Expresando la funci´on f(x, y) = x 2 y 2 x 2 + y 2 en coordenadas polares, x = r cos θ, y = r sin θ, obtenemos la funci´on g(r, θ) = r 2 cos 2 θ sin 2 θ cos 2 θ + sin 2 θ = r 2 cos 2 θ sin 2 θ cos 2 θ + sin 2 θ observese que ϕ(θ) = cos 2 θ sin 2 θ cos 2 θ + sin 2 θ = cos 2 θ sin 2 θ ≤ 1. est´ a acotada, y adem´as ψ (r) = r 2 Lo que implica que: l´ım r→0 g(r, θ) = l´ım r→0 ψ(r)ϕ(θ) = 0. Podemos concluir que si ϕ(θ) ≤ M est´ a acotada, en una vecindad con centro en el origen, y ψ(r) →0 cuando r −→0,entonces l´ım r→0 ψ(r)ϕ(∅) = 0. Continuidad en un conjunto abierto. Diremos una funci´ on es continua en un conjunto abierto U ⊆ R n ,si ella es continua en cada punto del conjunto U. 206 3.2.7. Derivadas Parciales Sea f : D ⊆ R 2 →R funci´ on de dos variables que est´a definida en una vecindad del puntos (x 0 , y 0 ). La derivada parcial de f respecto de x en (x 0 , y 0 ) se define por: ∂f ∂x (x 0 , y 0 ) = l´ım h→0 f(x 0 + h, y 0 ) −f(x 0 , y 0 ) h Similarmente, la derivada parcial de f respecto de y en (x 0 , y 0 ) se define por: ∂f ∂y (x 0 , y 0 ) = l´ım h→0 f(x 0 , y 0 + h) −f(x 0 , y 0 ) h siempre que estos l´ımites existan. Las derivadas de orden superior son una reiteraci´on de la definici´on anterior, es decir, derivadas sucesivas, as´ı: ∂ 2 f ∂x 2 = ∂ ∂x _ ∂f ∂x _ = l´ım h→0 f x (x 0 + h, y 0 ) −f x (x 0 , y 0 ) h ∂ 2 f ∂y 2 = ∂ ∂y _ ∂f ∂y _ = l´ım h→0 f y (x 0 , y 0 + h) −f y (x 0 , y 0 ) h ∂ 2 f ∂x∂y = ∂ ∂x _ ∂f ∂y _ = l´ım h→0 f y (x 0 + h, y 0 ) −f y (x 0 , y 0 ) h Ejemplo: Ejemplo: Calcular las derivadas parciales de: f(x, y) = x 4 sin y + cos xy Soluci´on: De acuerdo con la definici´on debemos calcular l´ım h→0 (x + h) 4 sin y + cos(x + h)y −(x 4 sin y + cos xy) h 207 Desarrollando el numerador se tiene: x 4 sin y + 4x 3 hsin y + 6x 2 h 2 sin y + 4xh 3 sin y + h 4 sin y +cos(xy) cos(hy) −sin(xy) sin(hy) −x 4 sin y −cos(xy) = 4x 3 hsin y + 6x 2 h 2 sin y + 4xh 3 sin y + h 4 sin y +cos(xy)(cos(hy) −1) −sin(xy) sin(hy) = Calculando l´ımite l´ım h→0 (x + h) 4 sin y + cos(x + h)y −(x 4 sin y + cos xy) h = 4x 3 sin y + cos xyl´ım h→0 (cos(hy) −1) h −sin(xy)l´ım h→0 sin(hy) h Como l´ım h→0 (cos(hy) −1) h = 0 y l´ım h→0 sin(hy) h = y Tenemos que este l´ımite y por tanto la derivada es ∂f ∂x (x, y) = 4x 3 sin y −y sin(xy) De igual forma se puede calcular por definici´on. ∂f ∂y (x, y) = x 4 cos y −x cos xy Derivadas Parciales Cruzadas Teorema 3.2.6. (Teorema de Schwarz) Sea f : D ⊆ R 2 →R una funci´on, D abierto. Si las derivadas ∂ 2 f ∂x∂y y ∂ 2 f ∂y∂x existen y son continuas en D, entonces: ∂ 2 f ∂x∂y = ∂ 2 f ∂y∂x 208 Demostraci´ on queda propuesta. Como ejemplo ilustrativo considere la funci´ on f(x, y) = _ x 3 y−xy 3 x 2 +y 2 si (x, y) ,= (0, 0) 0 si (x, y) = (0, 0) y verifique que: ∂ 2 f ∂x∂y (0, 0) = 1, ∂ 2 f ∂y∂x (0, 0) = −1 Observaci´ on: Use la definici´ on y encuentre estos resultados. 3.3. Diferenciabilidad en dos variables Sea f una funci´ on definida en una vecindad de (x 0 , y 0 ). Diremos que f es diferenciable en (x 0 , y 0 ) si existen n´ umeros A y B tales que: f(x 0 + h, y 0 + k) −f(x 0 , y 0 ) = Ah + Bk + α(h, k) |(h, k)| y si el residuo tiene la propiedad l´ım (h,k)→0 α(h, k) = 0. Diremos que f es diferenciable en una regi´on, si es diferenciable en cada punto de la regi´ on. Teorema 3.3.1. Si f es diferenciable en (x 0 , y 0 ), entonces f es con- tinua en (x 0 , y 0 ). Demostraci´ on: Sea (x, y) ∈ V δ ((x 0 , y 0 )) y f(x 0 +h, y 0 +k)−f(x 0 , y 0 ) = A(x −x 0 )+B(y −y 0 )+α(h, k) |(x, y) −(x 0 , y 0 )| Adem´ as sabemos que [x −x 0 [ ≤ |(x, y) −(x 0 , y 0 )| , [y −y 0 [ ≤ |(x, y) −(x 0 , y 0 )| entonces 209 |f(x 0 + h, y 0 + k) −f(x 0 , y 0 )| ≤ [Ah + Bk + α(h, k)[ |(x, y) −(x 0 , y 0 )| Si (x, y) →(x 0 , y 0 ) , tenemos que α(h, k) →0 y |(x, y) −(x 0 , y 0 )| → 0 por lo tanto |f(x 0 + h, y 0 + k) −f(x 0 , y 0 )| →0 ∴ l´ım (h,k)→(0,0) f(x 0 + h, y 0 + k) = f(x 0 , y 0 ) La diferenciabilidad es tambi´en una condici´ on m´ as fuerte que la exis- tencia de las derivadas parciales. Teorema 3.3.2. Si f es diferenciable en (x 0 , y 0 ), entonces las derivadas parciales de primer orden existen en (x 0 , y 0 ) y f x (x 0 , y 0 ) = A f y (x 0 , y 0 ) = B Demostraci´ on Sea f diferenciable en (x 0 , y 0 ) =⇒ f(x 0 + h, y 0 + k) −f(x 0 , y 0 ) = Ah + Bk + α(h, k) √ h 2 + k 2 dividiendo por h ,= 0 y haciendo k = 0 se tiene: f(x 0 + h, y 0 ) −f(x 0 , y 0 ) h = A + α [h[ h =⇒ ¸ ¸ ¸ ¸ f(x 0 + h, y 0 ) −f(x 0 , y 0 ) h −A ¸ ¸ ¸ ¸ = [α(h)[ ¸ ¸ ¸ ¸ [h[ h ¸ ¸ ¸ ¸ = [α(h)[ pero l´ım h→0 α(h) = 0 ∴ l´ım h→0 f(x 0 + h, y 0 ) −f(x 0 , y 0 ) h = A De manera analoga 210 ∴ l´ım k→0 f(x 0 , y 0 + k) −f(x 0 , y 0 ) k = B Se puede tener tambi´en el siguiente criterio para la diferenciabilidad de una funci´ on. Teorema 3.3.3. Sea f : D ⊆ R 2 →R una funci´on. Si f tiene primeras derivadas parciales continuas en una regi´on D. Entonces f es diferen- ciable en cada punto de D. Nota: Todas las ideas dadas para funciones de dos variables se pueden extender a funciones definidas en un espacio n dimensional. Diferencial Total En dos dimensiones: Si f es diferenciable en (x 0 , y 0 ) entonces la diferencial total es: df = ∂f ∂x dx + ∂f ∂y dy en (x 0 , y 0 ) En tres dimensiones:. Si f es diferenciable en (x 0 , y 0 , z 0 ) entonces la diferencial total es: df = ∂f ∂x dx + ∂f ∂y dy + ∂f ∂z dz en (x 0 , y 0 , z 0 ) 3.3.1. Derivada Direccional Gradiente Sea f : D ⊆ R n →R, P ∈ D abierto y f diferenciable en P. Entonces el vector gradiente de f en P se denota ∇f (P) y se define por la f´ ormula: ∇f (P) = _ ∂f ∂x 1 (P) , ∂f ∂x 2 (P) , ..., ∂f ∂x n (P) _ 211 Ejemplo: Sea f(x, y, z) = 3x 3 y 2 z. Calcular ∇f(1, 2, 3). Soluci´ on: La funci´on tiene derivadas parciales continuas, entonces ∇f(x, y, z) = _ 9x 2 y 2 z, 6x 3 yz, 3x 3 y 2 _ Evaluando, el gradiente de la funci´ on f en el punto (1, 2, 3) queda ∇f(1, 2, 3) = (108, 36, 12) = 12 (9, 3, 1) Propiedades Si f y g son funciones diferenciables demuestre que: ∇(fg) = f∇g + g∇f ∇ _ f g _ = g∇f −f∇g g 2 si g no es cero Ejemplo Si f es diferenciable en una variable y g = f (x 2 + y 2 + z 2 ) . Calcular ∇g ∇g. Soluci´on. Si ponemos u = x 2 + y 2 + z 2 al derivar parcialmente se tiene ∂g ∂x = 2xf (u) ; ∂g ∂y = 2yf (u) ; ∂g ∂z = 2zf (u). =⇒ ∇g = 2f (u)(x, y, z) =⇒∇g ∇g = 4 [f (u)] 2 [(x, y, z) (x, y, z)] = 4 [f´(u)] 2 (x 2 + y 2 + z 2 ) (Definici´ on Derivada direccional) Sea f : D ⊆ R n → R y P 0 un punto interior de D. Sea ´ u vector unitario |´ u| = 1. La derivada direccional de f en P 0 en la direcci´ on de ´ u se define por: l´ım h→0 f(P 0 + h´ u) −f(P 0 ) h Si este l´ımite existe f tiene derivada direccional en P 0 en la direcci´on de ´ u y la denotamos ∂f ∂´ u (P 0 ) . 212 Teorema 3.3.4. Si f es diferenciable en P, entonces ∂f ∂´ u (P) existe para todos los vectores ´ u, unitarios y ∂f ∂´ u (P) = ∇f (P) ´ u Demostraci´ on: Considerando la demostraci´ on en R 3 .Sean P 0 = (x, y, z) y ´ u = (u 1, u 2 , u 3 ) Por definici´ on de diferenciabilidad se tiene: f(P 0 + h´ u) −f(P 0 ) = f(x + hu 1 , y + hu 2 , z + hu 3 ) −f(x, y, z) = ∂f ∂x h u 1 + ∂f ∂y h u 2 + ∂f ∂z h u 3 + α[h[ = h _ ∂f ∂x u 1 + ∂f ∂y u 1 + ∂f ∂z u 3 _ + α[h[ Dividiendo por h y tomando l´ımite cuando h →0 se tiene α →0 y l´ım h→0 f(P 0 + h´ u) −f(P 0 ) h = ∂f ∂x u 1 + ∂f ∂y u 2 + ∂f ∂z u 3 ∴ ∂f ∂´ u (P 0 ) = ∇f (P 0 ) ´ u Nota: Para cada ´ u vector unitario fijo, la derivada direccional ∂f ∂´ u de- fine una nueva funci´on, a la cual a su vez se le puede aplicar la definici´on de derivada direccional y tenemos as´ı las derivadas direccionales de or- den superior. ∂ 2 f ∂´ u 2 = ∂ ∂´ u _ ∂f ∂´ u _ Ejemplo Sea f (x, y) = x 3 y 2 , calcular la derivada direccional de f en el punto P 0 (−1, 2) en la direcci´ on del vector − → u = (4, −3). Soluci´on. Como f (x, y) = x 3 y 2 en una funci´ on diferenciable ∀ (x, y) ∈ R 2 ,entonces 213 ∂f ∂´ u (P) = ∇f (P) ´ u En primer lugar calculemos el vector gradiente ∇f (P 0 ) = (3x 2 y 2 , 2x 3 y) En segundo lugar calculemos el vector unitario ´ u = − → u | − → u | = (4, −3) 5 Se deduce que ∂f ∂´ u (x, y) = _ 3x 2 y 2 , 2x 3 y _ (4, −3) 5 Por lo tanto, evaluando en P 0 (−1, 2) queda ∂f ∂´ u (−1, 2) = _ 3 (−1) 2 2 2 , 2 (−1) 3 2 _ (4, −3) 5 = 96 5 + 12 5 = 108 5 Teorema 3.3.5. Supongamos que f tiene segundas derivadas parciales continuas en una vecindad de un punto P. Entonces: ∂ 2 f ∂´ u 2 (P) = (´ u ∇) 2 f Demostraci´ on: ∂f ∂´ u = f 1 u 1 + f 2 u 2 + f 3 u 3 ∂ 2 f ∂´ u 2 = u 1 ∂f 1 ∂´ u + u 2 ∂f 2 ∂´ u + u 3 ∂f 3 ∂´ u ∂ 2 f ∂´ u 2 = u 1 (f 11 u 1 + f 12 u 2 + f 13 u 3 ) + u 2 (f 21 u 1 + f 22 u 2 + f 23 u 3 ) +u 3 (f 31 u 1 + f 32 u 2 + f 33 u 3 ) Como las derivadas cruzadas son iguales se tiene ∂ 2 f ∂´ u 2 = u 2 1 f 11 + 2u 1 u 2 f 12 + 2u 1 u 3 f 13 + u 2 2 f 22 + 2u 2 u 3 f 23 + u 2 3 f 33 = _ u 1 ∂ ∂x + u 2 ∂ ∂y + u 3 ∂ ∂z _ 2 f = (´ u ∇) 2 f 214 Consecuencias de ∂f ∂´ u (P) = ∇f (P) ´ u 1.) Si ∇f (P) = 0 =⇒ ∂f ∂ − → u (P) = 0 ∀´ u 2.) La direcci´ on de m´aximo crecimiento de la derivada direccional viene dada por ∂f ∂´ u (P) = |∇f (P)| en la direcci´ on del vector gradiente. 3.) La direcci´on de mayor decrecimiento (o m´ınimo crecimiento) viene dada en la direcci´on −∇f (P) y el valor m´ınimo es ∂f ∂´ u (P) = −|∇f (P)| . Demostraci´ on: Basta considerar ∂f ∂´ u (P) = ∇f (P) ´ u = |∇f (P)| cos θ 3.3.2. Plano tangente y recta normal Sea z = f(x, y) la ecuaci´ on de una superficie cualquiera. En lo que sigue conviene para mayor comprensi´ on del razonamiento usar la expresi´on F(x, y, z) = 0 para denotar la superficie de nivel de la funci´ on F(x, y, z) = f(x, y) −z = 0 ´ o F(x, y, z) = z −f (x, y) = 0 Sea entonces la superficie F(x, y, z) = 0 y P 0 = (x 0 , y 0 , z 0 ) un punto de ella. Sea − → r (t) = (x(t), y(t), z(t)) la ecuaci´ on param´etrica de una curva C en dicha superficie y que pasa por el punto (x 0 , y 0 , z 0 ). Tomando la diferencial de F(x, y, z) = 0 tenemos ∂F ∂x dx + ∂F ∂y dy + ∂F ∂z dz = 0 En P 0 = (x 0 , y 0 , z 0 ) de la curva donde − → r (t 0 ) = P 0 se tiene 215 ∂F(P 0 ) ∂x x´(t 0 )dt + ∂F(P 0 ) ∂y y´(t 0 )dt + ∂F(P 0 ) ∂z z´(t 0 )dt = 0 =⇒ ∂F(P 0 ) ∂x x´(t 0 ) + ∂F(P 0 ) ∂y y´(t 0 ) + ∂F(P 0 ) ∂z z´(t 0 ) = 0 Es decir _ ∂F(P 0 ) ∂x , ∂F(P 0 ) ∂y , ∂F(P 0 ) ∂z _ (x´(t 0 ), y´(t 0 ), z´(t 0 )) = 0 ∴ _F(P 0 ) − → r´(t 0 ) = 0 Esto nos dice que _F(P 0 ) es perpendicular a − → r´(t 0 ). Esto ocurre para toda curva C que pase por P 0 y como − → r´(t 0 ) es vector tangente a C se tiene que _F(P 0 ) es perpendicular a toda recta tangente a la superficie en P 0 . Por lo tanto, _F(P 0 ) es un vector normal a la superficie en P 0 . Plano Tangente La expresi´on de la forma: ∂F(P 0 ) ∂x (x −x 0 ) + ∂F(P 0 ) ∂y (y −y 0 ) + ∂F(P 0 ) ∂z (z −z 0 ) = 0 es la ecuaci´ on del plano tangente a la superficie F(x, y, z) = 0 en el punto P 0 = (x 0 , y 0 , z 0 ). Si la superficie es z = f(x, y) =⇒ F(x, y, z) = z − f (x, y) = 0 la ecuaci´ on del plano tangente se puede escribir, tambi´en como: z −z 0 = f x (x 0 , y 0 )(x −x 0 ) + f y (x 0 , y 0 )(y −y 0 ) Recta Normal Sean P 0 (x 0 , y 0 , z 0 ) un punto dado y el vector director de esta recta _F(P 0 ) La ecuaci´on vectorial es: 216 (x, y, z) = (x 0 , y 0 , z 0 ) + t( ∂F(P 0 ) ∂x , ∂F(P 0 ) ∂y , ∂F(P 0 ) ∂z ), t ∈ R Ecuaci´ on param´etrica: x = x 0 + t ∂F(P 0 ) ∂x y = y 0 + t ∂F(P 0 ) ∂y , t ∈ R x = z 0 + t ∂F(P 0 ) ∂z Eliminando el par´ametro t obtenemos la ecuaci´on cartesiana: x −x 0 F x (P 0 ) = y −y 0 F y (P 0 ) = z −z 0 F z (P 0 ) Ejemplo: Sea z = e x (cos y + 1) en el punto (0, π 2 , 1) ,calcular las ecuaciones del plano tangente y la recta normal a la superficie. Soluci´on. La componentes del vector gradiente son f x (x, y) = e x (cos y + 1), f y (x, y) = −e x (seny) f x (0, π 2 ) = 1, f y (0, π 2 ) = − 1 Reemplanzando t´erminos en la ecuaci´on del plano tenemos: z −1 = 1 (x −0) + (−1)(y − π 2 ) =⇒ z −1 = x −y + π 2 Finalmente, la ecuaci´on del plano tangente queda 217 x −y −z + π 2 + 1 = 0 y (x, y, z) = (0, π 2 , 1) + t(1, −1, −1) la ecuaci´on de la recta normal en su forma vectorial. 3.3.3. Funci´on Compuesta. La Regla de la Cadena. En este m´odulo abordaremos en forma b´ asica la siguiente problem´atica. Si u es una funci´ on diferenciable de variables x, y, z, , y a su vez estas ´ ultimas son funciones de otras variables nuevas t y/o s ¿podemos encontrar la primera derivada parcial de u con respecto a las nuevas variables t y/o s expresada en t´erminos de las derivadas parciales de las funciones dadas?. As´ı, por ejemplo, si un fen´omeno f´ısico est´ a ocur- riendo digamos en una regi´ on cil´ındrica, resultar´ıa mejor expresar las cantidades que interesen en t´erminos de coordenadas cilindricas y no en cartesianas. Caso particular: Sea f : D ⊆ R 2 −→ R tal que u = f(x, y), tiene dos variables independientes. Supongamos que cada una de estas variables es diferen- ciable de una simple variable independiente t. Si x = x(t) y y = y(t), la derivada de la funci´ on compuesta (regla de la cadena) con respecto a t es du dt = ∂u ∂x dx dt + ∂u ∂y dy dt du dt = ∇u − → r (t) 218 Caso particular de dos variable independientes simples. Sea f : D ⊆ R 2 −→ R tal que u = f(x, y), tiene dos variables independientes. Estas variables a su vez son funciones diferenciables de dos variables simples independientes t y s. La expresi´on de las derivadas de la funci´on compuesta (o regla de la cadena) es: ∂u ∂t = ∂u ∂x dx dt + ∂u ∂y dy dt ∂u ∂s = ∂u ∂x dx ds + ∂u ∂y dy ds Las expresiones anteriores las podemos expresar matricialmente como: _ ∂u ∂t , ∂u ∂s _ = _ ∂u ∂x , ∂u ∂y _ _ _ _ dx dt dy dt dx ds dy ds _ _ _ Ejemplo 1: Hallar dz dt , si z = x y , donde x = e t y y = ln t. Soluci´on. Aplicando la derivaci´ on compuesta, tenemos dz dt = ∂u ∂x dx dt + ∂u ∂y dy dt =⇒ dz dt = 1 y e t + _ − x y 2 _ 1 t dz dt = e t ln t − e t (ln t) 2 1 t = e t _ 1 ln t − 1 t(ln t) 2 _ Ejemplo 2: Hallar ∂u ∂t y ∂u ∂s si u = f(x, y), donde x = t 2 −s 2 , y = e ts Soluci´ on. 219 En este caso ∂u ∂t = f x (x, y) dx dt +f y (x, y) dy dt =⇒ ∂u ∂t = f x (x, y)(2t)+f y (x, y)(s e ts ) Similarmente ∂u ∂s = f x (x, y) dx ds +f y (x, y) dy ds =⇒ ∂u ∂s = f x (x, y)(−2s)+f y (x, y)(t e ts ) De forma m´as general si f : D ⊆ R n −→ R tal que z = f(y 1, y 2, y 3, , y n, ), es funci´on de n variables independientes. Supongamos tambi´en que cada una de estas variables independientes es funci´on diferenciable de otras m variables simples independientes x 1 , x 2 , x 3 , , x m . La expresi´ on de la derivada de la funci´on compuesta (regla de la cadena) es similar, pudiendose escribir cada una de estas derivadas por ∂z ∂x j = n i=1 ∂z ∂y i ∂y i ∂x j , j = 1, 2, ..., m Esta ecuaci´on se puede escribir utilizando matrices, como _ ∂z ∂x 1 , . . . , ∂z ∂x m _ = _ ∂z ∂y 1 , . . . , ∂z ∂y n _ _ _ _ _ _ _ ∂y 1 ∂x 1 . . . ∂y 1 ∂x m . . . . . . ∂y n ∂x 1 ∂y n ∂x m _ _ _ _ _ _ La matriz nm _ ∂y i ∂x j _ i=1...n, j=1...m se denomina matriz jacobiana de la transformaci´on y i = y i (x 1, x 2, x 3, , x m ). 220 Ejemplo 3: Sea f(x, y, z) = x + x 2 y + ze y , en donde x, y, y z , est´ an relacionadas con u y v a trav´es de la transformaci´on x = uv, y = u 2 −v 2 , z = usin v Calcule ∂f ∂u , ∂f ∂v , y ∂ 2 f ∂v∂u , en el punto (u, v) = (2, 2). Soluci´on. La matriz jacobiana para esta transformaci´on es _ _ x u x v y u y v z u z v _ _ = _ _ v u 2u −2v senv u cos v _ _ _ _ x u x v y u y v z u z v _ _ (2, 2) = _ _ 2 2 4 −4 sin 2 2 cos 2 _ _ Para (u, v) = (2, 2), los valores de x, y y z son 4, 0 , y 2sen2 respectivamente.Entonces la matriz regl´ on (f x f y f z ) es (f x f y f z ) = _ 1 + 2xy x 2 + ze y e y _ evaluando en el punto (4, 0, 2sen2) queda _ 1 + 2xy x 2 + ze y e y _ (4, 0, 2sen2) = (1 16 + 2sen2 1) De donde (f u f v ) = (f x f y f z ) _ _ x u x v y u y v z u z v _ _ = (f x x u + f y y u + f z z u f x x v + f y y v + f z z v ) Evaluando (f u f v ) (2, 2) = (66 + 9 sin 2 −62 −8 sin 2 + 2 cos 2) Por lo tanto ∂f ∂u (2, 2) = 66 + 9 sin 2 y ∂f ∂v (2, 2) = −62 −8 sin 2 + 2 cos 2 221 Ahora bien, como f u = f x x u + f y y u + f z z u = (1 + 2xy )v + (x 2 + ze y )2u + e y sin v Se tiene ∂ 2 f ∂v∂u = f uv = 2(xy v + yx v )v + 1 + 2xy + 2(2xx v + ze y y v + e y z v )u + y v e y senv + e y cos v Evaluando en el punto (2, 2) se tiene ∂ 2 f ∂v∂u (2, 2) = 1 −36sen2 + 9 cos 2 3.3.4. Funci´on Impl´ıcita Cuando definimos funciones en forma impl´ıcita decimos, por ejemplo, sea y = ϕ(x) una funci´ on diferenciable definida impl´ıcitamente por medio de la ecuaci´on F(x, y) = 0, o bien, sea z = f(x, y), definida impl´ıcitamente por la ecuaci´ on F(x, y, z) = 0. El primer caso lo podemos ejemplificar con una funci´ on y = ϕ(x) defini- da por la ecuaci´ on de la hip´erbola x 2 −4xy −3y 2 = 9. Muchas veces las ecuaciones que relacionan a las variables pueden ser tan complejas y no lineales, que no es posible esperar encontrar rela- ciones sencillas, si las hay, expl´ıcitas que expresen a una variable en t´erminos de las otras. En t´erminos generales, lo que se pide es que esta relaci´ on entre las variables exista localmente, es decir en alguna vecindad de un punto donde las ecuaciones se satisfacen. La mayor´ıa de las veces no se esperan resultados globales, es decir, no todos los puntos que satisfacen las relaciones impl´ıcitas satisfacen tambi´en las expl´ıcitas. Con el objeto de ilustrar estas ideas analicemos el siguiente ejemplo. Ejemplo: Pruebe que la ecuaci´on z 5 +z +xy = 0 define una funci´ on z = f(x, y) para todos los valores x e y. Soluci´on. 222 Se debe establecer que para cada x e y se puede resolver en forma ´ unica para z la ecuaci´ on de quinto grado. Ya que esto no es obvio, razonamos utilizando la gr´ afica de u = z 5 + z + a gr´ afica pendiente Como u´(z) = 5z 4 +1, ∀a =⇒ u es estrictamente creciente, entonces para cada a existen z 1 y z 2 tal que u(z 1 ) < 0 < u(z 2 ) por lo cual existe un ´ unico z tal que u(z) = 0 para cada a . Si se identifica a con la cantidad xy se ha establecido el hecho planteado. Luego, ”z = f(x, y) se define impl´ıcitamente por la ecuaci´ on z 5 +z + xy = 0” Derivaci´on impl´ıcita Sea F : U ⊆ R 2 → R definida en un conjunto abierto U. Sea P 0 = (x 0 , y 0 ) ∈ U un punto tal que: i) F(x 0 , y 0 ) = 0 ii) ∂F ∂x , ∂F ∂y son continuas en alguna vecindad V δ (P 0 ) y iii) ∂F ∂z (x 0 , y 0 ) ,= 0. Entonces existe una vecindad (x 0 −δ, x 0 +δ) de x 0 , (y 0 −a, y 0 +a) vecindad de y 0 , y una funci´ on ´ unica f (impl´ıcita) de clase C 1 tal que: a) f(x 0 ) = y 0 y f(x) ∈ (y 0 −a, y 0 + a) ∀x ∈ (x 0 −δ, x 0 + δ) b) F(x, f(x)) = 0 ∀x ∈ (x 0 −δ, x 0 + δ) c) ∀x, ∈ V δ (x 0 ) tiene derivadas que pueden calcularse como: 223 dy dx = − F x (x, y) F y (x, y) En el caso de funciones de dos variables definida impl´ıcitamente, el siguiente teorema afirma la existencia de funciones impl´ıcitas bajo cier- tas circunstancias y da f´ ormulas para obtener las derivadas parciales de estas funciones. Teorema 3.3.6. (Teorema de la funci´o impl´ıcita) Sea F : U ⊆ R 3 → R definida en un conjunto abierto U. Sea P 0 = (x 0 , y 0 , z 0 ) ∈ U un punto tal que: i) F(x 0 , y 0 , z 0 ) = 0 ii) ∂F ∂x , ∂F ∂y , ∂F ∂z son continuas en alguna vecindad V δ (P 0 ) y iii) ∂F ∂z (x 0 , y 0 , z 0 ) ,= 0. Entonces existe una vecindad V = V δ (x 0 , y 0 ), y una vecindad (z 0 −a, z 0 +a) de z 0 y una funci´on ´ unica f (impl´ıcita) de clase C 1 sobre V tal que: a) f(x 0 , y 0 ) = z 0 y f(x, y) ∈ (z 0 −a, z 0 + a) ∀(x, y) ∈ V. b) F(x, y, f(x, y)) = 0, ∀(x, y) ∈ V. c) ∀(x, y) ∈ V = V δ (x 0 , y 0 ) tiene derivadas que pueden calcularse como: ∂f(x, y) ∂x = − ∂F ∂x (x, y, f(x, y)) ∂F ∂z (x, y, f(x, y)) ∂f(x, y) ∂y = − ∂F ∂y (x, y, f(x, y)) ∂F ∂z (x, y, f(x, y)) Este teorema se puede generalizar a m´ as variables, de tal modo que si se dan las condiciones exigidas por el teorema y u = f(x, y, z, ) 224 est´ a definida impl´ıcitamente por F(x, y, z , , u) = 0 y F u (x, y, z , , u) ,= 0, entonces: ∂f ∂x = − ∂F ∂x ∂F ∂z , ∂f ∂y = − ∂F ∂y ∂F ∂z ,etc. Ejemplo. Calcular ∂z ∂x y ∂z ∂y si z se define impl´ıcitamente en la ecuaci´ on x 2 y −8xyz = yz + z 3 Soluci´on. Tenemos que: F(x, y, z) = −x 2 y + 8xyz + yz + z 3 Derivando parcialmente con respecto a x , y ,z. F x (x, y, z) = 8yz −2xy, F y (x, y, z) = 8xz + z −x 2 , F z (x, y, z) = 8xy + y + 3z 2 Claramente estas derivadas son continuas en R 3 , son funciones polin´ omicas, Por consiguiente, F pertenece a C 1 . Para todo (x, y, z) en que F z (x, y, z) ,= 0 se tiene. ∂z ∂x = − 8yz −2xy 8xy + y + 3z 2 ∂z ∂y = − 8xz + z −x 2 8xy + y + 3z 2 225 3.3.5. Jacobiano Si f 1 , f 2 , f 3, . . . , f n son funciones diferenciables de R n en R y si (x 1 , x 2 , x 3 , . . . , x n ) ∈ D su dominio tal que f 1 = f 1 (x 1 , x 2 , x 3 . . . , x n ) f 2 = f 2 (x 1 , x 2 , x 3 . . . , x n ) . . . f n = f n (x 1 , x 2 , x 3 . . . , x n ) El Jacobiano de las funciones f 1 , f 2 , f 3, . . . , f n , respecto de las variables x 1 , x 2 , x 3 . . . , x n se define por el determinante de las primeras derivadas parciales y se denota ∂(f 1 , f 2 , f 3, . . . , f n ) ∂(x 1 , x 2 , x 3 . . . , x n ) , ∂(f 1 , f 2 , f 3, . . . , f n ) ∂(x 1 , x 2 , x 3 . . . , x n ) = ¸ ¸ ¸ ¸ ¸ ¸ ¸ ¸ ¸ ∂f 1 ∂x 1 ∂f 1 ∂x 2 . . . ∂f 1 ∂x n ∂f 2 ∂x 1 ∂f 2 ∂x 2 . . . ∂f 2 ∂x n . . . ∂f n ∂x 1 ∂f n ∂x 2 . . . ∂f n ∂x n ¸ ¸ ¸ ¸ ¸ ¸ ¸ ¸ ¸ Dos funciones definidas impl´ıcitamente Teorema 3.3.7. Sean F , G: U ⊆ R 4 → R definidas en un conjunto abierto F(x, y, u, v) y G(x, y, u, v). Sea P 0 = (x 0 , y 0 , u 0 , v 0 ) ∈ U un punto tal que: i) F(x 0 , y 0 , u 0 , v 0 ) = 0 y G(x 0 , y 0 , u 0 , v 0 ) = 0 ii) F y G tienen derivadas parciales continuas en alguna vecindad V δ (P 0 ) y iii) ∂ (F, G) ∂ (u, v) (x 0 , y 0 , u 0 , v 0 ) ,= 0. Entonces las ecuaciones F(x, y, u, v) = 0 , G(x, y, u, v) = 0 definen funciones impl´ıcitas u = u(x, y), v = v(x, y) en alguna vecindad V δ (x 0 , y 0 ), las cuales tienen primeras derivadas parciales continuas con respecto a cada una de las variables, tal que : a) u 0 = u(x 0 , y 0 ), v 0 = v(x 0 , y 0 ). b) F(x, y, u (x, y) , v (x, y)) = 0 , G(x, y, u (x, y) , v (x, y)) = 0, V δ (x 0 , y 0 ) 226 c) Las derivadas parciales de u y v en V δ (P 0 ) est´an dadas por ∂u ∂x = − ∂(F, G) ∂(x, v) ∂(F, G) ∂(u, v) , ∂u ∂y = − ∂(F, G) ∂(y, v) ∂(F, G) ∂(u, v) ∂v ∂x = − ∂(F, G) ∂(u, x) ∂(F, G) ∂(u, v) , ∂v ∂y = − ∂(F, G) ∂(u, y) ∂(F, G) ∂(u, v) Ejemplo Calcule las derivadas parciales ∂u ∂x y ∂v ∂x donde u y v est´ an definidas por las ecuaciones impl´ıcitas x 2 + 2uv = 1, x 3 −u 3 + v 3 = 1 Soluci´on. En este caso F(x, u, v) = x 2 + 2uv −1 = 0 y G(x, u, v) = x 3 −u 3 + v 3 −1 = 0 ∂(F, G) ∂(u, v) = ¸ ¸ ¸ ¸ 2v 2u −3u 2 3v 2 ¸ ¸ ¸ ¸ = 6(u 3 + v 3 ) ∂(F, G) ∂(u, v) ,= 0 para todo (x, u, v) donde u ,= 0 y v ,= 0. ∂(F, G) ∂(x, v) = ¸ ¸ ¸ ¸ 2x 2u 3x 2 3v 2 ¸ ¸ ¸ ¸ = 6x(v 2 −xu) ∂(F, G) ∂(u, x) = ¸ ¸ ¸ ¸ 2v 2x −3u 2 3x 2 ¸ ¸ ¸ ¸ = 6x(xv + u 2 ) Entonces, derivando u y v con respecto a x, obtenemos ∂u ∂x = − 6x(v 2 −xu) 6(u 3 + v 3 ) = x(xu −v 2 ) u 3 + v 3 y 227 ∂v ∂x = − 6x(xv + u 2 ) 6(u 3 + v 3 ) = − x(xv + u 2 ) u 3 + v 3 3.3.6. M´aximos y M´ınimos M´ aximos y M´ınimos (extremos locales o relativos) para funciones de dos o m´ as variables. Sea f : U ⊆ R 2 →R, U conjunto abierto. i) f tiene un m´ aximo local en x 0 ∈ U si f(x 0 ) ≥ f(x) ∀x ∈ B δ (x 0 , y 0 ) ii) f tiene un m´ınimo local en x 0 ∈ U si f(x 0 ) ≤ f(x) ∀x ∈ B δ (x 0 , y 0 ) Ejemplo 1: La funci´ on f(x, y) = 4 −x 2 −y 2 tiene un m´aximo local en (0, 0), pues ∆f = f(0,0) − f(x, y) = x 2 + y 2 ≥ 0 ya que x 2 ≥ 0, y 2 ≥ 0 ∀(x, y) ∈ B δ (0, 0). Ejemplo 2: La funci´on z = f(x, y) = x 2 + y 2 Tiene un m´ınimo local en (0, 0) : f(0, 0) ≤ x 2 + y 2 = f(x, y) f(0, 0) = 0 es m´ınimo absoluto. Ejemplo 3: La funci´on z = f(x, y) = 1 − 3 _ x 2 + y 2 f(0, 0) = 1 es m´ınimo local y absoluto. Observaci´ on: Si las desigualdades son v´ alidas en todo U se tendr´ a ex- tremo absolutos. Punto Cr´ıtico: (Condici´on necesaria) Definici´ on. Un punto del dominio de f es un punto cr´ıtico si todas las derivadas parciales de f son cero en el punto o f no es diferenciable en el punto. Caso particular: Si f : R 2 →R, (x 0 , y 0 ) es punto cr´ıtico de f si y solo si : i) ∂f(x 0 , y 0 ) ∂x = 0, ∂f(x 0 , y 0 ) ∂y = 0 228 ´ o ii) ∂f(x 0 , y 0 ) ∂x , y/o ∂f(x 0 , y 0 ) ∂y Ejemplo : Sea z = f(x, y) = x 2 −y 2 =⇒ ∂f ∂x = 2x ∂f ∂y = −2y =⇒(0, 0) punto cr´ıtico. Punto silla: Definici´ on : Sea punto (x 0 , y 0 ) ∈ Dom(f) . Si cualquier B δ (x 0 , y 0 ) con- tiene puntos (x, y) ∈ B δ (x 0 , y 0 ) tales que f(x, y)− f(x 0 , y 0 ) > 0 y puntos (x, y) ∈ B δ (x 0 , y 0 ) tales que f(x, y)− f(x 0 , y 0 ) < 0 se llama punto ensilladura. Nota. Un punto cr´ıtico en que f no es m´ aximo ni m´ınimo se llama punto silla. Ejemplo: En la funci´on f(x, y) = x 2 −y 2 , (0, 0) es punto ensilladura, pues ∆f = f(0, 0) −f(x, y) = −x 2 +y 2 no se puede decir nada a´ un del signo, pero para ∆f = f(0,0) − f(x, 0) = −x 2 ≤ 0 y ∆f = f(0, 0) − f(0, y) = y 2 ≥ 0, permite concluir que (0, 0) es punto ensilladura. Observaci´ on: Las definiciones anteriores son extensibles a funciones de m´as variables. Teorema 3.3.8. Sea f una funci´on continua de dos variables definida en una regi´on cerrada y acotada R del plano XY. Entonces: a.) Al menos hay un punto (x 0 , y 0 ) ∈ R en que f alcanza su valor m´ınimo. b.) Al menos hay un punto (x 0 , y 0 ) ∈ R en que f alcanza su valor m´aximo. Observaci´ on: Este Teorema es extensible a m´ as variables. Definici´ on: Sea f : U ⊆ R n → R funci´ on definida en un conjunto abierto U ∈ R n y sea x ∈ U. Suponga que todas las derivadas parciales de segundo orden existen en x. A la matriz cuadrada de orden n. 229 A = (a ij ) i,j=1,...,n donde a ij = ∂ 2 f ∂x i ∂x j (x) x = (x 1 , x 2 , ..., x n ) se llama matriz Hessiana (o simplemente Hessiano) de la funci´on f en x y se denota H(x). Caso particular: Para f(x, y) H(x, y) = _ ∂ 2 f ∂x 2 ∂ 2 f ∂y∂x ∂ 2 f ∂x∂y ∂ 2 f ∂y 2 _ Caso particular: Para f(x, y, z) H(x, y, z) = _ _ _ ∂ 2 f ∂x 2 ∂ 2 f ∂y∂x ∂ 2 f ∂z∂x ∂ 2 f ∂x∂y ∂ 2 f ∂y 2 ∂ 2 f ∂z∂y ∂ 2 f ∂x∂z ∂ 2 f ∂y∂z ∂ 2 f ∂z 2 _ _ _ Definici´ on. Sea la matriz A n = _ _ _ _ _ a 11 a 12 . . . a 1n a 21 . . . . . . . . . a n1 a nn _ _ _ _ _ Consideremos las submatrices A k de A n (k = 1, 2, . . . , n) definidas de las siguientes maneras: A 1 = (a 11 ) , A 2 = _ a 11 a 12 a 21 a 22 _ , A 3 = _ _ a 11 a 12 a 13 a 21 a 22 a 23 a 31 a 32 a 33 _ _ , , A n = _ _ _ _ _ a 11 a 12 . . . a 1n a 21 . . . . . . . . . a n1 a nn _ _ _ _ _ 230 Teorema 3.3.9. (Consideraciones suficientes para la existencia de ex- tremos locales) Sea f : U ⊆ R n →R una funci´on definida en un conjunto abierto U de R n y x un punto cr´ıtico de f (es decir ∇f(x) = − → 0 ) y supongamos que las derivadas parciales de segundo orden son continuas en una vecindad abierta de x. Entonces: a.) Si todas las submatrices A k de la matriz Hessiana H(x) definidas de la forma anterior tienen determinante positivo, f tiene un m´ınimo local en x. b.) Si todas las submatrices A k de la matriz Hessiana H(x), definidas de la forma anterior, tienen determinantes de signo alternado comenzando por A 1 < 0, f tiene un m´aximo local en x. Ejemplo 1: Sea f : R 2 → R definida por f(x, y) = x 2 + 3y 2 − 2x − 12y + 13. Determine los valores extremos de f. Soluci´on Aplicando la condici´on necesaria de punto cr´ıtico, obtenemos ∇f(x, y) = (2x−2, 6y−12) = (0, 0) =⇒(1, 2) es el ´ unico punto cr´ıtico. Calculemos la matriz Hessiana H(x, y) = _ 2 0 0 6 _ Examinemos los determinantes de las submatrices A 1 = (2) =⇒det A 1 = 2 > 0 A 2 = _ 2 0 0 6 _ =⇒det A 2 = 12 > 0 ∴ f tiene m´ınimo local en (1, 2). Ejemplo 2: Sea f : R 2 →R definida por f(x, y) = (x 2 + 3y 2 ) e 1− ( x 2 +y 2 ) . ¿Cu´ ales son los puntos cr´ıticos de f? Soluci´on. De la condici´ on de punto cr´ıtico tenemos: ∂f ∂x = 2xe 1− ( x 2 +y 2 ) −2x (x 2 + 3y 2 ) e 1− ( x 2 +y 2 ) = 0 =⇒ 231 ∂f ∂x = e 1− ( x 2 +y 2 ) (2x −2x 3 −6xy 2 ) = 0 ∂f ∂y = 6ye 1− ( x 2 +y 2 ) −2y (x 2 + 3y 2 ) e 1− ( x 2 +y 2 ) = 0 =⇒ ∂f ∂y = e 1− ( x 2 +y 2 ) (6y −6y 3 −2x 2 y) = 0 De donde obtenemos, el sistema de ecuaciones x −x 3 −3xy 2 = 0 3y −3y 3 −x 2 y = 0 _ =⇒ x (1 −x 2 −3y 2 ) = 0 y (3 −3y 2 −x 2 ) = 0 _ =⇒ Puntos Cr´ıticos: (0, 0), (0, 1), (0, −1), (1, 0), (1, 0), (−1, 0) Determine en qu´e puntos hay m´aximos, m´ınimos o puntos sillas. Ejemplo 3: Sea f : R 4 − ¦0, 0, 0, 0¦ → R y f(x, y, z, u) = x + y x + z y + u z + 1 u Determine los valores extremos de f. Soluci´ on Determinemos los puntos cr´ıticos de la funci´ on f ∂f ∂x = 1 − y x 2 = 0 ∂f ∂y = 1 x − z y 2 = 0 ∂f ∂z = 1 y − u z 2 = 0 ∂f ∂y = 1 z − 1 u 2 = 0 =⇒(1, 1, 1, 1) Punto Cr´ıtico. La matriz Hessiana est´ a definida como H(x, y, z, u) = _ _ _ _ _ _ ∂ 2 f ∂x 2 ∂ 2 f ∂y∂x ∂ 2 f ∂z∂x ∂ 2 f ∂u∂x ∂ 2 f ∂x∂y ∂ 2 f ∂y 2 ∂ 2 f ∂z∂y ∂ 2 f ∂u∂y ∂ 2 f ∂x∂z ∂ 2 f ∂y∂z ∂ 2 f ∂z 2 ∂ 2 f ∂u∂z ∂ 2 f ∂x∂u ∂ 2 f ∂y∂u ∂ 2 f ∂z∂u ∂ 2 f ∂u 2 _ _ _ _ _ _ 232 H(x, y, z, u) = _ _ _ _ 2y x 3 − 1 x 2 0 0 − 1 x 2 2z y 3 − 1 y 2 0 0 − 1 y 2 2u z 3 − 1 z 2 0 0 − 1 z 2 2 u 3 _ _ _ _ H(1, 1, 1, 1) = _ _ _ _ 2 −1 0 0 −1 2 −1 0 0 −1 2 −1 0 0 −1 2 _ _ _ _ ´ 1 = 2, ´ 2 = 3, ´ 3 = 2, ´ 4 = 5 ∴Hay un m´ınimo local en (1, 1, 1, 1). Teorema 3.3.10. (Condiciones suficientes para la existencia de ex- tremos locales en dos variables.) Sea f : U ⊆ R 2 →R una funci´on de dos variables que tiene derivadas parciales de segundo orden continuas en el conjunto abierto U. Sea det(H (x, y)) = ¸ ¸ ¸ ¸ f xx f xy f yx f yy ¸ ¸ ¸ ¸ (x, y) ∀ (x, y) ∈ U. Sea (x 0 , y 0 ) ∈ U tal que ∇f (x 0 , y 0 ) = 0. Entonces i) Si det(H (x, y)) > 0 y f xx (x 0 , y 0 ) < 0, f tiene una m´aximo local en (x 0 , y 0 ) . ii) Si det(H (x, y)) > 0 y f xx (x 0 , y 0 ) > 0, f tiene un punto silla en (x 0 , y 0 ) . iii) Si det(H (x, y)) = 0 no podemos concluir nada. 3.3.7. Extremos Restringidos Pensemos, que se tiene una funci´ on f(x, y) sujeta a cierta condici´ on g(x, y) = 0. Se quiere maximizar o minimizar f(x, y) con la condici´on que (x, y) satisfaga la ecuaci´ on g(x, y) = 0. En este caso podemos elaborar una respuesta aplicando la condici´ on necesaria de valor extremo para f(x, y) df dx = ∂f ∂x + ∂f ∂y dy dx = 0 233 Aplicando el teorema de la funci´on impl´ıcita podemos calcular dy dx a partir de la restricci´ on: g(x, y(x)) = 0 =⇒ ∂g ∂x + ∂g ∂y dy dx = 0 =⇒ dy dx = − ∂g ∂x ∂g ∂y Sustituyendo en la primera ecuaci´ on tenemos: df dx = ∂f ∂x + ∂f ∂y _ − ∂g ∂x ∂g ∂y _ = 0 , ∂g ∂y ,= 0 =⇒ ∂f ∂x ∂g ∂y + ∂f ∂y ∂g ∂x = 0 =⇒ ∂(f, g) ∂(x, y) = 0 Lo que implica que los puntos cr´ıticos deben cumplir dos condiciones: g(x, y) = 0. ∂(f, g) ∂(x, y) = 0 Adem´ as, la decisi´ on de valor extremo resulta del signo de d 2 f dx 2 en cada punto cr´ıtico, ya que f(x, y) = f(x, y(x)) = f(x). Ejemplo 1. Obtenga las dimensiones de un rect´ angulo de per´ımetro dado que determinan la mayor y la menor ´ area de ´este. Soluci´on. Si x e y denotan las longitudes de los lados del rect´ angulo, el problema consistir´ a en buscar los extremos de la funci´ on f(x, y) = xy, x, y > 0 234 sujeta a la restricci´ on g(x, y) = x+y− L 2 = 0 , donde L es el per´ımetro dado. Entonces aplicando las condiciones necesaria para obtener los puntos cr´ıticos tenemos: g(x, y) = x + y − L 2 = 0. ∂(f, g) ∂(x, y) = _ y x 1 1 _ =⇒ y = x Sustituyendo el resultado de la segunda ecuaci´on en la primera, se tiene: x = y = L 4 Luego hemos encontrado que la funci´ on f tiene un ´ unico punto cr´ıtico en P = _ L 4 , L 4 _ . Adem´ as, determinemos la naturaleza del punto usando el criterio de la segunda derivada f(x, y (x)) = xy =⇒ df dx = y + x dy dx g(x, y) = x + y − L 2 = 0 =⇒ dy dx = −1 ∴ df dx = y −x d 2 f dx 2 = dy dx −1 = −2 < 0 Por tanto, en el punto ( L 4 , L 4 , L 2 16 ) hay un m´aximo relativo sobre la curva de intersecci´ on de las dos superficies z = xy y x + y = L 2 La situaci´on se puede generalizar para funciones de m´ as variables o m´as condiciones. 235 Por otra parte, para resolver este mismo tipo de problemas de m´aximo y m´ınimo podemos utilizar otro m´etodo que se fundamenta en el siguiente teorema. Teorema 3.3.11. (Multiplicadores de Lagrange) Sean f : U ⊆ R n →R, g : U ⊆ R n →R funciones de C 1 . Sean x 0 ∈ U tal que g(x 0 ) = 0 y S = ¦x ∈ U : g(x) = 0¦ supongamos adem´as que ∇g(x 0 ) ,= 0. Si f[ S (f restringida a S) tiene un m´aximo o m´ınimo local en S, en x 0 ,entonces existe λ ∈ R tal que: ∇f(x 0 ) + λ∇g(x 0 ) = 0 Nota: Lo anterior significa que x 0 es un punto cr´ıtico de f[ S . M´etodo: Construir F(x, y, λ) = f +λg y determinar puntos cr´ıticos de F Ejemplo 1. Utilice el procedimiento que se origina a partir de este teorema para obtener el m´ aximo local del ejemplo anterior. Soluci´ on. En este caso f(x, y) = xy, x, y > 0 sujeta a la condici´ on: g(x, y) = x + y − L 2 = 0 . Sea F(x, y, λ) = xy + λ(x + y − L 2 ), entonces F x = y + λ = 0 F y = x + λ = 0 F λ = x + y − L 2 = 0 de aqu´ı x = −λ, y = −λ y sustituyendo en la tercera ecuaci´on: −λ −λ − L 2 = 0 =⇒λ = − L 4 . El punto cr´ıtico es (x 0 , y 0 ) = ( L 4 , L 4 ). El punto ( L 4 , L 4 , L 2 16 ) es el m´ aximo relativo sobre la curva de intersecci´on de las dos superficies z = xy y x + y = L 2 236 Ejemplo 2. De todos los rect´ angulos inscritos en la elipse x 2 a 2 + y 2 b 2 = 1, a > 0 y b > 0 con lados paralelos al los ejes, determine el de mayor ´ area. Soluci´on. En este caso f(x, y) = 4xy, x, y > 0 y g(x, y) = x 2 a 2 + y 2 b 2 −1 = 0 Sea F(x, y, λ) = 4xy + λ _ x 2 a 2 + y 2 b 2 −1 = 0 _ , entonces F x = 4y + 2λ x a 2 = 0 =⇒ λ = − 2a 2 y x F y = 4x + 2λ y b 2 = 0 =⇒ λ = − 2b 2 x y ∴ x 2 a 2 = y 2 b 2 Reemplazando esta ´ ultima expresi´ on en la ecuaci´on F λ = x 2 a 2 + y 2 b 2 −1 = 0 =⇒ obtenemos a un ´ unico punto cr´ıtico: _ a √ 2 , b √ 2 _ Adem´ as, determinemos la naturaleza del punto usando el criterio de la segunda derivada f(x, y (x)) = 4xy =⇒ df dx = 4y + 4x dy dx =⇒ d 2 f dx 2 = 4 dy dx + 4x d 2 y dx 2 g(x, y) = x 2 a 2 + y 2 b 2 −1 = 0 =⇒ dy dx = − b 2 x a 2 y =⇒ d 2 y dx 2 = − b 2 a 2 _ 1 y + b 2 a 2 x 2 y 3 _ 237 Evaluando segunda derivada en el punto P _ a √ 2 , b √ 2 _ , pro- duce d 2 f dx 2 = −8 b 2 x a 2 y −4 b 4 a 4 _ x 3 y 3 _ =⇒ d 2 f dx 2 = −12 b a < 0 Por tanto, el punto _ a √ 2 , b √ 2 _ hay m´ aximo relativo cuyo valor es f _ a √ 2 , b √ 2 _ = 2ab Ejemplo 3: Obtenga los extremos posibles de f(x, y) = x 2 +24xy + 8y 2 con la restricci´ on x 2 + y 2 −25 = 0. Soluci´on: Sea F(x, y, λ) = x 2 + 24xy + 8y 2 + λ(x 2 + y 2 −25) . F x = 2x + 24y + 2λx = 0 =⇒ (1 + λ)x + 12y = 0 (1) F y = 24x + 16y + 2λy = 0 =⇒ 12x + (8 + λ)y = 0 (2) F λ = x 2 + y 2 −25 = 0 (3) Multiplicando la ecuaci´on (1) por 12 y (3) por(1 + λ) produce. 12(1 + λ)x + 144y = 0 12(1 + λ)x + (8 + λ)(1 + λ)y = 0 Restando ambas ecuaciones =⇒λ 2 + 9λ −136 = 0 =⇒λ 1 = 8; λ 2 = −17 Si λ = 8 =⇒ 18x + 24y = 0 24x + 32y = 0 _ =⇒3x + 4y = 0 =⇒y = − 3 4 x x 2 + _ 3 4 x _ 2 = 25 =⇒ x = ±4 Lo que da los puntos cr´ıticos (±4, ∓3) 238 Si λ = −17 en forma similar se obtiene que (±3, ±4) son puntos cr´ıticos. Localizaci´on del m´aximo y el m´ınimo absoluto de f sobre un dominio compacto. Para determinar m´ aximos y m´ınimos absolutos e una regi´ on cerrada y acotada encerrada por una curva suave, se debe: • Determinar todos los puntos cr´ıticos en el interior de la regi´on. • Usar Lagrange para determinar puntos cr´ıticos en la frontera. • Evaluar f en los puntos, cr´ıticos. • Comparar todos estos valores y seleccionar el m´ aximo y el m´ıni- mo. Criterio de la Segunda Derivada para Extremos Restringidos. Sea f(x, y) funci´on a maximizar o minimizar y g(x, y) = 0 la condici´on: S = ¦(x, y) ∈ R 2 [ g(x, y) = 0¦ .Sea F(x, y, λ) = f(x, y) + λg(x, y) Sea (x 0 , y 0 , λ) punto cr´ıtico de F ∂f ∂x [ (x 0 ,y 0 ) +λ ∂g ∂x [ (x 0 ,y 0 ) = 0 ∂f ∂y [ (x 0 ,y 0 ) +λ ∂g ∂y [ (x 0 ,y 0 ) = 0 g (x 0 , y 0 ) = 0 Se desea utilizar el criterio de la segunda derivada para identificar ex- tremos locales de la funci´on f(x, y) bajo la condici´on g (x, y) = 0 lo que conduce encontrar los m´aximos y m´ınimos de funciones de una variable f(x, y(x)) . De la condici´on, calculemos y = y(x) y reemplacemos en f [ S . Aqu´ı podemos considerar f como funci´ on de una variable, es decir f(x, y) = f(x, y(x)) en S. Nos proponemos calcular ∂ 2 f ∂x 2 df dx = ∂f ∂x + ∂f ∂y dy dx 239 d 2 f dx 2 = ∂ 2 f ∂x 2 + ∂ 2 f ∂y∂x dy dx + ∂ 2 f ∂x∂y dy dx + ∂ 2 f ∂y 2 _ dy dx _ 2 + ∂f ∂y d 2 y dx 2 = ∂ 2 f ∂x 2 + 2 ∂ 2 f ∂y∂x dy dx + ∂ 2 f ∂y 2 _ dy dx _ 2 + ∂f ∂y d 2 y dx 2 g(x, y(x)) = 0 =⇒ ∂g ∂x + ∂g ∂y dy dx = 0 =⇒ dy dx = − ∂g ∂x ∂g ∂y Derivando nuevamente g respecto de x ∂ 2 g ∂x 2 + ∂ 2 g ∂y∂x dy dx + ∂ 2 g ∂x∂y dy dx + ∂ 2 g ∂y 2 _ dy dx _ 2 + ∂g ∂y d 2 y dx 2 = 0 Sustituyendo dy dx se tiene: ∂ 2 g ∂x 2 + 2 ∂ 2 g ∂y∂x _ − ∂g ∂x ∂g ∂y _ + ∂ 2 g ∂y 2 _ − ∂g ∂x ∂g ∂y _ 2 + ∂g ∂y d 2 y dx 2 = 0 / _ ∂g ∂y _ ∂ 2 g ∂x 2 ∂g ∂y −2 ∂ 2 g ∂y∂x ∂g ∂x + ∂ 2 g ∂y 2 _ ∂g ∂x _ 2 1 _ ∂g ∂y _ 2 + _ ∂g ∂y _ 2 d 2 y dx 2 = 0 =⇒ d 2 y dx 2 = 1 _ ∂g ∂y _ 2 _ − ∂ 2 g ∂x 2 ∂g ∂y + 2 ∂ 2 g ∂y∂x ∂g ∂x − ∂ 2 g ∂y 2 _ ∂g ∂x _ 2 1 ∂g ∂y _ Reemplazando en d 2 f dx 2 240 d 2 y dx 2 = ∂ 2 f ∂x 2 −2 ∂ 2 f ∂y∂x _ ∂g ∂x ∂g ∂y _ + ∂ 2 f ∂y 2 _ ∂g ∂x ∂g ∂y _ 2 + ∂f ∂y 1 _ ∂g ∂y _ 2 _ − ∂ 2 g ∂x 2 ∂g ∂y + 2 ∂ 2 g ∂y∂x ∂g ∂x − ∂ 2 g ∂y 2 _ ∂g ∂x _ 2 1 ∂g ∂y _ y 0 = y(x 0 ), ∂f ∂x [ x 0 = −λ ∂g ∂x [ x 0 ; ∂f ∂y [ x 0 = −λ ∂g ∂y [ x 0 Luego (x 0 , y 0 ) , ∂f ∂x [ x 0 ∂g ∂x [ x 0 = −λ, ∂f ∂y [ x 0 ∂g ∂y [ x 0 = −λ d 2 f dx 2 = 1 _ ∂g ∂y _ 2 _ ∂ 2 f ∂x 2 _ ∂g ∂y _ 2 −2 ∂ 2 f ∂y∂x ∂g ∂x ∂g ∂y + ∂ 2 f ∂y 2 _ ∂g ∂x _ 2 +λ ∂ 2 g ∂x 2 _ ∂g ∂y _ 2 −2λ ∂ 2 g ∂y∂x ∂g ∂x ∂g ∂y + λ ∂ 2 g ∂y 2 _ ∂g ∂x _ 2 _ = 1 _ ∂g ∂y _ 2 _ _ ∂ 2 f ∂x 2 + λ ∂ 2 g ∂x 2 _ _ ∂g ∂y _ 2 −2 _ ∂ 2 f ∂y∂x + λ ∂ 2 g ∂y∂x _ ∂g ∂x ∂g ∂y + _ ∂ 2 f ∂y 2 + λ ∂ 2 g ∂y 2 _ _ ∂g ∂x _ 2 _ Podemos escribir entonces: d 2 f dx 2 = 1 _ ∂g ∂y _ 2 _ ∂ 2 F ∂x 2 _ ∂g ∂y _ 2 −2 ∂ 2 F ∂y∂x ∂g ∂x ∂g ∂y + ∂ 2 F ∂y 2 _ ∂g ∂x _ 2 _ En el par´entesis, cuadrado se tiene 241 ∗ = ∂g ∂x ¸ ¸ ¸ ¸ ¸ ∂g ∂x ∂g ∂y ∂ 2 F ∂y∂x ∂ 2 F ∂y 2 ¸ ¸ ¸ ¸ ¸ + ∂g ∂y ¸ ¸ ¸ ¸ ¸ ∂g ∂y ∂g ∂x ∂ 2 F ∂y∂x ∂ 2 F ∂x 2 ¸ ¸ ¸ ¸ ¸ = − ¸ ¸ ¸ ¸ ¸ ¸ ¸ 0 ∂g ∂x ∂g ∂y ∂g ∂x ∂ 2 F ∂x 2 ∂ 2 F ∂y∂x ∂g ∂y ∂ 2 F ∂y∂x ∂ 2 F ∂y 2 ¸ ¸ ¸ ¸ ¸ ¸ ¸ = − ¸ ¸ ¸ ¸ ¸ ¸ ¸ 0 ∂g ∂x ∂g ∂y ∂g ∂x ∂ 2 F ∂x 2 ∂ 2 F ∂y∂x ∂g ∂y ∂ 2 F ∂y∂x ∂ 2 F ∂y 2 ¸ ¸ ¸ ¸ ¸ ¸ ¸ De lo anterior se plantea la siguiente definici´on Si f y g son funciones como las definidas antes y F = f +λg. Llamamos hessiano limitado de la funci´on F a H F = ¸ ¸ ¸ ¸ ¸ ¸ ¸ 0 ∂g ∂x ∂g ∂y ∂g ∂x ∂ 2 F ∂x 2 ∂ 2 F ∂y∂x ∂g ∂y ∂ 2 F ∂y∂x ∂ 2 F ∂y 2 ¸ ¸ ¸ ¸ ¸ ¸ ¸ en (x 0 , y 0 ) De tal modo que d 2 f dx 2 = − H F (x 0 , y 0 ) _ ∂g ∂y (x 0 , y 0 ) _ 2 Teorema en extremos restringidos Consideremos nuevamente d 2 f dx 2 = − H F (x 0 , y 0 ) _ ∂g ∂y (x 0 , y 0 ) _ 2 De esta expresi´ on, dado que el signo de la segunda derivada d 2 f dx 2 depende s´ olo del determinante Hessiano limitado, ya que _ ∂g ∂y (x 0 , y 0 ) _ 2 > 0, inferimos el siguiente criterio de la segunda derivada para extremos restringidos. a) Si H F (x 0 , y 0 ) > 0, entonces d 2 f dx 2 < 0 y la funci´on tiene un m´ aximo local condicionado en (x 0 , y 0 ). 242 b) Si H F (x 0 , y 0 ) < 0, entonces d 2 f dx 2 > 0 y la funci´ on tiene un m´ınimo local condicionado en (x 0 , y 0 ) . c) Si H F (x 0 , y 0 ) = 0 no hay informaci´ on del punto(x 0 , y 0 ). Ejemplo: Hallar los extremos de f(x, y) = (x − y) 2 sujeta a la restricci´ on x 2 + y 2 −1 = 0 Soluci´on: Sea F(x, y, λ) = (x −y) 2 + λ(x 2 + y 2 −1) F x = 2 (x −y) + 2λx = 0 F y = −2 (x −y) + 2λy = 0 _ =⇒2λ(x + y) = 0 F λ = x 2 + y 2 = 1 a) Si escogemos λ = 0 , y (x + y) ,= 0 obtenemos los puntos cr´ıticos de f libre de restricci´on. b) Si λ ,= 0 =⇒ y = −x. Reemplazando en la tercera ecuaci´on , se obtienen dos puntos cr´ıticos de la funci´ on f sujetos a la restricci´ on. P 1 _ 1 √ 2 , − 1 √ 2 _ y P 2 _ − 1 √ 2 , 1 √ 2 _ Determinemos el Hessiano l´ımitado para decidir la naturaleza de estos puntos cr´ıticos H F (x, y) = ¸ ¸ ¸ ¸ ¸ ¸ 0 2x 2y 2x 2 + 2λ −2 2y −2 2 + 2λ ¸ ¸ ¸ ¸ ¸ ¸ = −8 _ x 2 + 2xy + y 2 _ −8λ _ x 2 + y 2 _ Sustituyendo y = −x en la primera ecuaci´on se tiene λ = −2 ,= 0,entonces el Hessiano se reduce a H F (x, y) = −8 _ x 2 + 2xy + y 2 _ + 16 _ x 2 + y 2 _ = 8 _ x 2 −2xy + y 2 _ Evaluando en el primer punto cr´ıticos produce 243 H F _ 1 √ 2 , − 1 √ 2 _ = 16 > 0 entonces d 2 f dx 2 _ 1 √ 2 , − 1 √ 2 _ < 0, luego en el punto _ 1 √ 2 , − 1 √ 2 _ la funci´ on f tiene un m´ aximo local condicionado. Por simetr´ıa, el segundo punto tambi´en corresponde a un m´ aximo local condicionado de f. El criterio de la segunda derivada para extremos restringidos se puede generalizar para funciones de m´ as de dos variables. En el caso de una funci´ on de tres variables x, y, z y sujeta a la sola restricci´ on g(x, y, z) = 0, formamos el Hessiano correspondiente a F(x, y, z, λ) = f(x, y, z) + λg (x, y, z) de la siguiente forma H F = ¸ ¸ ¸ ¸ ¸ ¸ ¸ ¸ ¸ 0 ∂g ∂x ∂g ∂y ∂g ∂z ∂g ∂x ∂ 2 F ∂x 2 ∂ 2 F ∂y∂x ∂ 2 F ∂x∂z ∂g ∂y ∂ 2 F ∂y∂x ∂ 2 F ∂y 2 ∂ 2 F ∂y∂z ∂g ∂z ∂ 2 F ∂x∂z ∂ 2 F ∂y∂z ∂ 2 F ∂z 2 ¸ ¸ ¸ ¸ ¸ ¸ ¸ ¸ ¸ y sea A 3 = ¸ ¸ ¸ ¸ ¸ ¸ ¸ 0 ∂g ∂x ∂g ∂y ∂g ∂x ∂ 2 F ∂x 2 ∂ 2 F ∂y∂x ∂g ∂y ∂ 2 F ∂y∂x ∂ 2 F ∂y 2 ¸ ¸ ¸ ¸ ¸ ¸ ¸ El criterio de la segunda derivada en este caso expresa: Sea (x 0 , y 0 , z 0 ) punto cr´ıtico. a) Si H F < 0 y A 3 > 0,entonces la funci´on f tendr´ a un m´ aximo condicionado en (x 0 , y 0 , z 0 ). b) Si H F < 0 y A 3 < 0,entonces la funci´on f tendr´ a un m´ınimo condicionado en (x 0 , y 0 , z 0 ). c) Si H F > 0,entonces la funci´on f no tiene extremos condicionado en (x 0 , y 0 , z 0 ). d.) Si H F = 0, no hay informaci´on acerca del punto (x 0 , y 0, z 0 ). 244 Ejemplo Hallar los extremos de la funci´ on f(x, y, z) = x 2 + y 2 + z 2 sujeta a la restrici´ on g (x, y, z) = z 2 + 2x −y 2 −1 = 0. Soluci´on: Sea la funci´ on F = x 2 + y 2 + z 2 + λ(z 2 + 2x −y 2 −1) F x = 2x + 2λ = 0 =⇒ 2 (x + λ) = 0 =⇒x = −λ F y = 2y −2λy = 0 =⇒ 2y(1 −λ) = 0 =⇒y = 0 F z = 2z + 2λz = 0 =⇒ 2z(1 + λ) = 0 =⇒z = 0 F λ = z 2 + 2x −y 2 −1 = 0 Reemplazando en F λ : 0 2 +2(−λ)−0 2 −1 = 0 =⇒λ = − 1 2 , y obtenemos el punto cr´ıtico _ 1 2 , 0, 0 _ H F = ¸ ¸ ¸ ¸ ¸ ¸ ¸ ¸ 0 2 −2y 2z 2 2 0 0 −2y 0 2 −2λ 0 2z 0 0 2 + 2λ ¸ ¸ ¸ ¸ ¸ ¸ ¸ ¸ = −4 _ 4 −4x 2 _ −2y 2 2y (2 −2λ) + 2z 2 (2 −2λ) (−2z) H F _ 1 2 , 0, 0, − 1 2 _ = −16 < 0 A 3 = 8y 2 −4 (2 −2λ) = 0 −12 < 0 ∴Hay un m´ınimo condicionado de f en el punto _ 1 2 , 0, 0 _ . Obs´ervese que las otras opciones no producen puntos cr´ıticos en IR 3 3.4. Problemas Resueltos 3.4.1. Continuidad y diferenciabilidad Problema Dada la funci´ on f : R 2 →R definida como f (x, y) = _ arctg xy x 2 + y 2 , si (x, y) ,= (0, 0) 0 , si (x, y) = (0, 0) . 245 a) Verificar si f es continua en IR 2 b) Calcular si existen las derivadas parciales ∂f ∂x , ∂f ∂y en IR 2 Soluci´ on. a) Tenemos que f (x, y) es continua ∀ (x, y) ,= (0, 0) puesto que es composici´ on de dos funciones continuas, como son arctg y xy x 2 + y 2 . Para estudiar la continuidad en el punto (0, 0) tenemos que calcular l´ım (x,y)→(0,0) f(x, y) lo que haremos a trav´es de la trayectoria y = mx, entonces l´ım (x,y)→(0,0) f(x, mx) = l´ım x→0 arctg mx 2 x 2 + y 2 = l´ım x→0 arctg m 2 + m 2 que depende de la pendiente m, por lo que este l´ımite no existe. Por lo tanto f no es continua en el punto(0, 0) b) Para (x, y) ,= (0, 0) la funci´ on admite derivadas parciales, que son: ∂f ∂x (x, y) = y y 2 −x 2 (x 2 + y 2 ) 2 + x 2 y 2 , ∂f ∂y (x, y) = x x 2 −y 2 (x 2 + y 2 ) 2 + x 2 y 2 Para (x, y) = (0, 0) ,se tiene ∂f ∂x (0, 0) = l´ım h→0 f(h, 0) −f (0, 0) h = l´ım h→0 arctg0 −0 h = l´ım h→0 0 = 0 Analogamente, derivando con respecto a y ∂f ∂y (0, 0) = l´ım h→0 f(0, h) −f (0, 0) h = l´ım h→0 arctg0 −0 h = l´ım h→0 0 = 0 Por lo tanto, existen las derivadas parciales en (x, y) = (0, 0) . 246 Problema Dada la funci´ on f : IR 2 →IR definida como f (x, y) = _ _ _ x 2 seny 2 x 2 + y 2 , si (x, y) ,= (0, 0) 0 , si (x, y) = (0, 0) , probar que es diferenciable en el punto P 0 = (0, 0) .¿Es continua la funci´on en ese punto? Soluci´ on. Tenemos que utilizar la definici´on y ver si el siguiente l´ımite es cero: L = l´ım (h,k)→(0,0) [∆f −df[ √ h 2 + k 2 donde ∆f = f(h, k) −f (0, 0) = h 2 senk 2 h 2 + k 2 , y df = ∂f ∂x (0, 0) h + ∂f ∂y (0, 0) k En este caso ∂f ∂x (0, 0) = l´ım h→0 f(h, 0) −f (0, 0) h = l´ım h→0 h 2 0 h 2 −0 h = 0 Asimismo ∂f ∂y (0, 0) = l´ım h→0 f(0, k) −f (0, 0) k = l´ım h→0 0 senk 2 k 2 −0 k = 0 Luego, df = 0, entonces L = l´ım (h,k)→(0,,0) h 2 senk 2 (h 2 + k 2 ) √ h 2 + k 2 g (h, k) = h 2 senk 2 (h 2 + k 2 ) 3/2 ≤ h 2 k 2 (h 2 + k 2 ) 3/2 ≤ (h 2 + k 2 )(h 2 + k 2 ) (h 2 + k 2 ) 3/2 = _ (h 2 + k 2 ) < ε 247 Si δ = ε . As´ı L = 0 y f es diferenciable en P 0 = (0, 0) . De lo anterior se deduce que f es es continua en (0, 0) ya que es diferenciable en dicho punto. 3.4.2. Regla de la cadena Problema 1 Sea la ecuaci´ on z xx + 2z xy + z y = 0 , donde x(u, v) = u + v 2 , y(u, v) = u −v 2 , z(u, v) = u 2 −v 2 4 −w(u, v) Muestre que al cambiar las variables independientes (x, y) por (u, v) y la funci´ on z por w la ecuaci´on se reduce a 2 −4w uu = 0. Soluci´on. En primer lugar, calculamos la aplicaci´on inversa u(x, y) = x + y, v (x, y) = x −y. Derivando parcialmente estas ´ ultimas expresiones se tiene: u x = 1, u y = 1; v x = 1, v y = −1 Usando estos resultados y la regla de la cadena, obtenemos z x = z u u x + z v v x = z u + z v z y = z u u y + z v v y = z u −z v Reiterando la derivaci´on parcial y usando la regla de la cadena por segunda vez z xx = (z x ) u u x + (z x ) v v x = z uu + z vu + z uv + z vv z xy = (z x ) u u y + (z x ) v v y = z uu + z vu −(z uv + z vv ) z yy = (z x ) u u y + (z v ) v v y = z uu −z vu −(z uv −z vv ) Suponiendo que z es una funci´ on continua con primeras derivadas par- ciales 248 continuas, entonces z xx + 2z xy + z y = 4z uu = 0 Finalmente, z u = 2u 4 −w u =⇒ z uu = 1 2 −w uu Por tanto, se tiene 1 2 −w uu = 0 Problema 2 Una funci´ on z = z (x, y) se dice que es arm´ onica si tiene derivadas parciales de segundo orden continuas y adem´as z xx + z yy = 0. Sean u = x x 2 + y 2 , v = y x 2 + y 2 . Pruebe que: i) u y v son arm´onicas ii) (u x ) 2 = (v y ) 2 iii) (u y ) 2 = (v x ) 2 iv) u x v x = −u y v y b) Si f (x, y) es una funci´ on arm´ onica, entonces la funci´on w(x, y) = f _ x x 2 + y 2 , y x 2 + y 2 _ es tambi´en arm´ onica Soluci´on. a) i) u = x x 2 + y 2 =⇒ u x = y 2 −x 2 (x 2 + y 2 ) 2 , u y = − 2xy (x 2 + y 2 ) 2 v = y x 2 + y 2 =⇒ v x = − 2xy (x 2 + y 2 ) 2 , v y = x 2 −y 2 (x 2 + y 2 ) 2 Derivando parcialmente por segunda vez se tiene u xx = −2x(x 2 + y 2 ) 2 −(y 2 −x 2 )2(x 2 + y 2 )2x (x 2 + y 2 ) 4 = 2x 3 −6xy 2 (x 2 + y 2 ) 3 249 u yy = −2x(x 2 + y 2 ) 2 −(2xy)2(x 2 + y 2 )2y (x 2 + y 2 ) 4 = −2x 3 + 6xy 2 (x 2 + y 2 ) 3 Lo anterior implica que u xx + u yy = 0 Analogamente para , v xx + v yy = 0 Adem´ as ii) (u x ) 2 = _ y 2 −x 2 (x 2 + y 2 ) 2 _ 2 = _ x 2 −y 2 (x 2 + y 2 ) 2 _ 2 = (v y ) 2 iii)(u y ) 2 = _ − 2xy (x 2 + y 2 ) 2 _ 2 = _ 2xy (x 2 + y 2 ) 2 _ 2 = (v x ) 2 iv) u x v x = _ y 2 −x 2 (x 2 + y 2 ) 2 __ − 2xy (x 2 + y 2 ) 2 _ = − _ 2xy(y 2 −x 2 ) (x 2 + y 2 ) 2 _ = −u y v y b) Aplicando derivaci´ on compuesta tenemos: w x = ∂f ∂u u x + ∂f ∂v v x , w y = ∂f ∂u u y + ∂f ∂v v y Derivando parcialmente por segunda vez, queda w xx = _ ∂ 2 f ∂u 2 u x + ∂ 2 f ∂v∂u v x _ u x + ∂f ∂u u xx + + _ ∂ 2 f ∂u∂v u x + ∂ 2 f ∂v 2 v x _ v x + ∂f ∂v v xx w yy = _ ∂ 2 f ∂u 2 u y + ∂ 2 f ∂v∂u v y _ u y + ∂f ∂u u yy + + _ ∂ 2 f ∂u∂v u y + ∂ 2 f ∂v 2 v y _ v y + ∂f ∂v v yy Finalmente 250 w xx = ∂ 2 f ∂u 2 (u x ) 2 + ∂ 2 f ∂v∂u v x u x + ∂f ∂u u xx + + ∂ 2 f ∂u∂v u x v x + ∂ 2 f ∂v 2 (v x ) 2 + ∂f ∂v v xx w yy = ∂ 2 f ∂u 2 (u y ) 2 + ∂ 2 f ∂v∂u u y v y + ∂f ∂u u yy + + ∂ 2 f ∂u∂v u y v y + ∂ 2 f ∂v 2 (v y ) 2 + ∂f ∂v v yy Sumando t´erminos y utilizando las igualdades establecidas en a) se tiene w xx + w yy = _ ∂ 2 f ∂u 2 + ∂ 2 f ∂v 2 _ (u x ) 2 + ∂f ∂u (u xx + u yy )+ + _ ∂ 2 f ∂u 2 + ∂ 2 f ∂v 2 _ (v x ) 2 + ∂f ∂v (v xx + v yy ) = 0 3.4.3. Derivaci´ on Impl´ıcita Problema 1 a) Sea f : R → R 2 una funci´on tal que grad f(1, 1) = (2, 4) y g : R 3 → R 2 una funci´ on tal que sus funciones coordenadas g i : R 3 → R, i = 1, 2 tienen los siguientes gradientes grad g 1 (1, 1, 1) = (2, 3, 1), grad g 2 (1, 1, 1) = (−5, 4, 2). Si g(1, 1, 1) = (1, 1). Obtener ∂(f ◦ g) ∂x (1, 1, 1). b) Utilizando el teorema de la funci´ on impl´ıcita determine si es posible escribir y en t´erminos de x para la funci´on F(x, y) = x 4 − e xy 3 −1 = 0 en una vecindad del punto (1, 1), y adem´ as encuentre su derivada. Soluci´on Sean las coordenadas cartesianas designadas en R 3 por (x, y, z) y en R 2 por (u, v), y tomando u = u(x, y, z), v = v(x, y, z) tenemos que: ∂f ◦ g ∂x (1, 1, 1) = ∂f ∂u (g(1, 1, 1)) ∂u ∂x (1, 1, 1) + ∂f ∂v (g(1, 1, 1)) ∂v ∂x (1, 1, 1) = ∂f ∂u (1, 1) ∂u ∂x (1, 1, 1) + ∂f ∂v (1, 1) ∂v ∂x (1, 1, 1) 251 Notemos que grad f(1, 1) = _ ∂f ∂u (1, 1), ∂f ∂v (1, 1) _ = (2, 4), grad g 1 = _ ∂g 1 ∂x , ∂g 1 ∂y , ∂g 1 ∂z _ = _ ∂u ∂x , ∂u ∂y , ∂u ∂z _ = (2, 3, 1), grad g 2 = _ ∂g 2 ∂x , ∂g 2 ∂y , ∂2 1 ∂z _ = _ ∂v ∂x , ∂v ∂y , ∂v ∂z _ = (−5, 4, 2), As´ı ∂f ◦ g ∂x (1, 1, 1) = 2 2 + 4 −5 = −16. Primeramente note que (1, 1) ∈ F −1 (0, 0) y adem´ as ∂F ∂x = 4x 3 −y 3 e xy 3 −1 , ∂F ∂y = −3xy 2 e xy 3 −1 las cuales son continuas en R 2 en particular para alguna vecindad V ((1, 1), δ) de (1, 1) donde ∂F ∂y (1, 1) = −3 ,= 0 por lo tanto podemos ocupar el teorema de la funci´on impl´ıcita, y definir f : V ((1, 1), δ) →R con y = f(x) y 1 = f(1) cuya derivada es y = 4x 3 −y 3 e xy 3 −1 3xy 2 e xy 3 −1 . Problema 2 a) Determine las derivadas parciales ∂u/∂x, ∂u/∂y, ∂v/∂x, ∂v/∂y, donde u, v son funciones definidas implicitamente por el sistema F(x, y, u, v) = xe u+v + uv −1 = 0, G(x, y, u, v) = ye u−v −2uv −1 = 0. alrededor del punto p = (1, 1, 0, 0). b) Sea la funci´ on z = f(u 2 + v 2 , u/v) obtener ∂ 2 z ∂u 2 . 252 Soluci´on. a) Verificando las hip´otesis del Teorema de la funci´on impl´ıcita podemos concluir que: ∂u ∂x = − (ye u−v + 2u)e u+v 2x u+v ye u−v + ye u−v u −2vxe u+v + 2xe u+v u + vye u−v ∂v ∂x = − e u+v (ye u−v −2v) 2x u+v ye u−v + ye u−v u −2vxe u+v + 2xe u+v u + vye u−v ∂u ∂y = − e ( u −v)(xe u+v + u) 2x u+v ye u−v + ye u−v u −2vxe u+v + 2xe u+v u + vye u−v ∂v ∂y = (xe u+v + v)e u−v 2x u+v ye u−v + ye u−v u −2vxe u+v + 2xe u+v u + vye u−v b) Definiendo x = x(u, v) = u 2 + v 2 , y = y(u, v) = u/v tenemos que z = f(x, y), entonces ∂z ∂u = ∂z ∂x ∂x ∂u + ∂z ∂y ∂y ∂u = ∂z ∂x 2u + ∂z ∂y 1 v , luego ∂ 2 z ∂u 2 = 2 ∂z ∂x + 2u _ ∂ 2 z ∂x 2 ∂x ∂u + ∂ 2 z ∂y∂x ∂y ∂u _ + 1 v _ ∂ 2 z ∂y 2 ∂y ∂u + ∂ 2 z ∂x∂y ∂x ∂u _ = 2 ∂z ∂x + 2u _ ∂ 2 x ∂x 2 2u + ∂ 2 z ∂y∂x 1 v _ + 1 v _ ∂ 2 z ∂y 2 2u + ∂ 2 z ∂x∂y 1 v _ Utilizando finalmente el teorema de Schwarz tenemos que ∂ 2 z ∂u 2 = 2 ∂z ∂x + 4u 2 ∂ 2 z ∂x 2 + 1 v ∂ 2 z ∂y 2 + 4u v ∂ 2 z ∂x∂y . Problema 3 a) Si u = f(x, y, z) define una funci´on diferenciable, y z se define im- plicitamente como una funci´on de x e y por la ecuaci´ on g(x, y, z) = 0 con los atributos pedido en el teorema de la funci´on impl´ıcita. Pruebe que u tiene primeras derivadas parciales de x e y dadas por: ∂u ∂x = ∂(f,g) ∂(x,z) ∂y ∂z ; ∂u ∂y = ∂(f,g) ∂(y,z) ∂y ∂z 253 b) Si u = x 2 y + z 2 , y z = g(x, y) se define impl´ıcitamente por la ecuaci´ on x 2 y −3z + 8yz 3 = 0 Calcule: ∂u ∂x (1, 0, 0) y ∂u ∂y (1, 0, 0) Soluci´on. a) Utilizando la regla de la cadena tenemos ∂u ∂x = ∂f ∂x + ∂f ∂z ∂z ∂x por otra parte si g(x, y, z) = 0 define impl´ıcitamente a z = z(x, y) entonces ∂z ∂x = − ∂g ∂x ∂g ∂z reemplazando en la ecuaci´ on anterior ∂u ∂x = ∂f ∂x + ∂f ∂z (− ∂g ∂x ∂g ∂z ) = ∂f ∂x ∂g ∂z − ∂f ∂z ∂g ∂x ∂g ∂z = ∂(f,g) ∂(x,z) ∂g ∂z Similarmente ∂u ∂y = ∂f ∂y + ∂f ∂z ∂z ∂y y ∂z ∂y = − ∂g ∂y ∂g ∂z De lo anterior se deduce ∂u ∂y = ∂f ∂y + ∂f ∂z (− ∂g ∂y ∂g ∂z ) = ∂f ∂y ∂g ∂z − ∂f ∂z ∂g ∂y ∂g ∂z = ∂(f,g) ∂(y,z) ∂g ∂z 254 b) En este caso u = f(x, y, z) = x 2 y + z 2 y z = z(x, y) se define impl´ıcitamente por g(x, y, z) = x 2 y −3z + 8yz 3 = 0 ,luego tenemos ∂g ∂x = 2xy , ∂g ∂y = x 2 + 8z 3 , ∂g ∂z = −3 + 24yz 2 derivadas que son todas continuas por lo que se afirma que g es de C 1 Adem´ as g(1, 0, 0) = 0 y ∂g ∂z (1, 0, 0) = −3 ,= 0 Entonces por el teorema de la funci´on impl´ıcita se tiene que existe V = V δ (1, 0) y una vecindad (−a, a) de z = 0 y una funci´ on z(x, y) de C 1 sobre V tal que z(1, 0) = 0 y z(1, 0)(−a, a) Calculemos el Jacobiano ∂(f, g) ∂(x, z) = ¸ ¸ ¸ ¸ 2xy 2z 2xy −3 + 24yz 2 ¸ ¸ ¸ ¸ = 2xy(−3 + 24yz 2 ) −2xy2z = 2xy(−3 + 24yz 2 −2z) Por otra parte ∂g ∂z = −3 + 24yz 2 Entonces ∂u ∂x = 2xy(−3 + 24yz 2 −2z) −3 + 24yz 2 ⇒ ∂u ∂x (1, 0, 0) = 0 −3 = 0 Tambi´en: ∂(f, g) ∂(y, z) = ¸ ¸ ¸ ¸ x 2 2z x 2 + 8z 3 −3 + 24yz 2 ¸ ¸ ¸ ¸ = −3x 2 + 24x 2 yz 2 −2x 2 z −16z 3 = x 2 (24yz 2 −2z −3) −16z 3 255 Finalmente ⇒ ∂u ∂y = x 2 (24yz 2 −2z −3) −16z 3 −3 + 24yz 2 ⇒ ∂u ∂y (1, 0, 0) = −3 −3 = 1 3.4.4. Plano Tangente a una Superficie Problema 1 Probar que los planos tangentes a la superficie S: xyz = a, a > 0 constante, en cualquier punto de S forma con los planos coordenados un tetraedro de volumen constante. Soluci´on. Sea la superficie S descrita por la funci´ on impl´ıcita F(x, y, z) = xyz − a = 0. los vectores normales a la superficie S satisfacen ∇F(x, y, z) = (yz, xz, xy) Si − → P 0 = (x 0 , y 0 , z 0 ) ∈ S,entonces − → N(P 0 ) = ∇F(x 0 , y 0 , z 0 ) = (y 0 z 0 , x 0 z 0 , x 0 y 0 ) La ecuaci´on del plano tangente al punto − → P 0 ∈ S, esta definida por ( − → r − − → P 0 ) − → N(P 0 ) = 0,luego y 0 z 0 (x −x 0 )+x 0 z 0 (y −y 0 )+x 0 y 0 (z −z 0 ) = 0. Entonces, las trazas de este plano con los ejes coordenados son i) Si x = α, y = 0, z = 0 =⇒ α = 3x 0 y 0 z 0 y 0 z 0 = 3x 0 ii) Si x = 0, y = β, z = 0 =⇒ β = 3x 0 y 0 z 0 x 0 z 0 = 3y 0 iii) Si x = 0, y = 0, z = γ =⇒ α = 3x 0 y 0 z 0 x 0 y 0 = 3z 0 El volumen del tetraedro es: V = αβγ 6 = (3x 0 ) (3y 0 ) (3z 0 ) 6 = 9 2 a, constante. 256 Problema 2 a) Probar que S 1 dada por F(x, y, z) = 0 y S 2 dada por G(x, y, z) = 0 son ortogonales en sus puntos de intersecci´on s´ı y solo si F x G x + F y G y + F z G z = 0. b) Probar que las superficies S 1 : x 2 + y 2 + z 2 − 2x + 4y − 8z = 0 y S 2 : 3x 2 −y 2 + 2z 2 −6x −4y −16z + 31 = 0 son ortogonales. Soluci´on. a) La normal a S 1 dada por F(x, y, z) = 0 es − → N 1 = ∇F(x, y, z) = (F x , F y , F z ). Del mismo modo, la normal a S 2 dada por G(x, y, z) = 0 es − → N 2 = ∇G(x, y, z) = (G x , G y , G z ) Por definici´ on, ambas superficies son ortogonales si ∇F ∇G = 0 ⇐⇒ (F x , F y , F z ) (G x , G y , G z ) = 0 =⇒ F x G x + F y G y + F z G z = 0 b) Sea S 1 : x 2 + y 2 + z 2 −2x + 4y −8z = 0 =⇒ (x −1) 2 + (y + 2) 2 + (z −4) 2 −21 = 0 y S 2 : 3x 2 −y 2 + 2z 2 −6x −4y −16z + 31 = 0 =⇒ 3(x −1) 2 −(y + 2) 2 + 2(z −4) 2 = 0 Entonces S 1 tiene normal − → N 1 = ∇F(x, y, z) = (2x −2, 2y + 4, 2z −8) Asimismo , S 2 tiene normal − → N 2 = ∇G(x, y, z) = (6x −6, −2y −4, 4z −16) =⇒ ∇F ∇G = (2x −2, 2y + 4, 2z −8) (6x −6, −2y −4, 4z −16) = (2x −2) (6x −6) + (2y + 4) (−2y −4) + (2z −8) (4z −16) = 4 [3(x −1) 2 −(y + 2) 2 + 2(z −4) 2 ] = 4 0 = 0 Por lo tanto, la superficie S 1 es ortogonal a S 2 . 257 3.4.5. Derivadas Direccionales Problema 1 Sea f (x, y) : _ _ _ 2xy 2 x 2 + y 4 , si (x, y) ,= (0, 0) 0 , si (x, y) = (0, 0) . Determine, si existe, la derivada direccional de f en P 0 = (0, 0). Soluci´on. Sea ´ e = (e 1 , e 2 ) un versor en una direcci´on cualquiera de IR 2 tal que D´ e f (0, 0) = l´ım λ→0 f((0, 0) + λ(e 1 , e 2 )) −f (0, 0) λ = l´ım λ→0 f(λe 1 , λe 2 ) −f (0, 0) λ D´ e f (0, 0) = l´ım λ→0 2 (λe 1 ) (λe 2 ) 2 (λe 1 ) 2 + (λe 2 ) 4 λ = l´ım λ→0 2λ 3 e 1 e 2 2 λ 2 (e 2 1 + λ 2 e 4 2 ) λ Por lo tanto D´ e f (0, 0) = l´ım λ→0 2e 2 2 e 1 ,este l´ımite existe si y solo si e 1 ,= 0 Problema 2 Sea f (x, y) = _ _ _ x (x + y) x 2 + y 2 , si (x, y) ,= (0, 0) 0 , si (x, y) = (0, 0) . Determine, si existe, la derivada direccional de f en P 0 = (0, 0). Soluci´ on. Sea ´ e = (e 1 , e 2 ) un versor en una direcci´on cualquiera de IR 2 tal que D´ e f (0, 0) = l´ım λ→0 f((0, 0) + λ(e 1 , e 2 )) −f (0, 0) λ = l´ım λ→0 f(λe 1 , λe 2 ) −f (0, 0) λ D´ e f (0, 0) = l´ım λ→0 e 1 (e 1 + e 2 ) λ(e 2 1 + e 2 2 ) Este l´ımite existe si y solo si e 1 (e 1 +e 2 ) = 0 =⇒ e 1 = 0 ´ o (e 1 +e 2 ) = 0 i) Si ´ e = (0, e 2 ) = 0, D´ e f (0, 0) = 0 ii) Si ´ e = (e 1 , −e 1 ) = 0, D´ e f (0, 0) = 0. 258 Problema 3 Hallar la derivada direccional de f (x, y, z) = x 2 yz 3 en el punto P 0 = (1, 1, −1) en la direcci´on de la tangente a la trayectoria : − → r (t) = (e −t , 1 + 2sen(t) , t −cos (t)) . Soluci´on. Como f (x, y, z) = x 2 yz 3 es una funci´ on diferenciable en R 3 ,entonces D´ t f (P 0 ) = ∇f (P 0 ) ´ t, donde ∇f (P) = (2xyz 3 , x 2 z 3 , 3x 2 yz 2 ) El punto P 0 que corresponde a t = t 0 es: − → r (t 0 ) = (e −t 0 , 1 + 2sen(t 0 ) , t 0 −cos (t 0 )) = (1, 1, −1) =⇒ e −t 0 = 1 As´ı, t 0 = ln (1) = 0 El vector tangente a la curva es: − → r (t) = (−e −t , 2 cos (t) , 1 + sen(t)) , entonces − → r (0) = (−1, 2, 1) y el vector tangente unitario en ´esta direcci´ on queda ´ t = −→ r (0) | −→ r (0) | = (−1,2,1) √ 6 Por tanto, la derivada direccional es D´ t f (P 0 ) = (−2, −1, 3) (−1,2,1) √ 6 = 3 √ 6 > 0 El valor positivo, significa que f aumenta en ´esta direcci´ on. Problema 4 Calcular la derivada direccional de f (x, y, z) = xy+xz−yz en el punto P 0 = _√ 2, − √ 2, 0 _ en direcci´ on de la tangente a la curva determinada por las superficies x 2 + y 2 + z 2 = 4, x + y + z = 0 Soluci´on. Como f (x, y, z) = xy + xz − yz es una funci´ on diferenciable en R 3 ,entonces D´ t f (P 0 ) = ∇f (P 0 ) ´ t, donde ∇f (P) = (y + z, x −z, x −y) =⇒ ∇f (P 0 ) = _ − √ 2, √ 2, 2 √ 2 _ 259 A´ un falta calcular el vector ´ t = −→ r (P 0 ) | −→ r (P 0 ) | , que es tangente a la curva determinada por las superficies dadas. Sea C dada por − → r (x) = (x, y (x) , z (x)) =⇒ − → r (x) = _ 1, y (x) , z (x) _ , donde y (x) , z (x) se calculan impl´ıcitamente a partir del sistema de ecuaciones por derivaci´ on con respecto a x, en el entendido que y = y (x) , z = z (x) . En efecto: 2x + 2yy + 2zz = 0 1 2 1 + y + z = 0 ⇐⇒ yy + zz = −x y + z = −1 Resolviendo el ´ ultimo sistema obtenemos y (x) = ¸ ¸ ¸ ¸ −x z −1 1 ¸ ¸ ¸ ¸ ¸ ¸ ¸ ¸ y z 1 1 ¸ ¸ ¸ ¸ = −x + z y −z , z (x) = ¸ ¸ ¸ ¸ y −x 1 −1 ¸ ¸ ¸ ¸ ¸ ¸ ¸ ¸ y z 1 1 ¸ ¸ ¸ ¸ = x −y y −z Evaluando y _√ 2 _ = − √ 2 − √ 2 = 1 y y _√ 2 _ = 2 √ 2 − √ 2 = −2 Luego =⇒ − → r _√ 2 _ = (1, 1, −2) =⇒ ´ t = − → r (P 0 ) | − → r (P 0 )| = (1, 1, −2) √ 6 Por lo tanto, se tiene que D´ t f (P 0 ) = (− √ 2, √ 2, 2 √ 2) (1, 1, −2) √ 6 = −4 √ 6 3.4.6. Valores extremos Problema 1 Sea f : U ⊆ R2 → R definida en el abierto U, dada por f(x, y) = x 4 −2px 2 −y 2 +3 donde p es una constante entera. Calcular los valores extremos de la funci´ on Soluci´on Apliquemos la condici´ on necesaria de punto cr´ıtico, calculando las derivadas parciales e igual´ andolas a cero, luego f x = 4x 3 −4px = 4x(x 2 −p); f y = −2y. 260 Tenemos que f y = −2y = 0 =⇒ y = 0, pero en la derivada parcial respecto de x hay que analizar el signo de p. i) Si p > 0, fx = 4x(x 2 − p) = 0 =⇒ x = 0, ± √ p y los puntos cr´ıticos son P 1 = (0, 0), P 2 = ( √ p, 0) y P 3 = (− √ p, 0). ii) Si p = 0, fx = 4x 3 = 0 =⇒ x = 0, y el ´ unico punto cr´ıtico es P 1 = (0, 0). iii) Si p < 0, fx = 4x(x 2 − p) = 0 =⇒ x = 0, y el ´ unico punto cr´ıtico es P 1 = (0, 0) Estudiemos el Hessiano para ver si los puntos obtenidos son m´aximos, m´ınimos o puntos silla. f xx = 12x 2 −4p, f xy = 0, f yx = 0, f yy = −2 Por tanto el determinante de la matriz Hessiana es H (x, y) = ¸ ¸ ¸ ¸ 12x 2 −4p 0 0 −2 ¸ ¸ ¸ ¸ Estudiemos los puntos cr´ıticos seg´ un los valores del par´ ametro p 1) Si p > 0: En este caso hay 3 puntos cr´ıticos. H (0, 0) = ¸ ¸ ¸ ¸ −4p 0 0 −2 ¸ ¸ ¸ ¸ = 8p > 0 y f xx = −4p < 0 Entonces, el punto P 1 = (0, 0) es un M´aximo Relativo. H (± √ p, 0) = ¸ ¸ ¸ ¸ 8p 0 0 −2 ¸ ¸ ¸ ¸ = −16p < 0 Por lo tanto, los puntos P 2 = ( √ p, 0) y P 3 = (− √ p, 0) son Puntos Silla. 2) Si p < 0: En este caso en P 1 = (0, 0) hay un punto cr´ıtico. H (0, 0) = ¸ ¸ ¸ ¸ −4p 0 0 −2 ¸ ¸ ¸ ¸ = 8p < 0, luego en P 1 = (0, 0) hay un Punto Silla. 3) Si p = 0: En este caso hay un ´ unico punto cr´ıtico en el origen, pero al calcular el determinante del Hessiano, obtenemos que H(0, 0) = 0, luego no se tiene informaci´on. 261 Sin embargo, podemos comprobar la naturaleza del punto aplicando la definici´ on de extremo local en las cercan´ıas del origen, siguiendo dos trayectorias distintas, para la funci´on: f(x, y) = x 4 −y 2 + 3. Entonces f(x, 0) −f(0, 0) = x 4 > 0 para todo (x, 0) ∈ V (0, 0) f(0, y) −f(0, 0) = −y 2 < 0 para todo (0, y) ∈ V (0, 0). Por lo tanto, P 1 = (0, 0) es un Punto de Silla Problema 2 Encuentre los valores extremos de la f (x, y) = x 2 + y 2 − 1 2 x 4 . Soluci´on. Derivando parcialmente con respecto a x e y tenemos: f x (x, y) = 2x −2x 3 f y (x, y) = 2y Esta claro que f x y f y son funciones continuas en R 2 Aplicando la condici´on necesaria para los puntos cr´ıticos de f tenemos: 2x −2x 3 = 0; 2y = 0. Al resolver el sistema obtenemos tres puntos cr´ıticos. P 0 = (0, 0) , P 1 = (1, 0) , P 2 = (−1, 0) Determinemos el Hessiano H (x, y) : H (x, y) = ¸ ¸ ¸ ¸ f xx (x, y) f xy (x, y) f yx (x, y) f yy (x, y) ¸ ¸ ¸ ¸ = ¸ ¸ ¸ ¸ 2 −6x 2 0 0 2 ¸ ¸ ¸ ¸ = 4 −12x 2 Evaluemos el Hessiano H (x, y) en cada uno de los puntos: i) Para P 0 = (0, 0) =⇒ H (0, 0) = 4 > 0 y f xx (0, 0) = 2 > 0 Entonces en P 0 hay un m´ınimo relativo f (0, 0) = 0. ii) Para P 1 = (1, 0) =⇒ H (1, 0) = −8 < 0 . Entonces, en P 1 hay punto silla de f. iii) Para P 2 = (−1, 0) =⇒ H (−1, 0) = −8 < 0 . As´ı, en P 2 tambien hay un punto silla de f. 262 Problema 3 Encuentre los valores extremos de la f (x, y) = x 2 + y 2 −xy + x + y en el dominio D = ¦(x, y) ∈ R 2 / x ≤ 0, y ≤ 0, x + y ≥ −3¦ . Soluci´on. En primer lugar, determinemos los valores extremos en el conjunto abierto: D ∗ = ¦(x, y) ∈ R 2 / x < 0, y < 0, x + y > −3¦ . Derivando parcialmente con respecto a x e y tenemos: f x (x, y) = 2x −y + 1 f y (x, y) = 2y −x + 1 Observe que f x y f y son continuas en R 2 Aplicando la condici´on necesaria para los puntos cr´ıticos de f tenemos el sistema: 2x −y = −1; −x + 2y = −1. Al resolver este sistema obtenemos un ´ unico punto cr´ıtico P 0 = (−1, −1) ∈ D ∗ Determinemos el Hessiano H (x, y) : H (x, y) = ¸ ¸ ¸ ¸ f xx (x, y) f xy (x, y) f yx (x, y) f yy (x, y) ¸ ¸ ¸ ¸ = ¸ ¸ ¸ ¸ 2 −1 −1 2 ¸ ¸ ¸ ¸ = 3 ∀ (x, y) ∈ D ∗ : As´ı, P 0 = (−1, −1) =⇒ H (−1, −1) = 3 > 0 y f xx (−1, −1) = 3 > 0 Se concluye, que en P 0 hay un m´ınimo relativo f (−1, −1) = −1. En segundo lugar, estudiemos la condici´on que se presenta en la frontera de D. a) Si y = 0, f (x, 0) = x 2 + x con x ∈ [−3, 0] Determinemos los puntos cr´ıticos en este borde f (x) = 2x + 1 = 0 =⇒ x = − 1 2 Luego, hay un punto crit´ıco en P 1 = _ − 1 2 , 0 _ ∈ D. Como f (x) = 2 > 0, ∀x ∈ [−3, 0] , entonces en P 1 = _ − 1 2 , 0 _ hay un m´ınimo f _ − 1 2 , 0 _ = − 1 4 . b) Si x = 0, f (0, y) = y 2 + y con y ∈ [−3, 0] Determinemos los puntos cr´ıticos en este borde 263 f (y) = 2y + 1 = 0 =⇒ y = − 1 2 Luego, se tiene un punto crit´ıco en P 2 = _ 0, − 1 2 _ ∈ D. Como f (y) = 2 > 0, ∀y ∈ [−3, 0] ,entonces en P 2 = _ 0, − 1 2 _ hay un m´ınimo f _ 0, − 1 2 _ = − 1 4 . c) Si x+ y = −3, f (x, −x −3) = 3x 2 + 9x + 6 con x ∈ [−3, 0] Determinemos los puntos cr´ıticos en este borde f (x) = 6x + 9 = 0 =⇒ x = − 3 2 =⇒ y = − 3 2 Luego, hay un punto crit´ıco en P 2 = _ − 3 2 , − 3 2 _ ∈ D. Como f (x, −x −3) = 6 > 0, ∀x ∈ [−3, 0] ,entonces P 2 = _ − 3 2 , − 3 2 _ en hay un m´ınimo f _ − 3 2 , − 3 2 _ = − 3 4 . 3.4.7. Multimplicadores de Lagrange para extremos restringidos Problema 1 Obtener los extremos de la funci´ on f(x, y) = x 3 + 2xy + y 2 que se encuentren en la recta x + y = 0. Soluci´ on En primer lugar consideremos la funci´ on de Lagrange y apliquemos la condici´ on necesaria de punto cr´ıtico: F(x, y, λ) = x 3 + 2xy + y 2 + λ(x + y) F x = 3x 2 + 2y + λ = 0 (1) Fy = 2x + 2y + λ = 0 (2) Fz = x + y = 0 (3) De la (3) ecuaci´ on obtenemos: y = −x que sustituida en la segunda: 2x −2x + λ = 0 =⇒ λ = 0 Sustituido este valor en la ecuaci´ on (1), queda: 3x 2 − 2x = 0, x = 0, x = 2/3 Entonces los puntos cr´ıticos del lagrangiano son. P 1 (0, 0), P 2 _ 2 3 , − 2 3 _ . Clasificaremos los puntos con el determinante del Hessiano Limitado: 264 H (x, y) = ¸ ¸ ¸ ¸ ¸ ¸ 0 1 1 1 6x 2 1 2 2 ¸ ¸ ¸ ¸ ¸ ¸ evalu´ ando en los puntos cr´ıticos obtenidos anteriormente. Consideremos H (0, 0) = ¸ ¸ ¸ ¸ ¸ ¸ 0 1 1 1 0 2 1 2 2 ¸ ¸ ¸ ¸ ¸ ¸ = 2 > 0, entonces d 2 f dx 2 < 0 y en P 1 (0, 0) hay un m´aximo local condicionado de f, cuyo valor es f(0, 0) = 0. Por otra parte H _ 2 3 , − 2 3 _ = ¸ ¸ ¸ ¸ ¸ ¸ 0 1 1 1 4 2 1 2 2 ¸ ¸ ¸ ¸ ¸ ¸ = −2 > 0, entonces d 2 f dx 2 < 0 y en P 2 _ 2 3 , − 2 3 _ hay un m´ınimo local condicionado de f, cuyo valor es f _ 2 3 , − 2 3 _ = − 4 27 . Problema 2 En que puntos de la elipse x 2 a 2 + y 2 b 2 = 1, la tangente a este lugar geom´etrico forma con los ejes coordenados un tri´ angulo de ´area m´ınima. Soluci´on. Sea la ecuaci´ on de la tangente a la elipse en el punto (x 0 , y 0 ) . x 0 x a 2 + y 0 y b 2 = 1 Sea f (x, y) = 1 2 x T y T , el ´ area que forma la recta tangente con los ejes coordenados, donde x T y T se determinan a partir de la ecuaci´ on de la tangente. Si y T = 0 =⇒ x T = a 2 x 0 x T = 0 =⇒ y T = b 2 y 0 As´ı f (x, y) = 1 2 a 2 b 2 x 0 y 0 es la funci´ on a estudiar, que verifica la condici´ on: x 2 0 a 2 + y 2 0 b 2 −1 = 0. Consideremos la funci´on : 265 L(x, y, λ) = 1 2 a 2 b 2 x 0 y 0 + λ _ x 2 0 a 2 + y 2 0 b 2 −1 _ L x (x, y, λ) = − 1 2 a 2 b 2 x 2 0 y 0 + 2λ x 0 a 2 = 0 1/y 0 L y (x, y, λ) = − 1 2 a 2 b 2 x 0 y 2 0 + 2λ y 0 b 2 = 0 /x 0 L λ (x, y, λ) = x 2 0 a 2 + y 2 0 b 2 −1 = 0 Multiplicando las ecuaciones anteriores por los coeficientes que se indican, tenemos 1,0) − 1 2 a 2 b 2 x 2 0 y 2 0 + 2λ a 2 x 0 y 0 = 0 1/y 0 2,0) − 1 2 a 2 b 2 x 2 0 y 2 0 + 2λ b 2 y 0 x 0 = 0 1/x 0 3,0) x 2 0 a 2 + y 2 0 b 2 −1 = 0 Restando 2,0 −1,0 se tiene 2λ a 2 x 0 y 0 = 2λ b 2 y 0 x 0 =⇒ x 2 0 = a 2 b 2 y 2 0 Sustituyendo este resultado en 3.0) se tiene un ´ unico punto cr´ıtico de f en P 0 = _ a √ 2 , b √ 2 _ Mediante el criterio de la segunda derivada se puede determinar la naturaleza del punto cr´ıtico f (x, y(x)) = 1 2 a 2 b 2 x 0 y 0 =⇒ f (x) = a 2 b 2 2 _ −y 0 −x 0 y 0 (x) (x 0 y 0 ) 2 _ donde a partir de la condici´on obtenemos 2x 0 a 2 + 2y 0 b 2 y 0 (x) = 0 =⇒ y 0 (x) = − b 2 x 0 a 2 y 0 =⇒ y 0 (x) = − b 2 a 2 _ y 0 −x 0 y 0 y 2 0 _ As´ı f (x) = − a 2 b 2 2 _ (x 0 y 0 ) 2 (2y 0 + x 0 y 0 (x)) −2(x 0 y 0 )(y 0 + x 0 y 0 (x)) 2 (x 0 y 0 ) 4 _ Produce 266 f _ a √ 2 , b √ 2 _ > 0 Por lo tanto, en el punto P 0 = _ a √ 2 , b √ 2 _ un m´ınimo de f Problema 3 Se desea construir una tolva para un silo, que tenga una capacidad de 100 m 3 y forma de cono circular recto de 2m de radio, coronado por un cilindro circular recto, empleando un m´ınimo de material para la superficie. Calcular las alturas x del cilindro e y del cono para tal objeto. Soluci´on: Sea la funci´ on superficie definida por f (x, y) = 2π _ 4 + y 2 +4πx Con la condici´ on que el volumen sea g (x, y) = 4 3 πy + 4πx −100 = 0 Entonces formemos la funci´ on: L(x, y, λ) = f (x, y) = 2π _ 4 + y 2 +4πx + λ _ 4 3 πy + 4πx −100 _ L x (x, y, λ) = 4π + 4πλ = 0 =⇒ λ = −1 L y (x, y, λ) = 4πy 2 _ 4 + y 2 + 4 3 πλ = 0 =⇒ y 2 _ 4 + y 2 = 1 3 L λ (x, y, λ) = 4 3 πy + 4πx −100 = 0 9y 2 = 4 (4 + y 2 ) =⇒ 5y 2 = 16 =⇒ y = 4 √ 5 Sustituyendo en la restricci´ on se tiene 4πx = 100 − 16 3 √ 5 π =⇒ x = 100 4π − 4 3 √ 5 En consecuencia, se tiene un ´ unico punto cr´ıtico en P 0 = _ 100 4π − 4 √ 5 , 4 √ 5 _ La condici´on de m´ınimo de f se estable mediante la segunda derivada f (x, y(x)) = 2π _ 4 + y 2 + 4πx =⇒ f (x) = 4πyy 2 _ 4 + y 2 + 4π 4 3 πy + 4πx −100 = 0 =⇒ y (x) = −3 Por lo tanto, sustituyendo y (x) , y derivando por segunda vez 267 f (x) = − 6πy _ 4 + y 2 + 4π =⇒ f (x) = −6π _ (−3(4 + y 2 ) −y 2 (4 + y 2 ) 3/2 _ f (P 0 ) > 0 =⇒ Valor m´ınimo As´ı el valor m´ınimo de la funci´on es: f _ 100 4π − 4 √ 5 , 4 √ 5 _ = 2π _ 4 + 16 5 +4π _ 100 4π − 4 √ 5 _ = 100 − 20π 3 √ 5 Problema 4 Determine la distancia m´ınima y m´axima del origen a la curva de in- tersecci´ on del paraboloide z = 7 4 −x 2 −y 2 y el plano x + y + z = 2. Soluci´ on: En este caso es conveniente los valores extremos del cuadrado de la distancia con respecto al origen en vez de la distancia misma. Por lo tanto, se deben hallar los valores extremos de la funci´on: f(x, y, z) = x 2 + y 2 + z 2 sujeta a las restriciones g (x, y, z) = z − 7 4 + x 2 + y 2 = 0 h(x, y, z) = x + y + z −2 = 0 Para aplicar el m´etodo de los multiplicadores de Lagrange se define F (x, y, z, λ 1 , λ 2 ) = x 2 +y 2 +z 2 +λ 1 _ z − 7 4 + x 2 + y 2 _ +λ 2 (x + y + z −2) F x = 2 (1 + λ 1 ) x + λ 2 = 0 (1,0) F y = 2 (1 + λ 1 ) y + 2λ 2 = 0 (2,0) F z = 2z + λ 1 + λ 2 = 0 (3,0) F λ 1 = z − 7 4 + x 2 + y 2 = 0 (4,0) F λ 2 = x + y + z −2 = 0 (5,0) 1,0) −2,0) : 2 (1 + λ 1 ) (x −y) = 0 =⇒λ 1 = −1 o y = x Si λ 1 = −1 ,entonces de 1) λ 2 = 0 y de 3) z = 1 2 268 Si z = 1 2 , entonces de 4) y 5) se obtiene: 2x 2 −3x + 1 = 0 Resolviendo la ecuaci´on anterior, sus soluciones son: x 1 = 1; x 2 = 1 2 x 1 = 1 =⇒ y 1 = 1 2 =⇒ _ 1, 1 2 , 1 2 _ es punto cr´ıtico. x 2 = 1 2 =⇒ y 2 = 1 =⇒ _ 1 2 , 1, 1 2 _ es punto cr´ıtico. Por otra parte:4) −5) =⇒ x 2 + y 2 −x −y + 1 4 = 0 Si y = x =⇒ 2x 2 −2x + 1 4 = 0, resolviendo la ecuaci´on x = 2 ± √ 2 4 y = x = 2 + √ 2 4 =⇒ z = 4 −2 √ 2 4 =⇒ _ 2 + √ 2 4 , 2 + √ 2 4 , 4 −2 √ 2 4 _ es punto cr´ıtico de f. y = x = 2 − √ 2 4 =⇒ z = 4 + 2 √ 2 4 =⇒ _ 2 − √ 2 4 , 2 − √ 2 4 , 4 + 2 √ 2 4 _ es punto cr´ıtico de f. As´ı f m´ax _ 2 ± √ 2 4 , 2 ± √ 2 4 , 4 ∓2 √ 2 4 _ = 1 4 _ 9 + 2 √ 2 _ f m´ın _ 1 2 , 1, 1 2 _ = 3 2 Como la curva intersecci´ on del paraboloide y el plano es una curva cerrada, la distancia m´ınima y la distancia m´ axima al origen son re- spectivamente _ 3 2 y 1 2 _ _ 9 + 2 √ 2 _ .No necesitamos m´as pruebas por las caracteristicas geom´etricas del problema. Problema 5 Demuestre que las distancias m´ axima y m´ınima desde el origen a la curva de intersecci´ on definida por x 2 4 + y 2 5 + z 2 25 = 1, z = x + y. Soluci´on: 269 Debenos encontrar los valores extremos de la funci´ on f(x, y, z) = x 2 + y 2 + z 2 sujeta a las restriciones g (x, y, z) = x 2 4 + y 2 5 + z 2 25 −1 = 0 h(x, y, z) = x + y −z = 0 Para aplicar el m´etodo de los multiplicadores de Lagrange se define F (x, y, z, λ 1 , λ 2 ) = x 2 +y 2 +z 2 +λ 1 _ x 2 4 + y 2 5 + z 2 25 −1 _ +λ 2 (x + y −z) Aplicando la condici´on necesaria de punto cr´ıtico F x = 2 _ 1 + λ 1 4 _ x + λ 2 = 0 (1,0) F y = 2 _ 1 + λ 1 5 _ y + λ 2 = 0 (2,0) F z = 2(1 + λ 1 5 + λ 2 = 0 (3,0) F λ 1 = x 2 4 + y 2 5 + z 2 25 −1 = 0 (4,0) F λ 2 = x + y −z = 0 (5,0) Despejando de estas ecuaciones x, y, z se tiene x = − 2λ 2 λ 1 + 4 ; y = − 5λ 2 2λ 1 + 10 ; z = − 25λ 2 2λ 1 + 50 ; 6,0) Al dividir 5,0 por λ 2 ,= 0 (lo cual est´ a justificado porque de otro modo de 1,0, 2,0 y 3,0, se tendr´ıa x = y = z = 0). 2 λ 1 + 4 + 5 2λ 1 + 10 + 25 2λ 1 + 50 = 0. Multiplicando por 2 (λ 1 + 4) (2λ 1 + 10) (2λ 1 + 50) y simplificando da 17λ 2 1 + 245λ 1 + 750 = 0 =⇒ (λ 1 + 10)(17λ 1 + 75) = 0 de donde: λ 1 = −10, λ 1 = − 75 17 Caso i) Si λ 1 = −10, entonces de 6,0 : x = λ 2 3 ; y = λ 2 2 ; z = 5λ 2 6 . Sutituyendo en 4.0 da: λ 2 2 36 + λ 2 2 20 + 5λ 2 2 66 −1 = 0 =⇒ λ 2 2 = 180 19 =⇒ 270 λ 2 = ±6 _ 5 19 Por lo tanto, se tienen dos puntos cr´ıticos. P 1 = _ 2 _ 5 19 , 3 _ 5 19 , 5 _ 5 19 _ y P 2 = _ −2 _ 5 19 . −3 _ 5 19 , −5 _ 5 19 _ Evaluando en la funci´ on se tiene f _ ±2 _ 5 19 , ±3 _ 5 19 , ±5 _ 5 19 _ = 10 Caso ii) Si λ 1 = − 75 17 , entonces de 6,0 : x = 34λ 2 7 ; y = − 17λ 2 4 ; z = 17λ 2 28 . Sutituyendo en 4.0 da: λ 2 2 36 + λ 2 2 20 + 5λ 2 2 66 −1 = 0 =⇒ λ 2 2 = (140) 2 (17) 2 (646) =⇒ λ 2 = ±14017 √ 646 Por lo tanto, se tienen otros dos puntos cr´ıticos m´ as. P 1 = _ 40 √ 646 , − 35 √ 646 5 √ 646 _ y P 2 = _ − 40 √ 646 , 35 √ 646 , − 5 √ 646 _ Evaluando en la funci´on se tiene f _ ± 40 √ 646 , ∓ 35 √ 646 , ± 5 √ 646 _ = 75 17 Asi el valor m´ aximo buscado es 10 y el valor m´ınimo es 75 17 Problema 6 Se desea construir un silo, que tenga una capacidad de V 0 con forma de cilindro circular recto de altura h y radio basal r . Calcular la altura h del cilindro y radio basal r de manera que la superficie total sea m´ınima. Soluci´on: Sea la funci´ on superficie definida por f (r, h) = 2πr 2 +2πrh Con la condici´ on que el volumen sea g (x, y) = πr 2 h −V 0 = 0 Entonces formemos la funci´ on: L(r, h, λ) = 2πr 2 + 2πrh + λ(πr 2 h −V 0 ) 271 L r (r, h, λ) = 4πr + 2πh + 2λπrh = 0 1,0) L h (r, h, λ) = 2πh + λπr 2 = 0 2,0) L λ (r, h, λ) = πr 2 h −V 0 = 0 3,0) De 2,0) se tiene: λ = − 2 r y sustituyendo este valor en 1,0) obtenemos h = 2r Si h = 2r, entonces de 3.0) r = 3 _ V 0 2π ; h = 2 3 _ V 0 2π En consecuencia, se tiene un ´ unico punto cr´ıtico en P 0 = _ 3 _ V 0 2π , 2 3 _ V 0 2π _ La condici´on de m´ınimo de f se establece mediante la segunda derivada f (r, h(r)) = 6πr 2 =⇒ f (r) = 12πr =⇒ f (r) = 12π > 0 Por lo tanto, se tiene un valor m´ınimo de f si h = 2r As´ı el valor m´ınimo de la superficie es: f _ 3 _ V 0 2π , 2 3 _ V 0 2π _ = 6π _ V 0 2π _ 2/3 Problema 7: Determinar los extremos absolutos de la funci´ on f (x, y) = y 3 + x 2 y + 2x 2 + 2y 2 −4y −8 en el conjunto D = ¦(x, y) ∈ IR 2 /x 2 + y 2 ≤ 1¦ . Soluci´on: En primer lugar estudiemos los puntos del interior de D, para ver si existen m´aximos o m´ınimos locales. La condicion necesaria, de los puntos interiores candidatos a extremos, es ∇f (x, y) = (0, 0) =⇒ ∂f ∂x = 2x(y + 2) = 0 ∂f ∂y = 3y 2 + x 2 + 4y −4 = 0 i) La primera ecuaci´ on implica que x = 0 ´o y = −2. Si y = −2, la segunda ecuaci´on implica que x = 0, luego se tiene un punto cr´ıtico en P 0 = (0, −2) , sin embargo , P 0 / ∈ D. 272 ii) Si x = 0, la segunda ecuaci´ on es 3y 2 + 4y − 4 = 0 =⇒ y = −2, y = 2 3 . Las coordenadas del punto P 1 = _ 0, 2 3 _ verifican 4 9 < 1,entonces P 1 ∈ D. Adem´ as f (P 1 ) = 8 27 + 8 9 − 8 3 −8 = 256 27 = −9, 48. En segundo lugar, estudiemos los puntos de la frontera de D usando la funci´ on f (x, y) = y 3 + x 2 y + 2x 2 + 2y 2 − 4y − 8 bajo la restrici´ on g (x, y) = x 2 + y 2 −1 = 0. Usemos el m´etodo de los multiplicadores de Lagrange. Sea L(x, y, λ) = y 3 + x 2 y + 2x 2 + 2y 2 −4y −8 + λ(x 2 + y 2 −1), y obtenemos: ∂L ∂x = 2x(y + 2) + λ2x = 0 (1,0) ∂L ∂y = 3y 2 + x 2 + 4y −4 + λ2y = 0 (2,0) ∂L ∂λ = x 2 + y 2 −1 = 0 (3,0) De la ecuaci´ on 1,0 se tiene que x = 0 ´ o (y + 2) + λ = 0 a) Si x = 0,en 3.0 se tiene y 2 − 1 = 0 =⇒ y = ±1.Luego se tienen otros dos puntos cr´ıticos P 2 = (0, 1) y P 2 = (0, −1) que satisfacen las ecuaciones 1,0 y 3,0. Para comprobar que tambi´en satisfacen la ecuaci´ on 2,0 , sustituyamos el ella P 1 = (0, 1) =⇒ λ = 3 2 ∈ IR P 2 = (0, −1) =⇒ λ = 5 2 ∈ IR. Si evaluamos las funci´on en los puntos encontrados obtenemos: f (P 2 ) = 1 + 2 −4 −8 = −9 f (P 3 ) = −1 + 2 + 4 −8 = −3 b) Si (y + 2) + λ = 0 ⇐⇒ λ = −(y + 2), en 2,0 se tiene 3y 2 + x 2 + 4y −4 + 2y(y + 2) = 0 =⇒ x 2 +y 2 = 4, resultado que contradice la ecuaci´ on 3,0, x 2 +y 2 = 1. Luego, esta condici´on no produce un punto cr´ıtico. 273 Por lo tanto, comparando los valores de la funci´ on en los tres puntos encontrados, podemos inferir que el m´ aximo absoluto se alcanza en P 3 y que el m´ınimo absoluto se alcanza en P 1 Problema 8: Determine las dimensiones de una caja rectangular, sin tapa superior,que ha de tener un volumen dado V 0 ,de manera que su superficie sea m´ınima. Soluci´on: Sea la funci´ on superficie definida por f (x, y, z) = xy + 2xz + 2yz Con la condicion que el volumen sea g (x, y, z) = xyz −V 0 = 0 Entonces formemos la funci´ on: L(x, y, z, λ) = f (x, y) = xy + 2xz + 2yz + λ(xyz −V 0 ) L x (x, y, z, λ) = y + 2z + λyz = 0 (1,0) L y (x, y, z, λ) = x + 2z + λxz = 0 (2,0) L z (x, y, z, λ) = 2x + 2y + λxy = 0 (3,0) L λ (x, y, z, λ) = xyz −V 0 = 0 (4,0) Despejando y y x de 1,0 y 2,0 se tiene 5.0) y = x = − 2z 1+λz , sutituyendo en 3,0) produce: − 8z 1+λz + 4λz 2 (1+λz) 2 = 0 =⇒ z = 1 λ 6,0) Reemplazando en 6,0) en 5,0) se tiene y = x = − 4 λ . Sustituyendo en 4.0 _ − 4 λ _ _ − 4 λ _ _ 1 λ _ = V 0 =⇒ λ = _ 16 V 0 _ 1/3 Por lo tanto, se tiene un ´ unico punto cr´ıtico de f en P 0 = _ 4 _ V 0 16 _ 1/3 , 4 _ V 0 16 _ 1/3 , _ V 0 16 _ 1/3 _ . Examinemos la naturaleza del punto cr´ıtico usando el Hessiano limita- do: H (x, y, z) = ¸ ¸ ¸ ¸ ¸ ¸ ¸ ¸ ¸ 0 ∂g ∂x ∂g ∂y ∂g ∂z ∂g ∂x ∂ 2 L ∂x 2 ∂ 2 L ∂x∂y ∂ 2 L ∂x∂z ∂g ∂y ∂ 2 L ∂x∂y ∂ 2 L ∂y 2 ∂ 2 L ∂y∂z ∂g ∂z ∂ 2 L ∂x∂z ∂ 2 L ∂y∂z ∂ 2 L ∂z 2 ¸ ¸ ¸ ¸ ¸ ¸ ¸ ¸ ¸ =⇒ 274 H (x, y, z) = ¸ ¸ ¸ ¸ ¸ ¸ ¸ ¸ 0 yz xz xy yz 0 1 + λz 2 + λy xz 1 + λz 0 2 + λx xy 2 + λy 2 + λx 0 ¸ ¸ ¸ ¸ ¸ ¸ ¸ ¸ Evaluemos el Hessiano H _ 4 _ V 0 16 _ 1/3 , 4 _ V 0 16 _ 1/3 , _ V 0 16 _ 1/3 _ H = ¸ ¸ ¸ ¸ ¸ ¸ ¸ ¸ ¸ ¸ 0 4 _ V 0 16 _ 2/3 4 _ V 0 16 _ 2/3 16 _ V 0 16 _ 2/3 4 _ V 0 16 _ 2/3 0 2 6 4 _ V 0 16 _ 2/3 2 0 6 16 _ V 0 16 _ 2/3 6 6 0 ¸ ¸ ¸ ¸ ¸ ¸ ¸ ¸ ¸ ¸ Usando propiedades de determinantes, obtenemos que el valor de Hes- siano es: H = 16 _ V 0 16 _ 4/3 ¸ ¸ ¸ ¸ ¸ ¸ ¸ ¸ 0 1 1 4 1 0 2 6 1 2 0 6 4 6 6 0 ¸ ¸ ¸ ¸ ¸ ¸ ¸ ¸ H = −2048,0 (0,062 5V 0 ) 4 3 < 0 Adem´ as: ∆ 3 = ¸ ¸ ¸ ¸ ¸ ¸ 0 1 1 1 0 2 1 2 0 ¸ ¸ ¸ ¸ ¸ ¸ = 4 > 0 Entonces la funci´ on f tendr´ a un m´aximo condicionado en el punto P 0 = _ 4 _ V 0 16 _ 1/3 , 4 _ V 0 16 _ 1/3 , _ V 0 16 _ 1/3 _ 3.4.8. Aplicaci´on al c´alculo de errores Problema 1: El periodo T de un p´endulo simple depende de la longitud l y de la aceleraci´ on de gravedad g del lugar y est´ a dado por: T = 2π _ l g . Hallar a a) el error absoluto y b) el error relativo , al calcular T con l = 0, 6 m y g = 10m/s 2 si los valores verdaderos eran l = 58, 5cm y g = 9, 8m/s 2 . Soluci´on: a) Sea T = 2π _ l g . el per´ıodo de un p´endulo simple. 275 El error absoluto de T es ∆T, que en este caso es aproximadamente dT. as´ı se tiene: El error absoluto de T = dT = ∂T ∂l dl + ∂T ∂g dg = π √ lg dl −π _ l g 3 dg Error de l = ∆l = dl = (0, 6 −0, 585) m = 0, 015m Error de g = ∆g = dg = (10 −9, 8) m/s 2 = 0, 2m/s 2 El error absoluto de T = dT = π √ 0, 6x10 (0, 015) −π _ 0, 6 1000 (0, 2) b) El error relativo de T = dT T = 1 2π _ l g _ π √ lg dl −π _ l g 3 dg _ El error relativo de T = _ 1 2l dl − 1 2g dg _ . El error relativo de T = dT T = 1 2π _ l g _ π √ lg dl −π _ l g 3 dg _ 3.5. Ejercicios Propuestos 3.5.1. L´ımites Problema 1 Determinar si existen los siguientes limites, y si existen, calcular su valor a) l´ım (x,y)→(0,0) ln xseny ysenx b) l´ım (x,y)→(0,0) x 2 −y 2 x 2 + y 2 Soluci´on: a) l´ım (x,y)→0 ln xseny ysenx = 0, b) El l´ımite de la funci´ on f no existe 276 Problema 2 En los siguientes casos, usar coordenadas polares para concluir que el l´ımite de f (x, y) cuando (x, y) tiende a (0, 0) existe y vale cero. a) l´ım (x,y)→(0,0) y 3 x 2 + y 2 b) l´ım (x,y)→(0,0) x 3 y 4 x 4 + y 4 3.5.2. Diferenciabilidad, continuidad Problema 1 Dada la funci´on f (x, y) = _ _ _ xysen 1 y si y ,= 0 0 ∀ (x, 0) , probar que es diferenciable en el punto P 0 = (0, 0) ,¿es continua en este punto? Soluci´on: La funci´ on f es diferenciable en P 0 = (0, 0) , implica que es continua en (0, 0) Problema 2 Dada la funci´ on f (x, y) = _ _ _ [x[ y _ x 2 + y 2 si (x, y) ,= (0, 0) 0 si (x, y) = (0, 0) . a) Estudiar su continuidad en IR 2 b) Estudiar la existencia de derivadas parciales en IR 2 y determinarlas, si es que existen. c) Estudiar su diferenciabilidad en IR 2 Soluci´on: a) La funci´ on f es en continua en IR 2 b) Existen derivadas parciales en IR 2 −¦(o, a) /a ,= 0¦ ,No existe ∂f ∂x (0, a) , ∂f ∂y (0, a) = 0, 277 ∂f ∂x (0, 0) = 0, ∂f ∂y (0, 0) = 0 c) La funci´ on f no es diferenciable en P 0 = (0, 0) ,puesto que l´ım (h,k)→(0,0) [∆f −df[ √ h 2 + k 2 ,= 0 Problema 3 Verificar que f (x, y) = _ [xy[ es continua y no diferenciable en (0, 0) . Problema 4 Dada la funci´ on f (x, y) = _ arctg x 4 +y 4 x 2 +y 2 si (x, y) ,= (0, 0) a si (x, y) = (0, 0) a) Determinar el valor de a para que la funci´on sea continua en el origen. b) Para este valor de a calcular f x (0, 0), f y (0, 0) c) Hallar la derivada direccional D u f(1, 0), siendo ´ u un vector unitario que la forma un ´angulo de 60 o con la parte positiva del eje OX. Soluci´on: a) f(x,y) es continua en (0,0) si a= f(0,0) b) f x (0,0)=0 , f y (0,0)=0 c) D u f(1, 0) = 1 2 3.5.3. Derivadas parciales Problema 1 Sea f (x, y) = _ _ _ xy x 2 −y 2 x 2 + y 2 si (x, y) ,= (0, 0) 0 si (x, y) = (0, 0) a) Deducir que ∂f ∂x (0, y) = −y ∀y; ∂f ∂y (x, 0) = x ∀x b) Verificar que ∂ 2 f ∂y∂x (0, 0) ,= ∂ 2 f ∂x∂y (0, 0) 278 Problema 2 Dada la funci´ on f (x, y) = _ x + y si x = 0 ´o y = 0 1 si x ,= 0, y ,= 0 a) Verifique que ∂f ∂x (0, 0) = 1 ; ∂f ∂y (0, 0) = 1 b) Muestre que la funci´on f no es continua en (0, 0) Problema 3 Sea u = xy x + y , muestre que u satisface la ecuaci´ on: x 2 ∂ 2 u ∂x 2 + 2xy ∂ 2 u ∂y∂x + y 2 ∂ 2 u ∂y 2 = 0 Problema 4 Sea u = 1 _ x 2 + y 2 + z 2 , probar que u satisface la ecuaci´ on: ∂ 2 u ∂x 2 + ∂ 2 u ∂y 2 + ∂ 2 u ∂z 2 = 0 Problema 5 Enuncie las condiciones del teorema de Schwarz de igualdad de las derivadas cruzadas. Problema 6 Justifique si es cierto que una funci´on f : IR 3 → IR diferenciable en IR 3 tiene derivadas parciales. Soluci´ on Si una funci´ on f es diferenciable en − → x o y la diferencial es Df ( − → x o ) , entonces para todo vector unitario ´ v ,= 0 existe D v f ( − → x o ) que verifica D v f ( − → x o ) = Df ( − → x o ) ´ v. En particular, esto sucede para los vectores (1, 0, 0) , (0, 1, 0) , (0, 0, 1) , por lo que existen las derivadas parciales 279 Problema 7 Suponga que z es funci´ on de las variables x e y , que satisface la ecuaci´on que se da en cada caso . Encontrar las derivadas parciales de primer orden ∂z ∂x y ∂z ∂y a) x 3 + y 3 + z 3 + senxz + cos yz = 15 b) e z + x 2 ln z + y = 0. Soluci´on: a) ∂z ∂x = − (3x 2 + z cos xz) 3z 2 + x cos xz −ysenyz , ∂z ∂y = − (3y 2 −zsenxz) 3z 2 + x cos xz −ysenyz b) ∂z ∂x = − 2xz ln z ze z + x 2 , ∂z ∂y = − z ze z + x 2 Problema 8 Suponga que existen funciones u y v , que satisfacen las siguientes ecuaciones. _ ucos v = x + 1 usin y = x + y Calcular ∂u ∂x , ∂u ∂y , ∂v ∂x , ∂v ∂y Soluci´on: ∂u ∂x = cos v + senv , ∂u ∂y = senv, ∂v ∂x = cos v −senv u , ∂v ∂y = cos v u 3.5.4. Derivadas Direccionales Problema 1 Calcule la derivada direccional de la funci´ on dada en la direcci´on del vector indicado. a) f (x, y, z) = 2x 2 −y 2 −z 2 , en (1, 2, 2) hacia (4, 5, 0) . b) f (x, y, z) = 2x 2 −8xy + z 2 , en el punto (4, 4, 1) en la direcci´ on de la 280 normal exterior a la superficie x 2 + y 2 + z = 17 Soluci´on: a) D v f (1, 2, 2) = 8 √ 22 , b) D v f (4, 4, 1) = − 446 √ 129 Problema 2 Determine la ecuaci´on del plano tangente al paraboloide x 2 +y 2 +z−1 = 0 y que pasa por los puntos _ 1, 0, 1 2 _ y _ 0, 1, 1 2 _ . Soluci´on: La ecuaci´on de plano tangente es: x + y + z = 3 2 Problema 3 Sea S una superficie dada por la ecuaci´on x 3 +y 3 +z 3 −a 3 = 0, con a constante, y Π un plano tangente en P 0 = (x o , y o, z o ) ∈ S que intersec- ta los ejes coordenados X,Y,Z en α, β, γ respectivamente. Probar que α −3/2 + β −3/2 + γ −3/2 = a −3/2 es constante. Problema 4 Determinar el plano tangente a la superficie S √ x + √ y + √ z = √ a, (a > 0) en el punto P 0 = (x o , y o, z o ) ∈ S y demostrar que este plano corta los ejes coordenados en segmentos cuya suma de longitudes es constante. Soluci´on: x −x o √ x o + y −y o √ y o + z −z o √ z o = 0 l = √ a( √ x o + √ y o + √ z o ) = a, (a > 0) 281 Problema 5 Muestre que si el punto P 0 = (x o , y o, z o ) pertenece al elipsoide de ecuaci´ on: π : x 2 a 2 −α + y 2 b 2 −α + z 2 c 2 −α = 1 con (α < c 2 < b 2 < a 2 ) y al manto del hiperboloide de ecuaci´ on: θ : x 2 a 2 −β + y 2 b 2 −β + z 2 c 2 −β = 1 con (c 2 < β < b 2 < a 2 ) Entonces las superficies π y θ se cortan ortogonalmente en P 0 = (x o , y o, z o ) . Regla de la cadena Problema 1 Sean f : R 2 → R y g : R → R, con f (x, g (x)) = x sin (g (x)) . Sabiendo que g (1) = 0 y que f (1, 0) = 1, calcular g (1) . Soluci´ on: El valor de la derivada es g (1) = 1 Problema 2 Sea f : D ⊂ IR 2 → IR una funci´ on diferenciable homog´enea de grado p, es decir que verifica la condici´on f (tx, ty) = t p f (x, y) ∀t > 0, ∀ (x, y) ∈ IR 2 . a) Demuestre la ecuaci´ on x ∂f ∂x + y ∂f ∂y = pf. b) Pruebe que la siguientes funciones son homog´eneas f (x, y) = xy, f (x, y) = x 2 + 3xy + y 2 y luego verifique la ecuaci´ on anterior. Problema 3 Sea g(t) = f (x (t) , y (t)) de clase C 2 , deduzca que: g (t) = f xx (x, y) (x ) 2 +2f xy (x, y) x y +f yy (x, y) (y ) 2 +f x (x, y) x +f y (x, y) y 282 Problema 4 Sea f = f (x, y) de clase C 2 con f x (0, 1) = 2, f y (0, 1) = 1; f xx (0, 1) = 0, f xy (0, 1) = −1, f yy (0, 1) = 1.Si h(t) = f (t 2 , 1 + t 3 ) . Calcule la derivada compuesta (f ◦ − → r ) (t) y evalue h´ ´(0) = 4 Soluci´on: h (t) = f xx (x, y) (x ) 2 + 2f xy (x, y) x y +f yy (x, y) (y ) 2 +f x (x, y) x + f y (x, y) y h (0) = 4 Derivaci´on impl´ıcita Sea z = f(x, y) definida por z = u + vdonde u = u(x, y) y v = v(x, y) son funciones definidas de manera impl´ıcita por las ecuaciones F = u + e u+v −x = 0 G = v + e u−v −y = 0 Si u = v = 0 entonces x = y = 1: Calcular z x (1, 1). Soluci´ on: z x (1, 1) = 1 Problema 5 Verifique que la funci´on f (x, y) = g _ x y 2 _ satisface la ecuaci´ on: 2x ∂f ∂x + y ∂f ∂y = 0 Problema 6 Sea g _ xy z , x 2 + y 2 _ = 0 una ecuaci´ on que define a z como una funci´ on de x e y. Verifique que si g x ; g y y g z existen y son continuas en toda la regi´on en la que g z ,= 0; entonces 283 yz x −xz y = z(x 2 −y 2 ) xy Problema 7 Si F(xz, yz) = 0 define a z como funci´ on impl´ıcita de x e y y adem´ as cumple con las condiciones del teorema de la funci´on impl´ıcita en cada punto de una regi´ on R; entoncesverifique que, en R; se satisface la ecuaci´ on xz x + yz y = −z 3.5.5. Puntos cr´ıticos m´aximos y m´ınimos Problema 1 Sea f : U ⊆ R 2 →R definida en el abierto U, dada por: f(x, y) = x 4 + y 4 + 4axy + 8a 4 , (a ∈ R). Calcular los valores extremos de la funci´ on. Soluci´on: Si a = 0, entonces, el ´ unico punto cr´ıtico es P 0 = (0, 0) (m´ınimo) . Si a > 0, tenemos tres puntos cr´ıtico es P 0 = (0, 0) (punto silla), P 1 = ( √ a, − √ a), P 2 = (− √ a, √ a) (m´ınimos). Si a < 0, tenemos tres puntos cr´ıtico es P 0 = (0, 0) (punto silla), P 1 = ( √ −a, √ −a), P 2 = (− √ −a, − √ −a) (m´ınimos). Problema 2 Hallar los extremos absolutos de la funci´on f (x, y, z) = x +y +z en el conjunto D = ((x, y, z) ∈ IR 3 /x 2 + y 2 ≤ z ≤ 1) . Soluci´on: El m´ınimo de f se alcanza en _ − 1 2 , − 1 2 , 1 2 _ y vale 1 2 , y el m´ aximo se alcanza en _ − 1 √ 2 , − 1 √ 2 , 1 2 _ y vale 1 + √ 2 284 Problema 3 Sea la funci´ on f (x, y) = xy (y 2 −x 2 ) a) Determine y clasifique los puntos cr´ıticos de f b) Detemine los m´ aximos y los m´ınimos de f en la regi´ on 0 ≤ x ≤ 1; 0 ≤ y ≤ 1. Soluci´on: a) En P 0 = (0, 0) hay un punto silla. En P 1 = ( 1 √ 3 , 1) hay m´ aximo de f , y en P 2 = (1, 1 √ 3 , ) hay un m´ınimo de f b) El m´ aximo es f( 1 √ 3 , 1) = 2 3 √ 3 y el m´ınimo f(1, 1 √ 3 ) = − 2 3 √ 3 Problema 4 Calcular la distancia m´ınima del punto (0, b) a la par´abola x 2 −4y = 0. Soluci´on: d = 2 Problema 5 Calcular el paralelep´ıpedo de volumen m´ aximo que tiene una diagonal de longitud 1. Soluci´on: V _ 1 √ 3 , 1 √ 3 , 1 √ 3 _ = 1 3 √ 3 Problema 6 Hallar el punto de la elipse 3x 2 −2xy+3y 2 = 8 para la cual la respectiva recta tangente est´a a la menor distancia del origen. Soluci´on: Se tienen cuatro puntos cr´ıticos P 0 = (1, −1) , P 1 = (−1, 1) , P 2 = _√ 2, √ 2 _ , P 3 = _ − √ 2, − √ 2 _ . 285 Los puntos donde esta la menor distancia son: d _√ 2, √ 2 _ = d _ − √ 2, − √ 2 _ = 2 Los puntos donde est´ a la mayor distancia : d (1, −1) = d (−1, 1) = √ 2 Problema 7 Un canaleta cuya secci´ on transversal tiene forma de trapecio, con ´angu- los iguales en la base, se fabrica doblando bandas iguales a lo largo de ambos lados de una larga plancha de metal,de 12 pulgadas de ancho. Encuentre los ´angulos de la base y el ancho de los lados que producen la m´axima capacidad de la canaleta. Soluci´on: ´ angulos 2π 3 , ancho de lados 4 pulgadas Problema 8 Pruebe que para ´ angulos x, y, z cuya suma es x + y + z = π 2 , se cumple la desigualdad sin x sin y sin z ≤ 1 8 3.6. Aplicaciones Derivada Direccional Ejemplo 1 Suponga que la temperatura en grados Celsuis en el punto (x, y) cerca de un aeropuerto est´ a dada por f(x, y) = 1 180 [7400 −4x −9y −0, 03xy] ( con las distancias x e y medidas en kilometros). a) Suponga que un avi´ on despega del aeropuerto en la ubicaci´on P(200, 200) y se dirige al noreste en la direcci´on especificada por el vector − → v = (3, 4) .¿Cu´ al es la tasa de cambio inicial de la temperatura en la direc- ci´ on dada?. 286 b) Si el avi´on sale del aeropuerto en P y vuela en la direcci´ on − → v con una rapidez v = ds dt = 5 km/min. ¿Cu´ al es la tasa de cambio inicial de la temperatura en la direcci´on dada por unidad de tiempo? Soluci´on: a) Como f es una funci´on diferenciable en IR 2 , la derivada direccional de la funci´ on f en el punto P en la direcci´ on − → v est´ a dada por: D u f (P) = ∇f (P) ´ u Determinemos el versor ´ u = − → u | − → u | = (3, 4) √ 3 2 + 4 2 = _ 3 5 , 4 5 _ Ahora calculemos el gradiente ∇f (P) = _ 1 180 (−4 −0, 03y) , 1 180 (−9 −0, 03x) _ La derivada direccional en cualquier punto es D v f (P) = _ 1 180 (−4 −0, 03y) , 1 180 (−9 −0, 03x) _ _ 3 5 , 4 5 _ Evaluemos la derivada direccional el punto P(200, 200), se encuentra que: D v f (200, 200) = _ 3 5 _ − 10 180 _ + 4 5 _ − 15 180 __ = − 18 180 D v f (200, 200) = −0, 1 o C km Lo que significa que se observar´a una disminuci´ on de 0, 1 o C de temper- atura en esa direcci´ on por cada km de viaje. b) La tasa de variaci´on de la temperatura con respecto al tiempo est´ a dada por df dt = df ds ds dt = D v f (P) | − → v | = _ −0, 1 o C km _ (5km/min) df dt = −0, 5 km m´ın 287 Ejemplo 2 Suponga ahora que consideramos que la temperatura en grados Celsuis en el punto (x, y, z) cerca de un aeropuerto est´ a dada por f(x, y, z) = 1 180 [7400 −4x −9y −0, 03xy] −2z , donde f es la tem- peratura por kilometro de altitud. a) Si un halc´ on esta inm´ ovil en el aire, en el punto P(200, 200, 5) y sobre el aeropuerto desciende en forma s´ ubita con una rapidez de 3 km/min en la direcci´ on dada por el vector (3,4,-12) . ¿Cu´ al es la tasa de cambio instant´ anea que experimenta el ave? b) ¿En qu´e direcci´ on debe descender el halc´ on que est´ a en el punto P(200,200) a una altitud de 5 km, a fin de calentarse lo m´ as r´apido? c) ¿Qu´e tan r´ apido s´ ubira su temperatura conforme el ave baje a una rapidez de 3 km/min? d) ¿Cu´ al ser´ a la direcci´ on de la br´ ujula y el ´ angulo de descenso conforme vuele en esa direcci´ on particular? Soluci´on: a) La derivada direccional de la funci´on f en el punto P en la direcci´on − → v est´ a dada por: D u f (P) = ∇f (P) ´ u Determinemos el versor ´ u = − → u | − → u | = (3, 4, −12) _ 3 2 + 4 2 + (−12) 2 = _ 3 5 , 4 5 , − 12 13 _ El vector gradiente de temperatura es ∇f (P) = _ 1 180 (−4 −0, 03y) , 1 180 (−9 −0, 03x) , −2 _ El gradiente en la posici´on inicial es ∇f(200, 200, 5) = _ − 10 180 , − 15 180 , −2 _ Por tanto , la tasa de cambio inicial de la temperatura del ave respecto de la distancia en la direcci´ on dada es df ds = D u f (P) = ∇f (P) ´ u 288 Evaluemos la derivada direccional la posici´ on inicial P(200, 200, 5), D v f (200, 200, 5) = _ 3 13 _ − 10 180 _ + 4 13 _ − 15 180 _ −2 _ − 12 13 __ = 47 26 Como la rapidez del ave es ds dt =3 km/min, entonces la tasa de cambio por unidad de tiempo que experimenta el halc´on es df dt = df ds ds dt = D v f (P) | − → v | = _ 47 26 o C km _ (3km/min) ≈ 5, 4 o C m´ın c) El valor m´ aximo est´a dado por df ds = [D v f (P)] m´ax = |∇f (P)| = ¸ _ − 10 180 _ 2 + _ − 15 180 _ 2 + (−2) 2 = 2,0025 Y se alcanza en en la direcci´ on dada por el gradiente ´ n = ∇f (P) |∇f (P)| = (−10, −15, −360) 360, 45 La rapidez del halc´ on es 3 , por lo que la tasa temporal de cambio de la temperatura en esa direcci´on experimentada por el halcon es df dt = df ds ds dt = D v f (P) | − → v | = _ 2, 0025 o C km _ (3km/min) ≈ 6, 0075 o C m´ın 3.7. Aplicaciones de M´aximos y M´ınimos 3.7.1. Aplicaci´on al campo de la mec´anica Sea − → F : D →R 3 un campo de Fuerza definido en cierto dominio D de R 3 . Consideremos ahora una part´ıcula de masa m que se mueve a lo largo de una trayectoria − → r (t) bajo la acci´ on de este campo de fuerza. 289 La ecuaci´on de movimiento de la part´ıcula esta dada por la segunda ley de Newton: − → F ( − → r (t)) = m − → r ” (t) (∗) Si el campo vectorial − → F es conservativo, esto es, − → F = −∇V , entonces: 1 2 m| − → r (t)| + V ( − → r (t)) = c donde c es una constante. El primer t´ermino se llama energ´ıa cin´etica y el segundo corresponde a la funci´ on potencial V. Si diferenciamos la expresi´ on anterior usando la regla de la cadena: d dt _ 1 2 m| − → r (t)| + V ( − → r (t)) _ = m − → r (t) − → r (t)+∇V ( − → r (t)) − → r (t) = 0 _ m − → r (t) +∇V ( − → r (t)) ¸ − → r (t) = 0 ⇐⇒ m − → r (t) +∇V ( − → r (t)) = 0 Por tanto, m − → r (t) = −∇V ( − → r (t)) = − → F ( − → r (t)) lo que demuestra la ecuaci´ on de Newton (∗) . Un punto − → r o ∈ D se llama posici´on de equilibrio si la fuerza en ese punto es cero: − → F ( − → r o ) = 0.Un punto − → r o que sea de equilibrio se llama estable si para todo ρ > 0 y ε > 0, podemos escoger n´ umeros ρ o > 0 y ε o > 0 tales que un punto situado en cualquier lugar a una distancia menor que ρ o de − → r o ,despu´es de recibir inicialmente energ´ıa cin´etica en una cantidad menor que ε o , permanecer´ a para siempre a una distancia de − → r o menor que ρ y poseera energ´ıa cin´etica menor que ε. As´ı, si tenemos una posici´on de equilibrio, la estabilidad en − → r o sig- nifica que una part´ıcula que se mueve lentamente cerca de − → r o siempre permanecer´ a cerca de − → r o y se mantendr´a moviendose lentamente. Ahora, si tenemos un punto de equilibrio inestable − → r o , entonces − → r (t) = − → r o resuelve la ecuaci´on de Newton − → F ( − → r (t)) = m − → r ” (t) , pero las soluciones cercanas pueden alejarse de − → r o conforme trascurra el tiempo. Proposici´on 3.7.1. i) Los puntos cr´ıticos de un potencial son posi- ciones de equilibrio. ii) En un campo conservativo, un punto − → r o en el cual el potencial alcance un m´ınimo local estricto, es una posici´on de equilibrio estable. 290 Demostraci´ on 1) La primera afirmaci´ on es bastante obvia debido a la definici´ on de campo conservativo: − → F = −∇V, los puntos de equilibrios − → r o son exactamente los puntos cr´ıticos de V, en los cuales ∇V ( − → r o ) = 0 2) Para probar la afirmaci´ on ii), haremos uso de la ley de conservaci´ on de energ´ıa . Tenemos 1 2 m| − → r (t)| + V ( − → r (t)) = 1 2 m| − → r (0)| + V ( − → r (0)) Escojamos un peque˜ na vecindad de − → r o , y asumamos que la part´ıcula tienen poca energ´ıa cin´etica. Conforme t crece, la part´ıcula se aleja de − → r o sobre una trayectoria − → r (t) y V ( − → r (t)) crece pues V ( − → r (0)) es un m´ınimo estricto , de modo que la energ´ıa cin´etica debe decrecer . Si la energ´ıa cinetica inicial es suficientemente peque˜ na, entonces, para que la part´ıcula escape de la vecindad de − → r o , fuera de la cual V ha crecido en una cantidad definida, la energia cinetica tendria que volverse negativa, lo cual es imposible. Asi, la particula no puede escapar de la vecindad. Sea una part´ıcula en un campo de potencial V restringido a manten- erse sobre la superficie de nivel S dada por la ecuaci´on φ(x, y, z) = 0 con ∇φ ,= 0. Si en la ecuaci´ on de Newton − → F ( − → r (t)) = m − → r ” (t) (∗) , reemplazamos − → F con la componente de − → F paralela a S, aseguramos que la part´ıcula permanecer´a en S. Proposici´on 3.7.2. i) Si en un punto P sobre la superficie S el poten- cial V [ S tiene un valor extremo, entonces el punto P es una posici´on de equilibrio sobre la superficie. ii) Si un punto P ∈ S es un m´ınimo local estricto del potencial V[ S , entonces el punto P es una posicion de equilibrio estable. Ejemplo Sea el campo gravitacional cerca de la superficie de la tier- ra; esto es, sea − → F = (0, 0, −mg) donde g es la aceleraci´on de gravedad . Determine la funci´on potencial gravitacional y ¿cu´ ales son las posi- ciones de equilibrio, si una part´ıcula con masa m esta restrigida a la esfera g (x, y, z) = x 2 +y 2 +z 2 −r 2 = 0, (r > 0)?¿Cu´ ales son estables?. Soluci´ on: 291 Tenemos que F z = − ∂V ∂z = mg =⇒ V (x, y, z) = mgz Usando el m´etodo de los multiplicadores de Lagrange podemos localizar los extremos posibles, tenemos que: L(x, y, z, λ) = V (x, y, z) + λg (x, y, z) L(x, y, z, λ) = mgz + λ _ x 2 + y 2 + z 2 −r 2 _ L x (x, y, z, λ) = 2λx = 0 =⇒λ ,= 0 y x = 0 (1,0) L y (x, y, z, λ) = 2λy = 0 =⇒λ ,= 0 y y = 0 (2,0) L z (x, y, z, λ) = mg + 2λz = 0 =⇒ λ = − mg 2z y z ,= 0 (3,0) L λ (x, y, z, λ) = x 2 + y 2 + z 2 −r 2 = 0 ( 4,0) Reemplazando (1,0), (2,0), (3,0) en ( 4,0) z 2 −r 2 = 0 =⇒ z = ±r y λ = − mg 2r Luego, se deduce que los puntos P 1 = (0, 0, r) y P 2 = (0, 0, −r) son posiciones de equilibrio. 3.7.2. Aplicaciones a la geometr´ıa Ejemplo 1 Determine la distancia m´ınima desde el origen (0,0,0) a la superficie S del elipsoide x 2 + y 2 4 + z 2 9 = 1. Soluci´on: Sabemos que la distancia entre un punto P y el origen est´a dada por la funci´on d(x, y, z) = _ x 2 + y 2 + z 2 . Sin embargo, por razones de simplicidad en los c´ alculos , en lugar de la funci´ on anterior vamos a considerar la funci´ on f(x, y, z) = x 2 + y 2 + z 2 en atenci´on a que f tendra un m´ınimo en un punto si y solo si d lo tiene. Se trata de obtener los extremos condicionados de la funci´on distancia f(x, y, z) = x 2 + y 2 + z 2 sujeta a la condici´ on x 2 + y 2 4 + z 2 9 = 1. Formemos la funci´ on auxiliar de Lagrange F(x, y, z, λ) = (x 2 + y 2 + z 2 ) + λ(x 2 + y 2 4 + z 2 9 −1) 292 y consideremos entonces el sistema. F x (x, y, z, λ) = 2x + 2λx = 0 F y (x, y, z, λ) = 2y + 1 2 λy = 0 F z (x, y, z, λ) = 2z + 2 9 λz = 0 F λ (x, y, z, λ) = x 2 + y 2 4 + z 2 9 −1 = 0 De las tres primeras ecuaciones obtenemos 2x(1 + λ) = 0 =⇒ x = 0 ∨ λ = −1 y(2 + 1 2 λ) = 0 =⇒ y = 0 ∨ λ = −4 2z(1 + 1 9 λ) = 0 =⇒ z = 0 ∨ λ = −9 Sustituyendo y = z = 0 en la cuarta ecuaci´ on produce x 2 −1 = 0 =⇒ x = ±1 . Reemplazando x = z = 0 en la cuarta ecuaci´ on produce y 2 4 −1 = 0 =⇒ y = ±2 Sustituyendo x = y = 0 en la cuarta ecuaci´ on produce z 2 9 −1 = 0 =⇒ z = ±3 Luego , se obtienen seis puntos cr´ıticos P 0 = (1, 0, 0) , P 1 = (−1, 0, 0) , P 2 = (0, 2, 0) , P 3 = (0, −2, 0) P 4 = (0, 0, 3) , P 5 = (0, 0, −3) . Evaluando la funci´ on en los puntos encontrados deber´a haber un m´ axi- mo y un m´ınimo f(±1, 0, 0) = 1, f(0, ±2, 0) = 4, f(0, 0, ±3) = 9 Se tiene que el m´ınimo de f se encuentra en los puntos (±1, 0, 0) y es igual 1. y el m´aximo est´ a localizado en los puntos (0, 0, ±3) y vale 9. 293 Ejemplo 2 Determine la distancia m´ınima y m´axima del origen a la curva de interseccion del paraboloide x 2 + y 2 + z − 7 4 = 0 y el plano x + y + z −2 = 0. Soluci´on: Igual que el ejemplo anterior resulta mas conveniente hallar los ex- tremos del cuadrado de la distancia respecto del origen en vez de la funci´ on distancia d (x, y, z) = _ x 2 + y 2 + z 2 Por consiguiente, se trata de obtener los extremos condicionados de la funci´on distancia f(x, y, z) = x 2 + y 2 + z 2 sujeta a las condiciones g(x, y, z) = x 2 + y 2 + z − 7 4 = 0 y h(x, y, z) = x + y + z −2 = 0 Formemos la funci´ on auxiliar de Lagrange F(x, y, z, λ 1 , λ 2 ) = (x 2 +y 2 +z 2 )+λ 1 _ x 2 + y 2 + z − 7 4 _ +λ 2 (x + y + z −2) y consideremos entonces el sistema. F x (x, y, z, λ 1 , λ 2 ) = 2x(1 + λ 1 ) + 2λ 2 = 0 (1) F y (x, y, z, λ 1 , λ 2 ) = 2y(1 + λ 1 ) + 2λ 2 = 0 (2) F z (x, y, z, λ 1 , λ 2 ) = 2z + λ 1 + λ 2 = 0 (3) F λ 1 (x, y, z, λ 1 , λ 2 ) = x 2 + y 2 + z − 7 4 = 0 (4) F λ 2 (x, y, z, λ 1 , λ 2 ) = x + y + z −2 = 0 (5) De las dos primeras ecuaciones se obtiene λ 1 = −1 ´o y = x Consideremos primero el caso λ 1 = −1.A partir de (1) se obtiene λ 2 = 0 . Sustituyendo estos valores en (3) z = 1 2 . Reemplazando z en (4) y (5), produce x 2 + y 2 − 5 4 = 0 x + y − 3 2 = 0 Resolviendo el sistema se obtienen los puntos cr´ıticos 294 P 0 = _ 1, 1 2 , 1 2 _ , P 1 = _ 1 2 , 1, 1 2 _ Al evaluar ambos puntos en la funci´on distancia, obtenemos d (x, y, z) = _ 1 + 1 4 + 1 4 = _ 3 2 Consideremos ahora y = x , a partir de las ecuaciones (4) y (5) se obtiene 2x 2 + z − 7 4 = 0 2x + z −2 = 0 Resolviendo el sistema se obtienen los puntos cr´ıticos P 3 = _ 1 2 ± √ 2 4 , 1 2 ± √ 2 4 , 1 ∓ √ 2 2 _ , Al evaluar todos estos puntos en la funci´ on distancia, obtenemos d _ 1 2 ± √ 2 4 , 1 2 ± √ 2 4 , 1 ∓ √ 2 2 _ = 1 2 _ 9 ∓2 √ 2 Como la curva de intersecci´ on del paraboloide y el plano es cerrada, las distancias m´axima y minima absoluta del la curva al origen son: d m´ ax = 1 2 _ 9 + 2 √ 2, d m´ın = _ 3 2 Ejemplo 3 Encuentre el volumen m´aximo de una caja rectangular inscrita en el elipsoide x 2 a 2 + y 2 b 2 + z 2 c 2 = 1 con sus caras paralelas a los planos coordenados. Soluci´on: Sea P(x,x,z) el v´ertice de la caja que esta en el primer octante donde x > 0, y > 0, z > 0. Por la simetria del problema se desea maximizar la funci´on volumen f(x, y, z) = 8xyz sujeta a la condici´on g(x, y, z) = x 2 a 2 + y 2 b 2 + z 2 c 2 −1 = 0 Formemos la funci´ on auxiliar de Lagrange F(x, y, z, λ) = 8xyz + λ _ x 2 a 2 + y 2 b 2 + z 2 c 2 −1 _ 295 y consideremos entonces el sistema. F x (x, y, z, λ) = 8yz + 2λ a 2 x = 0 (1) F y (x, y, z, λ) = 8xz + 2λ b 2 y = 0 (2) F z (x, y, z, λ) = 8xy + 2λ c 2 z = 0 (3) F λ (x, y, z, λ) = x 2 a 2 + y 2 b 2 + z 2 c 2 −1 = 0 (4) Multiplicando las ecuaciones (1), (2)y (3) por x, y, x respectivamente, produce 8xyz + 2λ a 2 x 2 = 0 (1,1) 8xyz + 2λ b 2 y 2 = 0 (2,2) 8xyz + 2λ c 2 z 2 = 0 (3,3) Entonces , obtenemos 2λ a 2 x 2 = 2λ b 2 y 2 = 2λ c 2 z 2 = −8xyz Para obtener el volumen m´ aximo se requiere que x, y, x ,= 0 y λ ,= 0 Concluimos entonces que x 2 a 2 = y 2 b 2 = z 2 c 2 Sustituyendo esta expresi´on en la ecuaci´ on (4), obtenemos 3 x 2 a 2 −1 = 0 =⇒ x = a √ 3 As´ı, sucesivamente se tiene un ´ unico punto cr´ıtico P 0 = _ a √ 3 , b √ 3 , c √ 3 _ Por lo tanto , la caja tiene un volumen m´ aximo dado por f m´ax _ a √ 3 , b √ 3 , c √ 3 _ = 8 3 √ 3 abc 296 3.7.3. Aplicaci´on al campo de la econom´ıa Supongase que la producci´ on de cierto producto de una compa˜ n´ıa man- ufacturera es una cantidad Q, donde Qes una funci´ on de f(K, L) donde K es la cantidad de capital (o inversi´on) y L es la cantidad de trabajo realizado. Si el precio del trabajo es p, el precio del capital es q y la compa˜ nia no puede gastar m´as de B d´ olares, ¿c´ omo podemos hallar la cantidad de capital y de trabajo que maximice la producci´ on Q? Soluci´on: Se esperar´ıa que si se incrementa la cantidad de capital o de trabajo, entonces la producci´on deber´a incrementarse; esto es: ∂Q ∂K ≥ 0 y ∂Q ∂L ≥ 0 Tambi´en se esperaria que conforme se a˜ nada trabajo a una cantidad dada de capital, obtendremos menos productos adicionales por nuestro esfuerzo, esto es: ∂ 2 Q ∂K 2 < 0 De manera an´ aloga, ∂ 2 Q ∂L 2 < 0 Con estas hip´ otesis sobre Q , es razonable esperar que las curvas de nivel de la producci´on- llamadas isocuantas- Q(K, L) = c, se vean como las esbozadas en la figura, con c 1 < c 2 < c 3. pendiente imagen Podemos interpretar la convexidad de las isocuantas como sigue: si nos movemos hacia la derecha a lo largo de una isocuanta dada , se em- plea m´ as capital para reemplazar una unidad de trabajo y producir la misma cantidad. La restricci´ on de presupuesto significa que debe- mos mantenernos dentro del tri´angulo acotado por los ejes y la recta pL + qK = B. Geometricamente, es claro que producimos m´as al gas- tar nuestro dinero de tal manera que seleccionemos la isocuanta que solamente toca, pero no cruza, la recta presupuesto. 297 Como el punto m´ aximo est´ a en la frontera de nuestro dominio, aplicare- mos el m´etodo de los multiplicadores de Lagrange para hallar el m´ axi- mo. Para maximizar Q = f(K, L) sujeto a la restricci´on pL+qK = B, buscamos los puntos cr´ıticos de la funci´ on auxiliar H (K, L, λ) = f (K, L) + λ(pL + qK −B) As´ı queremos: H K (K, L, λ) = f K (K, L) + λq = 0 =⇒ f K (K, L) = Q K (K, L) = −λq H L (K, L, λ) = f L (K, L) + λp = 0 =⇒ f L (K, L) = Q L (K, L) = −λp H λ (K, L, λ) = pL + qK −B = 0 =⇒ pL + qK −B = 0 Con estas ecuaciones podemos encontrar los puntos cr´ıticos de la fun- ci´ on Q. Luego, usando derivaci´ on implicita podemos determinar el pun- to donde se maximiza la producci´ on. En el ejemplo anterior , λ representa algo interesante. M´as adelante vamos a interpretar λ. Sean k = qK y l = pL, de modo que k es el valor en d´ olares del capital empleado, y l es el valor en dolares del trabajo empleado, entonces las ecuaciones se convierten en: H (k, l, λ) = f (k, l) + λ(l + k −B) H k (k, l, λ) = f k (k, l) + λ = 0 =⇒ f k (k, l) = Q k (k, l) = −λ H l (k, l, λ) = f l (k, l) + λ = 0 =⇒ f l (k, l) = Q l (k, l) = −λ H λ (k, l, λ) = λl + k −B = 0 =⇒ λl + k −B = 0 Comparando con las primeras dos ecuaciones del caso anterior se tiene que: ∂Q ∂k = 1 q ∂ ∂K = −λ = ∂Q ∂l = 1 p ∂ ∂K As´ı , en el punto ´ optimo de producci´ on , el cambio marginal en la pro- ducci´ on por d´olar de inversi´ on de capital adicional, es igual al cambio marginal de la producci´on por d´ olar de trabajo adicional, y λ es este 298 valor com´ un. En el punto ´optimo, el intercambio de un d´ olar de capi- tal por un d´olar de trabajo no cambia la producci´ on. Fuera del punto ´ optimo, la producci´on marginal es distinta, y un intercambio, o el otro, incrementan la producci´on. Ejemplo Suponga que la producci´on total de una compa˜ nia est´ a da- da por la funci´ on P = P (K, L) = K 1/4 L 1/2 donde K denota el n´ umero de unidades de capital usado y L es el n´ umero de unidades de trabajo usado. a) Suponga que cada unidad de capital K cuesta 1 mill´ on de d´olares y cada unidad de trabajo L cuesta 7 millones de d´olares.Considere el problema de maximizar la producci´on total P si el presupuesto total es de 10 millones de d´ olares. b) Suponga que el capital K aumenta en una raz´ on de 4 % por a˜ no, y el trabajo L aumenta en una raz´on de 5 % por a˜ no. Determine la raz´on de crecimiento de la producci´ on P por a˜ no. Soluci´on: Tenemos que extremar P = P (K, L) = K 1/4 L 1/2 bajo la condici´ on K + 7L −10 = 0 Sea la funci´ on auxiliar de Lagrange F (K, L.λ) = K 1/4 L 1/2 + λ(K + 7L −10) y consideremos entonces el sistema. F K (K, L.λ) = 1 4 K −3/4 L 1/2 + λ = 0 (1) F L (K, L.λ) = 1 2 K 1/4 L −1/2 + 7λ = 0 (2) F λ (K, L.λ) = K + 7L −10 = 0 (3) Multiplicando (1,0) por 4K y (2,0) por 2L, produce: K 1/4 L 1/2 + 4Kλ = 0 (1,1) K 1/4 L −1/2 + 14Lλ = 0 (2,1) Restando las ´ ultimas ecuaciones tenemos: 299 4Kλ −14Lλ = 0 Entonces 2λ(2K −7L) = 0 =⇒ λ = 0´ o (2K −7L) = 0 Si λ = 0 nos quedamos sin restricion, luego λ ,= 0y(2K −7L) = 0 =⇒ K = 7 2 L Reemplazando el resultado anterior en (3,0) ,queda 7 2 L + 7L = 10 =⇒ L = 20 21 entonces K = 10 3 Asi, se tienen un ´ unico punto cr´ıtico en P 0 = _ 10 3 , 20 21 _ . Por tanto, la produccion maximizada ser´a P _ 10 3 , 20 21 _ = _ 10 3 _ 1/4 _ 20 21 _ 1/2 ii) Supongamos que K y L aumentan en una raz´on de 4 % y 5 %, respectivamente, por a˜ no dK dt = 0, 04K (1,0) dL dt = 0, 05L (2,0) Usando la regla de la cadena sobre la funci´on P (K, L) tenemos: dP dt = ∂P ∂K ∂K ∂t + ∂P ∂L ∂L ∂t dP dt = 1 4 K −3/4 L 1/2 ∂K ∂t + 1 2 K 1/4 L −1/2 ∂L ∂t (3,0) Sustituyendo (1,0) y (2,0) en (3,0) dP dt = 1 4 K −3/4 L 1/2 (0, 04K) + 1 2 K 1/4 L −1/2 (0, 05L) dP dt = K 1/4 L 1/2 _ 1 4 0, 04 + 1 2 0, 05 _ 300 Reemplazado en la expresion anterior P = K 1/4 L 1/2 Finamente, se tiene: dP dt = 0, 035P,lo que significa que la producci´on aumenta 3,5 % por a˜ no. Ejemplo Una compa˜ nia planea gastar 10.000 d´ olares en publicidad. Cuesta 3.000 d´olares un minuto de publicidad en la televisi´ on y 1.000 d´ olares un minuto de publicidad en la radio. Si la empresa compra x minutos de comerciales en televisi´ on e y minutos de comerciales en la radio, su ingreso, en miles de d´ olares, esta dado por f (x, y) = −2x 2 − y 2 + xy + 8x + 3y. ¿Como puede la empresa maximizar su ingreso invirtiendo en la publicidad? Soluci´on: Se desea maximizar la funci´on f (x, y) = −2x 2 −y 2 +xy + 8x + 3y bajo la restricci´ on g(x, y) = 3x + y −10 = 0. Consideremos la funci´on auxiliar L(x, y, λ) = −2x 2 −y 2 + xy + 8x + 3y + λ(3x + y −10) Primero,apliquemos la condici´ on necesaria de punto cr´ıtico ∇L(x, y, λ) = 0. L x (x, y, λ) = −4x + y + 8 −3λ = 0 (1) L y (x, y, λ) = −2y + x + 3 −λ = 0 (2) L λ (x, y, λ) = 3x + y −10 = 0 (3) De la ecuaciones (1) y (2)se obtiene y = 3λ + 4x −8 (1,1) , x = λ + 2y −3 (2,1) Reemplazando (2,1) en (1,1) queda: y = 3λ + 4 (λ + 2y −3) −8 =⇒ y = 7λ + 8y −80 Luego, se tiene y = 20 7 −λ (1,2) Asi, (1,2) en (2,1) produce: 301 x = 19 7 −λ (2,2) Sustituyendo (1,2) y (2,2) en (3,0) , obtenemos: 4λ −1 = 0 =⇒ λ = 1 4 . Entonces (1,2) y (2,2) nos da un ´ unico punto cr´ıtico P 0 = _ 73 28 , 69 28 _ . Determinemos la naturaleza de este punto critico usando el criterios de la segunda derivada. El hessiano para f (x, y) es H _ 73 28 , 69 29 _ = ¸ ¸ ¸ ¸ f xx f xy f xy f yy ¸ ¸ ¸ ¸ = ¸ ¸ ¸ ¸ −4 1 1 −2 ¸ ¸ ¸ ¸ = 7 > 0 y f xx _ 73 28 , 69 28 _ = −4 < 0 Por lo tanto, la funci´ on es m´ axima en el punto P 0 = _ 73 28 , 69 28 _ . As´ı , la empresa tendria que comprar 73 28 minutos de comerciales en televisi´ on y 69 28 minutos de comerciales en radio. 3.7.4. Problemas Propuestos de Aplicaciones 1.- Sea una part´ıcula que se mueve en un campo de potencial en R 2 dado por V (x, y) = 3x 2 + 2xy + 2x + y 2 + y + 4. Hallar los puntos de equilibrio estable si los hay. Soluci´on: Hay un ´ unico punto de equilibrio estable en _ − 1 4 , 1 4 _ . 2.- Sea una part´ıcula movi´endose en un campo de potencial en R 2 dado por V (x, y) = x 2 + 4xy − y 2 − 8x − 6y. Hallar todos los puntos de equilibrio.¿Cu´ ales, si los hay, son estable? Soluci´on : Hay un ´ unico punto de equilibrio inestable en (2, 1) . 3.- Sea una part´ıcula restringida a moverse sobre la esfera x 2 +y 2 +z 2 = 1, sujeta a fuerzas gravitacionales, asim como al potencial adicional V (x, y, z) = x + y.Hallar los puntos de equilibrio estable, si los hay. 302 Soluci´on : Hay un ´ unico punto de equilibrio estable en (2 + m 2 g 2 ) −1/2 (−1, −1, −mg) . 4.- Usando la informaci´on anterior , encuentre el punto ´ optimo para la funci´ on de producci´ on Q(K, L) = AK α L 1−α , donde A y α son con- stantes positivas y 0 < α < 1, que se usa para modelar la econonom´ıa nacional. Q es, entonces, la producci´ on agregada de la econom´ıa para una entrada de capital y trabajo dada . Soluci´on: En el ´ optimo: KQ α = pL 1 −α 5.- Una compa˜ nia usa aluminio, hierro y magnesio para producir acc- cesorios de autom´ oviles. La cantidad de accesorios que puede producir usando x toneladas de aluminio, y toneladas de hierro y z toneladas de magnesio es Q(x, y, z) = xyz. El costo de la materia prima es: aluminio 6 d´olares por tonelada; hierro 4 d´olares por tonelada ; y magnesio 8 dolares por tonelada. ¿Cu´ antas toneladas de aluminio, hierro y magne- sio deber´ an usarse para manufacturar 1000 accesorios al menor costo posible? Soluci´on: x = 20 3 3 √ 3 , y = 10 3 √ 3, z = 5 3 √ 3 6.- Una Pyme cuenta con 8.000 d´olares para importar dos tipos de bebidas energ´eticas . Si x son la unidades de bebidas energ´eticas que se importar´ an desde Holanda, y se estima que vender´ an 12x x + 6 unidades de esta bebidas a un precio de 200 d´olares cada una. Si y son la unidades de bebidas energ´eticas que se importar´an desde Alemania, estimandose que vender´ an 24y y + 3 unidades, a un precio de 200 d´ olares cada una. Si el costo por unidad vendida de cada bebida es de 50 d´ olares. a) Determine cuantas unidades de cada bebida energ´etica deben im- portar para maximizar su utilidad. b) Determine la utilidad m´ axima. Soluci´on:La funci´on utilidad esta dada por la diferencia entre el precio de venta de las bebidas y el costo de importaci´ on a) 78, 5 bebidas Holandesas y 81, 5 bebidas Alemanas. b) Utilidad U(78, 5; 81, 5) = 5144, 7 d´olares. 303 7.- La funci´on de producci´ on de una compa˜ n´ıa es Q(x, y) = xy. El costo de producci´on es C (x, y) = 2x+3y.Si esta compa˜ n´ıa puede gastar C (x, y) = 10, ¿cu´ al es la m´axima cantidad que puede producir? Soluci´on: P _ 5 2 , 5 3 _ = 25 6 3.8. Auto evaluaciones Autoevaluaci´ on N o 1 El estudiante: 1) Determinar´ a si una funci´ on f : D ⊆ R 2 −→ R definida en el conjun- to abierto D, es o no diferenciable en un punto del dominio, empleando la definici´ on de diferenciabilidad. 2) Calcular´ a la derivada direccional m´ axima de una funci´on f diferenciable en un punto evaluando el gradiente y el vector unitario necesario. 3) Utilizar´ a el m´etodo de los multiplicadores de Lagrange para localizar los posibles valores extremos de una funci´on f : Ω ⊆ R N −→R, definida en un conjunto abierto Ω, sujeta a una o dos condiciones g m (x) = 0 , m = 1,2 y m < N Tiempo: 2 horas Pregunta 1 Estudie la diferenciabilidad de la funci´ on f(x; y) = x 2 +2y 2 , en el punto (x 0 ; y 0 ) = (1; 2). Pregunta 2 Hallar los coeficientes a y b para que la derivada direccional m´ axima de la funci´ on e ax+by cos(x + y) − z = 0 en el punto (0, 0) sea 3 √ 2 en la direcci´ on de la bisectriz del primer cuadrante. Pregunta 3 Calcular la m´ınima distancia desde el origen (0, 0) hasta la recta 2x + y = 1,usando el teorema de los Multiplicadores de Lagrange. Pauta Autocorrecci´ on 304 Pregunta 1 En primer lugar, sus derivadas parciales en el punto (x 0 ; y 0 ) = (1; 2) existen y valen ∂f ∂x (x, y) = 2x =⇒ ∂f ∂x (1, 2) = 2 ∂f ∂y (x, y) = 4y =⇒ ∂f ∂y (1, 2) = 8 En segundo lugar, para mostrar que f es diferenciable tenemos que demostrar que se cumple l´ım (x,y)→(x 0 ,y 0 ) f (x, y) −f (x 0 , y 0 ) + ∂f ∂x (x 0 , y 0 ) (x −x 0 ) + ∂f ∂y (x 0 , y 0 ) (y −y 0 ) _ (x −x 0 ) 2 + (y −y 0 ) 2 = 0 En efecto, tenemos que mostrar que l´ım (x,y)→(1,2) (x 2 + 2y 2 ) −(9 + 2(x −1) + 8(y −2)) _ (x −1) 2 + (y −2) 2 = 0 Para demostrar esto , traslademos el origen usando el cambio de variables u = (x −1) y v = (y −2) .De este modo, tenemos que l´ım (u,v)→(0,0) ((u + 1)2 + 2(v + 2)2) −(9 + 2u + 8v) √ u 2 + v 2 si se desarrollan los par´entesis, se obtiene l´ım (u,v)→(0,0) u 2 + 2v 2 √ u 2 + v 2 Si nos acercamos al origen por una recta v = ku, obtenemos l´ım (u,v)→(0,0) u 2 + 2v 2 √ u 2 + v 2 = l´ım u→0 u 2 (1 + 2k 2 ) u √ 1 + k 2 = 0 Esto no prueba que el valor del l´ımite sea cero. Lo que nos indica es que si tal l´ımite existe, debe ser cero. As´ı, se requiere la aplicaci´on directa de la definici´on de l´ımite para concluir que ´este existe y vale cero Para aplicar la definici´ on se tiene que ¸ ¸ u 2 + 2v 2 ¸ ¸ ≤ ¸ ¸ 2u 2 + 2v 2 ¸ ¸ ≤ 2 ¸ ¸ u 2 + v 2 ¸ ¸ 305 Luego ¸ ¸ ¸ ¸ u 2 + 2v 2 √ u 2 + v 2 −0 ¸ ¸ ¸ ¸ ≤ ¸ ¸ ¸ ¸ 2(u 2 + v 2 ) √ u 2 + v 2 ¸ ¸ ¸ ¸ = 2 √ u 2 + v 2 vemos que √ u 2 + v 2 < δ =⇒ ¸ ¸ ¸ ¸ 2(u 2 + v 2 ) √ u 2 + v 2 ¸ ¸ ¸ ¸ = √ u 2 + v 2 < δ 2 Es decir, con δ = 2 nos queda |(u, v) −(0, 0)| < δ =⇒ ¸ ¸ ¸ ¸ u 2 + 2v 2 √ u 2 + v 2 −0 ¸ ¸ ¸ ¸ < lo que nos dice que efectivamente l´ım (u,v)→(0,0) u 2 + 2v 2 √ u 2 + v 2 = 0 Por lo tanto la funci´ on f es diferenciable en el punto (1, 2) . Pregunta 2 La funci´on z = e ax+by cos(x + y) es continua en todo IR 2 por ser com- posici´on de funciones continuas. z/ x = ∂f ∂x = ae ax+by cos(x + y) −e ax+by sen(x + y) z/ y = ∂f ∂y = be ax+by cos(x + y) −e ax+by sen(x + y) Adem´ as, las derivadas parciales son continuas en todo IR 2 .Por tanto la funci´ on es diferenciable en todo en todo IR 2 . Esto significa que la derivada direccional en un punto P (x, y) en una direcci´ on ´ u se puede obtener como el producto escalar del gradiente de la funci´ on en el punto y el versor que se˜ nala la direcci´ on en el punto considerado D u f (x, y) = ∇f (x, y) ´ u = 3 √ 2 Por otro lado, el gradiente de f nos da la direcci´on hacia donde la derivada direccional es m´axima, que en este caso corresponde a la bisectriz del primer cuadrante, luego tenemos: |∇f (0, 0)| = 3 √ 2 y ´ u = ∇f (0, 0) |∇f (0, 0)| = _ √ 2 2 , √ 2 2 _ 306 Calculando el gradiente en el punto P(x, y) se tiene: ∇f (x, y) = (ae ax+by cos(x + y) −e ax+by sen(x + y), be ax+by cos(x + y) −e ax+by sen(x + y)) Evaluando en el origen, obtenemos: ∇f (0, 0) = a ´ i + b ´ j se tiene que cumplir que: √ a 2 + b 2 = 3 √ 2 y ´ u = _ a 3 √ 2 , b 3 √ 2 _ = _ √ 2 2 , √ 2 2 _ =⇒ a = b Por lo tanto, al sustituir el resultado en la primera ecuaci´ on, queda: a = b = 3 Pregunta 3 Hay que calcular los m´ınimos de la funci´on d (x, y) = _ x 2 + y 2 bajo la condici´ on 2x + y −1 = 0. Sin embargo, podemos simplificar c´ alculos si se trabaja con la funci´ on f (x, y) = x 2 + y 2 , puesto que los m´aximos y min´ımos de ambas funciones coinciden. Determinemos los puntos cr´ıticos usando la funci´ on de Lagrange Sea la funcion de Lagrange F (x, y, λ) = x 2 + y 2 + λ(2x + y −1) , aplicando la condicion necesaria de punto critico , se tiene F x (x, y, λ) = 2x + 2λ = 0 =⇒ x = −λ F y (x, y, λ) = 2y + λ = 0 =⇒ y = − λ 2 F λ (x, y, λ) = 2x + y −1 = 0 Sustituyendo los dos primeros resultados en la tercera ecuaci´ on, produce x = 2 5 , y = 1 5 , λ = − 2 5 . Entonces, existe un ´ unico punto cr´ıtico, que es P = _ 2 5 , 1 5 _ . Determinemos si es m´ aximo o m´ınimo, usando la funci´on de Lagrange para el valor λ obtenido F (x, y) = x 2 + y 2 + 2 3 (2x + y −1) Derivando parcialmente, queda F x (x, y) = 2x + 4 3 =⇒ F xx (x, y) = 2, F xy (x, y) = 0 F y (x, y) = 2y + 2 3 =⇒ F yy (x, y) = 2 307 Luego, el determinante de la matriz Hessiana es: H (x, y) = ¸ ¸ ¸ ¸ 2 0 0 2 ¸ ¸ ¸ ¸ = 4 ,= 0. Como F xx = 2 > 0 , se trata de un m´ınimo de F. Autoevaluaci´ on N o 2 El estudiante: 1) Determinar´ a la continuidad de una funci´ on f : R 2 → R, dada f(x, y) y un punto (x 0 , y 0 ) del dominio. 2) Evaluar´ a las derivadas parciales, de primer y segundo orden, de una funci´ on f(x, y) en un punto dado del dominio , por definici´ on. 3) Calcular´ a las derivadas parciales de primer y segundo orden de una funci´ on f(x, y) usando el ´ algebra de derivadas. 4) Emplear´ a la definici´on de derivada direccional para determinar la ex- istencia y variaci´ on de una funci´ on f en un punto (x, y) de su dominio en una direcci´on ´ v, 5) Emplear´a el teorema de la funci´ on implicita, en la vecindad de un punto (x 0 , y 0 ) del dominio y calcular´a las derivadas de una funci´ on impl´ıcita de primer y segundo orden. 6) Usar´ a la regla de la cadena para probar que una funci´ on compuesta es soluci´ on de una ecuaci´on diferencial parcial. Tiempo: 2 horas Pregunta 1 Dada la funci´ on f (x, y) = _ xsin(xy) x 2 +y 2 si (x, y) ,= (0, 0) 0 si (x, y) = (0, 0) a) estudie la continuidad de f en todo IR 2 b) calcular ∂f ∂x y ∂f ∂y en todo IR 2 c) estudie la derivada direccional de f en (0, 0) d) determine si f es diferenciable en (0, 0). Pregunta 2 Dada F(x, y, z) = x 2 + y 2 + z 2 + xy + 2z −1 = 0 a) Verificar si F(x, y, z) = 0 define en el punto P(0, −1, 0) a z como funci´ on impl´ıcita de x e y, es decir, z = f(x, y) b) Calcular z x (0, −1) , z y (0, −1) , z xx (0, −1) y z yy (0, −1) Pregunta 3 308 La ecuaci´on de onda ∂ 2 u ∂t 2 = a ∂ 2 u ∂x 2 donde a es una constante, describe el movimiento de una onda, que puede ser una onda de sonido, una onda de luz o una onda que viaja a lo largo de una cuerda vibrante.Si f y g son funciones de una sola variable dos veces derivables, compruebe que la funci´on u(x, t) = f(x+at) +g(x−at) satisface la ecuaci´ on de onda. Pauta Autocorrecci´ on Pregunta 1 (a) En los puntos de IR 2 distintos de (0, 0) la funci´ on es continua por ser cuociente de funciones continuas con denominador no nulo. En (0, 0) estudiemos la continuidad de f: i) Existe f (0, 0) = 0 ii) lim (x,y)→(0,0) f (x, y) = lim (x,y)→(0,0) x sin (xy) x 2 + y 2 Al usar coordenadas polares se tiene lim (x,y)→(0,0) x sin (xy) x 2 + y 2 = l´ım r→0 r cos θ sin (r 2 cos θ sin θ) r 2 = l´ım r→0 r cos 2 θ sin θ = 0 = f (0, 0) Puesto que sin α · α si α →0,y θ ∈ [0, 2π] , −1 < cos 2 θ sin θ < 1,en el ´ ultimo l´ımite se tiene que el producto de una cantidad infinit´esimal por una funci´ on acotada es cero. Por lo tanto la funci´ on es continua en todo IR 2 b) Las derivadas parciales ∀ (x, y) ,= (0, 0) son: ∂f ∂x = [sin (xy) + xy cos (xy)] (x 2 + y 2 ) −2x [x sin (xy)] (x 2 + y 2 ) 2 ∂f ∂y = [x 2 cos (xy)] (x 2 + y 2 ) −2y [x sin (xy)] (x 2 + y 2 ) 2 Adem´ as, las derivadas parciales para (x, y) = (0, 0) son ∂f ∂x (0, 0) = l´ım h→0 f (h, 0) −f (0, 0) h = l´ım h→0 hsin 0 h 2 −0 h = l´ım h→0 0 h = 0 ∂f ∂y (0, 0) = l´ım k→0 f (0, k) −f (0, 0) k = l´ım k→0 0 sin 0 k 2 −0 k = l´ım k→0 0 k = 0 309 Por lo tanto las derivadas parciales de primer orden existen en todo IR 2 c) Calculamos la derivada direccional en cualquier direcci´ on ´ u = (a, b) tal que |´ u| = √ a + b = 1utilizando, la definici´ on D u f (0, 0) = l´ım t→0 f (ta, tb) −f (0, 0) t = l´ım t→0 ta sin ( t 2 ab ) t 2 −0 t = l´ım h→0 ta ( t 2 ab ) t 2 −0 t D u f (0, 0) = a 2 b Por tanto, existe la derivada direccional de la funci´on f en el origen en cualquier direcci´on ´ u = (a, b). d) Examinemos si funci´ on es diferenciable en el origen. Utilizando la definici´on de diferenciabilidad, se tiene: l´ım (h,k)→(0,0) f (h, k) −f (0, 0) −f x (0, 0) h −f y (0, 0) k |(h, k)| = l´ım (h,k)→(0,0) hsin (hk) h 2 + k 2 −0 −0 h −0 k √ h 2 + k 2 = l´ım (h,k)→(0,0) hsin (hk) (h 2 + k 2 ) 3/2 Calculemos este l´ımite utilizando coordenadas polares l´ım (h,k)→(0,0) hsin (hk) (h 2 + k 2 ) 3/2 = l´ım r→0 r cos θ sin (r 2 cos θ sin θ) r 3 = l´ım r→0 r cos θ (r 2 cos θ sin θ) r 3 = cos 2 θ sin θ ,= 0 El l´ımite no es cero y por lo tanto la funci´ on no es diferenciable en (0, 0). Pregunta 2 a) F(x, y, z) = 0 define a z = f(x, y) en una vecindad de P(0, −1, 0) si: i) El punto P es un punto de la superficie, es decir, F(0, −1, 0) = 0. En efecto, al evaluar F(0, −1, 0) = 0 + 1 + 0 + 0 + 0 −1 = 0. ii) F x , F y , F z son continuas en una vecindad de P. En efecto: F x (x, y, z) = 2x + y , F y (x, y, z) = 2y + x, F z (x, y, z) = 2z + 2 son funciones polin´ omicas y estas son continuas en IR 2 . iii) Adem´as, se debe cumplir F z (0, −1, 0) ,= 0 Como F z (x, y, z) = 2z + 2 entoncesF z (0, −1, 0) = 0 + 2 = 2 ,= 0. 310 El teorema de la funci´on impl´ıcita garantiza una vecindad V (0, −1) en la cual podemos definir una funci´on z = f(x, y) tal que F (x, y, f(x, y)) = 0. b) La funci´ on tiene derivadas continuas en V (0, −1) que pueden calcularse por: z x (x, y) = − F x (x, y, z) F z (x, y, z) = − 2x + y 2z + 2 =⇒ z x (0, −1) = − F x (0, −1, 0) F z (0, −1, 0) = 1 2 z y (x, y) = − F y (x, y, z) F z (x, y, z) = − 2y + x 2z + 2 =⇒ z y (0, −1) = − F y (0, −1, 0) F z (0, −1, 0) = 1 Para calcular las derivadas de segundo orden basta derivar (1) y (2) re- specto a x e y respectivamente: z xx (x, y) = − [(2z + 2)2 −(2x + y) (2z x )] (2z + 2) 2 =⇒ z xx (0, −1) = − 5 4 z yy (x, y) = − [(2z + 2) 2 −(2y + x) (2z y )] (2z + 2) 2 =⇒ z yy (0, −1) = −2 Pregunta 3 Derivando u(x, t) con respecto a x se tiene que : ∂u ∂x = f (x + at) + g (x −at) ∂ 2 u ∂x 2 = f (x + at) + g (x + at) Asimismo las derivadas u(x, t) con respecto a t est´ an dadas por : ∂u ∂t = af (x + at) −ag (x −at) ∂ 2 u ∂t 2 = a 2 f (x + at) + a 2 g (x −at) Sustituyendo obtenemos que ∂ 2 u ∂t 2 = a 2 f (x +at) +a 2 g (x −at) = a 2 (f (x +at) +g (x −at)) = a 2 ∂ 2 u ∂x 2 Autoevaluaci´ on N o 3 El estudiante: 1) Resolver´a problemas de aplicaci´on empleando la derivada direccional de una funci´on f diferenciable en un punto (x, y) en una direcci´ on dada por vector unitario ´ u, 2) Emplear´a el teorema de la derivada implicita para obtener las derivadas parciales de primer y segundo orden de una funci´on implicita. 3) Hallar´ a los puntos cr´ıticos de una funci´ on f : Ω ⊆ R 2 → R aplicando la condici´on necesaria de punto cr´ıtico. 311 4) Utilizar´ a el criterio de la segunda derivada para determinar los ex- tremos locales de funciones de dos variables. Tiempo: 2 horas Pregunta 1 El conjunto de los puntos (x, y) tal que 0 ≤ x ≤ 5 , 0 ≤ y ≤ 5 es un cuadrado colocado en el primer cuadrante del plano XY . Supongamos que se calien- ta ese cuadrado de tal manera que T(x, y) = x 2 +y 2 es la temperatura en el puntoP(x, y). ¿En qu´e direccion se establecer´a el flujo de calor en el punto P 0 (3, 4) ?. Pregunta 2 Sea g _ xy z , x 2 + y 2 _ = 0 una ecuaci´on que define a z como una funci´ on de x e y. Verifique que si g x ; g y y g z existen y son continuas en toda la regi´ on en la que g z ,= 0; entonces yz x −xz y = − z (x 2 −y 2 ) xy Pregunta 3 Hallar los valores extremos locales, absolutos y puntos sillas de f(x, y) = xy(1 −x 2 −y 2 ) en [0, 1] [0, 1]. Pauta Autocorrecci´ on Pregunta 1 El flujo de calor en la regi´ on est´ a dado por una funci´ on vectorial − → f (x, y) y su valor en cada punto depende de las coordenadas de ´este. Sabemos que − → f (x, y) es perpendicular a las curvas de isonivel T(x, y) = c donde c es constante. Por consiguiente, el gradiente de la funci´ on T (x, y) verifica esta condici´ on. Entonces − → f (x, y) = −κ∇T (x, y) donde κ es una constante positiva ,llamada conductividad t´ermica. N´ otese que el signo negativo indica que el calor fluye desde puntos de mayor temperatura a puntos de menor temperatura. Como T(3, 4) = 25 el punto P est´ a en la isoterma T(x, y) = 25 , que es un cuadrante de la circunferencia x 2 + y 2 = 25. Sabemos que el flujo 312 de calor en P 0 (3, 4) es − → f (3, 4) = −K∇T (3, 4) . Apartir de ∇T (x, y) = 2x ´ i + 2y ´ jse tiene que ∇T (3, 4) = 6 ´ i + 8 ´ j.As´ı, el flujo de calor en P 0 es: − → f (3, 4) = −κ(6 ´ i + 8 ´ j). Como la conductividad t´ermica es positiva se puede afirmar que el calor fluye en P 0 en la direcci´ on del vector unitario. ´ u = −(6 ´ i + 8 ´ j) √ 36 + 64 = − _ 3 5 ´ i + 4 5 ´ j _ Pregunta 2 Sea g _ xy z , x 2 + y 2 _ = 0 y u = xy z , v = x 2 + y 2 . Entonces z x = − g x g z = − _ g u y z + g v 2x _ _ −g u xy z 2 _ z y = − g y g z = − _ g u x z + g v 2y _ _ −g u xy z 2 _ Sumando t´erminos yz x −xz y = y _ g u y z + g v 2x _ g u xy z 2 −x _ g u x z + g v 2y _ g u xy z 2 = − g u (x 2 −y 2 ) z g u xy z 2 = − z (x 2 −y 2 ) xy Pregunta 3 Sea f(x, y) = xy(1 −x 2 −y 2 ) en el interior del cuadrado (0, 1) (0, 1). Obtengamos sus puntos cr´ıticos ∇f (x, y) = 0 ⇐⇒ f x (x, y) = y(1 −x 2 −y 2 ) + xy(−2x) = y −3x 2 y −y 3 = 0 f y (x, y) = x(1 −x 2 −y 2 ) + xy(−2y) = x −x 3 −3xy 2 = 0 Factorizando las ecuaciones anteriores, se tiene y(1 −3x 2 −y 2 ) = 0 =⇒ y = 0 ´o 1 −3x 2 −y 2 = 0 x(1 −x 2 −3y 2 ) = 0 =⇒ x = 0 ´ o 1 −x 2 −3y 2 = 0 Consideremos y = 0, x = 0 =⇒ P 0 (0, 0) no pertenece al interior del cuadrado. Tomemos ahora las expresiones y = 0; 1 −x 2 −3y 2 = 0 =⇒dos puntos criticos P 1 (1, 0); P 2 (−1, 0) estos puntos no pertenecen al interior del cuadrado. Resolvamos las ecuaciones 313 x = 0; 1 −3x 2 −y 2 = 0 =⇒ P 3 (0; 1); P 4 (0; −1) estos puntos tampoco pertenecen al interior del cuadrado. Examinemos, las ecuaciones 1 −x 2 −3y 2 = 0 1 −3x 2 −y 2 = 0 _ =⇒ y 2 = 1 −3x 2 =⇒ 1 −x 2 −3(1 −3x 2 ) = 0 4x 2 = 1 =⇒ x = 1 2 ; x = − 1 2 esta ´ ultima coordenada no pertenece al interior del cuadrado, luego , al sustituir en la segunda ecuaci´ on anterior produce: x = 1 2 ; 1 −x 2 −3y 2 = 0 =⇒ y = 1 2 ; y = − 1 2 =⇒ dos punto P 5 _ 1 2 ; 1 2 _ y P 6 _ 1 2 ; − 1 2 _ / ∈ al interior del cuadrado. Entonces estudiemos el Hessiano en el punto P 5 _ 1 2 ; 1 2 _ f xx (x, y) = −6xy, f xy (x, y) = 1 −3x 2 −3y 2 , f yy (x, y) = −6xy H _ 1 2 , 1 2 _ = ¸ ¸ ¸ ¸ − 3 2 − 1 2 − 1 2 − 3 2 ¸ ¸ ¸ ¸ = 2 > 0 y f xx _ 1 2 , 1 2 _ = − 3 2 < 0 =⇒ hay un m´ınimo local de f en P 5 _ 1 2 , 1 2 _ cuyo valor es f _ 1 2 , 1 2 _ = 1 8 . Finalmente, estudiemos la funci´on en la frontera del cuadrado: Para x ∈ [0, 1]; y = 0 ; f(x, 0) = 0 =⇒ m´ınimo en este segmento abierto . Ahora, en los v´ertices, f(0; 0) = 0 =⇒ m´ınimo local y f(1, 0) = 0. Para y ∈ [0, 1]; x = 1; f(1; y) = −y 3 =⇒ f (y) = −3y 2 = 0 ⇐⇒ y = 0; f (y) = −6y; f (0) = 0 no existen valores extremos en el segmento abierto . Ahora, en los v´ertices f(1, 0) = 0; f(1, 1) = −1. Por lo tanto hay m´ınimos locales en (1, 0),(1, 1). Para x ∈ [0, 1]; y = 1; f(x; 1) = −x 3 =⇒ f (x) = −3x 2 = 0 ⇐⇒ x = 0; f (x) = −6x; f (0) = 0 entonces no existe ni m´ aximo ni m´ınimo en el segmento abierto. En los v´ertices x = 0; x = 1 tenemos f(0, 1) = 0; f(1, 1) = −1 =⇒ m´ınimos locales. Para x = 0; y ∈ [0, 1]; f(0; y) = 0 =⇒ m´ınimo local en el segmento abierto. 314 Cap´ıtulo 4 Integraci´ on Multiple Falta una introducci´ on 4.1. Integrales dobles y triples 4.1.1. Integrales Dobles Aspectos geom´etricos Sea R un rect´angulo representado por R = [a, b] [c, d] y f una funci´on continua definida sobre R, es decir f : R ⊆ R 2 →R Caso de funciones no negativas: Supongamos que f(x, y) ≥ 0 y ∀(x, y) ∈ R tal que la gr´afica de z = f(x, y) est´ a arriba del plano xy ,determinando una regi´on V del espacio R 3 , bajo la superficie z = f(x, y) y sobre la regi´on R. Antes de dar una definici´ on en el lenguaje de las Sumas de Riemann, podemos decir que; bajo las condiciones anteriores el volumen de la regi´ on V corresponde en este caso a lo que llamaremos integral doble de f sobre R y que denotaremos: _ _ R f(x, y)dA o _ _ R f(x, y)dxdy 315 Ejemplo: Sea f(x, y) = x 2 + y 2 y R = [0, 1] [0, 2]. Figura 4.1: paraboloide El volumen bajo el paraboloide z = x 2 + y 2 sobre el rect´angulo R corre- sponde a la integral doble de f sobre R en este caso. _ _ R (x 2 + y 2 )dA Debemos estar claros eso si, que el concepto de integral doble es mucho m´ as que esta interpretaci´ on geom´etrica Integral doble sobre un rect´angulo Sea R = [a, b] [c, d] un rect´ angulo y f una funci´ on acotada definida sobre R es decir existe M > 0 tal que −M ≤ f(x, y) ≤ M para (x, y) ∈ R Observaci´ on: Una funci´on continua sobre un rect´ angulo cerrado siempre es acotada. Partici´ on Sean P 1 = ¦x 0 , x 1 , ..., x n ¦ partici´ on de [a, b] P 2 = ¦y 0 , y 1 , ..., y n ¦ partici´ on de [c, d] Al conjunto P = P 1 P 2 = ¦(x i , y j ) / 0 ≤ i ≤ n, 0 ≤ j ≤ n¦ lo llamare- mos partici´on de R de valor n n. Sea |P 1 | = m´ ax ¦´ x i = x i −x i−1 / i = 1, 2, ..., n¦ |P 2 | = m´ ax ¦´ y i = y i −y i−1 / j = 1, 2, ..., n¦ 316 Norma de la partici´ on. La norma de P denotada |P| se define por |P| = m´ ax ¦|P 1 | , |P 2 |¦ |P| = m´ ax ¦l ij : diagonal de R ij ¦ (no es ´ unica forma de definir norma de P, pero ´esta es la que usaremos) Sumas superiores y sumas inferiores Sean R ij rect´ angulo [x i−1 , x i ] [y j−1 , y j ] 1 ≤ i, j ≤ n y ´ ij = ´ area del rect´ angulo R ij = (x i −x i−1 ) (y j −y j−1 ) Definimos ahora, sumas inferiores y sumas superiores de Riemann de f respecto de la partici´ on dada, por s P (f) = n i,j=1 m ij (f)´ ij , S P (f) = n i,j=1 M ij (f)´ ij Como consecuencia de estas definiciones podemos decir de estas sumas: i) Si P es una partici´ on cualquiera de R s P (f) ≤ S P (f) ii) Si P es partici´on mas fina que P ( P ⊆ P ) entonces s P (f) ≤ s P (f) y S P (f) ≤ S P (f) iii) Si P 1 y P 2 son dos particiones cualquiera de R s P 1 (f) ≤ S P 2 (f) Con estas sumas formamos los respectivos conjuntos: Conjunto de sumas inferiores. ¦s P (f)/P es partici´on de R¦ Conjunto de sumas superiores. ¦S P (f)/P es partici´on de R¦ 317 Si m y M son cortas inferior y superior respectivamente de f en R en- tonces si A = (b −a) (d −c). i) s P (f) ≥ m A para todo P partici´ on de R es decir el conjunto de sumas inferiores es acotado inferiormente ii) S P (f) ≤ M A ∀ P partici´ on de R, es decir, el conjunto de sumas superiores es acotado superiormente. Por lo tanto, haciendo uso del axioma del supremo (o del ´ınfimo) de la axiom´ atica de los n´ umeros reales podemos definir. Si R es un rect´ angulo de R 2 y f una funci´ on acotada sobre R definimos: a) Integral Inferior de f sobre R por _ _ R fdA = sup ¦s P (f) : P es partici´on de R¦ b) Integral Superior de f sobre R por _ _ R fdA =´ınf ¦S P (f) : P es partici´on de R¦ Sumas e Integrales Las definiciones de estas respectivas integrales permiten afirmar que para toda partici´on P de R y toda funci´ on acotada definida sobre R s P (f) ≤ _ _ R fdA ≤ _ _ R fdA ≤ S P (f) Estamos ahora en condiciones de formular la definici´on de integral doble sobre un rect´ angulo en base a sumas superiores e inferiores. Una funci´ on f(x, y) definida y acotada sobre un rect´ angulo R se dice que es Riemann integrable sobre R si _ _ R fdA = _ _ R fdA Si f es integrable sobre R, entonces la integral doble definida de f sobre R se denota por _ _ R fdA o _ _ R fdxdy y en tal caso _ _ R fdA = _ _ R fdA = _ _ R fdA 318 Nota: Alternativamente en cursos de c´ alculo se define: _ _ R fdA = l´ım p→0 n i,j=1 f(x i , y j )A ij, (x i , y j ) ∈ R ij lo que no es contradictorio sino que complementario y resultan planteamien- tos equivalentes. Esto es la definici´ on utilizando el concepto de sumas inter- medias de Riemann. Teorema 4.1.1. Cualquier funci´on continua definida en un rect´angulo cer- rado R es integrable Demostraci´ on: La demostraci´on de este hecho no resulta de inter´es en este curso a pesar de su enorme importancia, dejemos las cosas aqu´ı a la imaginaci´ on del estudiante. Proposici´on 4.1.1. (Propiedades b´asicas de la Integral Doble) De la definici´on se desprende que: 1) Si f(x, y) = 1 todo (x, y) ∈ R,la integral resulta el ´area de la regi´on A = ´ Area de R = _ _ R dA 2) Si f es integrable en R _ _ R cfdA = c _ _ R fdA 3) Si f y g son funciones integrables en R _ _ R (f + g)dA = _ _ R fdA + _ _ R gdA 4) Si f y g son funciones integrables en R y f(x, y) ≤ g(x, y) para todo (x, y) ∈ R entonces _ _ R fdA ≤ _ _ R gdA 5) Si f es integrable sobre R, entonces [f[ es integrable sobre R y ¸ ¸ ¸ ¸ _ _ R fdA ¸ ¸ ¸ ¸ ≤ _ _ R [f[ dA 319 Teorema 4.1.2. (Teorema del Valor Medio para Integrales Dobles) Si f(x, y) es continua sobre rect´angulo R con ´area A(R), entonces existe un punto (ε, η) en el interior de R tal que _ _ R f(x, y)dA = f(ε, η) A(R) Demostraci´ on: Sea m = m´ın ¦f(x, y) : (x, y) ∈ R¦ M = m´ax ¦f(x, y) : (x, y) ∈ R¦ y supongamos que m < M. Entonces m ≤ f(x, y) ≤ M, y si f no es identicamente igual a m o M, entonces m A(R) < _ _ R f(x, y)dA < M A(R) El teorema del valor medio de las funciones continuas asegura que existe un punto (ε, η) en el interior R tal que f(ε, η) = __ _ R f(x, y)dA _ A(R) =⇒∴ f(ε, η) A(R) = _ _ R f(x, y)dA 4.1.2. Integrales sobre conjuntos acotados de R 2 En este caso extenderemos la definici´ on de integral doble a regiones que no son necesariamente rect´ angulos, sino que regiones acotadas en general. Supongamos que S es una regi´ on cerrada y acotada de R 2 , por ejemplo un circulo, un tri´angulo, un rombo etc. , cualquier regi´ on con estas caracter´ısticas se puede poner dentro de un rect´ angulo R Sea R rect´ angulo que contiene a regi´ on cerrada y acotada S y f una funci´ on definida y acotada en S, extendemos f a R de la siguiente forma f R (x, y) = _ f(x, y), (x, y) ∈ S 0, (x, y) ∈ R −S f R la consideraremos como la extensi´ on de f a todo R. 320 Sea S una regi´ on acotada de R 2 y f una funci´ on definida y acotada sobre S, si R es en rect´ angulo tal que R ⊇ S y f R la extensi´on de f a R del tipo definido aqu´ı, entonces si existe _ _ R f R dA , definimos _ _ S fdA = _ _ R f R dA Importante. Las propiedades enunciadas, de la integral doble en rect´angulos siguen siendo v´alidas en conjuntos m´ as generales lo que se puede justificar por la definici´ on anterior Integrales Iteradas Una integral de la forma _ b a _ h(x) g(x) f(x, y)dydx se llama integral iterada y se interpreta como _ b a F(x)dx donde para cada x ∈ [a, b],con F(x) = _ h(x) g(x) f(x, y)dy. Si f(x, y) es funci´ on continua sobre ¦(x, y) : a ≤ x ≤ b, g(x) ≤ y ≤ h(x)¦ y G(x, y) una primitiva en la segunda variable, de f(x, y), es decir , ∂G(x, y) ∂y = f(x, y) para cada x ∈ [a, b] y todo g(x) ≤ y ≤ h(x), el teorema Fundamental del c´ alculo permite que F(x) = _ h(x) g(x) f(x, y)dy = G(x, y) [ y=h(x) y=g(x) = G(x, h(x)) −G(x, g(x)) Se puede interpretar entonces la integral iterada como un proceso sucesivo de integraci´ on as´ı _ b a _ h(x) g(x) f(x, y)dydx = _ b a _ _ h(x) g(x) f(x, y)dy _ dx De manera similar se tiene _ d c _ h(y) g(y) f(x, y)dxdy = _ d c _ _ h(y) g(y) f(x, y)dx _ dy 321 Ejemplo. _ 1 0 _ x 0 (x 2 + 4xy)dydx = _ 1 0 __ x 0 (x 2 + 4xy)dy _ dx = _ 1 0 _ x 2 y + 2xy 2 ¸ x 0 dx = _ 1 0 (x 3 + 2x 3 )dx = _ 1 0 3x 3 dx = 3 4 x 4 ¸ ¸ ¸ ¸ 1 0 = 3 4 Otros ejemplos: Interprete y eval´ ue: i) _ 4 0 _ 2 0 x √ ydxdy ii) _ 2 0 _ 4 0 x √ ydydx iii) _ 1 0 _ x 0 sin(x 2 )dydx iv) _ 2 0 _ 3 √ x (x 2 + y) dydx v) _ 5 0 _ x 2 2x (x + y)dydx Algunas Respuestas i) _ 4 0 _ 2 0 x √ ydxdy = _ 4 0 _ _ 2 0 x √ ydx _ dy = _ 4 0 _ x 2 2 √ y _ 2 0 dy = = _ 4 0 2 √ ydy = _ 2 2 3 y 3 2 _ 4 0 = 32 3 iii) _ 1 0 _ x 0 sin(x 2 )dydx = 1−cos 1 2 iv) _ 2 0 _ 3 √ x (x 2 + y) dydx 16 _ 1 − √ 2 7 _ A continuaci´ on examinaremos la evaluaci´on de la integral doble por medio de integrales iteradas. 4.1.3. Teorema de Fubini Sea f una funci´ on continua en una regi´ on R cerrada y acotada, entonces a) Si R = ¦(x, y) a ≤ x ≤ b, g 1 (x) ≤ y ≤ g 2 (x)¦ y g y son funciones continuas en [a, b] se tiene: _ _ R fdA = _ b a _ g 2 (x) g 1 (x) f(x, y)dydx b) Si R = ¦(x, y) c ≤ y ≤ d, h 1 (y) ≤ x ≤ h 2 (y)¦ y h 1 y h 2 son 322 Figura 4.2: Regi´on tipo a Figura 4.3: Regi´on tipo b funciones continuas en [c, d] se tiene _ _ R fdA = _ d c _ h 2 (y) h 1 (y) f(x, y)dxdy Ejemplo Este ejemplo ilustra como este teorema se adapta a la situaci´ on del prob- lema, en este caso se pide calcular _ _ R xydA y R es la regi´on tri´angular del plano con v´ertices en los puntos A(-6,-2), B(-1,3) y C(9,-7). Soluci´on: 323 Figura 4.4: Regi´on tri´angular del plano La regi´ on se debe subdividir en dos subregiones del tipo (a)., tal como lo muestra la figura siguiente Los segmentos de recta AB, BC Y AC tienen ecuaciones y = x + 4, y = −x + 2 e y = − 1 3 x − 4 respectivemente. Las regiones pueden escribirse. I : −6 ≤ x ≤ −1, − 1 3 x −4 ≤ y ≤ x + 4 II : −1 ≤ x ≤ 9, − 1 3 x −4 ≤ y ≤ −x + 2 Entonces aplicando T. de Fubini a ambas regiones se tiene _ _ R xydA = _ −1 −6 _ x+4 − 1 3 x−4 xydydx + _ 9 −1 _ −x+2 − 1 3 x−4 xydydx = 1 2 _ −1 −6 _ xy 2 ¸ x+4 − 1 3 x−4 dx + 1 2 _ 9 −1 _ xy 2 ¸ −x+2 − 1 3 x−4 dx = 1 9 _ −1 −6 (4x 3 + 24x 2 )dx + 1 9 _ 9 −1 (4x 3 −30x 2 −54x)dx = − 1025 27 Se observa que R tambi´en puede subdividirse en dos regiones del tipo (b) mediante una recta paralela al eje horizontal que pase por A. Proposici´on 4.1.2. Suponga que S es una regi´on acotada y sea C una curva la cual divide a S en dos subregiones S 1 y S 2 . Si f es continua en S , lo es tambien en S 1 y S 2 , y _ _ S f(x, y)dA = _ _ S 1 f(x, y)dA + _ _ S 2 f(x, y)dA 324 Figura 4.5: Regi´on tri´angular del plano, tipo b Demostraci´on.- Directamente de la definici´on eligiendo un rectangulo sufi- cientemente grande que contenga a S y extendiendo f de S a R, de S 1 a R, S 2 a R. Ejemplo. Calcular _ _ S (x 2 + y)dA, donde S es la regi´on limitada por la recta y = x y la curva y = x 3 Figura 4.6: Regi´on comprendida entre las curvas y = x y y = x 3 325 Soluci´on.- En este caso S 1 = _ (x, y) ∈ R 2 : −1 ≤ x ≤ 0, x ≤ y ≤ x 3 _ S 2 = _ (x, y) ∈ R 2 : 0 ≤ x ≤ 1, x 3 ≤ y ≤ x _ _ _ S (x 2 + y)dA = _ _ S 1 (x 2 + y)dA + _ _ S 2 (x 2 + y)dA _ _ S 1 (x 2 + y)dA = _ 0 −1 _ x 3 x (x 2 + y)dydx = _ 0 −1 _ x 2 y + y 2 2 _ x 3 x dx = _ 0 −1 [(x 5 + x 6 2 ) −(x 3 + x 2 2 )]dx = _ x 6 6 + x 7 14 − x 4 4 − x 3 6 _ 0 −1 = − (−1) 6 6 − (−1) 7 14 + (−1) 4 4 + (−1) 3 6 = − 1 84 _ _ S 2 (x 2 + y)dA = _ 1 0 _ x x 3 (x 2 + y)dydx = _ 1 0 _ x 2 y + y 2 2 _ x x 3 dx = _ 1 0 [(x 3 + x 2 2 ) −(x 5 + x 6 2 )]dx = _ ( x 4 4 + x 3 6 ) −( x 6 6 + x 7 14 ) _ 1 0 = ( 1 4 4 + 1 3 6 ) −( 1 6 6 + 1 7 14 ) = 5 28 Por lo tanto _ _ S (x 2 + y)dA = − 1 84 + 5 28 = 1 6 326 4.1.4. ´ Areas y Volumenes ´ Area Como se dijo en la introducci´on y de acuerdo a la idea geom´etrica si R es una regi´ on plana entonces el ´area de R se calcula con la integral doble. A(R) = _ _ R dA Ejemplo. Calcule el ´ area de la regi´ on interior a la circunferencia x 2 +y 2 = 2ax arriba de la par´ abola ay = x 2 , a > 0. Soluci´ on: Figura 4.7: Regi´on en el plano xy, acotada por la circunferencia (x−a) 2 +y 2 = a 2 y la par´abola y = x 2 a 2 Sea R = _ (x, y) ∈ IR 2 / 0 ≤ x ≤ a, x 2 a ≤ y ≤ √ 2ax −x 2 _ A = _ _ R dA = _ a 0 _ √ 2ax−x 2 x 2 a dydx = _ a 0 ( √ 2ax −x 2 − x 2 a )dx = a 2 12 (3π −4) Volumen Si R es una regi´ on plana, z = f(x, y), z = g(x, y) son dos superficies tal que f(x, y) ≥ g(x, y) ∀(x, y) ∈ R , el volumen entre ambas superficies 327 al interior de la regi´ on se puede calcular usando la siguiente integral doble V = _ _ R [f(x, y) −g(x, y)]dA Ejemplo 1) Use la integral doble para determinar el volumen del tetraedro acotado por los planos coordenados y el plano 3x + 6y + 4z −12 = 0. Soluci´on: Para determinar la regi´ on R hacemos z = 0 y encontramos su intersecci´ on con el plano dado xy . As´ı z = 0 =⇒ 3x + 6y = 12 =⇒ y = − 1 2 x + 2 La regi´ on en el plano xy est´a acotada por el eje x, el eje y y la recta y = − 1 2 x + 2, por lo tanto Figura 4.8: Regi´on en el plano xy, acotada por el x = 0, y = 0 y la recta y = − x 2 +2 Figura 4.9: Tetraedro R = _ (x, y) ∈ IR 2 / 0 ≤ x ≤ 4, 0 ≤ y ≤ − 1 2 x + 2 _ 328 z = f(x, y) = 12 −3x −6y 4 = 3 − 3 4 x − 3 2 y y z = g(x, y) = 0 V = _ _ R f(x, y)dA = _ _ R _ 3 − 3 4 x − 3 2 y _ dA = _ 4 0 _ − 1 2 x+2 0 _ 3 − 3 4 x − 3 2 y _ dydx = = _ 4 0 __ 3 − 3 4 x _ y − 3y 2 4 _ − 1 2 x+2 0 dx = _ 4 0 _ 3 16 x 2 − 3 2 x + 3 _ dx = _ x 3 16 − 3x 2 4 + 3x _ 4 0 = 4 Otros ejemplos: 1) Usando integral iterada calcule _ _ s (x 2 + 2y) dA, donde S: regi´ on comprendida entre y = x 2 e y = √ x 2) Calcule el volumen del s´ olido limitado por los cilindros x 2 +z 2 = 16 y y 2 + z 2 = 16 . Este es un interesante ejercicio, puede empezar por bosquejar el s´ olido o una parte de ´el, pues se puede aprovechar su simetr´ıa. 4.1.5. Cambio de variable Un cambio de variables adecuado puede no s´olo simplificar el integrando sino tambi´en la regi´ on donde se eval´ ua la integral. Sea f una funci´ on continua definida sobre la region R cerrada y acotada. Considerese la integral doble _ _ R f(x, y)dxdy Definimos T, transformaci´on invertible x = x(u, v), (u, v) ∈ S y = y(u, v), (u, v) ∈ S tal que ∂(x, y) ∂(u, v) ,= 0, la que produce una correspondencia biun´ıvoca entre R y S donde R es una regi´ on en xy y S es la nueva regi´ on en plano uv. resultado de la transformaci´ on T. Si P es una partici´on definida en R la transformaci´on induce a su vez una correspondiente partici´ on en S de tal modo que si R ij es un subrect´ angulo 329 generado por la partici´on P en R, denotaremos por S ij el correspondiente subrect´ angulo en S Si ´A ij = ´ Area de R ij y ´A ij = ´ Area de S ij Se tiene la siguiente raz´ on entre las ´areas. ´A ij ´A ij ≈ ¸ ¸ ¸ ¸ ∂(x, y) ∂(u, v) ¸ ¸ ¸ ¸ =⇒´A ij ≈ ¸ ¸ ¸ ¸ ∂(x, y) ∂(u, v) ¸ ¸ ¸ ¸ ´A ij Entonces f(x, y) ´A xy ≈ f(x, y) ¸ ¸ ¸ ¸ ∂(x, y) ∂(u, v) ¸ ¸ ¸ ¸ ´A uv De esta relaci´ on y la definici´ on de integral doble se tiene el siguiente teorema de cambio de variable. Teorema 4.1.3. Sea R una regi´on en el plano xy acotado por una curva simple cerrada y suave y que S es la imagen de R bajo la transformaci´on T invertible, definida: x = x(u, v), (u, v) ∈ S y = y(u, v), (u, v) ∈ S donde x(u, v), y(u, v) son continuamente diferenciables en un dominio que contiene a S en cual J ≡ ∂(x, y) ∂(u, v) ,= 0 Si f(x, y) define una funci´on continua sobre R se tiene: _ _ R f(x, y)dxdy = _ _ S f (x (u, v) , y (u, v)) ¸ ¸ ¸ ¸ ∂(x, y) ∂(u, v) ¸ ¸ ¸ ¸ dudv Ejemplo Calcular _ _ R 3xydA Sea R la regi´on limitada por las rectas x−2y = 0, x−2y = −4, x+y = 4, x + y = 1. Soluci´on: Sea u = x + y, v = x −2y Resolviendo el sistema lineal obtenemos 330 Figura 4.10: Aplicaci´on definida por una ecuaci´on vetorial Figura 4.11: Aplicaci´on definida por una transformaci´on lineal u = x + y v = x −2y _ =⇒ x = 1 3 (2u + v) y = 1 3 (u −v) Adem´ as el jacobiano de la transformaci´ on es ¸ ¸ ¸ ¸ ∂(x, y) ∂(u, v) ¸ ¸ ¸ ¸ = ¸ ¸ ¸ ¸ ∂x ∂u ∂x ∂v ∂y ∂u ∂y ∂v ¸ ¸ ¸ ¸ = ¸ ¸ ¸ ¸ 2 3 1 3 1 3 − 1 3 ¸ ¸ ¸ ¸ = 2 9 − 1 9 = − 1 3 Aplicando el teorema del cambio de variable , obtenemos 331 _ _ R 3xydA = _ _ S 3( 1 3 (2u + v) 1 3 (u −v)) ¸ ¸ ¸ ¸ − 1 3 ¸ ¸ ¸ ¸ dA = 1 9 _ _ S (2u + v) (u −v) dA = 1 9 _ 4 1 _ 0 −4 (2u + v) (u −v) dvdu = 104 9 Ejemplo Calcular _ _ R _ x 2 + y 2 dA R regi´on del plano xy limitada por x 2 + y 2 = 4, x 2 + y 2 = 9 Figura 4.12: Regi´on del plano xy limitada por x 2 + y 2 = 4, x 2 + y 2 = 9 Soluci´on: Sea x = r cos θ, y = r sin θ el cambio de variable a polares entonces ¸ ¸ ¸ ¸ ∂(x, y) ∂(r, θ) ¸ ¸ ¸ ¸ = ¸ ¸ ¸ ¸ ∂x ∂r ∂x ∂θ ∂y ∂r ∂y ∂θ ¸ ¸ ¸ ¸ = ¸ ¸ ¸ ¸ cos θ −r sin θ sin θ r cos θ ¸ ¸ ¸ ¸ = r 332 _ _ R _ x 2 + y 2 dA = _ _ S r [r[ drdθ = _ 2π 0 _ 3 2 r 2 drdθ = _ 2π 0 19 3 dθ = 38 3 π 4.2. Aplicaciones de la integral doble 4.2.1. Masa de una regi´ on plana de densidad variable. Sea δ(x, y) funci´ on positiva y definida sobre un conjunto cerrado y aco- tado S con ´area no nula, que indica la densidad en cada punto (x, y) de S. La masa de S es la integral de la funci´on densidad. M(S)= _ _ S δ(x, y)dA En el caso que la densidad es constante δ = k, la masa es el producto del ´ area por la densidad: M(S) = k A(S). Ejemplo. Encuentre la masa de un c´ırculo de radio a si su densidad es λ veces la distancia al centro. Soluci´on: Con el uso de coordenadas polares el c´alculo de la integral resultante es m´ as sencillo. 333 Figura 4.13: M(S) = λ _ _ S _ x 2 + y 2 dydx = λ _ 2π 0 _ a 0 ρ 2 dρdθ = 2λπa 3 3 4.2.2. Momentos y centroide de una regi´ on plana Para un conjunto S acotado y de ´ area positiva , y una funci´ on densidad definida en S, tenemos las siguientes definiciones. Primer momento con respecto al eje y: M y = _ _ S δ(x, y)xdA Primer momento con respecto al eje x: M x = _ _ S δ(x, y)ydA Segundo momento con respecto al eje y: I y = _ _ S δ(x, y)x 2 dA 334 Segundo momento con respecto al eje x: I x = _ _ S δ(x, y)y 2 dA Segundo momento Polar con respecto al origen: I 0 = _ _ S δ(x, y)(x 2 + y 2 )dA Centroide: (x, y) = _ M y M , Mx M _ Cuando la funci´ on densidad es variable y est´a asociada con la distribuci´on de la masa, los segundos momentos se llaman tambi´en momentos de inercia y el centroide se le llama tambi´en centro de masas. En una forma m´ as general, el primer y segundo momento de un conjunto S se puede definir con respecto a una linea recta cualquiera L. M L = _ _ S δ(x, y)D(x, y)dA I L = _ _ S δ(x, y) [D(x, y)] 2 dA Siendo D(x, y) la distancia de la recta L al punto (x,y). Ejemplo. Una l´ amina triangular tiene los v´ertices (0, 0), (1, 0) y (1, 2), y tiene densidad δ(x, y) = x 2 y. Halle su centro de masa. Soluci´ on: En este caso R = ¦(x, y) ∈ R 2 / 0 ≤ x ≤ 1, 0 ≤ y ≤ 2x¦ Debemos calcular M = M(S), M y y M x : M = _ _ S x 2 ydA = _ 1 0 _ 2x 0 x 2 ydydx = _ 1 0 _ x 2 y 2 2 _ 2x 0 dx =⇒ M = _ 1 0 2x 4 dx = _ 2x 5 5 _ 1 0 = 2 5 335 Figura 4.14: Calculemos ahora M y = _ _ S x 2 yxdA = _ 1 0 _ 2x 0 x 3 ydydx = _ 1 0 2x 5 dx = _ 2x 6 6 _ 1 0 =⇒ M y = 1 3 Por ´ ultimo M x = _ _ S x 2 yydA _ 1 0 _ 2x 0 x 2 y 2 dydx = _ 1 0 _ x 2 y 3 3 _ 2x 0 dx = _ 1 0 8x 5 3 dx =⇒ M x = _ 8x 6 18 _ 1 0 = 4 9 As´ı tenemos que (x, y) = _ M y M , Mx M _ = ( 5 6 , 10 9 ) 336 4.3. Integrales triples 4.3.1. Ideas preliminares Los conceptos a desarrollar en lo referente a la Integral en su g´enesis, su significado y su c´ alculo en el caso de tres variables es similar al mismo tema en funciones de una variable y de dos variables. El tratamiento hecho en el caso de la integral doble se extiende en forma natural a las integrales triples en sus mecanismos conceptuales y m´etodos de c´ alculo involucrados solo hay un cambio en el escenario, el espacio R 3 . Atendiendo a la declaraci´ on anterior no haremos el detalle de la gen- eraci´ on del concepto porque como ya lo dijimos se trata de una generalizaci´ on. Para mantener el marco de referencia, pensemos en sumas superiores, sumas inferiores, integral superior e integral inferior para funciones definidas sobre una caja rectangular de tipo: h(x, y, z) : ¦a 1 ≤ x ≤ b 1 , a 2 ≤ y ≤ b 2 , a 3 ≤ z ≤ b 3 ¦ Con este trasfondo se plantea el teorema de Fubini que permitir´ a el c´ alculo de la integral triple. 4.3.2. Teorema de Fubini Si f(x, y, z) esta definida sobre una regi´ on R = ¦a 1 ≤ x ≤ b 1 , a 2 ≤ y ≤ b 2 , a 3 ≤ z ≤ b 3 ¦ , entonces: _ _ R _ f(x, y, z)dv = _ b 1 a 1 _ b 2 a 2 _ b 3 a 3 f(x, y, z)dzdydx siempre que estas integrales existan. Observaci´ on: Hay otras cinco formas de calcular la integral triple de- pendiendo del orden de integraci´ on en la integral iterada. Ejemplo: Calcule _ _ R _ (x 2 + yz) dv, donde R = ¦0 ≤ x ≤ 2, −1 ≤ y ≤ 2, −1 ≤ z ≤ 3¦ Soluci´ on: 337 _ _ R _ _ x 2 + yz _ dv = _ 2 0 _ 2 −1 _ 3 −1 _ x 2 + yz _ dzdydx = _ 2 0 _ 2 −1 _ x 2 z + y z 2 2 _¸ ¸ ¸ ¸ 3 −1 dydx = _ 2 0 _ 2 −1 __ 3x 2 + 9 2 y _ − _ −x 2 + y 2 _ _ dydx = _ 2 0 _ 2 −1 _ 4x 2 + 4y _ dydx = _ 2 0 _ 4x 2 y + 2y 2 _¸ ¸ 2 −1 dx = _ 2 0 _ 12x 2 + 6 _ dx = 4x 3 + 6x ¸ ¸ 2 0 = 44 4.3.3. Teorema de la integral triple (Para dominios m´as generales) Si f(x, y, z) esta definida sobre un conjunto acotado R formado por todos los puntos tales que a 1 ≤ x ≤ b 1 , y 1 (x) ≤ y ≤ y 2 (x) y z 1 (x, y) ≤ z ≤ z 2 (x, y)entonces: _ _ R _ f(x, y, z)dv = _ b 1 a 1 _ y 2 (x) y 1 (x) _ z 2 (x) z 1 (x) f(x, y, z)dzdydx siempre que ambas integrales existan. Importante. Hay otras cinco formas de calcular la integral triple dependiendo el orden de integraci´on para el calculo de la integral iterada. Si S es un conjunto acotado el cual tiene ´ area en el plano XY y f(x, y, z) es una funci´ on definida, acotada y no negativa sobre S, y si R es el conjunto de todos los (x, y, z) tal que (x, y) ∈ S y 0 ≤ z ≤ f(x, y), entonces la regi´ on R tiene volumen si y solo si f(x, y) es integrable sobre S y en tal caso 338 V (R) = _ _ S f(x, y)dA Si R es una rigi´ on de R 3 que tiene volumen, entonces V (R) = __ R _ f(x, y, z)dv con f(x, y, z) = 1 Ejemplo Sea R la regi´on acotada por los paraboloides z = x 2 + y 2 y 2z = 12 −x 2 −y 2 . Usando integral triple calcule el volumen de R. La representaci´ on gr´ afica de estas superficies es. Figura 4.15: Regi´on acotada por los paraboloides z = x 2 + y 2 y 2z = 12 −x 2 −y 2 Por separado, tenemos Soluci´on: La curva de intersecci´on es el c´ırculo x 2 + y 2 = 4, z = 4 Si f(x, y, z) = 1 se tiene el volumen 339 Figura 4.16: v(R) = __ R _ f(x, y, z)dv = _ 2 −2 _ √ 4−x 2 − √ 4−x 2 _ 12−x 2 −y 2 2 x 2 +y 2 dzdydx = 6 _ 2 0 _ √ 4−x 2 0 _ 4 − _ x 2 + y 2 _¸ dydx = 4 _ 2 0 _ 4 −x 2 _3 2 dx = 12π Ejemplo Calcular el volumen de la regi´ on del espacio limitada por las superficies c´ılindricas x 2 + z 2 = 1, y 2 + z 2 = 1 Soluci´on: La representaci´ on gr´ afica de estas superficies es Utilizaremos la simetr´ıa del problema y proyectaremos la regi´on al plano xz ( tambi´en se podr´ıa proyectar al plano yz ). La proyecci´ on nos da un c´ırculo de radio 1 340 Figura 4.17: Regi´on acotada por los cilindros x 2 + z 2 = 1, y 2 + z 2 = 1 La regi´on se puede expresar: −1 ≤ x ≤ 1 − √ 1 −z 2 ≤ y ≤ √ 1 −z 2 − √ 1 −x 2 ≤ z ≤ √ 1 −x 2 Expresando el c´ alculo del volumen como una integral triple tenemos V = _ _ R _ dxdydz usando integrales iteradas V = _ 1 −1 _ √ 1−x 2 − √ 1−x 2 _ √ 1−z 2 − √ 1−z 2 dydzdx = _ 1 −1 _ √ 1−x 2 − √ 1−x 2 2 √ 1 −z 2 dzdx Si seguimos por este camino llegamos a una expresi´ on d´ıficil de resolver ( intentelo), recurriremos entonces al cambio de orden de integraci´ on que es un recurso siempre disponible 341 V = _ 1 −1 _ √ 1−x 2 − √ 1−x 2 2 √ 1 −z 2 dzdx = _ 1 −1 _ √ 1−z 2 − √ 1−z 2 2 √ 1 −z 2 dxdz = _ 1 −1 4(1 −z 2 )dxdz = (4z −4 z 3 3 ) ¸ ¸ ¸ ¸ 1 −1 = 16 3 El volumen calculado es V = 16 3 unidades de volumen 4.3.4. Cambio de variable para integrales triples Sea T : U ⊆ R 3 →R 3 una transformaci´ on de clase C 1 definida por: x = x (u, v, w) y = y (u, v, w) z = z (u, v, w) Recordando el jacobiano de la transformaci´on se tiene: J = ∂ (x, y, z) ∂ (u, v, w) = ¸ ¸ ¸ ¸ ¸ ¸ ∂x ∂u ∂x ∂v ∂x ∂w ∂y ∂u ∂y ∂v ∂y ∂w ∂z ∂u ∂z ∂v ∂z ∂w ¸ ¸ ¸ ¸ ¸ ¸ Como en el caso anterior de dos variables, el jacobiano mide como la curva la transformaci´on distorsiona su dominio. Formula de cambio de variable para integrales triples Sea R una regi´on en el espacio xyz y S una regi´on en el espacio uvw que corresponde a R bajo la transformaci´ on T definida por x = x (u, v, w) , y = y (u, v, w) y z = z (u, v, w) siempre que T sea de clase C 1 y uno a uno, ∂(x,y,z) ∂(u,v,w) ,= 0 en S. Entonces: __ R _ f(x, y, z)dv = __ S _ f(x (u, v, w) , y (u, v, w) , z (u, v, w)) [J[ dudvdw Donde J = ∂ (x, y, z) ∂ (u, v, w) Los cambios m´ as usados en integrales triples es a coordenadas cilindricas y coordenadas esf´ericas dependiendo de la naturaleza del problema. 342 Coordenadas cil´ındricas El cambio de variable es: x = r cos θ y = r sin θ z = z Figura 4.18: Cambio a coordenadas cil´ındricas Supongamos que: P es un punto del espacio de coordenadas xyz, P 1 proyecci´on de P en plano xy, r radio vector de O a P 1 y θ el ´ angulo entre eje x y −−→ OP 1 , medido del lado positivo del eje x entonces r = _ x 2 + y 2 y θ = arctan y x Tenemos J = ∂ (x, y, z) ∂ (r, θ, z) = ¸ ¸ ¸ ¸ ¸ ¸ cos θ −r sin θ 0 sin θ r cos θ 0 0 0 1 ¸ ¸ ¸ ¸ ¸ ¸ = r cos 2 θ + r sin 2 θ = r __ R _ f(x, y, z)dv = __ S _ f(r cos θ, r sin θ, z) r drdθdz Ejemplo: Use coordenadas cil´ındricas para calcular el volumen del s´ olido limitado por el paraboloide z = x 2 + y 2 y el plano z = 4. Soluci´on: En el espacio xyz, la figura es al interior del elipsoide y limitado por arriba por el plano z = 4 que es un plano paralelo al plan xy 343 Figura 4.19: S´olido limitado por el paraboloide z = x 2 + y 2 y el plano z = 4 Aprovechando la simetr´ıa del s´ olido calculamos la cuarta parte de ´el (por conveniencia). En esta situaci´on la regi´on transformada viene descrita por: 0 ≤ r ≤ 2, 0 ≤ θ ≤ π 2 , r 2 < z < 4 La descripci´on de la regi´ on en las nuevas variables es fundamental para el acertado planteamiento de la integral doble como integral iterada, en este caso esta circunstancia es evidente V 4 = _ π 2 0 _ _ 2 0 _ 4 r 2 rdzdrdθ = _ π 2 0 _ 2 0 _ 4r −r 3 _ drdθ = _ π 2 0 4dθ = 2π ∴ V = 8π Coordenadas Esf´ericas El cambio de variable es: x = ρ cos θ sin φ y = ρ sin θ sin φ z = ρ cos φ 344 Figura 4.20: Cambio a coordenadas esfericas Supongamos que P es un punto del espacio de coordenadas xyz P 1 proyecci´on de P en plano xy ρ magnitud del radio vector −→ OP θ el ´ angulo entre eje x y −−→ OP 1 , medido del lado positivo del eje x φ angulo formado por −→ OP y el eje Z, medido del lado positivo del eje z. J = ∂ (x, y, z) ∂ (ρ, θ, φ) = ¸ ¸ ¸ ¸ ¸ ¸ cos θ sin φ sin θ sin φ cos φ −ρ sin θ sin φ ρ cos θ sin φ 0 ρ cos θ cos φ ρ sin θ cos φ −ρ sin φ ¸ ¸ ¸ ¸ ¸ ¸ = −ρ 2 sin φ 4.3.5. Formula del cambio de variable __ R _ f(x, y, z)dv = __ S _ f(ρ cos θ sin φ, ρ sin θ sin φ, ρ cos φ) ¸ ¸ _ −ρ 2 sin φ _¸ ¸ drdθdz Las coordenadas esf´ericas se usan preferentemente en el caso en que uno o ambas superficies que acotan la regi´on de integraci´ on es una esfera centrada en el origen, esto se observa en los siguientes ejemplos. Ejemplo 1 345 Figura 4.21: S´olido limitado imitada inferiormente por el semicono z 2 = x 2 + y 2 , z ≥ 0 y superiormente por la esfera x 2 + y 2 + z 2 = 9. Hallar el volumen de la regi´on s´ olida limitada inferiormente por el semi- cono z 2 = x 2 + y 2 , z ≥ 0 y superiormente por la esfera x 2 + y 2 + z 2 = 9. Soluci´ on: Haciendo la intersecci´on de x 2 + y 2 + z 2 = 9 y z = x 2 + y 2 resulta que la intersecci´ on de estas superficies es una circunferencia en el plano z = 3 √ 2 definida por las ecuaciones ⇒x 2 + y 2 = 9 2 , z = 3 √ 2 esto permite visualizar que (0, 3 √ 2 , 3 √ 2 ) es un punto de la intersecci´ on por lo que 0 ≤ φ ≤ π 4 , la esfera tiene radio 3 por lo cual 0 ≤ ρ ≤ 3, y 0 ≤ θ ≤ 2π. Como se est´ a calculando el volumen de una regi´on que es sim´etrica respecto del eje z, la cuarta parte de la regi´ on queda descrita por 0 ≤ ρ ≤ 3, 0 ≤ θ ≤ π 2 , 0 ≤ φ ≤ π 4 346 esto implica ∴ V 4 = __ R _ f(x, y, z)dV = _ π 2 0 _ π 4 0 _ 3 0 ρ 2 sin φdρdφdθ V 4 = _ π 2 0 _ π 4 0 9 sin φdφdθ V 4 = _ π 2 0 _ −9 √ 2 + 9 _ dθ = 9 2 _ √ 2 −1 √ 2 _ π Por lo tanto V = 18 _ √ 2 −1 √ 2 _ π Ejemplo 2 Utilice coordinadas esf´ericas para hallar el volumen del s´ olido que est´ a ar- riba del cono z = _ x 2 + y 2 y debajo de la esfera x 2 + y 2 + z 2 = z Figura 4.22: S´olido que est´a arriba del cono z = _ x 2 + y 2 y debajo de la esfera x 2 + y 2 + z 2 = z Soluci´on: x 2 + y 2 + z 2 = z ⇐⇒x 2 + y 2 + (z − 1 2 ) 2 = 1 4 es una esfera que pasa por el origen y tiene centro en (0, 0, 1 2 ). La ecuaci´on de la esfera en coordinadas esf´ericas es ρ = cos φ,a su vez de la ecuaci´on del cono se infiere que 0 ≤ φ ≤ π 4 . 347 Por lo que la regi´ on en coordenadas esfericas esta descrita por , 0 ≤ φ ≤ π 4 , 0 ≤ θ ≤ 2π, 0 ≤ ρ ≤ cos φ El volumen de la regi´ on es V = __ R _ f(x, y, z)dV = _ 2π 0 _ π 4 0 _ cos φ 0 ρ 2 sin φdρdφdθ = _ 2π 0 _ π 4 0 sin φ _ ρ 3 3 _¸ ¸ ¸ ¸ cos φ 0 dφdθ = 2π 3 _ π 4 0 sin φcos 3 φdφ = 2π 3 ¸ ¸ ¸ ¸ . _ − cos 4 φ 4 _¸ ¸ ¸ ¸ π 4 0 = π 8 Por lo tanto V = π 8 4.3.6. Masa, Momentos, y Centroide de una Regi´ on del Espacio Como en el caso de dos dimensiones, si δ(x, y, z) funci´ on positiva y con- tinua, definida sobre una region compacta (conjunto cerrado y acotado) W con volumen, que indica la densidad en cada punto (x, y, z) de W. La masa de W es dada por la integral de la funci´on densidad. M(W)= _ _ _ W δ(x, y, z)dV El primer momento de W se define respecto de algun plano, y el segundo momento (o momento de inercia) con respecto a alg´ un plano, l´ınea o punto. Daremos aqui solo las formulas t´ıpicas planos coordenados, ejes y el ori- gen. Primer momento con respecto al plano yz. M yz = _ _ _ W δ(x, y, z)xdV Segundo momento con repecto al plano yz. I yz = _ _ _ W δ(x, y, z)x 2 dV 348 Segundo momento con repecto al eje x I x = _ _ _ W δ(x, y, z)(y 2 + z 2 )dV Segundo momento polar con respecto del origen I 0 = _ _ _ W δ(x, y, z)(x 2 + y 2 + z 2 )dV Centroide: ( x, y, z) = _ M yz M , M zx M , M xy M _ . Ejemplo Determinar el centroide de la porci´ on de la esfera x 2 + y 2 + z 2 ≤ a 2 , en el primer octante, asumiendo densidad constante. Soluci´on: El problema no pierde generalidad si suponemos que δ = 1, y claramente x = y = z. Necesitamos calcular solamente M xy = _ _ _ W δ(x, y, z)zdV y usando coordenadas esfericas esta integral queda M xy = _ π 2 0 _ π 2 0 _ a 0 (ρ cos φ)ρ 2 senφdρdφdθ = πa 4 16 Como V = πa 3 6 , el centroide es _ 3 8 a, 3 8 a, 3 8 a _ Ejemplo Encontrar el momento de inercia I L de un cilindro circular recto co a radio de la base, h altura y densidad proporcional a la distancia al eje del cilindro, con respecto a una recta L paralela al eje del cilindro y a una distancia b de ´el. 349 Soluci´on: La recta L se define por: x = b, y = 0 El cilindro es descrito por: 0≤ r ≤ a, 0 ≤ z ≤ h La densidad es δ = kr Entonces I L = _ _ _ W δ(x, y, z)((x −b) 2 + y 2 )dV = _ 2π 0 _ a 0 _ h 0 kr((x −b) 2 + y 2 )rdzdrdθ = _ 2π 0 _ a 0 _ h 0 kr 2 (r 2 + b 2 )dzdrdθ + 0 = 2πka 3 h _ a 2 5 + b 2 3 _ 4.4. Ejercicios resueltos integrales triples y dobles 4.4.1. C´alculo de integrales dobles en coordenadas rect´angu- lares cartesianas Problema 1 Calcular __ D √ x + ydxdy si D es la regi´ on acotada por las respectivas rectas y = x, y = −x y x = 1 Soluci´on: Se tiene que la regi´on D = ¦(x, y) ∈ IR 2 / 0 ≤ x ≤ 1; −x ≤ y ≤ x¦ 350 Figura 4.23: D regi´on acotada por y = x, y = −x y x = 1 __ D √ x + ydxdy = _ 1 0 _ x −x √ x + ydydx = 2 3 _ 1 0 (x + y) 3/2 ¸ ¸ ¸ x −x dx = 2 3 _ 1 0 (2x) 3/2 dx = 2 5/2 3 2 5 (x) 5/2 ¸ ¸ ¸ 1 0 = 8 √ 2 15 Problema 2 Calcular __ D _ x 2 −y 2 dxdy si D es el dominio limitado por el tri´ angulo de v´ertices A(0, 0) , B(1, −1), C (1, 1) . V´ease figura 4.23 Soluci´on Entonces se tiene que el dominio est´ a delimitado por las rectas y = x, y = −x y x = 1. Luego el dominio de integraci´ on es: D = _ (x, y) ∈ IR 2 / 0 ≤ x ≤ 1; −x ≤ y ≤ x _ . Integrando a franjas verticales, resulta 351 __ D _ x 2 −y 2 dxdy = _ 1 0 _ x −x _ x 2 −y 2 dydx = _ 1 0 _ x −x x _ 1 − _ y x _ 2 dydx Hacemos el cambio de variables y x = sent =⇒ dy = x cos tdt y determinemos los limites. Para y = x =⇒ arcsen _ x x _ = arcsen(1) = π 2 . Para y = −x =⇒ arcsen _ −x x _ = arcsen(−1) = − π 2 Por tanto _ 1 0 _ x −x x _ 1 − _ y x _ 2 dydx = _ 1 0 _ π 2 − π 2 x 2 √ 1 −sen 2 tdtdx = _ 1 0 _ π 2 − π 2 x 2 cos 2 tdtdx = _ 1 0 _ π 2 − π 2 x 2 ( 1 + cos 2t 2 )dtdx = _ 1 0 x 2 _ t 2 + sen2t 4 _π 2 − π 2 dx = π 2 _ 1 0 x 2 dx = π 2 _ x 3 3 _ 1 0 = π 6 352 Problema 3 Calcular __ D _ y −2x 2 _ dxdy si D es la regi´ on acotada por [x[ +[y[ = 2 Figura 4.24: D regi´on acotada por [x[ +[y[ = 2 Soluci´on: Se tiene que la regi´on D = ¦(x, y) ∈ IR 2 / [x[ +[y[ ≤ 2¦ Si escogemos la regi´ on con una partici´on de tipo I, es necesario utilizar dos integrales iterativas porque para −2 ≤ x ≤ 0 , la frontera inferior de la regi´ on es la gr´afica de y = −x−2, y la superior es y = x+2;y para 0 ≤ x ≤ 2 la frontera inferior de la regi´ on es la gr´afica de y = x − 2, y la superior es y = −x + 2 Entonces se tiene D = D 1 ∪ D 2 tal que D 1 ∪ D 2 = φ. donde D 1 = ¦(x, y) ∈ IR 2 / −2 ≤ x ≤ 0, −x −2 ≤ y ≤ x + 2¦ D 2 = ¦(x, y) ∈ IR 2 / 0 < x ≤ 2, x −2 ≤ y ≤ −x + 2¦ Por otra parte la funcion del integrando f (x, y) = y−2x 2 es sim´etrica con respecto al eje y, es decir ∀ (x, y, z) ∈ D existe (−x, y, z) tal que f (−x, y) = y −2(−x) 2 = f (x, y) . Por lo tanto 353 __ D _ y −2x 2 _ dxdy = 2 _ 2 0 _ −x+2 x−2 _ y −2x 2 _ dydx = 2 _ 2 0 _ y 2 2 + 2x 2 y _¸ ¸ ¸ ¸ −x+2 x−2 dx = 2 _ 1 0 _ 4x 3 −8x 2 _ dx = _ x 4 − 8 3 x 3 _¸ ¸ ¸ ¸ 2 0 = 2 _ 16 − 64 3 _ = − 32 3 Problema 4 Calcular __ D _ x 2 + y 2 _ dxdy si D = ¦(x, y) ∈ IR 2 / x 2 + y 2 ≤ 1¦ .Usando coordenadas cartesianas Figura 4.25: D regi´on acotada por x 2 + y 2 ≤ 1 Soluci´on: Usando coordenadas cartesianas, la regi´on de integraci´ on es un c´ırculo centrado en el origen de radio uno Por lo tanto D = _ (x, y) ∈ IR 2 / −1 ≤ x ≤ 1, − √ 1 −x 2 ≤ y ≤ √ 1 −x 2 _ 354 __ D _ x 2 + y 2 _ dxdy = _ 1 −1 _ √ 1−x 2 − √ 1−x 2 (x 2 + y 2 )dydx = _ 1 −1 (x 2 y + y 3 3 ) ¸ ¸ ¸ ¸ √ 1−x 2 − √ 1−x 2 dx = 2 _ 1 −1 (x 2 √ 1 −x 2 + 1 3 _ (1 −x 2 ) 3 )dx = 2 _ 1 −1 x 2 √ 1 −x 2 dx + 2 3 _ 1 −1 _ (1 −x 2 ) 3 dx Con ayuda de una tabla de integrales obtenemos que: _ 1 −1 x 2 √ 1 −x 2 dx = (− x 4 √ 1 −x 2 + 1 8 (x √ 1 −x 2 + arcsenx) ¸ ¸ ¸ ¸ 1 −1 = 1 8 (arcsen(1) −arcsen(−1) = 1 8 ( π 2 + π 2 ) = π 8 _ 1 −1 _ (1 −x 2 ) 3 dx = ( x 4 _ (1 −x 2 ) 3 + 3x 8 _ (1 −x 2 ) + 3 8 arcsenx) ¸ ¸ ¸ ¸ 1 −1 = 3π 8 Por lo tanto: __ D _ x 2 + y 2 _ dxdy = 2π 8 + 2 3 3π 8 = π 2 Notese que la soluci´ on del problema usando coordenadas cartesianas es bastante compleja Problema 5 Calcular __ D xydxdy si D es la regi´on acotada por y = √ x, y = √ 3x −18, y ≥ 0.Usando coordenadas cartesianas. Soluci´on. Si escogemos la regi´ on con una partici´on de tipo I, es necesario utilizar dos integrales iterativas porque para 0 ≤ x ≤ 6 , la frontera inferior de la regi´ on es la gr´ afica de y = 0, y la superior es y = √ x;y para 6 ≤ x ≤ 9 la 355 Figura 4.26: D y = √ x, y = √ 3x −18, y ≥ 0. Regi´on de tipo I frontera inferior de la regi´on es la gr´afica de y = √ 3x −18, y la superior es y = √ x Luego tenemos que D = D 1 ∪ D 2 tal que D 1 ∪ D 2 = φ. Entonces D 1 = ¦(x, y) ∈ IR 2 / 0 ≤ x ≤ 6, 0 ≤ y ≤ √ x¦ D 2 = _ (x, y) ∈ IR 2 / 6 < x ≤ 9, √ 3x −18 ≤ y ≤ √ x _ Por lo tanto __ D xydxdy = __ D 1 xydxdy + __ D 2 xydxdy = _ 6 0 _ √ x 0 xydydx + _ 9 6 _ √ x √ 3x−18 xydydx = _ 6 0 x _ y 2 2 _ √ x 0 dx + _ 9 6 x _ y 2 2 _ √ x √ 3x−18 dx = 1 2 _ 6 0 x 2 dx + 1 2 _ 9 6 (−2x 2 + 18x)dx = _ 1 6 x 3 _ 6 0 + _ − x 3 3 + 9 x 2 2 _ 9 6 = 185 2 Si escogemos la regi´ on con una partici´ on de tipo II, es necesario utilizar s´ olo una integral iterativa porque para 0 ≤ y ≤ 3 , la frontera izquierda de la regi´ on es la gr´ afica de x = y 2 mentras que la frontera derecha queda determinada por la gr´ afica x = y 2 3 + 6, obteniendo as´ı la regi´on D 1 = _ (x, y) ∈ IR 2 / y 2 ≤ x ≤ y 2 3 + 6, 0 ≤ y ≤ 3 _ 356 Figura 4.27: D y = √ x, y = √ 3x −18, y ≥ 0. Regi´on de tipo II la integral iterativa queda __ D xydxdy = _ 3 0 _ (y 2 /3)+6 y 2 xydxdy = _ 3 0 _ x 2 2 _ (y 2 /3)+6 y 2 ydy = 1 2 _ 3 0 _ _ y 2 + 18 3 _ 2 −y 4 _ (y 2 /3)+6 y 2 ydy = 1 18 _ 3 0 _ −8y 5 + 36y 3 + 324y ¸ dy = 1 18 _ − 4 3 y 6 + 9y 4 + 162y 2 _ 3 0 = 1 18 _ − 4 3 3 6 + 3 6 + 2 3 6 _ = 185 2 357 Problema 6 Encontrar el ´area de la regi´on determinada por las desigualdades: xy ≥ 4, y ≤ x, 27y ≥ 4x 2 . Figura 4.28: Regi´on determinada por xy ≥ 4, y ≤ x, 27y ≥ 4x 2 Soluci´on. Sabemos que xy = 4 tiene por gr´afica una hip´erbola equil´atera, y = x es la recta bisectriz del primer cuadrante y 27y = 4x 2 corresponde a una par´ abola. Veamos cuale son los puntos de intersecci´ on de estas curvas con el proprosito de configurar el dominio de integraci´on xy = 4 y = x _ =⇒ x 2 = 4 =⇒ x = ±2 =⇒ y = ±2 27y = 4x 2 y = x _ =⇒ 27x = 4x 2 =⇒ x = 0 x = 27 4 _ =⇒ y = 0, y = 27 4 xy = 4 27y = 4x 2 _ =⇒ x = 3, y = 4 3 Para calcular el ´ area A(R) = __ D dxdy, podemos escoger una partici´ on del dominio de tipo I ´ o de tipo II. Consideremos dos subregiones de tipo I D 1 = _ (x, y) ∈ IR 2 / 2 ≤ x ≤ 3, 4 x ≤ y ≤ x _ D 2 = _ (x, y) ∈ IR 2 / 3 ≤ x ≤ 27 4 , 4 27 x 2 ≤ y ≤ x _ 358 Si proyectamos sobre eje x A(R) = __ D dxdy = __ D 1 dxdy + __ D 2 dxdy A(R) = _ 3 2 _ x 4 x dydx + _ 27/4 3 _ x 4 27 x 2 dydx = _ 3 2 y[ x 4 x dx + _ 27/4 3 y[ x 4 27 x 2 dx = _ 3 2 _ x − 4 x _ dx + _ 27/4 3 _ x − 4 27 x 2 _ dx = _ x 2 2 −4 ln x _ 3 2 + _ x 2 2 − 4 81 x 3 _ 27/4 3 = 5 2 −4 ln 3 2 + 729 32 − 9 2 − 4 81 27 3 4 3 + 4 81 3 3 = −2 −4 ln 3 2 + 729 32 − 243 16 + 4 3 = 665 96 −4 ln 3 2 Si proyectamos sobre eje y D I = _ (x, y) ∈ IR 2 / 4 y ≤ x ≤ 3 2 √ 3y, 4 3 ≤ y ≤ 2 _ D I = _ (x, y) ∈ IR 2 / y ≤ x ≤ 3 2 √ 3y, 2 ≤ y ≤ 27 4 _ A(R) = __ D dxdy = __ D 1 dxdy + __ D 2 dxdy A(R) = _ 2 4 3 _ 3 2 √ 3y 4 y dxdy + _ 27/4 2 _ 3 2 √ 3y y dxdy = _ 2 4 3 _ _ 3y −4 ln y _ dy + _ 27/4 2 _ 3 2 _ 3y −y _ dy = _ 3 2 _ 3y 3 − 4 y _ 2 4 3 + _ _ 3y 3 − y 2 2 _ 27/4 2 = − 8 3 −4 ln 3 2 + 9 27 8 − 729 32 + 2 = 665 96 −4 ln 3 2 359 Problema 7 Encontrar el volumen de la regi´ on acotada por los tres planos coordenados y el plano x + 2y + 3z = 6 Soluci´on. Figura 4.29: Regi´on determinada por los tres planos coordenados y el plano x + 2y + 3z = 6 Usando integrales dobles y proyectando la regi´on sobre el plano xy ten- emos: V = __ D 6 −x −2y 3 dxdy , D = _ (x, y) ∈ IR 2 / 0 ≤ x ≤ 6, 0 ≤ y ≤ 6 −x 2 _ V = 1 3 _ 6 0 _ 6−x 2 0 (6 −x −2y) dydx = 1 3 _ 6 0 _ (6 −x)y −y 2 ¸6−x 2 0 dx = 1 3 _ 6 0 _ (6 −x) 2 2 − (6 −x) 2 4 _ dx = 1 12 _ 6 0 (6 −x) 2 dx = _ − 1 36 (6 −x) 3 _ 6 0 = 6 Usando integrales dobles y proyectando la regi´ on sobre el plano yz ten- emos: V = __ R (6 −3z −2y) dzdy , R = _ (y, z) ∈ IR 2 / 0 ≤ y ≤ 3, 0 ≤ z ≤ 6 −2y 3 _ 360 V = _ 3 0 _ 6−2y 3 0 (6 −2y −3z) dzdy = _ 3 0 _ (6 −2y)z − 3 2 z 2 _ 6−2y 3 0 dy = _ 3 0 _ (6 −2y) 2 3 − (6 −2y) 2 6 _ dy = 1 6 _ 3 0 (6 −2y) 2 dy = _ − 1 12 (6 −x) 3 3 _ 3 0 = 6 4.4.2. Cambios de orden de Integraci´on Problema 1 Invierta el orden de integraci´ on y eval´ ue la integral resultante . I = _ 1 0 _ 2 2x e y 2 dydx Soluci´on. El dominio de integracion dado es D = ¦(x, y) ∈ IR 2 / 0 ≤ x ≤ 1, 2x ≤ y ≤ 2¦ . Si se invierte el orden de integraci´on tenemos que modificar la partici´ on del dominio. D = _ (x, y) ∈ IR 2 / 0 ≤ x ≤ y 2 , 0 ≤ y ≤ 2 _ ,entonces la inte- gral se puede escribir. I = _ 1 0 _ 2 2x e y 2 dydx = _ 2 0 _ y 2 0 e y 2 dxdy = _ 2 0 xe y 2 ¸ ¸ ¸ y 2 0 dy = _ 2 0 y 2 e y 2 dy = 1 4 e y 2 ¸ ¸ ¸ 4 0 = 1 4 _ e 16 −1 _ 361 Problema 2 Invierta el orden de integraci´ on y eval´ ue la integral resultante . I = _ 2 0 _ 4 x 2 √ y cos ydydx Soluci´on. El dominio de integraci´on dado es D = ¦(x, y) ∈ IR 2 / 0 ≤ x ≤ 2, x 2 ≤ y ≤ 4¦ . Si se invierte el orden de integraci´on tenemos que modificar la partici´ on del dominio, D = _ (x, y) ∈ IR 2 / 0 ≤ x ≤ √ y, 0 ≤ y ≤ 4 _ ,entonces la inte- gral se puede escribir _ 2 0 _ 4 x 2 √ y cos ydydx = _ 4 0 _ √ y 0 √ y cos ydxdy = _ 4 0 √ y cos(y)x[ √ y 0 dy = _ 4 0 y cos(y)dy Integrando esta ´ ultima integral por partes se tiene: _ 4 0 y cos(y)dy = ysen(y)[ 4 0 − _ 4 0 sen(y)dy = ysen(y)[ 4 0 + cos(y)[ 4 0 = 4sen(4) + cos(4) −1 Problema 3 Invierta el orden de integraci´ on y eval´ ue la integral resultante . I = _ e 1 _ ln x 0 ydydx Soluci´on. El dominio de integraci´on dado es D = ¦(x, y) ∈ IR 2 / 1 ≤ x ≤ e, 0 ≤ y ≤ ln x¦ . Si se invierte el orden de integraci´on tenemos que el dominio, D = ¦(x, y) ∈ IR 2 / e y ≤ x ≤ e, 0 ≤ y ≤ 1¦ ,entonces la integral se puede escribir 362 _ e 1 _ ln x 0 ydydx = _ 1 0 _ e e y ydxdy = _ 4 0 y x ¸ ¸ ¸ e e y dy = _ 4 0 y(e −e y )dy = e _ y 2 2 _ 4 0 −e y [y −e y ] 4 0 = 8e −4e 4 −1 4.4.3. Cambios de variables: Coordenadas polares Problema 1 Calcular __ D _ x 2 + y 2 _ dxdy si D = ¦(x, y) ∈ IR 2 / x 2 + y 2 ≤ 1¦ ,usando coordenadas polares. V´ease figura 4.25 Soluci´on. A partir de la coordenadas polares tenemos: x = rcosθ, y = rsenθ =⇒ x 2 + y 2 = r 2 El valor absoluto del Jacobiano de transformaci´on a polares es: ¸ ¸ ¸ ¸ ∂ (x, y) ∂ (r, θ) ¸ ¸ ¸ ¸ = r Reemplazando t´erminos en la integral, produce __ D _ x 2 + y 2 _ dxdy = __ D r 2 ¸ ¸ ¸ ¸ ∂ (x, y) ∂ (r, θ) ¸ ¸ ¸ ¸ drdθ = _ 1 0 _ 2π 0 r 3 dθdr = _ 1 0 _ 2π 0 r 3 θ[ 2π 0 dr = 2π _ 1 0 r 3 dr = 2π r 4 4 ¸ ¸ ¸ ¸ 1 0 = π 2 Las coordenadas polares dieron una solucion m´as simple del problema. La simplicidad depende de la naturaleza del problema y de la simetria que presenta el dominio. 363 Problema 2 Calcular el ´ area de la regi´on interior a la circunferencia x 2 + y 2 = 8y y exterior a la circunferencia x 2 + y 2 = 9. Soluci´on. Determinemos el centro y radio de la circunsferencia x 2 + y 2 = 8y =⇒ x 2 + y 2 −8y = 0 =⇒ x 2 + (y −4) 2 = 16 El ´area de la regi´ on D es: A(D) __ D dxdy V´ease figura 4.30 Figura 4.30: Regi´on interior a la circunferencia x 2 + y 2 = 8y y exterior a la circunferencia x 2 + y 2 = 9 Por simetr´ıa, podemos calcular el ´area de la regi´on D en el primer cuad- rante y multiplicar por 2. A fin de conocer los l´ımites de integraci´ on en coordenadas polares necesitamos conocer el ´ angulo que forma la recta OT con el eje x. x 2 + y 2 = 8y =⇒ r 2 = 8rsenθ =⇒ r = 8senθ x 2 + y 2 = 9 =⇒ r = 3 Como T pertenece a ambas circunferencias se cumple 8senθ = 3 =⇒ θ = arcsen 3 8 Luego, la mitad de la regi´ on D ∗ = _ (r, θ) /3 ≤ r ≤ 8senθ; arcsen 3 8 ≤ θ ≤ π 2 _ __ D dxdy = __ D ∗ ¸ ¸ ¸ ¸ ∂ (x, y) ∂ (r, θ) ¸ ¸ ¸ ¸ drdθ 364 2 _ π/2 arcsen 3 8 _ 8senθ 3 rdrdθ = 2 _ π/2 arcsen 3 8 r 2 2 ¸ ¸ ¸ ¸ 8senθ 3 dθ _ π/2 arcsen 3 8 _ 64sen 2 θ −9 _ dθ = _ 64 _ θ 2 − sen2θ 4 _ − 9 2 θ _ π/2 arcsen 3 8 = _ 55 2 θ −16sen2θ _ π/2 arcsen 3 8 = _ 55 4 π − 55 2 arcsen 3 8 + 16sen(2arcsen 3 8 ) _ ≈ 38, 42 Problema 3 Calcular __ D x 2 + y 2 x + _ x 2 + y 2 dxdy , si D es el interior del cardioide r = a (1 + cos θ) Figura 4.31: D es la regi´on interior del cardioide r = a (1 + cos θ) Soluci´on. Cambiando a cordenadas polares, tenemos: __ D x 2 + y 2 x + _ x 2 + y 2 dxdy = __ D ∗ r 2 r cos θ + r ¸ ¸ ¸ ¸ ∂ (x, y) ∂ (r, θ) ¸ ¸ ¸ ¸ drdθ = __ D ∗ r 2 r cos θ + r rdrdθ 365 = _ 2π 0 _ a(1+cos θ) 0 r 2 1 + cos θ drdθ = _ 2π 0 1 1 + cos θ r 3 3 ¸ ¸ ¸ ¸ a(1+cos θ) 0 dθ = a 3 3 _ 2π 0 (1 + cos θ) 2 dθ = a 3 3 _ 2π 0 _ 1 + 2 cos θ + cos 2 θ _ dθ = a 3 3 _ θ + 2senθ + θ 2 + sen2θ 4 _ 2π 0 = πa 3 Observacion si deseamos se rigurosos debemos hacer notar que la integral es impropia cuando x ≤ 0, e y = 0, pues en tal caso el denominador es cero. Luego: I = l´ım α→π − ε→0 _ α 0 _ a(1+cos θ) ε r 2 1 + cos θ drdθ + l´ım β→π+ ε→0 _ 2π β _ a(1+cos θ) ε r 2 1 + cos θ drdθ = l´ım α→π − a 3 3 _ α 0 (1 + cos θ) 2 dθ + l´ım β→π+ a 3 3 _ 2π β (1 + cos θ) 2 dθ = l´ım α→π − a 3 3 _ 3 2 α + 2senα + sen2α 4 _ + l´ım β→π+ a 3 3 _ 3π − 3 2 β −2senβ − sen2β 4 _ = πa 3 Problema 4 Calcular el volumen V del s´ olido acotado por las gr´ aficas z = 9 −x 2 −y 2 y z = 5. Soluci´on. Como el s´olido es sim´etrico, basta encontrar su volumen en el primer octante y multiplicar su resultado por cuatro. Usando integrales dobles y proyectando la regi´on sobre el plano xy ten- emos: V = 4 _ _ D _ 9 −x 2 −y 2 −5 ¸ dxdy 366 Figura 4.32: s´olido acotado por las gr´aficas z = 9 −x 2 −y 2 y z = 5 D = ¦(x, y) ∈ IR 2 / x ≥ 0, y ≥ 0, 0 ≤ x 2 + y 2 ≤ 4¦ A partir de la coordenadas polares, obtenemos: x = rcosθ y = rsenθ _ =⇒ f (x, y) = 4 −x 2 −y 2 = 4 −r 2 0 ≤ x 2 + y 2 = r 2 ≤ 4 ⇐⇒ 0 ≤ r ≤ 2 y 0 ≤ θ ≤ π 2 D ∗ = _ (r, θ) / 0 ≤ r ≤ 2, 0 ≤ θ ≤ π 2 _ El valor absoluto del Jacobiano de transformaci´on a polares es: ¸ ¸ ¸ ¸ ∂ (x, y) ∂ (r, θ) ¸ ¸ ¸ ¸ = r Reemplazando t´erminos en la integral, produce: V = 4 _ _ D ∗ _ 4 −r 2 ¸ rdrdθ = 4 _ π/2 0 _ 2 0 _ 4 −r 2 ¸ rdrdθ = 4 _ π/2 0 _ 4 2 r 2 − 1 4 r 4 _ 2 0 dθ = 8π 4.4.4. Cambios de variables. Coordenadas curvil´ıneas Problema 1 Calcular I = __ D 3xydxdy, donde D es la regi´ on acotada por las rectas x −2y = 0, x −2y = −4 x + y = 4, x + y = 1 (1) Soluci´on. Podemos usar el cambio de variables 367 Figura 4.33: D regi´on acotada por x − 2y = 0, x − 2y = −4, x + y = 4, x + y = 1 u = x −2y v = x + y _ (1) =⇒ x = 1 3 (2u + v) y = 1 3 (u −v) (2) Asi,x −2y = −4 se transforma en u = −4 x −2y = 0 se transforma en u = 0 x + y = 1 se transforma en v = 1 x + y = 4 se transforma en v = 4 Para calcular el Jacobiano ¸ ¸ ¸ ¸ ∂ (x, y) ∂ (u, v) ¸ ¸ ¸ ¸ tenemos dos posibilidades. La primera, es usar la transformaci´ on inversa (2) x e y en t´erminos de u y v . La segunda, mucho m´ as simple, es calcular a partir de (1) ¸ ¸ ¸ ¸ ∂ (u, v) ∂ (x, y) ¸ ¸ ¸ ¸ y luego usar la propiedad ¸ ¸ ¸ ¸ ∂ (x, y) ∂ (u, v) ¸ ¸ ¸ ¸ = _¸ ¸ ¸ ¸ ∂ (u, v) ∂ (x, y) ¸ ¸ ¸ ¸ _ −1 . En efecto ¸ ¸ ¸ ¸ ∂ (u, v) ∂ (x, y) ¸ ¸ ¸ ¸ = ¸ ¸ ¸ ¸ 1 −2 1 1 ¸ ¸ ¸ ¸ = 1 + 2 = 3 =⇒ ¸ ¸ ¸ ¸ ∂ (x, y) ∂ (u, v) ¸ ¸ ¸ ¸ = 1 3 Por lo tanto, del teorema del cambio e variables se deduce que: 368 I = __ D 3xydxdy = __ D ∗ 3 _ 1 3 (2u + v) 1 3 (u −v) _¸ ¸ ¸ ¸ ∂ (x, y) ∂ (u, v) ¸ ¸ ¸ ¸ dudv = _ 4 1 _ 0 −4 1 9 _ 2u 2 −uv −v 2 _ dvdu = 1 9 _ 4 1 _ 2u 2 v − uv 2 2 − v 3 3 _ 0 −4 du = 1 9 _ 4 1 _ 8u 2 + 8u − 64 3 _ du = 1 9 _ 8u 3 3 + 4u 2 − 64 3 u _ 4 1 du = 164 9 Problema 2 Calcular el ´ area de la regi´ on D, que esta acotada por las curvas x 2 −y 2 = 1, x 2 −y 2 = 9 x + y = 4, x + y = 6 (1) Figura 4.34: D regi´on acotada por x 2 −y 2 = 1, x 2 −y 2 = 9, x+y = 4, x+y = 6 Soluci´on. Teniendo en cuenta el cambio de variables que transforma la regi´ on D en la regi´on D ∗ u = x 2 −y 2 v = x + y _ (1) =⇒ La imagen D ∗ de la regi´ on D est´ a acotada por la rectas verticales; x 2 −y 2 = 1 se transforma en u = 1 x 2 −y 2 = 9 se transforma en u = 9 y las rectas horizontales x + y = 4 se transforma en v = 4 369 x + y = 6 se transforma en v = 6 Es decir, D ∗ = ¦(u, v) /1 ≤ u ≤ 9, 4 ≤ v ≤ 6¦ Vamos a calcular ¸ ¸ ¸ ¸ ∂ (x, y) ∂ (u, v) ¸ ¸ ¸ ¸ a partir de (1) ¸ ¸ ¸ ¸ ∂ (u, v) ∂ (x, y) ¸ ¸ ¸ ¸ y usar la propiedad ¸ ¸ ¸ ¸ ∂ (x, y) ∂ (u, v) ¸ ¸ ¸ ¸ = _¸ ¸ ¸ ¸ ∂ (u, v) ∂ (x, y) ¸ ¸ ¸ ¸ _ −1 . En efecto ¸ ¸ ¸ ¸ ∂ (u, v) ∂ (x, y) ¸ ¸ ¸ ¸ = ¸ ¸ ¸ ¸ 2x −2y 1 1 ¸ ¸ ¸ ¸ = 2 (x + y) = 2v =⇒ ¸ ¸ ¸ ¸ ∂ (x, y) ∂ (u, v) ¸ ¸ ¸ ¸ = 1 2v El teorema del cambio variables afirma que: A(D) = __ D dxdy = __ D ∗ ¸ ¸ ¸ ¸ ∂ (x, y) ∂ (u, v) ¸ ¸ ¸ ¸ dudv = _ 9 1 _ 6 4 1 3v dvdu = 1 2 _ 9 1 [ln v] 6 4 du = 1 2 _ ln 6 4 __ 9 1 du = 1 2 ln 3 2 [u] 9 1 = 4 ln 3 2 Problema 3 Calcular I = __ D x 3 + y 3 xy dxdy, donde D es la regi´on del primer cuad- rante acotada por: y = x 2 , y = 4x 2 x = y 2 , x = 4y 2 (1) Soluci´ on. El c´alculo de I ser´ıa bastante complejo si usamos coordenadas cartesianas por la simetr´ıa que tiene el dominio.Sin embargo, una cambio de variables simplifica la regi´ on D y la transforma en D ∗ . Sean u = x 2 y , v = y 2 x Luego D ∗ esta acotada por la rectas verticales; y = x 2 se transforma en u = 1. y = 4x 2 se transforma en u = 1 4 . y las rectas horizontales x = y 2 se transforma en v = 1. 370 Figura 4.35: D regi´on acotada por y = x 2 , y = 4x 2 , x = y 2 , x = 4y 2 x = 4y 2 se transforma en v = 1 4 . Es decir, D ∗ = _ (u, v) /1 ≤ u ≤ 1 4 , 1 ≤ v ≤ 1 4 _ Para calcular ¸ ¸ ¸ ¸ ∂ (x, y) ∂ (u, v) ¸ ¸ ¸ ¸ tenemos dos posibilidades, la primera es despejar x e y en t´erminos de u y v a partir de (1) . La segunda, es calcular ¸ ¸ ¸ ¸ ∂ (u, v) ∂ (x, y) ¸ ¸ ¸ ¸ y usar la propiedad ¸ ¸ ¸ ¸ ∂ (x, y) ∂ (u, v) ¸ ¸ ¸ ¸ = _¸ ¸ ¸ ¸ ∂ (u, v) ∂ (x, y) ¸ ¸ ¸ ¸ _ −1 . En efecto ¸ ¸ ¸ ¸ ∂ (u, v) ∂ (x, y) ¸ ¸ ¸ ¸ = ¸ ¸ ¸ ¸ ¸ ¸ ¸ 2x y − x 2 y 2 − y 2 x 2 2y x ¸ ¸ ¸ ¸ ¸ ¸ ¸ = 4 −1 = 3 =⇒ ¸ ¸ ¸ ¸ ∂ (x, y) ∂ (u, v) ¸ ¸ ¸ ¸ = 1 3 Calculemos ahora la integral 371 I = __ D x 3 + y 3 xy dxdy = __ D _ x 2 y + y 2 x _ dxdy = _ 1 1/4 _ 1 1/4 (u + v) 1 3 dvdu = 1 3 _ 1 1/4 _ uv + v 2 2 _ 1 1/4 du = 1 3 _ 1 1/4 _ 3 4 u + 15 32 _ du = 1 3 _ 3 8 u 2 + 15 32 u _ 1 1/4 = 1 3 _ 3 8 15 16 + 15 32 3 4 _ = 15 64 Problema 4 Evaluar la integral I = __ D [x + y] 2 dxdy, donde D es la regi´ on del plano xy acotado por las curvas x + y = 2, x + y = 4, y = x, x 2 −y 2 = 4, (1) Figura 4.36: D regi´on acotada por x + y = 2, x + y = 4, y = x y x 2 −y 2 = 4 Soluci´on. Observese que las ecuaciones de la curvas de la frontera de D s´ olo incluyen a x e y en las combinaciones de x±y,y el integrando incluye solamentenlas mismas combinaciones. Aprovechando estas simetr´ıas, sean las coordenadas u = x + y, v = x −y Luego, la imagen D ∗ de la regi´ on D est´ a acotada por las curvas; 372 x + y = 2 se transforma en u = 2. x + y = 4 se transforma en u = 4. A su vez x −y = 0 se transforma en v = 0. x 2 −y 2 = (x + y) (x −y) = 4 se transforma en uv = 4. Es decir, D ∗ = _ (u, v) / 2 ≤ u ≤ 4, 0 ≤ v ≤ 4 u _ El jacobiano de la transformaci´ on es ¸ ¸ ¸ ¸ ∂ (x, y) ∂ (u, v) ¸ ¸ ¸ ¸ = _¸ ¸ ¸ ¸ ∂ (u, v) ∂ (x, y) ¸ ¸ ¸ ¸ _ −1 . En efecto ∂ (u, v) ∂ (x, y) = ¸ ¸ ¸ ¸ 1 1 1 −1 ¸ ¸ ¸ ¸ = −2 =⇒ ¸ ¸ ¸ ¸ ∂ (x, y) ∂ (u, v) ¸ ¸ ¸ ¸ = 1 2 Entonces: __ D [x + y] 2 dxdy = 1 2 __ D ∗ u 2 dudv = 1 2 _ 4 2 _ 4/u 0 u 2 dvdu = 1 2 _ 4 2 u 2 v[ 4/u 0 du = 1 2 _ 4 2 4udu = 4 2 u 2 2 ¸ ¸ ¸ ¸ 4 2 = 12 4.4.5. C´alculo de integrales triples en coordenadas rect´angu- lares cartesianas Problema 1 Sea R la regi´on en IR 3 acotada por: z = 0, z = 1 2 y, x = 0; x = 1, y = 0, y = 2 Calcular ___ R (x + y −z) dxdydz. Soluci´on. Del gr´ afico de la regi´on (V´ease figura 4.37) , tenemos que 0 ≤ z ≤ 1 2 y.Proyectando la regi´ on R sobre el plano xy. As´ı D = ¦(x, y) ∈ IR 2 / 0 ≤ x ≤ 1, 0 ≤ y ≤ 2¦ . Por lo tanto; 373 Figura 4.37: R regi´on acotada por z = 0, z = 1 2 y, x = 0; x = 1, y = 0, y = 2 ___ R (x + y −z) dxdydz = __ D ( _ 1 2 y 0 (x + y −z) dz)dxdy _ 1 0 _ 2 0 ( _ 1 2 y 0 (x + y −z) dz)dydx = _ 1 0 _ 2 0 _ xz + yz − z 2 2 _1 2 y 0 dydx _ 1 0 _ 2 0 _ 1 2 (x + y)y − y 2 8 _ dydx = _ 1 0 _ 2 0 _ 1 2 xy + 3 8 y 2 _ dydx _ 1 0 _ 1 4 xy 2 + 1 8 y 3 _ 2 0 dx = _ 1 0 [(x + 1)] dx = _ 1 2 x 2 + x _ 1 0 = 3 2 Tambi´en es posible resolver el problema anterior proyectando la regi´on R sobre el plano xz.En tal caso, 2z ≤ y ≤ 2 y D = ¦(x, z) ∈ IR 2 / 0 ≤ x ≤ 1, 0 ≤ z ≤ 1¦ ___ R (x + y −z) dxdydz = _ 1 0 _ 1 0 ( _ 2 2z (x + y −z) dy)dzdx 374 _ 1 0 _ 1 0 _ xy + y 2 2 −zy _ 2 2z dzdx = 2 _ 1 0 _ 1 0 [x + 1 −z −xz] dzdx 2 _ 1 0 _ xz + z − z 2 2 −x z 2 2 _ 1 0 dx = 2 _ 1 0 _ x + 1 − 1 2 − x 2 _ dx _ 1 0 [(x + 1)] dx = _ 1 2 x 2 + x _ 1 0 = 3 2 Una tercera posibilidad de soluci´on consiste en proyectar la regi´ on R sobre el plano yz. Esta se deja como ejercicio. Problema 2 Calcular ___ D x 2 dxdydz si D es la regi´on acotada por y 2 + z 2 = 4ax, y 2 = ax, x = 3a Figura 4.38: D regi´on acotada por y 2 + z 2 = 4ax, y 2 = ax, x = 3a Soluci´on. La superficie y 2 + z 2 = 4ax corresponde a un paraboloide de revoluci´on como el bosquejado en la figura. En dos variables el gr´ afico de y 2 = ax es una par´ abola, pero es tres variables es la superficie de un manto parab´ olico. Finalmente, el gr´ afico x = 3 es un plano paralelo al plano xz a la distancia 3a. Luego el gr´ afico de la regi´ on es La proyecci´ on de la region sobre el plano xy es: D = _ (x, y, z) ∈ IR 3 /D 1 ∪ D 2 , − _ 4ax −y 2 ≤ z ≤ _ 4ax −y 2 _ 375 Por simetr´ıa se tiene: I = ___ D x 2 dxdydz = 2 __ D 1 _ √ 4ax−y 2 − √ 4ax−y 2 x 2 dzdxdy = 2 _ 3a 0 _ 2 √ ax √ ax _ √ 4ax−y 2 − √ 4ax−y 2 x 2 dzdydx = 2 _ 3a 0 _ 2 √ ax √ ax _ x 2 z ¸ √ 4ax−y 2 − √ 4ax−y 2 dydx = 4 _ 3a 0 _ 2 √ ax √ ax x 2 _ 4ax −y 2 dydx De una tabla de integrales obtenemos _ √ a 2 −u 2 du = 1 2 (u √ a 2 −u 2 + a 2 arcsen u a ) As´ı al integrar la expresi´on: _ 2 √ ax √ ax _ 4ax −y 2 dy = _ 1 2 _ y _ 4ax −y 2 + 4ax arcsen y 2 √ ax __ 2 √ ax √ ax = 2ax arcsen(1) − 1 2 _ √ ax √ 3ax + 4ax arcsen 1 2 _ = 2ax π 2 + 1 2 ax √ 3 −2ax π 6 = 2π 3 ax + √ 3 2 ax Por lo tanto al sustituir en la integral anterior, queda 4 _ 3a 0 _ 2π 3 + √ 3 2 _ ax 3 dx = __ 2π 3 + √ 3 2 _ ax 4 _ 3a 0 = 27a 5 _ 2π + 3 √ 3 2 _ Problema 3 Calcular el volumen del s´ olido Ω acotado por la superficie y = x 2 y los planos y + z = 4 ; z = 0. 376 Figura 4.39: Ω acotado por la superficie y = x 2 y los planos y + z = 4, z = 0 Soluci´on. Consideremos que la regi´ on Ω est´a acotada inferiormente por la frontera z = 0 y superiomente por z = 4 −y. Si Proyectamos la regi´on Ω sobre el plano xy, se tiene: Ω = ¦(x, y, z) ∈ IR 3 / (x, y) ∈ D, 0 ≤ z ≤ 4 −y¦ D = ¦(x, y) ∈ IR 2 / −2 ≤ x ≤ 2, x 2 ≤ y ≤ 4¦ Luego el volumen de la regi´on es V (Ω) = ___ Ω dxdydz = _ 2 −2 _ 4 x 2 _ 4−y 0 dzdydx = _ 2 −2 _ 4 x 2 (4 −y) dydx = _ 2 −2 _ 4y − y 2 2 _ 4 x 2 dx = _ 2 −2 _ 8 −4x 2 + x 4 2 _ dx = _ 8x − 4 3 x 3 + x 4 10 _ 2 −2 = 256 15 4.4.6. Coordenadas esf´ericas Problema 1 Resolver I = ___ D _ x 2 + y 2 + z 2 e − ( x 2 +y 2 +z 2 ) dxdydz si D es la regi´on de IR 3 limitada por las superficies x 2 + y 2 + z 2 = a 2 x 2 + y 2 + z 2 = b 2 con 0 < b < a anillo esf´erico. Soluci´ on. Por la simetr´ıa del dominio y la forma del integrando 377 Figura 4.40: D es la regi´on de IR 3 limitada por las superficies x 2 + y 2 + z 2 = a 2 x 2 + y 2 + z 2 = b 2 con 0 < b < a usaremos coordenadas esf´ericas: x = rsenθ cos φ y = rsenθsenφ z = r cos θ _ _ _ =⇒ b 2 ≤ x 2 + y 2 + z 2 ≤ a 2 =⇒ b ≤ r ≤ a tgθ = y z = 0 =⇒ 0 ≤ θ ≤ π tgφ = y x = 0 =⇒ 0 ≤ φ ≤ 2π Recordando que el valor absoluto del Jacobiano a esf´ericas es : ¸ ¸ ¸ ¸ ∂ (x, y, z) ∂ (r, θ, φ) ¸ ¸ ¸ ¸ = r 2 senθ se tiene: I = _ 2π 0 _ π 0 _ a b re −r 2 ¸ ¸ ¸ ¸ ∂ (x, y, z) ∂ (r, θ, φ) ¸ ¸ ¸ ¸ drdθdφ = _ 2π 0 _ π 0 _ a b r 3 e −r 2 senθ drdθdφ = _ 2π 0 _ π 0 _ − 1 2 r 2 e −r 2 −e −r 2 _ a b senθ dθdφ = _ 1 2 b 2 e −b 2 + 1 2 e −b 2 − 1 2 a 2 e −a 2 −e −a 2 __ 2π 0 _ π 0 senθ dθdφ = _ 1 2 b 2 e −b 2 + 1 2 e −b 2 − 1 2 a 2 e −a 2 −e −a 2 __ 2π 0 −cos θ[ π 0 dφ = 2 _ 1 2 b 2 e −b 2 + 1 2 e −b 2 − 1 2 a 2 e −a 2 −e −a 2 __ 2π 0 dφ = 4π _ 1 2 b 2 e −b 2 + 1 2 e −b 2 − 1 2 a 2 e −a 2 −e −a 2 _ 378 Problema 2 Encontrar el volumen de la regi´ on determinada por x 2 +y 2 +z 2 ≤ 16 , z 2 ≥ x 2 + y 2 . Figura 4.41: Regi´on determinada por x 2 + y 2 + z 2 ≤ 16 , z 2 ≥ x 2 + y 2 Soluci´on. x 2 + y 2 + z 2 = 16 es una esfera con centro en el origen y radio 4 z 2 = x 2 +y 2 es un cono con v´ertice en el origen y eje de simetr´ıa coincidente con el eje z. Como z ≥ 0 , s´ olo debemos considerar s´olo la regi´on sobre el plano xy. La intersecci´ on de la esfera con el cono se obtiene mediante el sistema: x 2 + y 2 + z 2 = 16 x 2 + y 2 = z 2 _ =⇒ z = √ 8 x 2 + y 2 = 8 Usaremos coordenadas esf´ericas: x = rsenθ cos φ y = rsenθsenφ z = r cos θ _ _ _ =⇒ 0 ≤ x 2 + y 2 + z 2 ≤ 16 =⇒ 0 ≤ r ≤ 4 tgθ = y z = √ 8 √ 8 = 1 =⇒ 0 ≤ θ ≤ π 4 tgφ = y x = 0 =⇒ 0 ≤ φ ≤ 2π Recordando que el valor absoluto del Jacobiano a esf´ericas es : ¸ ¸ ¸ ¸ ∂ (x, y, z) ∂ (r, θ, φ) ¸ ¸ ¸ ¸ = r 2 senθ se tiene: 379 V = ___ D dxdydz = _ 2π 0 _ π 4 0 _ 4 0 r 2 senθ drdθdφ V = _ 2π 0 _ π 4 0 r 3 3 ¸ ¸ ¸ ¸ 4 0 senθ dθdφ V = 4 3 3 _ 2π 0 −cos θ[ π 4 0 dφ V = 4 3 3 _ 2π 0 _ 1 − √ 2 2 _ dφ = 4 3 3 _ 1 − √ 2 2 _ 2π Otra opci´ on para resolver este problema es usar coordenadas cil´ındricas,en tal caso x = r cos θ y = rsenθ z = z _ _ _ =⇒ x 2 + y 2 + z 2 = 16 =⇒ z = 16 −r 2 . x 2 + y 2 = z 2 =⇒ z = r 2 Ten´ıamos que el Jacobiano de transformaci´ on a cil´ındricas es: ¸ ¸ ¸ ¸ ∂ (x, y, z) ∂ (r, θ, z) ¸ ¸ ¸ ¸ = r luego: V = ___ D dxdydz = _ 2π 0 _ √ 8 0 _ √ 16−r 2 r 2 rdzdrdθ = _ 2π 0 _ √ 8 0 rz[ √ 16−r 2 r 2 drdθ = _ 2π 0 _ √ 8 0 _ r √ 16 −r 2 −r 2 _ drdθ = _ 2π 0 _ − 1 3 _ (16 −r 2 ) 3 − r 3 3 _ √ 8 0 dθ = − 2π 3 _ 2 √ 8 3 − √ 16 3 _ = 2π 3 _ 64 −32 √ 2 _ 4.4.7. Coordenadas Cil´ındricas Problema 1 Usando integrales triples calcular el volumen de la regi´ on acotada por z = x 2 + y 2 y z = 27 −2x 2 −2y 2 . Soluci´ on. Por la simetr´ıa del volumen los resolveremos usando coordenadas cil´ındri- cas. 380 Figura 4.42: Regi´on acotada por z = x 2 + y 2 y z = 27 −2x 2 −2y 2 x = r cos θ y = rsenθ z = z _ _ _ =⇒ z = x 2 + y 2 =⇒ z = r 2 . z = 27 −2x 2 −2y 2 =⇒ z = 27 −2r 2 x 2 + y 2 = 9 =⇒ r = 3. Como el Jacobiano de transformaci´ on a cil´ındricas es: ¸ ¸ ¸ ¸ ∂ (x, y, z) ∂ (r, θ, z) ¸ ¸ ¸ ¸ = r se tiene: V = ___ D dxdydz = _ 2π 0 _ 3 0 _ 27−2r 2 r 2 rdzdrdθ = _ 2π 0 _ 3 0 r z[ 27−2r 2 r 2 drdθ = _ 2π 0 _ 3 0 r _ 27 −3r 2 _ drdθ = _ 2π 0 _ 27 2 r 2 − 3 4 r 4 _ 3 0 dθ = 243 4 _ 2π 0 dθ = 243 4 2π = 243 2 π Problema 2 Calcular el volumen de la regi´on acotada por la esfera x 2 +y 2 +z 2 = 13 y el cono (z −1) 2 = x 2 + y 2 , z ≥ 1 Soluci´on. El volumen pedido es V = ___ R dxdydz donde la regi´ on R est´ a dada por 381 Figura 4.43: Regi´on acotada por la esfera x 2 +y 2 +z 2 = 13 y el cono (z −1) 2 = x 2 + y 2 , z ≥ 1 R = _ (x, y, z) ∈ IR 3 / (x, y) ∈ D; 1 + _ x 2 + y 2 ≤ z ≤ _ 4 −x 2 −y 2 _ D corresponde a la proyecci´ on de R sobre el plano xy. D = ¦(x, y, z) ∈ IR 2 /x 2 + y 2 ≤ 13¦ Por la simetr´ıa del volumen conviene usar coordenadas cil´ındricas. x = r cos θ y = rsenθ z = z _ _ _ =⇒ x 2 + y 2 + z 2 ≤ r 2 + z 2 ≤ 13 , Determinemos la imagen R ∗ de R (z −1) 2 = x 2 + y 2 ⇐⇒ z ≥ 1 + r =⇒ 1 + r ≤ z ≤ √ 13 −r 2 Luego R ∗ = _ (r, θ, z) ∈ IR 3 / (r, θ) ∈ D; 1 + r ≤ z ≤ √ 13 −r 2 _ La regi´on R al ser proyectada sobre el plano xy. produce z = 0 =⇒ x 2 + y 2 = 13 D ∗ 1 = _ (r, θ) ∈ IR 3 / ≤ r ≤ 2 ; − π 2 ≤ θ ≤ π 2 _ Como el Jacobiano de transformaci´ on a cil´ındricas es: ¸ ¸ ¸ ¸ ∂ (x, y, z) ∂ (r, θ, z) ¸ ¸ ¸ ¸ = r se tiene: 382 V = ___ R dxdydz = _ 2 0 _ 2π 0 _ √ 13−r 2 1+r rdzdθdr = _ 2 0 _ 2π 0 rz √ 13−r 2 1+r dθdr = _ 2 0 _ 2π 0 r _ √ 13 −r 2 −(1 + r) _ dθdr = 2π _ 2 0 _ r √ 13 −r 2 − _ r + r 2 _ _ dr = 2π _ − 1 3 _ 13 −r 2 _ 3/2 − _ r 2 2 + r 3 3 __ 2 0 = 2π _ 1 3 _ 13 3/2 −7 3/2 _ − _ 4 2 + 8 3 __ Problema 3 Calcular utilizando coordenadas cil´ındricas el volumen de la regi´on R , donde R es el interior a la esfera x 2 +y 2 +z 2 = 4, z ≥ 0,y exterior al cilindro (x −1) 2 + y 2 = 1. Figura 4.44: R, regi´on interior a la esfera x 2 + y 2 + z 2 = 4, z ≥ 0,y exterior al cilindro (x −1) 2 + y 2 = 1 Soluci´on. La regi´on R se describe en coordenadas cartesianas mediante R = _ (x, y, z) ∈ IR 3 / (x, y) ∈ D; 0 ≤ z ≤ _ 4 −x 2 −y 2 _ donde D es la proyecci´ on de R sobre el plano xy. 383 D = ¦(x, y) ∈ IR 3 /x 2 + y 2 ≤ 4 ; (x −1) 2 + y 2 ≥ 1¦ Transformemos la regi´ on R a coordenadas cilindricas definidas por x = r cos θ y = rsenθ z = z _ _ _ =⇒ x 2 + y 2 + z 2 = r 2 (cos 2 θ + sen 2 θ) + z 2 ≤ 4 ⇐⇒ 0 ≤ z ≤ √ 4 −r 2 La regi´ on R al ser proyectada sobre el plano xy da origen a dos subregiones x 2 + y 2 ≤ r 2 ≤ 4 ⇐⇒ 0 ≤ r ≤ 2 si π 2 ≤ θ ≤ 3π 2 (x −1) 2 + y 2 ≥ 1 ⇐⇒ r ≥ 2 cos θ y r ≤ 2 si - π 2 ≤ θ ≤ π 2 Entonces, la regi´on R ∗ puede describirse mediante R ∗ = _ (r, θ, z) / (r, θ) ∈ D ∗ = D ∗ 1 ∪ D ∗ 1 ; 0 ≤ z ≤ √ 4 −r 2 _ D ∗ 1 = _ (r, θ) ∈ IR 3 /2 cos θ ≤ r ≤ 2 ; − π 2 ≤ θ ≤ π 2 _ D ∗ 2 = _ (r, θ) ∈ IR 3 /0 ≤ r ≤ 2 ; π 2 ≤ θ ≤ 3π 2 _ Ademas, el Jacobiano de la transformaci´on a cil´ındricas es: ¸ ¸ ¸ ¸ ∂ (x, y, z) ∂ (r, θ, z) ¸ ¸ ¸ ¸ = r 384 En consecuencia la integral puede describirse por I = ___ R (r) drdθdz = _ π/2 −π/2 _ 2 2 cos θ _ √ 4−r 2 0 rdzdrdθ + _ 3π/2 π/2 _ 2 0 _ √ 4−r 2 0 rdzdrdθ = _ π/2 −π/2 _ 2 2 cos θ r [z] √ 4−r 2 0 drdθ + _ 3π/2 π/2 _ 2 0 r [z] √ 4−r 2 0 drdθ = _ π/2 −π/2 _ 2 2 cos θ r √ 4 −r 2 drdθ + _ 3π/2 π/2 _ 2 0 r √ 4 −r 2 drdθ = _ π/2 −π/2 _ − 1 3 _ 4 −r 2 _ 3/2 _ 2 2 cos θ dθ + _ 3π/2 π/2 _ − 1 3 _ 4 −r 2 _ 3/2 _ 2 0 dθ = 8 3 _ π/2 −π/2 _ 1 −cos 2 θ _ 3/2 dθ + 8 3 _ 3π/2 π/2 dθ = 8 3 _ π/2 −π/2 sen 3 θdθ + 8 3 _ 3π/2 π/2 dθ = 8 3 _ −cos θ + cos 3 θ 3 _ π/2 −π/2 + 8 3 π = 8 3 π Problema 4 Calcular I = ___ D _ x 2 a 2 + y 2 b 2 + z 2 c 2 _ dxdydz. En la regi´on D = _ (x, y, z) ∈ IR 3 / x 2 a 2 + y 2 b 2 + z 2 c 2 ≤ 1 _ a > 0, b > 0, c > 0 Figura 4.45: D es la regi´on interior al elipsoide x 2 a 2 + y 2 b 2 + z 2 c 2 = 1 385 Soluci´on. La regi´on de integraci´ on es un elipsoide de semieejes a,b,c. Efectuemos un primer cambio de variables: x = au, y = bv, z = cw. Con ello, D se transforma en la bola. D ∗ = ¦(u, v, w) /u 2 + v 2 + w 2 ≤ 1¦ y el valor absoluto del Jacobiano que- da : ¸ ¸ ¸ ¸ ∂ (x, y, z) ∂ (u, v, w) ¸ ¸ ¸ ¸ = ¸ ¸ ¸ ¸ ¸ ¸ a 0 0 0 b 0 0 0 c ¸ ¸ ¸ ¸ ¸ ¸ = abc Luego, aplicando el teorema del cambio de variables y obtenemos la inte- gral I = ___ D _ x 2 a 2 + y 2 b 2 + z 2 c 2 _ dxdydz. = ___ D ∗ _ u 2 + v 2 + w 2 _ ¸ ¸ ¸ ¸ ∂ (x, y, z) ∂ (u, v, w) ¸ ¸ ¸ ¸ dudvdw = ___ D ∗ _ u 2 + v 2 + w 2 _ ¸ ¸ ¸ ¸ ∂ (x, y, z) ∂ (u, v, w) ¸ ¸ ¸ ¸ dudvdw = ___ D ∗ (u 2 + v 2 + w 2 ) (abc) dudvdw Ahora, transformamos a coordenadas esf´ericas. u = rsenθ cos φ v = rsenθsenφ w = r cos θ _ _ _ =⇒ 0 ≤ u 2 + v 2 + w 2 ≤ 1 =⇒ 0 ≤ r ≤ 1 tgθ = v w =⇒ 0 ≤ θ ≤ π tgφ = v u =⇒ 0 ≤ φ ≤ 2π Quedando, la region D ∗∗ = ¦(r, θ, φ) /0 ≤ r ≤ 1, 0 ≤ θ ≤ π, 0 ≤ φ ≤ 2π¦ abc ___ D ∗ (u 2 + v 2 + w 2 )dudvdw = abc _ 2π 0 _ π 0 _ 1 0 _ r 2 _ r 2 senθ drdθdφ = abc _ 2π 0 _ π 0 r 5 5 ¸ ¸ ¸ ¸ 1 0 senθ dθdφ = abc 5 _ 2π 0 −cos θ[ π 0 dφ = 2abc 5 _ 2π 0 dφ = 4πabc 5 Observaci´ on 386 Es claro que la integraci´ on se podr´ıa haber efectuado usando directamente la trasformaci´on compuesta. x = arsenθ cos φ y = brsenθsenφ z = cr cos θ _ _ _ =⇒ ¸ ¸ ¸ ¸ ∂ (x, y, z) ∂ (r, θ, φ) ¸ ¸ ¸ ¸ = abcr 2 senθ Problema 5 Calcular I = ___ D dxdydz. _ (x −a) 2 + (y −b) 2 + (z −c) 2 , en la regi´on D = ¦(x, y, z) ∈ IR 3 /x 2 + y 2 + z 2 ≤ R 2 ¦ , (a, b, c) es un punto fijo no peteneciente a la esfera x 2 + y 2 + z 2 ≤ R 2 . Figura 4.46: D es la regi´on interior a la esfera x 2 + y 2 + z 2 = R 2 Soluci´on. Si usamos coordenadas cartesianas los l´ımites de integraci´ on son dificul- tosos, pues en tal caso tendr´ıamos. I = ___ D dxdydz. _ (x −a) 2 + (y −b) 2 + (z −c) 2 I = _ r −r _ √ r 2 −x 2 − √ r 2 −x 2 _ √ r 2 −x 2 −y 2 − √ r 2 −x 2 −y 2 dzdydx. _ (x −a) 2 + (y −b) 2 + (z −c) 2 Es claro que si usamos este camino las cosas no ser´an f´ aciles. Sin embargo , dada la simetria esf´erica del dominio y observando que el integrando no es nada m´as que el reciproco de la distancia desde (a, b, c) / ∈ D hasta (x, y, z) ∈ D,nos damos cuenta que el resultado no puede depender m´ as que de la distancia d entre dichos puntos.Por ello, el resultado no puede 387 variar si ubicamos el eje z pasando por el punto (a, b, c). Si (0, 0, d) son las nuevas coordenadas del punto fijo tenemos. I = ___ D dxdydz. _ x 2 + y 2 + (z −d) 2 Observaci´ on El razonamiento anterior es muy usado el c´ alculo de integrales que apare- cen aplicaciones a la F´ısica pues en dicha Ciencia son comunes las leyes en que aparece una distacia o el cuadrado de una distancia en el denominador del integrando. Para calcular I en (*) usamos coordenadas esf´ericas. Obtenemos: I = _ R 0 _ π 0 _ 2π 0 r 2 senθ dφdθdr √ r 2 + d 2 −2dr cos θ = 2π _ R 0 _ π 0 r 2 senθ dθdr √ r 2 + d 2 −2dr cos θ Para calcular J = _ π 0 r 2 senθ dθdr √ r 2 + d 2 −2dr cos θ podemos hacer s = r 2 + d 2 −2dr cos θ ds = 2drsenθdθ Adem´ as, θ = 0 =⇒ s = r 2 + d 2 −2dr = (d −r) 2 θ = π =⇒ s = r 2 + d 2 + 2dr = (d + r) 2 Reemplazando en la integral anterior produce J = r 2d _ (d+r) 2 (d−r) 2 s −1/2 ds = r 2d 2s 1/2 ¸ ¸ (d+r) 2 (d−r) 2 = r 2d [2 (d + r) −2 (d −r)] = r 2d [4r] = 2r 2 d Por lo tanto 388 I = 2π _ R 0 2r 2 d dr I = 4π d r 3 3 ¸ ¸ ¸ ¸ R 0 I = 4π 3d R 3 4.5. Ejercicios propuestos integrales dobles y triples 4.5.1. Integrales dobles Integrales dobles en coordenadas cartesianas 1.- Resolver las integrales: a) _ 1 0 _ 1 √ y 2x 3 dxdy b) _ 4 −5 _ y+2 9 2− √ 4−y dxdy c) _ 2 −2 _ √ 4−x 2 − √ 4−x 2 xydydx Respuestas a) _ 1 0 _ 1 √ y 2x 3 dxdy = 1 3 b) _ 4 −5 _ y+2 3 2− √ 4−y dxdy = 9 2 c) _ 2 −2 _ √ 4−x 2 − √ 4−x 2 xydydx = 0 2.- Calcular la integral __ D xydxdy, donde D es la regi´on acotada por y 2 = x, y 2 = 3x −18, y ≥ 0 Respuesta: __ D xydxdy = 135 2 V´ease Figura 4.47 389 Figura 4.47: Regi´on acotada por y 2 = x, y 2 = 3x −18, y ≥ 0 3.- Calcular la integral __ D x x 2 + y 2 dxdy, donde D = _ (x, y) ∈ IR 2 /1 ≤ x ≤ 2, x 2 2 ≤ y ≤ x 2 _ Figura 4.48: Regi´on acotada por y 2 = x, y 2 = 3x −18, y ≥ 0 Respuesta: I = __ D x x 2 + y 2 dxdy = arctan 2 − 3 2 ln 5 − 1 4 π + 7 2 ln 2 V´ease Figura 4.48 4.- Eval´ ue la integral resultante: _ 8 0 _ 2 3 √ y y √ 16 + x 7 dxdy Respuesta _ 2 0 _ x 3 0 y √ 16 + x 7 dydx = 8 7 V´ease Figura 4.49 390 Figura 4.49: Regi´on acotada por y 2 = x, y 2 = 3x −18, y ≥ 0 5.- Calcule el volumen del s´ olido acotado por las gr´aficas de: x 2 + y 2 = 9, y 2 + z 2 = 2. Figura 4.50: S´olido acotado por las gr´aficas de x 2 + y 2 = 9, y 2 + z 2 = 2. Respuesta V (R) = 8 _ 3 0 _ √ 9−y 2 0 _ 9 −y 2 dxdy 6.- Calcule el volumen del s´olido acotado por las gr´aficas de las ecuaciones dadas por z = x 2 + 4, y = 4 −x 2 , x + y = 2; z = 0. Respuesta V (R) = 423 20 391 Figura 4.51: S´olido acotado por las gr´aficas de las ecuaciones dadas por z = x 2 +4, y = 4 −x 2 , x + y = 2 z = 0. 4.5.2. C´alculo de Integrales dobles usando transforma- ci´on de coordenadas 1.- Calcular __ R y x 2 + y 2 dxdy,donde R es a regi´on plana determinada por los puntos del primer cuadrante que son interiores a la circun- fererencia x 2 −4x+y 2 = 0 y exteriores a la circunferencia x 2 +y 2 = 4. Figura 4.52: Regi´on plana determinada por los puntos del primer cuadrante que son interiores a la circunfererencia x 2 −4x+y 2 = 0 y exteriores a la circunferencia x 2 + y 2 = 4 Respuesta: I = __ R y x 2 + y 2 dxdy = 1 2 392 2.- Calcular la integral __ R _ (x −y) 2 + 2 (x + 2) + 1 ¸ −1/2 dxdy, donde R es la regi´on plana acotada por y = 0, x = 2, y = x. a) Utilice la transformaci´ on u = x −y, v = x + y. Figura 4.53: R es la regi´on plana acotada por y = 0, x = 2, y = x y las transfor- maciones u = x −y, v = x + y. y x = u(1 + v) , y = v (1 + u) b) Utilice la sustituci´on x = u(1 + v) , y = v (1 + u) . Respuesta. I = __ D _ (x −y) 2 + 2 (x + 2) + 1 ¸ −1/2 dxdy = 2 ln 2 − 1 2 3.- Resolver las integrales usando las coordenadas m´as convenientes a) _ 2a 0 _ √ 2ax−x 2 0 _ x 2 + y 2 _ dydx b) _ 2 −2 _ √ 4−y 2 0 _ x 2 + y 2 dxdy c) _ √ π − √ π _ √ π−y 2 − √ π−y 2 sen _ x 2 + y 2 _ dxdy d) _ 2 −2 _ 2+ √ 4−x 2 2− √ 4−x 2 _ x 2 + y 2 dxdy e) __ R e y x dxdy, si R esta acotado por x = a, x = b, y = px, y = qx, con a < b, p < q. 393 Respuestas. a) _ 2a 0 _ √ 2ax−x 2 0 _ x 2 + y 2 _ dydx = 3 4 πa 2 b) _ 2 −2 _ √ 4−y 2 0 _ x 2 + y 2 dxdy = 4 3 π c) _ √ π − √ π _ √ π−y 2 − √ π−y 2 sen _ x 2 + y 2 _ dxdy = 2π d) _ 2 −2 _ 2+ √ 4−x 2 2− √ 4−x 2 _ x 2 + y 2 dxdy = 64 9 (3π −4) e) __ R e y x dxdy = 1 2 _ b 2 −a 2 _ (e q −e p ) 4.5.3. Integrales triples 4.5.4. Integrales triples iteradas 1.- Resolver las integrales triples por iteraciones: a) _ 2 0 _ 2 √ x 0 _ √ 4x−y 2 2 0 xdzdydx b) _ a 0 _ √ a 2 −x 2 0 _ √ a 2 −x 2 −y 2 0 _ a 2 −x 2 −y 2 −z 2 dzdydx c) _ 1 −1 _ √ 1−x 2 − √ 1−x 2 _ √ 1 2 −x 2 −y 2 0 _ x 2 + y 2 _ dzdydx Respuestas a) _ 2 0 _ 2 √ x 0 _ √ 4x−y 2 2 0 xdzdydx = 4 3 π √ 2 b) _ a 0 _ √ a 2 −x 2 0 _ √ a 2 −x 2 −y 2 0 _ a 2 −x 2 −y 2 −z 2 dzdydx = π 2 8 a 2 c) _ 1 −1 _ √ 1−x 2 − √ 1−x 2 _ √ 1 2 −x 2 −y 2 0 _ x 2 + y 2 _ dxdy = 4 15 π 394 2.- Calcular las ´areas de las regiones D⊂ R 2 acotadas por: a) xy ≤ 4, y ≤ x, 27y ≥ 4x 2 . Figura 4.54: D es la regi´on plana acotada por xy ≤ 4, y ≤ x, 27y ≥ 4x 2 b) x 2 + y 2 ≤ 9, y ≤ x + 3, y ≤ −x. Figura 4.55: D es la regi´on plana acotada por x 2 + y 2 ≤ 9, y ≤ x + 3, y ≤ −x Respuestas a) A(D) = 2 3 + 4 ln 3 2 . b) A(D) = 9 8 (3π + 2) . 395 4.5.5. Integrales triples en coordenadas rect´angulares cartesianas. 1.- Calcular la integral triple ___ D xyzdxdydz, siendo el dominio de integraci´ on D el tetaedro delimitado por los planos coordenados y el plano de ecuaci´ on x + y 2 + z 3 = 1. Respuesta. I = __ D xyzdxdydz = 1 20 . 2.- Calcular la integral triple ___ D x 2 dxdydz, siendo el dominio de integraci´ on D acotado por y 2 + z 2 = 4ay, y 2 = ax, x = 3a,con a > 0 Respuesta. I = ___ D x 2 dxdydz = 27a 2 _ 2π − 3 √ 3 2 _ 3.- Calcular la integral triple ___ D xdxdydz, si D es la regi´ on de IR 3 acotada por las superficies 2x + 3y = 6, 2x + 3z = 6, y = 3 + 2x − x 2 , z = 0 Figura 4.56: D es la regi´on de IR 3 acotada por las superficies 2x+3y = 6, 2x+3z = 6, y = 3 + 2x −x 2 , z = 0 Respuesta. I = 1 9 _ 6 5 x 5 − 34 4 x 4 + 42 3 x 3 + 9x 2 _ 3 −1/3 396 4.5.6. Calcular las integrales dadas usando las coorde- nadas adecuadas: a) ___ D (x + y + z) −3 dxdydz,si D esta acotado por x = 0, y = 0, z = 0, x + y + z = 0. Figura 4.57: D acotado por x = 0, y = 0, z = 0, x + y + z = 0 b) ___ D (x+y+z) 2 dxdydz,si D esta acotado por 2 z = x 2 +y 2 , x 2 +y 2 +z 2 = 3 Figura 4.58: D acotado por 2 z = x 2 + y 2 , x 2 + y 2 + z 2 = 3 397 c) ___ D z 2 dxdydz,si D esta limitado por x 2 +y 2 +z 2 = a 2 , x 2 +y 2 +z 2 = 2az Figura 4.59: D esta limitado por x 2 + y 2 + z 2 = a 2 , x 2 + y 2 + z 2 = 2az d) ___ D zdxdydz,si D esta acotado por z = 0, x 2 a 2 + y 2 b 2 + z 2 c 2 = 1 Figura 4.60: D esta acotado por z = 0, x 2 a 2 + y 2 b 2 + z 2 c 2 = 1 398 e) ___ D zdxdydz,si D esta limitado por z = h, z 2 = h 2 (x 2 + y 2 ) Figura 4.61: D esta limitado por z = h, z 2 = h 2 _ x 2 + y 2 _ Respuestas a) ___ D (x + y + z) −3 dxdydz = 1 2 ln 2 − 5 16 b) ___ D (x + y + z) 2 dxdydz = π 5 _ 18 √ 3 − 97 6 _ c) ___ D z 2 dxdydz = 59 480 πa 5 d) ___ D zdxdydz = πabc 2 4 e) ___ D zdxdydz = πh 2 4 4.5.7. Resolver las integrales usando coordenadas esf´eri- cas: a) _ a −a _ √ a 2 −x 2 − √ a 2 −x 2 _ √ a 2 −x 2 −y 2 0 _ x 2 + y 2 _ dzdydx. b) _ _ _ D _ x 2 + y 2 _ dxdydz, si D es la regi´on limitada por r 2 ≤ x 2 + y 2 + z 2 ≤ R 2 , z > 0. Respuestas: 399 a) _ a −a _ √ a 2 −x 2 − √ a 2 −x 2 _ √ a 2 −x 2 −y 2 0 _ x 2 + y 2 _ dzdydx = 4 15 πa 5 . b) _ _ _ D _ x 2 + y 2 _ dxdydz = 4 15 _ R 5 −r 5 _ . Calcular los volumenes de las regiones D⊂ R 3 acotado por: a) z = x, z = x 2 + y 2 Figura 4.62: Regi´on limitada por z = x, z = x 2 + y 2 b) x 2 + y 2 = 4, x 2 + y 2 −z 2 = 1 Figura 4.63: Regi´on limitada por x 2 + y 2 = 4, x 2 + y 2 −z 2 = 1 c) z 2 = x 2 + y 2 , y = 0, y = x, x = a. 400 Figura 4.64: Regi´on limitada por z 2 = x 2 + y 2 , y = 0, y = x, x = a d) z = x + y, z = 0, x 2 + y 2 = 1, x = 1, y = 1. Respuestas. Figura 4.65: Regi´on limitada por z = x + y, z = 0, x 2 + y 2 = 1, x = 1, y = 1 a) V (D) = π 32 b) V (D) = 4π √ 3 c) V (D) = a 2 3 _√ 2 + ln _ 1 + √ 2 __ d) V (D) = 1 3 4.6. Aplicaciones integrales dobles y triples 4.6.1. Volumenes de cuerpos en el espacio Si la funci´on z = f (x, y) es continua tal que f (x, y) ≥ 0, entonces la integral doble: V (Ω) = _ _ R f (x, y) dA 401 define el volumen Ω bajo la superficie de la gr´ afica de la funci´ on f (x, y) sobre la regi´on R ⊂ IR 2 Problema 1 Calcular el volumen acotado por el paraboloide z = x 2 + y 2 entre los planos z = 1 y z = 4. Figura 4.66: Volumen acotado por el paraboloide z = x 2 + y 2 y los planos z = 1 y z = 4 Soluci´ on. Es claro que el volumen a calcular esta dado por V (Ω) = _ _ R (x 2 + y 2 )dA donde R = ¦(x, y) ∈ IR 2 /1 ≤ x 2 + y 2 ≤ 1¦ Cambiando a coordenadas polares, queda x = r cos θ y = rsenθ _ =⇒ 1 ≤ x 2 + y 2 ≤ 4 =⇒ 1 ≤ r ≤ 2, 0 ≤ θ ≤ 2π Luego , el dominio de integraci´on se transforma en: R ∗ = ¦(r, θ) /1 ≤ r ≤ 2, 0 ≤ θ ≤ 2π¦ Adem´ as, el jacobiano de transformacion es ¸ ¸ ¸ ¸ ∂(x, y) ∂(r, θ) ¸ ¸ ¸ ¸ = r Reemplazando t´erminos en la ´ ultima integral se tiene V = _ 2π 0 _ 2 1 r 2 (r) drdθ = _ 2π 0 _ r 4 4 _ 2 1 dθ = _ 2π 0 _ 2 4 4 − 1 4 _ dθ = 15 4 (2π) = 15 2 π 402 Problema 2 Calcular el volumen acotado por z ≤ 36 −3x 2 −y 2 , y z ≥ x 2 + 3y 2 . Figura 4.67: Volumen acotado por el paraboloide z ≤ 36−3x 2 −y 2 , y z ≥ x 2 +3y 2 Soluci´on. Ambas ecuaciones z = 36 − 3x 2 − y 2 , y z = x 2 + 3y 2 representan paraboloides de revoluci´on. La regi´on de integraci´on est´a definida por R = _ (x, y, z) ∈ IR 3 / x 2 + 3y 2 ≤ z ≤ 36 −3x 2 −y 2 _ . Determinemos la proyecci´ on de la intersecci´ on de ambas superficies sobre el plano xy. z = 36 −3x 2 −y 2 z = x 2 + 3y 2 _ =⇒ 36−3x 2 −y 2 = x 2 +3y 2 =⇒ 32 = 4x 2 +4y 2 Por tanto, x 2 + y 2 = 9, corresponde a una circunferencia centrada en el origen de radio 3. En consecuencia, el volumen de la regi´ on es V = _ _ D __ 36 −3x 2 −y 2 _ −(x 2 + 3y 2 ) ¸ dxdy = _ _ D _ 36 −4x 2 −4y 2 ¸ dxdy donde D = ¦(x, y) ∈ IR 2 / 0 ≤ x 2 + y 2 ≤ 9¦ Como la funci´ on del integrando es sim´etrica con respecto al plano xy, podemos calcular el volumen en el primer octante y multiplicar por cuatro. 403 V = 4 _ _ D + _ 36 −4x 2 −4y 2 ¸ dxdy donde D ∗ = _ (x, y) ∈ IR 2 / 0 ≤ x ≤ 3, 0 ≤ y ≤ √ 9 −x 2 _ . V = 4 _ 3 0 _ √ 9−x 2 0 __ 36 −4x 2 −4y 2 _¸ dydx = 4 _ 3 0 _ √ 9−x 2 0 4 _ 9 −x 2 −y 2 ¸ dydx Para simplificar los calculos usemos coordenadas polares x = r cos θ y = rsenθ _ =⇒ 0 ≤ x 2 + y 2 ≤ 9 =⇒ 0 ≤ r ≤ 3, 0 ≤ θ ≤ π 2 Luego , el dominio de integraci´on se transforma en: D ∗∗ = _ (r, θ) /0 ≤ r ≤ 3, 0 ≤ θ ≤ π 2 _ Adem´ as, el jacobiano de transformaci´on es ¸ ¸ ¸ ¸ ∂(x, y) ∂(r, θ) ¸ ¸ ¸ ¸ = r Reemplazando t´erminos en la ´ ultima integral se tiene V = 16 _ π 2 0 _ 3 0 _ 9 −r 2 ¸ (r) drdθ = 16 _ π 2 0 _ 9 2 r 2 − r 4 4 _ 3 0 dθ = 16 _ π 2 0 _ 3 4 2 − 3 4 4 _ dθ = 16 _ 3 4 4 _ π 2 = 162π 4.6.2. ´ Area de figuras planas. Sea D una regi´on acotada en el plano xy. El ´area de D se define por: A(D) = __ D dxdy Si se tiene una regi´on D de tipo I, D = _ (x, y) ∈ IR 2 / a ≤ x ≤ b, φ 1 (x) ≤ y ≤ φ 2 (x) _ 404 obtendr´ıamos que A(D) = __ D dxdy = _ b a _ φ 2 (x) φ 1 (x) dydx = _ b a (φ 2 (x) −φ 1 (x))dx Problema 1 Calcular el ´area D de la regi´ on situada en el primer cuadrante, acotada por: y = x 2 , y = 4x 2 x = y 2 , x = 4y 2 (1) Figura 4.68: ´area D de la regi´on situada en el primer cuadrante, acotada por y = x 2 , y = 4x 2 , x = y 2 y x = 4y 2 Soluci´on. El c´alculo del ´ area ser´ıa bastante engorroso si usamos una regi´ on de tipo I en coordenadas cartesianas por la simetr´ıa que tiene la regi´ on. Sin embargo, un cambio de variables simplifica la regi´ on D y la transforma en D ∗ . Sean u = x 2 y , v = y 2 x Entonces las curvas que acotan la regi´on se tranforman en y = x 2 =⇒ u = 1. y = 4x 2 =⇒ u = 1 4 . Luego D ∗ est´ a acotada por la rectas verticales; y las rectas horizontales x = y 2 =⇒ v = 1. 405 x = 4y 2 =⇒ transforma en v = 1 4 . Es decir, D ∗ = _ (u, v) /1 ≤ u ≤ 1 4 , 1 ≤ v ≤ 1 4 _ Para calcular ¸ ¸ ¸ ¸ ∂ (x, y) ∂ (u, v) ¸ ¸ ¸ ¸ tenemos dos posibilidades, la primera es despejar x e y en t´erminos de u y v a partir de (1) . La segunda, es calcular ¸ ¸ ¸ ¸ ∂ (u, v) ∂ (x, y) ¸ ¸ ¸ ¸ y usar la propiedad ¸ ¸ ¸ ¸ ∂ (x, y) ∂ (u, v) ¸ ¸ ¸ ¸ = _¸ ¸ ¸ ¸ ∂ (u, v) ∂ (x, y) ¸ ¸ ¸ ¸ _ −1 . En efecto ¸ ¸ ¸ ¸ ∂ (u, v) ∂ (x, y) ¸ ¸ ¸ ¸ = ¸ ¸ ¸ ¸ ¸ ¸ ¸ 2x y − x 2 y 2 − y 2 x 2 2y x ¸ ¸ ¸ ¸ ¸ ¸ ¸ = 4 −1 = 3 =⇒ ¸ ¸ ¸ ¸ ∂ (x, y) ∂ (u, v) ¸ ¸ ¸ ¸ = 1 3 Calculemos ahora el ´ area pedida: A(D) = __ D dxdy = _ _ 1 D 1 3 dvdu = 1 3 _ 1 1/4 _ 1 1/4 dvdu = 1 3 _ 1 1/4 [v] 1 1/4 du = 1 4 [u] 1 1/4 = 3 16 4.6.3. Momentos y centros de masa para placas planas delgadas Definici´ on: Sea L una l´ amina con la forma de una regi´on R del plano xy. Si la densidad en (x, y) es δ (x, y) , y δ es continua en R, entonces la masa M y los primeros momentos de la l´amina con respecto al eje x e y respectivamente son M x , M y y los centros de masa (x, y) se definen: 406 M = _ _ R δ (x, y) dA M x = _ _ R yδ (x, y) dA M y = _ _ R xδ (x, y) dydx x = M y M = _ _ R xδ (x, y) dA _ _ R δ (x, y) dA y = M x M = _ _ R yδ (x, y) dA _ _ R δ (x, y) dA Si una part´ıcula de masa m se encuentra en el punto (x, y, z) , entonces su distancia al eje z es Sea L una l´ amina con la forma de una regi´on R del plano xy. Si la densidad en (x, y) es δ (x, y) , y δ es continua en R, entonces los segundos momentos de la l´amina o momentos de inercia con respecto al eje x e y respectivamente son I x , I y y los momentos de inercia con respecto a una recta L y respecto al origen se definen: I x = _ _ R y 2 δ (x, y) dA I y = _ _ R x 2 δ (x, y) dydx I L = _ _ R r 2 (x, y) δ (x, y) dA I 0 = _ _ R (x 2 + y 2 )δ (x, y) dA = I x + I y 4.6.4. Centroide de figuras geom´etricas Cuando δ (x, y) es igual a uno, la posici´ on del centro de masa x, y se convierte en una carater´ıstica de la forma del objeto y no del material que est´ a hecho y se define como el centroide de una placa. Problema 1 Dada una placa delgada que cubre la regi´on triangular acotada por el eje x y las rectas x = 1, y = 2x en el primer cuadrante si la densidad de la placa en en cada punto de la regi´ on esta dada por δ (x, y) = 6x + 6y + 6.Calcular: 407 a) la masa de la placa. b) los primeros momentos c) el centro de masa. d) momentos de inercia(segundos momentos) con respecto a los ejes x e y e) radios de giro con respecto a los ejes x e y respectivamente f) radio de giro con respecto al origen Figura 4.69: Regi´on triangular acotada por el eje x y las rectas x = 1, y = 2x en el primer cuadrante Soluci´on. Consideremos una partici´ on de la regi´on que delimita la placa como una regi´ on de tipo I: R = ¦(x, y) / 0 ≤ x ≤ 1, 0 ≤ y ≤ 2x¦ . a) La masa de la placa es M = _ 1 0 _ 2x 0 δ (x, y) dydx = _ 1 0 _ 2x 0 (6x + 6y + 6) dydx = _ 1 0 _ 6xy + 3y 2 + 6y ¸ 2x 0 dx = _ 1 0 _ 24x 2 + 12x ¸ dx = _ 8x 3 + 6x 2 ¸ 1 0 = 14 b) El primer momento con respecto al eje x es 408 M x = _ 1 0 _ 2x 0 yδ (x, y) dydx = _ 1 0 _ 2x 0 _ 6xy + 6y 2 + 6y _ dydx = _ 1 0 _ 3xy 2 + 2y3 + 6y 2 ¸ 2x 0 dx = _ 1 0 _ 28x 3 + 12x 2 ¸ dx = _ 7x 4 + 4x 1 ¸ 1 0 = 11 Por otra parte, el primer momento con respecto al eje y es: M y = _ 1 0 _ 2x 0 xδ (x, y) dydx = _ 1 0 _ 2x 0 _ 6x 2 + 6xy + 6x _ dydx = _ 1 0 _ 6x 2 y + 3xy 2 + 6xy ¸ 2x 0 dx = _ 1 0 _ 12x 3 + 12x 3 + 12x 2 ¸ dx = _ 6x 4 + 4x 3 ¸ 1 0 = 10 c) As´ı, las coordenadas del centro de masa son: x = M y M = 10 14 = 5 7 y = M x M = 11 14 d) El momento de inercia con respecto al eje x es 409 I x = _ 1 0 _ 2x 0 y 2 δ (x, y) dydx = _ 1 0 _ 2x 0 _ 6xy 2 + 6y 3 + 6y 2 _ dydx = _ 1 0 _ 6xy 3 + 3 2 y 4 + 2y 3 _ 2x 0 dx = _ 1 0 _ 40x 4 + 116x 3 ¸ dx = _ 8x 5 + 4x 4 ¸ 1 0 = 12 Por otra parte, el momento de inercia con respecto al eje y es I y = _ 1 0 _ 2x 0 x 2 δ (x, y) dydx = _ 1 0 _ 2x 0 _ 6x 3 + 6x 2 y + 6x 2 _ dydx = _ 1 0 _ 6x 3 y + 3x 2 y 2 + 6x 2 y ¸ 2x 0 dx = _ 1 0 _ 12x 4 + 12x 4 + 12x 3 ¸ dx = _ 24 5 x 5 + 3x 4 _ 1 0 = 39 5 Los radios de giros con respecto a los ejes x e y son R x = _ I x M = _ 12 14 = _ 6 7 R y = _ I y M = _ 39 70 El radio de giro con respecto al origen es R 0 = _ I 0 M donde el momento de inercia con respecto al origen esta dado por: I 0 = _ _ (x 2 + y 2 )δ (x, y) dxdy = I x + I y 410 R 0 = ¸ 99 5 14 Problema 2 Calcular el momento de inercia con respecto al eje x de la l´ amina plana R situada en el primer cuadrante, acotada por las curvas y 2 = x, y 2 = 2x, x 2 = y, x 2 = 3y, si la densidad en cada punto de la regi´ on es δ (x, y) = x 3 y Figura 4.70: Soluci´on: El momento de inercia con respecto al eje x en coordenadas cartesianas queda I x = __ R y 2 δ (x, y) dxdy = __ R y 2 (x 3 y)dxdy = __ R (xy) 3 dxdy Sin embargo, conviene hacer un cambio de variables para simplificar la geometr´ıa de la regi´on u = y 2 x v = x 2 y _ ¸ _ ¸ _ =⇒ J = ¸ ¸ ¸ ¸ ∂ (x, y) ∂ (u, v) ¸ ¸ ¸ ¸ = _¸ ¸ ¸ ¸ ∂ (u, v) ∂ (x, y) ¸ ¸ ¸ ¸ _ −1 = = ¸ ¸ ¸ ¸ ¸ ¸ ¸ −y 2 x 2 2y x 2x y −x 2 y 2 ¸ ¸ ¸ ¸ ¸ ¸ ¸ −1 = 1 3 411 Examinemos ahora como se transforma la regi´ on y 2 = x, y 2 = 2x =⇒ y 2 x = 1; y 2 x = 2 x 2 = y, x 2 = 3y =⇒ x 2 y = 1; x 2 y = 3 Luego, R ∗ = ¦(u, v) / 1 ≤ u ≤ 2, 1 ≤ v ≤ 3¦ . Por otra parte, el integrando se puede expresar uv = y 2 x x 2 y = xy I x = __ R (xy) 3 dxdy = _ 2 1 _ 3 1 (uv) 3 _ 1 3 _ dvdu = 1 3 _ 2 1 u 3 du _ 3 1 v 3 dv = 1 3 _ u 4 4 _ 2 1 _ v 4 4 _ 3 1 = 1 3 _ 4 − 1 4 _ _ 81 4 − 1 4 _ = 1 3 15 4 20 = 25 4.6.5. Momentos y Centros de masa de un s´olido Sea Q un s´ olido una regi´ on del espacio en IR 3 . Si la densidad en (x, y, z) es δ (x, y, z) , y δ es continua en Q, entonces la masa M y los primeros mo- mentos del s´olido con respecto a los planos xy, xz e yz respectivamente son M xy , M xz , M yz y las coordenadas del centro de masa (x, y, z) se definen: 412 M = _ _ _ Q δ (x, y, z) dV M xy = _ _ _ Q yδ (x, y) dV M xz = _ _ _ Q yδ (x, y, z) dV M yz = _ _ _ Q xδ (x, y, z) dV x = M xy M = _ _ _ Q zδ (x, y, z) dV _ _ _ Q δ (x, y, z) dV y = M xz M = _ _ _ Q yδ (x, y, z) dV _ _ _ Q δ (x, y, z) dV z = M xy M = _ _ _ Q xδ (x, y, z) dV _ _ _ Q δ (x, y, z) dV Sea Q un s´ olido una regi´ on del espacio en R 3 . Si la densidad en (x, y, z) es δ (x, y, z) , y δ es continua en Q, entonces los segundos momentos del s´olido o momentos de inercia con respecto al eje x, al eje y,y al eje z respectivamente son I x , I y , I z se definen: I x = _ _ _ Q (y 2 + z 2 )δ (x, y, z) dV I y = _ _ _ Q (x 2 + z 2 )δ (x, y, z) dV I z = _ _ _ Q (x 2 + y 2 )δ (x, y, z) dV 4.6.6. Masa de un s´ olido Problema 1 La densidad en un cubo de lado a es proporcional al cuadrado de la dis- tancia respecto a uno de los v´ertices ( al que denotaremos por 0). Demostrar que la masa del cubo es igual a su volumen multiplicada por la densidad en un v´ertice adyacente a 0. 413 Soluci´on. Si R es una regi´ on cerrada y acotada en IR 3 y la densidad en el punto (x, y, z) ∈ R es δ (x, y, z) , entonces la masa de R est´a dada por M = _ _ R δ (x, y, z) dV Situando el cubo con el v´ertice en el origen del sistema de coordenadas y tres de sus caras coincidiendo con los planos coordenados, se tiene: δ (x, y, z) = x 2 + y 2 + z 2 y por lo tanto M = _ _ R _ x 2 + y 2 + z 2 _ dxdydz donde R = ¦(x, y, z) ∈ IR 3 /0 ≤ x ≤ a; 0 ≤ y ≤ a; 0 ≤ z ≤ a¦ . Luego M = _ a 0 _ a 0 _ a 0 _ x 2 + y 2 + z 2 _ dzdydx = _ a 0 _ a 0 _ (x 2 + y 2 )z + z 3 3 _ a 0 dydx = _ a 0 _ a 0 _ x 2 a + y 2 a + a 3 3 _ dydx = _ a 0 _ ax 2 y + a y 3 3 + a 3 3 y _ a 0 dx = _ a 0 _ a 2 x 2 + 2a 4 3 _ dx = _ a 3 x 3 3 + 2a 4 3 x _ a 0 = a 5 Por otra parte la distancia entre 0 y cualquier v´ertice adyacente a ´el es a. Entonces la densidad de un v´ertice adyacente es δ (a, 0, 0) = a 2 , δ (0, a, 0) = a 2 , δ (0, 0, a) = a 2 . Por lo tanto, queda V (R) δ (a, 0, 0) = a 3 a 2 = a 5 = M Problema 2 Un s´ olido tiene forma de cilindro circular recto con radio de la base R y altura h. Si la densidad en un punto es directamente proporcional a la distancia a una de las bases. Encontrar: a) las coordenadas del centro de masa b) el momento de inercia con respecto al eje axial de simetr´ıa del s´ olido cil´ındrico Soluciones La regi´on del s´olido cil´ındrico est´ a acotado por Q = ¦(x, y, z) ∈ IR 3 / 0 ≤ x 2 + y 2 ≤ R 2 , 0 ≤ z ≤ h¦ Calculemos en primer lugar la masa total del cil´ındro, usando coorde- nadas cil´ındricas , la regi´ on del s´ olido se transforma en Q∗ = ¦r, θ, z)/ 0 ≤ r ≤ R, 0 ≤ θ ≤ 2π, 0 ≤ z ≤ h¦ 414 M = ___ Q δ (x, y, z) dV = ___ Q ∗ kzrdrdθdz = _ R 0 _ 2π 0 _ h 0 kzrdzdθdr = _ R 0 _ 2π 0 k _ z 2 2 _ h 0 dθdr = _ R 0 _ 2π 0 k _ h 2 2 _ rdθdr = k h 2 2 _ R 0 [θ] 2π 0 rdr = πkh 2 _ r 2 2 _ R 0 = 1 2 πkh 2 R 2 El momento de inercia del s´olido con respecto al plano xy es M xy = ___ Q zδ (x, y, z) dV = ___ Q z (kz) dV = ___ Q ∗ kz 2 (r) drdθdz = _ R 0 _ 2π 0 _ h 0 kz 2 rdzdθdr = _ R 0 _ 2π 0 k _ z 3 3 _ h 0 dθdr = k h 3 3 _ R 0 [θ] 2π 0 rdr = 2 3 πkh 3 _ r 2 2 _ R 0 = 1 3 πkh 3 R 2 Finalmente, las coordenadas del centro de masa son: x = M y M = 0 y = M x M = 0 z = M xy M = 1 3 πkh 3 R 2 1 2 πkh 2 R 2 = 2 3 h 415 b) El momento de inercia con respecta al eje de simetr´ıa z es: I z = ___ Q (x 2 + y 2 )δ (x, y, z) dV = ___ Q ∗ (r 2 )kz (r) drdθdz = _ R 0 _ 2π 0 _ h 0 (r 3 )kzdzdθdr = k h 2 2 _ R 0 _ 2π 0 (r 3 )dθdr = πkh 2 _ R 0 (r 3 )dr = πkh 2 _ r 4 4 _ R 0 = 1 4 πkh 2 R 4 Problema 3 Calcular el momento respecto al eje de simetr´ıa de un cono homog´eneo aco- tado por las gr´ aficas x 2 − y 2 + z 2 = 0 e y = 3, si tiene una densidad δ (x, y, z) constante Figura 4.71: Soluci´on Sea la regi´ on del cono circular recto : Q = _ (x, y, z) ∈ IR 3 / −y ≤ x ≤ y, 0 ≤ y ≤ 3, − _ y 2 −x 2 ≤ z ≤ _ y 2 −x 2 _ Sea δ (x, y, z) = k, la densidad es constante. Luego, el momento con respecto al eje y es: I z = ___ Q (x 2 + y 2 )δ (x, y, z) dV 416 Usando coordenadas cil´ındricas tenemos: x = r cos θ y = y z = rsenθ ⇒Q ∗ = ¦(r, θ, y) / 0 ≤ r ≤ 3, 0 ≤ θ ≤ 2π, r ≤ y ≤ 3¦ El Jacobiano de transformaci´ on es ∂ (x, y, z) ∂ (r, θ, z) = r Reemplazando en la expresi´ on anterior se tiene I z = ___ Q ∗ (r 2 )k (r) drdθdy = k _ 3 0 _ 2π 0 _ 3 r (r 3 )dydθdr = k _ 3 0 _ 2π 0 (r 3 ) [y] 3 r dθdr = k _ 3 0 (r 3 ) [3 −r] [θ] 2π 0 dr = 2πk _ 3 0 (3r 3 −r 4 )dr = 2πk _ 3 4 r 4 − 1 5 r 5 _ 3 0 = 11 10 3 5 πk Problema 4 Calcular el volumen y el centroide de la regi´on acotada arriba por la esfera ρ = a, y abajo por el cono θ = θ 0 , donde 0 < θ 0 < π 2 , que tiene una densidad δ (x, y, z) = 1 Soluci´on. Sea la regi´ on Q = _ (x, y, z) ∈ IR 3 / 0 ≤ x 2 + y 2 ≤ a 2 2 , _ y 2 + x 2 ≤ z ≤ _ a 2 −y 2 −x 2 _ El volumen de esta regi´ on en coordenadas esf´ericas es V = _ 2π 0 _ θ 0 0 _ a 0 ρ 2 senθdρdθdφ = _ 2π 0 _ θ 0 0 1 3 a 3 senθdρdθdφ = 1 3 a 3 _ 2π 0 [−cos θ] θ 0 0 dφ = 2 3 πa 3 [1 −cos θ 0 ] Por simetr´ıa, el centroide est´ a en el eje z, luego determinemos el momento del s´olido respecto al plano xy 417 M xy = ___ Q zδ (x, y, z) dV M xy = ___ Q ∗ (ρ cos θ) ρ 2 senθdρdθdφ = _ 2π 0 _ θ 0 0 _ a 0 ρ 3 cos θsenθdρdθdφ = _ 2π 0 _ θ 0 0 1 4 a 4 cos θsenθdθdφ = 1 4 a 4 _ 2π 0 _ sen 2 θ 2 _ θ 0 0 dφ = 1 8 a 4 sen 2 θ 0 _ 2π 0 dφ = 1 4 πa 4 sen 2 θ 0 As´ı, las coordenadas del centro de masa son: x = M yz M = 0 y = M xz M = 0 z = M xy M = 3 8 a (1 + cos θ 0 ) Problema 5 Se tiene una regi´on s´ olida que se encuentra al exterior de la esfera x 2 + y 2 +z 2 = a 2 y dentro de la esfera x 2 +y 2 +z 2 = b 2 ,suponiendo que la densidad de un punto es directamente proporcional al cuadrado de la distancia de P al centro de las esferas. a) Calcule la masa del s´ olido b) Calcule su momento de inercia con respecto a una recta que pasa por el centro Soluci´on. a) La regi´ on s´ olida esta acotada por Q = ¦(x, y, z) ∈ IR 3 / a 2 ≤ x 2 + y 2 + z 2 ≤ b 2 ¦ y la densidad del punto es δ (x, y, z) = k (x 2 + y 2 + z 2 ) . La masa total del casquete esf´erico en coordenadas cartesianas es M = ___ Q δ (x, y, z) dV = ___ Q k _ x 2 + y 2 + z 2 _ dxdydz Para calcular la masa usaremos coordenadas esf´ericas: 418 Figura 4.72: Regi´on s´olida que se encuentra al exterior de la esfera x 2 +y 2 +z 2 = a 2 y dentro de la esfera x 2 + y 2 + z 2 = b 2 x = ρsenθ cos φ y = ρsenθsenφ z = ρ cos θ _ _ _ =⇒ ¸ ¸ ¸ ¸ ∂ (x, y, z) ∂ (ρ, θ, φ) ¸ ¸ ¸ ¸ = ρ 2 senθ La regi´on de integraci´ on queda: Q ∗ = ¦(ρ, θ, φ) / a ≤ ρ ≤ b, 0 ≤ θ ≤ π, 0 ≤ φ ≤ 2π¦ Al sustituir en la expresi´on anterior tenemos que la masa total es M = _ 2π 0 _ π 0 _ b a (kρ 2 )ρ 2 senθdρdθdφ = _ 2π 0 _ π 0 k _ ρ 5 5 _ b a senθdθdφ = 1 5 k _ b 5 −a 5 _ _ 2π 0 [−cos θ] π 0 dφ = 2 5 k _ b 5 −a 5 _ _ 2π 0 dφ = 4 5 πk _ b 4 −a 4 _ Por la simetr´ıa, el momento de inercia con respecto al eje z es 419 I z = ___ Q _ x 2 + y 2 _ δ (x, y, z) dV = _ 2π 0 _ π 0 _ b a _ ρ 2 sen 2 θ _ (kρ 2 )ρ 2 senθdρdθdφ = _ 2π 0 _ π 0 _ b a kρ 6 sen 3 θdρdθdφ = _ 2π 0 _ π 0 k _ ρ 7 7 _ b a sen 3 θdθdφ = 1 7 k _ b 7 −a 7 _ _ 2π 0 _ π 0 senθ _ 1 −cos 2 θ _ dθdφ = 1 7 k _ b 7 −a 7 _ _ 2π 0 _ −cos θ + 1 3 cos 3 θ _ π 0 dφ = 4 21 k _ b 7 −a 7 _ _ 2π 0 dφ = 8π 21 k(b 7 −a 7 ) Problema 6 Calcular el momento de inercia con respecto al eje z del s´olido acotado entre las esferas x 2 + y 2 + z 2 ≥ a 2 , x 2 + y 2 + z 2 ≤ 9a 2 . Si la densidad en cada punto de la regi´ on es δ (x, y, z) = x 2 + y 2 + z 2 x 2 + y 2 . Figura 4.73: S´olido acotado entre las esferas x 2 +y 2 +z 2 ≥ a 2 , x 2 +y 2 +z 2 ≤ 9a 2 . Soluci´on Por la simetr´ıa, el momento de inercia con respecto al eje z es 420 I z = ___ Q _ x 2 + y 2 _ δ (x, y, z) dV = ___ Q _ x 2 + y 2 _ x 2 + y 2 + z 2 x 2 + y 2 dV = = ___ Q _ x 2 + y 2 + z 2 _ dxdydz donde la regi´ on s´olida est´ a acotada por Q = ¦(x, y, z) ∈ IR 3 / a 2 ≤ x 2 + y 2 + z 2 ≤ 9a 2 ¦. Para calcular la ´ ultima integral usaremos coordenadas esf´ericas: x = ρsenθ cos φ y = ρsenθsenφ z = ρ cos θ _ _ _ =⇒ ¸ ¸ ¸ ¸ ∂ (x, y, z) ∂ (ρ, θ, φ) ¸ ¸ ¸ ¸ = ρ 2 senθ La regi´on de integraci´ on queda: Q ∗ = ¦(ρ, θ, φ) / a ≤ ρ ≤ 3a, 0 ≤ θ ≤ π, 0 ≤ φ ≤ 2π¦ Al sustituir en la expresi´on anterior tenemos que I z queda: I z = _ 2π 0 _ π 0 _ 3a a (ρ 2 )(ρ 2 senθ)dρdθdφ = _ 2π 0 _ π 0 _ 3a a ρ 4 senθdρdθdφ = _ 2π 0 _ π 0 _ ρ 5 5 _ 3a a senθdθdφ = 1 5 _ (3a) 5 −a 5 _ _ 2π 0 _ π 0 senθdθdφ = 8 5 _ a 5 _ _ 2π 0 [−cos θ] π 0 dφ = 16 5 _ a 5 _ _ 2π 0 dφ = 32 5 πa 5 Problema 7 Encontrar el centro de masa del s´ olido homog´eneo , cuya densidad es δ (x, y, z) = 1, que est´ a acotado por arriba por z 2 = 64 + x 2 + y 2 y por abajo z 2 = 2( x 2 + y 2 ), z ≥ 0. 421 Figura 4.74: Soluci´on. Como la densidad es constante δ (x, y, z) = 1,y de acuerdo a la simetr´ıa ,que presenta el problema las coordenadas del centro de masa son x = M yz M = 0 y = M xz M = 0 z = M xy M = ___ Q zdV V (Q) Para representar el problema, usaremos coordenadas cil´ındricas. x = r cos θ y = rsenθ z = z _ _ _ =⇒ ¸ ¸ ¸ ¸ ∂ (x, y, x) ∂ (r, θ, z) ¸ ¸ ¸ ¸ = r es el Jacobiano de transforma- ci´ on. Primero determinemos la funci´ on intersecci´ on de las superficies: z 2 = 64 + x 2 + y 2 z 2 = 2(x 2 + y 2 ) _ =⇒ x 2 + y 2 = 64 Pero en cil´ındricas describen: x 2 + y 2 = r 2 =⇒ 0 ≤ r 2 ≤ 64 =⇒ 0 ≤ r ≤ 8 Por otra parte 0 ≤ θ ≤ 2π Ahora determinemos las cotas para z z 2 = 64 + x 2 + y 2 = 64 −r 2 =⇒ z 2 = 2(x 2 + y 2 ) = 2r 2 =⇒ z = √ 64 + r 2 z = √ 2r ,con z ≥ 0 Luego la regi´ on de integraci´ on queda: Q ∗ = _ (r, θ, z) / 0 ≤ r ≤ 8, 0 ≤ θ ≤ 2π, √ 2r ≤ z ≤ √ 64 + r 2 _ Calculemos el volumen de la regi´on: 422 V (Q ∗ ) = ___ Q ∗ rdrdθdz = _ 2π 0 _ 8 0 _ √ 64+r 2 √ 2r r dzdrdθ = _ 2π 0 _ 8 0 [z] √ 64+r 2 √ 2r rdrdθ = _ 2π 0 _ 8 0 _ √ 64 + r 2 − √ 2r _ rdrdθ = _ 2π 0 __ 8 0 r √ 64 + r 2 dr − √ 2 _ 8 0 r 2 dr _ dθ = _ 2π 0 _ 1 2 _ _ 2 3 (64 + r 2 ) 3/2 _ 8 0 − √ 2 _ r 3 3 _ 8 0 _ dθ _ = 2π _ 2 21/2 3 − 2 18/2 3 − √ 2 8 3 3 _ ∴ V (Q ∗ ) = 2 3 π _ 2 9 _ 2 3/2 −1 _ − √ 22 9 _ Por otro lado, el momento de inercia es M = ___ Q δ (x, y, z) zdV = ___ Q ∗ z rdrdθdz = _ 2π 0 _ 8 0 _ √ 64+r 2 √ 2r r zdzdθdr = _ 2π 0 _ 8 0 r _ z 2 2 _ √ 64+r 2 √ 2r drdθ = 1 2 _ 2π 0 _ 8 0 _ 64 + r 2 −2r 2 ¸ rdrdθ = 1 2 _ 2π 0 _ 8 0 _ 64r −r 3 ¸ drdθ = 1 2 _ 2π 0 _ 32r 2 − r 4 4 _ 8 0 dθ = 1 2 2π _ 8 2 (32 −16) ¸ = 1024π Por tanto, la tercera coordenada del centro de masa del s´ olido es: 423 z = ___ Q zdV V (Q) = 1024π 1024π _√ 2 −1 _ 3 = 3 _ √ 2 + 1 _ 4.6.7. Determinaci´ on del centroide dee un s´olido Problema 1 Encontrar el centroide del s´ olido encerrado por el cilindro x 2 + y 2 = 4 , acotado por arriba por el paraboloide z = x 2 + y 2 , y por abajo por el plano XY. Para efectos de c´ alculo suponga que la densidad es δ (x, y, z) = 1 constante. Figura 4.75: S´olido encerrado por el cilindro x 2 +y 2 = 4 , acotado por arriba por el paraboloide z = x 2 + y 2 , y por abajo por el plano XY Soluci´on El centroide del s´ olido (x, y, z) ,est´ a en su eje de simetr´ıa,en este caso el eje z,. Esto hace que x = y = 0. Para hallar z, dividimos el primer momento M xy por la masa M del s´ olido, es decir z = M xy M Donde el primer momento se define M xy = ___ Ω zδ (x, y, z) dxdydz y la masa del s´ olido es M = ___ Ω δ (x, y, z) dxdydz siendo Ω = ¦(x, y, z) ∈ IR 3 / x 2 + y 2 ≤ 4, 0 ≤ z ≤ x 2 + y 2 ¦ 424 Por la simetr´ıa del dominio, es necesario hacer un cambio de variables a coordenadas cil´ındricas x = r cos θ y = rsenθ z = z _ _ _ =⇒el jacobiano de transformaci´ on es ∂ (x, y, z) ∂ (r, θ, z) = ¸ ¸ ¸ ¸ ¸ ¸ cos θ −rsenθ 0 senθ r cos θ 0 0 0 1 ¸ ¸ ¸ ¸ ¸ ¸ = r Entonces la regi´on Ω se tranforma en Ω ∗ = ¦(r, θ, z) / 0 ≤ r ≤ 2, 0 ≤ θ ≤ 2π, 0 ≤ z ≤ r 2 ¦ El valor de M xy es: M xy = ___ Ω zδ (x, y, z) dxdydz = ___ Ω ∗ zrdrdθdz = _ 2π 0 _ 2 0 _ r 2 0 zrdzdrdθ = _ 2π 0 _ 2 0 z 2 2 ¸ ¸ ¸ ¸ r 2 0 rdrdθ = _ 2π 0 _ 2 0 r 5 2 drdθ = _ 2π 0 r 6 12 ¸ ¸ ¸ ¸ 2 0 dθ = _ 2π 0 16 3 dθ = 32 3 π El valor de la masa M es M = ___ Ω δ (x, y, z) dxdydz = ___ Ω ∗ rdrdθdz = _ 2π 0 _ 2 0 _ r 2 0 rdzdrdθ = _ 2π 0 _ 2 0 z[ r 2 0 rdrdθ = _ 2π 0 _ 2 0 r 3 drdθ = _ 2π 0 r 4 4 ¸ ¸ ¸ ¸ 2 0 dθ = _ 2π 0 4dθ = 8π Por lo tanto z = M xy M = 4 3 425 En consecuencia, el centroide se localiza en _ 0, 0, 4 3 _ . Problema 2 Calcular el momento de inercia del s´ olido con respecto al eje z , que est´ a acotado por arriba por la esfera s´ olida x 2 + y 2 + z 2 = a 2 y por abajo por el cono θ = π 3 ,y cuya densidad es constante δ = 1 Soluci´on: En coordenadas rectangulares, el momento de inercia con respecto al eje z es I z = ___ Q _ x 2 + y 2 _ δ (x, y, z) dV En coordenadas esf´ericas, queda I z = _ 2π 0 _ π/3 0 _ 1 0 _ ρ 2 sen 2 θ _ ρ 2 senθdρdθdφ _ 2π 0 _ π/3 0 _ 1 0 ρ 4 sen 3 θdρdθdφ = _ 2π 0 _ π/3 0 _ ρ 5 5 _ 1 0 sen 3 θdθdφ 1 5 _ 2π 0 _ π/3 0 senθ _ 1 −cos 2 θ _ dθdφ = 1 5 _ 2π 0 _ −cos θ + 1 3 cos 3 θ _ π/3 0 dφ = 1 5 _ 2π 0 5 24 dφ = 1 24 (2π) I z = π 12 4.7. Autoevaluaci´ on Integrales dobles y triples Tiempo 2 horas Pregunta 1 a) Dada la integral _ 2 0 _ 4−x 2 0 xe 2y 4 −y dydx, dibuje la regi´ on de integraci´ on y eval´ ue la integral. b) Calcular el volumen del s´olido limitado por las superficies x 2 + y 2 + z 2 = 1, x 2 + y 2 + z 2 = 4, z 2 = x 2 + y 2 , con z ≥ 0 Pregunta 2 426 Calcular _ _ _ R (x 2 +y 2 )dxdydz, siendo R un cono recto de revoluci´ on de altura h, base situada en el plano XY, eje de simetr´ıa en z, dado por la ecuaci´on a 2 (h −z) 2 = h 2 (x 2 + y 2 ). Pregunta 3 Calcular la integral I = _ _ _ R xyzdxdydz, en la regi´on del primer octante limitada por los planos coordenados y el plano x + y + z = 1. Pauta de Correcci´ on Pregunta 1 a) Por la forma del integrando es conveniente cambiar el orden de inte- graci´ on, en efecto _ 2 0 _ 4−x 2 0 xe 2y 4 −y dydx = _ 4 0 _ √ 4−y 0 xe 2y 4 −y dxdy = _ 4 0 _ x 2 e 2y 4 (4 −y) _ √ 4−y 0 dy = _ 4 0 e 2y 2 dy = _ e 2y 4 _ 4 0 = e 8 −1 4 b) El volumen del s´ olido est´ a dado por V (D) = _ _ _ R dxdydz Usando coordenadas esf´ericas, tenemos x = r sin θ cos φ y = r sin θ sin φ z = r cos θ que tiene por Jacobiano de la transformaci´ on ¸ ¸ ¸ ¸ ∂ (x, y, z) ∂ (r, θ, φ) ¸ ¸ ¸ ¸ = r 2 sin θ. Entonces ´esta regi´on R corresponde en el espacio rθφ a la regi´on R ∗ dada por R ∗ = _ (r, θ, φ) /1 ≤ r ≤ 2; 0 ≤ θ ≤ π 4 ; 0 ≤ φ ≤ 2π _ . 427 Des esta manera V (D) = _ 2π 0 _ π 4 0 _ 2 1 r sin θdrdθdφ = _ 2π 0 _ π 4 0 _ r 3 3 _ 2 1 sin θdθdφ = 7 3 _ 2π 0 [−cos θ] π 4 0 dφ = 7 3 _ 1 − √ 2 2 _ 2π = 14 3 _ 1 − √ 2 2 _ π Pregunta 2 Queremos calcular la integral la integral de la funci´on f (x, y) = y 2 + z 2 sobre la regi´on R limitada por el cono a 2 (h −z) 2 = h 2 (x 2 + y 2 ). Este es un caso t´ıpico que por la simetr´ıa de la regi´ on y del integrando, bajo cualquier punto de vista conviene hacer el cambio a coordenadas cil´ındricas para evaluar la integral. Sea x = ρ cos θ y = ρ sin θ z = z que tiene por Jacobiano de la transformaci´ on ¸ ¸ ¸ ¸ ∂ (x, y, z) ∂ (ρ, θ, z) ¸ ¸ ¸ ¸ = ρ . Obs´ervese que la regi´ on de integraci´ on R se describe, proyectando sobre el plano XY z = 0 =⇒ (x 2 + y 2 ) ≤ a 2 =⇒ 0 ≤ ρ ≤ a y 0 ≤ θ ≤ 2π. De modo que la proyeci´on de la regi´ on de integraci´on R ∗ se ve en el plano (ρ, θ) como el rect´ angulo 0 ≤ ρ ≤ a y 0 ≤ θ ≤ 2π. Por otra parte, sustituyendo en a 2 (h −z) 2 ≤ h 2 (x 2 + y 2 ) =⇒ 0 ≤ z ≤ h a (a −ρ) . De esta manera, la regi´on de integraci´ on en el nuevo 428 sistema queda como R ∗ = _ (ρ, θ, z) /0 ≤ ρ ≤ a ; 0 ≤ θ ≤ 2π; 0 ≤ z ≤ h a (a −ρ) _ La funci´on a integrar f (y, z) = x 2 + y 2 se reduce a f (ρ sin θ, z) = ρ 2 .Se tiene entonces que _ _ _ R (x 2 + y 2 )dxdydz = _ _ _ R ∗ _ ρ 2 _ (ρ) dρdθdz = _ a 0 _ 2π 0 _ h a (a−ρ) 0 ρ 3 dzdθdρ = _ a 0 _ 2π 0 ρ 3 h a (a −ρ) dθdρ = 2πh a _ a 0 _ aρ 3 −ρ 4 _ dρ = 2πh a _ a ρ 4 4 − ρ 5 5 _ a 0 = πh 10 a 4 Pregunta 3. El plano corta a los ejes coordenados en los puntos (1, 0, 0) , (0, 1, 0) y (0, 0, 1) . Si se proyecta la regi´ on sobre el plano XY, se tiene z = 0 =⇒ x+y = 1, luego R xy = ¦(x, y) /0 ≤ x ≤ 1, 0 ≤ y ≤ 1 −x¦ . Por consiguiente, una partici´ on de la regi´ on est´a dada por R = ¦(x, y, z) /0 ≤ x ≤ 1, 0 ≤ y ≤ 1 −x, 0 ≤ x ≤ 1 −x −y¦ 429 Luego la integral se puede expresar: _ _ _ R xyzdxdydz = _ 1 0 _ 1−x 0 _ 1−x−y 0 xyzdzdydx = _ 1 0 _ 1−x 0 xy 1 2 (1 −x −y) 2 dydx = _ 1 0 _ 1−x 0 xy 1 2 _ y(1 −x) 2 −2(1 −x)y 2 + y 3 ¸ dydx = 1 2 _ 1 0 x _ (1 −x) 4 2 −2 (1 −x) 4 3 + (1 −x) 4 4 _ dx = 1 2 _ 1 0 x(1 −x) 4 dx = 1 720 Autoevaluaci´ on N o 2 Tiempo 2 horas Pregunta 1 Encontrar el volumen del s´ olido, en el primer octante, acotado por los planos coordenados, el cil´ındro x 2 + y 2 = 4 y el plano z + y = 3. Pregunta 2. Calcular _ _ _ R 1 [ 1+(x 2 +y 2 +z 2 ) 3/2 ] 3/2 dxdydz, siendo R la esfera x 2 + y 2 + z 2 = a 2 . Pregunta 3. Utilice la sustituci´ on u = y x , v = y + x 2 para evaluar la integral _ _ R xy y + 2x 2 x 2 + xy dydx en donde R xy es la regi´ on del primer cuadrante acotada por las curvas y = 3 −x 2 , y = 8 −x 2 , e y = 2x. Nota: Usar la propiedad ∂ (x, y) ∂ (u, v) = _ ∂ (u, v) ∂ (x, y) _ −1 en un dominio en que ambos Jacobianos son distintos de cero. Pauta de Correcci´ on. 430 Pregunta 1 El volumen de la regi´ on R est´a dado por V = _ _ R f(x, y)dxdy En nuestro caso f(x, y) = z = 3−y queda definida en la regi´on del primer octante dada por R = _ (x, y) /0 ≤ x ≤ 2, 0 ≤ y ≤ √ 4 −x 2 _ . Entonces, al sustituir t´erminos en el integrando, obtenemos V = _ 2 0 _ √ 4−x 2 0 (3 −y) dydx = _ 2 0 _ 3y − y 2 2 _ √ 4−x 2 0 dx = _ 2 0 _ 3 √ 4 −x 2 − _ 4 −x 2 2 __ dx = _ 3 2 √ 4 −x 2 + 6arcsen _ x 2 _ −2x + x 3 6 _ 2 0 = 6 _ π 2 _ −4 + 8 6 = 9π −8 3 Pregunta 2. Calculemos la integral triple de la funci´on f (x, y, z) = 1 [ 1+(x 2 +y 2 +z 2 ) 3/2 ] 3/2 sobre la regi´ on R limitada por la esfera x 2 + y 2 + z 2 = a 2 . Aunque el problema se puede plantear en coordenadas directamente en coordenadas cartesianas , por la forma de la regi´ on y del integrando, ser´ a dific´ıl resolverlo directamente. En consecuencia, lo resolveremos en coordenadas esf´ericas. Sea x = r sin θ cos φ y = r sin θ sin φ z = r cos θ que tiene por Jacobiano de la transformaci´ on ¸ ¸ ¸ ¸ ∂ (x, y, z) ∂ (r, θ, φ) ¸ ¸ ¸ ¸ = r 2 sin θ. Entonces esta regi´on R corresponde en el espacio rθφ a la regi´ on R ∗ dada por R ∗ = ¦(r, θ, φ) /0 ≤ r ≤ a; 0 ≤ θ ≤ π; 0 ≤ φ ≤ 2π¦ El integrando f (x, y, z) = 1 [ 1+(x 2 +y 2 +z 2 ) 3/2 ] 3/2 en las nuevas coordenadas 431 queda f (r sin θ cos φ, r sin θ sin φ, r cos θ) = 1 (1 + r 3 ) 3/2 ,de modo que la integral por calcular toma la forma: _ _ _ R 1 _ 1 + (x 2 + y 2 + z 2 ) 3/2 _ 3/2 dxdydz = _ _ _ R ∗ (r 2 sin θ) [1 + r 3 ] 3/2 drdθdφ _ 2π 0 _ π 0 _ a 0 r 2 sin θ [1 + r 3 ] 3/2 drdθdφ = _ 2π 0 _ π 0 −2 (1 + r 3 ) 1/2 ¸ ¸ ¸ ¸ ¸ a 0 sin θdθdφ 2 _ 1 − 1 (1 + a) 1/2 _ _ 2π 0 −cos θ[ π 0 dφ = 4 _ 1 − 1 (1 + a) 1/2 _ φ[ 2π 0 = 8π _ 1 − 1 (1 + a) 1/2 _ Pregunta 3. Vamos a evaluar la integral _ _ R xy y + 2x 2 x 2 + xy dydx, en la regi´ on R xy del primer cuadrante acotada por las curvas y = 3 −x 2 , y = 8 −x 2 , e y = 2x. Usando el cambio de variables u = y x , v = y + x 2 , la regi´ on R xy se transforma en la regi´ on del primer cuadrante acotada por las curvas: y + x 2 = 3 =⇒ v = 3 y + x 2 = 8 =⇒ x = 8, y x = 2 =⇒ u = 2; y x = 0 =⇒ u = 0 Luego, R uv = ¦(u, v) /0 ≤ u ≤ 2, 3 ≤ v ≤ 8¦ . Calculemos ahora el Jacobiano de la transformaci´ on usando la propiedad ∂ (x, y) ∂ (u, v) = _ ∂ (u, v) ∂ (x, y) _ −1 = _¸ ¸ ¸ ¸ − y x 2 1 x 2x 1 ¸ ¸ ¸ ¸ _ −1 =⇒ ¸ ¸ ¸ ¸ ∂ (x, y) ∂ (u, v) ¸ ¸ ¸ ¸ = x 2 y + 2x 2 Sustituyendo t´erminos en la integral, obtenemos _ _ R xy y + 2x 2 x 2 + xy dydx = _ _ R uv 1 1 + u dudv = _ 2 0 _ 8 3 1 1 + u dudv = _ 2 0 5 1 + u du = 5 ln [1 + u[ 2 0 = 5 ln 3 432 Autoevaluaci´ on N o 3 Tiempo 2 horas Pregunta 1 Considere la integral I = _ 1 0 _ 1 y e −x 2 dxdy, dibuje la regi´on de integraci´ on, cambie el orden de integraci´ on y calc´ ulela. Pregunta 2 Utilice la sustituci´ on u = x+y, y = x−y para evaluar I = _ _ R e − ( x−y x+y ) dA, donde R es la regi´ on tri´angular acotada por las rectas x = 0, y = 0 y x = 1 −y. Pregunta 3 En la esfera x 2 + y 2 + z 2 = a 2 los planos y = x , y = √ 3x determinan un sector esf´erico D, con x ≥ 0, y ≥ 0, z ≥ 0. Calcular el volumen de D Pauta Autoevaluaci´on Pregunta 1 La integral I = _ 1 0 _ 1 y e −x 2 dxdy est´ a definida sobre la regi´ on de tipo II dada por R II = ¦(x, y) ∈ R 2 /y ≤ x ≤ 1, 0 ≤ y ≤ 1¦ . Invirtiendo la partici´on de la regi´ on a una de tipo I, tenemos: R I = _ (x, y) ∈ R 2 /0 ≤ x ≤ 1, 0 ≤ y ≤ x _ entonces _ 1 0 _ 1 y e −x 2 dxdy = _ 1 0 _ x 0 e −x 2 dydx _ 1 0 _ ye −x 2 _ x 0 dx = − 1 2 _ e −x 2 _ 1 0 = 1 2 _ 1 − 1 e _ Pregunta 2 Usando la sustituci´ on u = x + y v = x −y _ (1)resolviendo x, y en t´erminos de u, v produce 433 x = u+v 2 y = u−v 2 _ (2) determinemos como tranforma la regi´on tri´ angular R = _ (x, y) ∈ R 2 /0 ≤ x ≤ 1, 0 ≤ y ≤ 1 −x _ Reemplazando valores en (2) x + y = 1 x = 0 y = 0 _ _ _ =⇒ u = 1 u + v = 0 v = u _ _ _ Luego, la proyecci´ on de la regi´on en el plano (u, v) es R ∗ = ¦(u, v) ∈ R 2 /0 ≤ u ≤ 1, −u ≤ v ≤ u¦ Calculemos el Jacobiano de la transformaci´on, nos queda ¸ ¸ ¸ ¸ ∂ (x, y) ∂ (u, v) ¸ ¸ ¸ ¸ = ¸ ¸ ¸ ¸ 1 2 1 2 1 2 − 1 2 ¸ ¸ ¸ ¸ = ¸ ¸ − 1 2 ¸ ¸ = 1 2 La funci´on a integrar es, en t´erminos de las nuevas variables _ _ R e − ( x−y x+y ) dA = _ _ R ∗ e − ( v u ) _ 1 2 _ dudv _ 1 0 _ u −u e − ( v u ) _ 1 2 _ dudv = 1 2 _ 1 0 _ −ue − ( v u ) _ u −u du (e −1 −e) 2 _ 1 0 −udu = (e −e −1 ) 2 _ u 2 2 _ 1 0 = (e −e −1 ) 4 Pregunta 3 El volumen de la regi´ on R est´a dado por V (D) = _ _ _ R dxdydz Usando coordenadas esf´ericas, tenemos x = r sin θ cos φ y = r sin θ sin φ z = r cos θ que tiene por Jacobiano de la transformaci´ on ¸ ¸ ¸ ¸ ∂ (x, y, z) ∂ (r, θ, φ) ¸ ¸ ¸ ¸ = r 2 sin θ. Entonces ´esta regi´ on R corresponde en el espacio rθφ a la regi´ on R ∗ 434 dada por R ∗ = _ (r, θ, φ) /0 ≤ r ≤ a; 0 ≤ θ ≤ π 2 ; π 4 ≤ φ ≤ π 3 _ . De ´esta manera V (D) = _ π 3 π 4 _ π 2 0 _ a 0 r 2 sin θdrdθdφ = _ π 3 π 4 _ π 2 0 a 3 3 sin θdθdφ = a 3 3 _ π 3 π 4 dφ = a 3 3 _ π 3 − π 4 _ = a 3 36 π 435 Cap´ıtulo 5 Integral de Linea Es una forma de generalizar la integral de Riemann en una variable, conocida en el curso anterior, a una integral definida sobre una curva del espacio bi o tridimensional. Planteamiento Supongamos que tenemos: a) f : U ⊂ IR 3 →IR funci´ on escalar continua en U y b) − → c : [a, b] → IR 3 , − → c (t) = (x(t), y(t), z(t)) una trayectoria de clase C 1 , es decir una curva en IR 3 . La integral de la funci´on f calculada sobre la trayectoria descrita por la curva C se define de la forma dada a continuaci´ on. La integral de trayectoria de f(x, y, z) a lo largo de la trayectoria − → c , est´ a definida cuando − → c : [a, b] → IR 3 es de clase C 1 y adem´as la funci´ on compuesta f(t) = f(x(t), y(t), z(t)) es continua en I = [a, b]; por: _ −→ c f ds = _ b a f(x(t), y(t), z(t)) | − → c (t)| dt Alternativamente se denota como: _ −→ c fds = _ −→ c f(x, y, z)ds = _ b a f( − → c (t)) | − → c (t)| dt Observaciones i) Si f(x, y, z) = 1, tenemos la longitud del arco. 436 En efecto, como f(x, y, z) = 1, _ −→ c f(x, y, z) ds = _ −→ c ds = _ b a | − → c (t)| dt = _ b a _ (x (t)) 2 + (y (t)) 2 + (z (t)) 2 dt ii) Si C la curva definida por − → c (t) es de clase C 1 por tramos o f( − → c (t)) es continua por tramos, entonces calculamos _ −→ c f ds en segmentos sobre los cuales _ b a f( − → c (t)) | − → c (t)| dt es continua y sumamos las integrales resultantes. Por ejemplo si − → c = − → c 1 ∪ − → c 2 ∪ − → c 3 con a lo sumo sus extremos en com´ un _ −→ c f(x, y, z)ds = _ −→ c 1 f(x, y, z)ds + _ −→ c 2 f(x, y, z)ds + _ −→ c 2 f(x, y, z)ds Algunos ejemplos donde aplicamos esta definici´on; son los siguientes. Ejemplo 1 Sea la funci´ on escalar f(x, y, z) = x 2 + y 2 + z 2 continua en R y − → c : [0, 2π] →IR 3 , definida por − → c (t) = (cos t, sin t, t) una trayectoria de clase C 1 . Calcular _ −→ c f(x, y, z)ds Soluci´ on: En este caso tenemos la ecuaciones param´etricas : x(t) = cos t, y(t) = sin t, z(t) = t Luego la funci´ on compuesta es f(x(t), y(t), z(t)) = cos 2 t + sin 2 t + t 2 = 1 + t 2 Adem´ as − → c (t) = (−sin t, cos t, 1) =⇒| − → c (t)| = √ 2 Por lo tanto, sutituyendo t´erminos en la integral, queda _ −→ c f(x, y, z)ds = _ 2π 0 f(x(t), y(t), z(t)) | − → c (t)| dt = _ 2π 0 _ 1 + t 2 _ √ 2dt √ 2 _ t + t 3 3 _ 2π 0 = 2 √ 2π 3 _ 3 + 4π 2 _ Ejemplo 2 437 Calcular la siguiente integral de trayectoria _ −→ c f(x, y, z)ds donde a) f(x, y, z) = x + y + z y − → c (t) = (cos t, sin t, t), t ∈ [0, 2π] Soluci´on: En este caso se tiene que x(t) = cos t, y(t) = sin t, z(t) = t Que produce la funci´ on compuesta f( − → c (t)) = cos t + sent + t, Adem´ as − → c (t) = (−sin t, cos t, 1) y | − → c (t)| = √ 2. Entonces _ c f(x, y, z)ds = _ 2π 0 √ 2(cos t + sent + t)dt = √ 2 _ sent −cos t + t 2 2 _ 2π 0 =⇒ _ c f(x, y, z)ds = 4 √ 2 2 π 2 Como taller adicional, evaluar las siguientes integrales de trayectoria _ −→ c f(x, y, z)ds donde: b) f(x, y, z) = cos z y − → c (t) = (cos t, sin t, t), t ∈ [0, 2π] c) f(x, y, z) = x cos z y − → c (t) = (t, t 2 , 0), t ∈ [0, 1] Ejemplo 3 a) Demostrar que la integral de l´ınea de f(x, y), a lo largo de una trayec- toria dada en coordenadas polares por r = r(θ), con θ 1 ≤ θ ≤ θ 2 es: I = _ θ 2 θ 1 f (r cos θ, r sin θ) ¸ r 2 + _ dr dθ _ 2 dθ b) Calcular la longitud de arco de r = 1 + cos θ, con 0 ≤ θ ≤ 2π Soluci´on: a) En primer lugar, determinemos la ecuaciones param´etricas de la trayectoria en coordenadas polares x = r cos θ =⇒ x = r(θ) cos θ y = r sin θ =⇒ y = r (θ) sin θ =⇒ − → c (θ) = (r(θ) cos θ, r (θ) sin θ) Derivando la ecuaci´ on de la trayectoria, queda 438 − → c (θ) = (r (θ) cos θ −r (θ) sin θ, r (θ) sin θ + r(θ) cos θ) Lo que implica | − → c (θ)| = _ (r (θ) cos θ −r (θ) sin θ) 2 + (r (θ) sin θ + r(θ) cos θ) 2 = _ r (θ) 2 + r (θ) 2 Por tanto I = _ −→ c f(x, y) ds = _ θ 2 θ 1 f (r cos θ, r sin θ) _ r (θ) 2 + r (θ) 2 dθ b) En este caso se tiene f(x, y) = 1, y como r = 1 + cos θ, con 0 ≤ θ ≤ 2π Obtenemos _ r (θ) 2 + r (θ) 2 = _ (1 + cos θ) 2 + (−sin θ) 2 Entonces: s = _ 2π 0 _ (1 + cos θ) 2 + (−sin θ) 2 dθ = _ 2π 0 √ 2 + 2 cos θdθ = 2 _ π 0 [ cos θ 2 [dθ = 2 __ π 0 cos θ 2 dθ + _ π π _ −cos θ 2 _ dθ _ = 8 Es ´ util tener en cuenta que la integral de trayectoria se puede aplicar a problemas de f´ısica e ingenier´ıa 1. Si f(x, y, z) representa la masa por unidad de longitud de un alambre delgado con la forma de C, entonces _ −→ c fds es la masa total del alambre. 2. Si f(x, y, z) es la componente tangencial de una fuerza en (x, y, z) de C, entonces _ −→ c fds representa el trabajo realizado por la fuerza al mover una part´ıcula a lo largo de la curva C. 3.- Si f : I ⊂ IR 2 → IR funci´ on escalar continua en I = [a, b] y f(x, y) ≥ 0,entonces _ −→ c f(x, y)ds = _ b a f(x(t), y(t)) | − → c (t)| dt representa el ´ area de una pared cuya base es la imagen de − → c y la altura f(x, y). 439 Figura 5.1: Pared zigzagueante Ejemplo 4 a) Calcular anal´ıticamente la masa M de un resorte que tiene la forma de una h´elice dada por la ecuaci´on: − → r (t) = (5cost; 5sent; 4t); t ∈ [0; 2π] si la densidad en cada punto (x, y, z) es el cuadrado de la distancia en cada punto al origen de coordenadas. Figura 5.2: H´elice circular b) Si el centro de masa de un resorte con forma de alambre delgado de masa M, se define por (X M ; Y M ; Z M ) con 440 X M = 1 M _ −→ r xδ(x, y, z)ds Y M = 1 M _ −→ r yδ(x, y, z)ds Z M = 1 M _ −→ r zδ(x, y, z)ds Calcular la componente Z M del resorte helicoidal Soluci´on. a) La f´ ormula para el c´ alculo de la masa del resorte es: M = _ −→ r δ(x, y, z)ds = _ b a δ( − → r (t)) | − → r (t)| dt donde se tiene que: − → r (t) = (5cost; 5sent; 4t); t ∈ [0; 2π] =⇒ − → r (t) = (−5 sin t; 5 cos t; 4) =⇒ | − → r (t)| = √ 25 sin 2 t + 25 cos 2 t + 16 = √ 41 y la densidad δ(x; y; z) = x 2 + y 2 + z 2 =⇒ δ ( − → r (t)) = (25 + 16t 2 ) Reemplazando en el integrando, queda M = _ −→ r δ(x, y, z)ds = _ 2π 0 (25 + 16t 2 ) √ 41dt = √ 41 _ 25t + 16 3 t 3 _ 2π 0 = √ 41 _ 50π + 128 3 π 3 _ b) La componente Z M del centro de masa es Z M = 1 M _ −→ r zδ(x, y, z)ds = 1 M _ b a z(t)δ( − → r (t)) | − → r (t)| dt = 1 M _ 2π 0 t(25 + 16t 2 ) √ 41dt = √ 41 M _ 25 2 t 2 + 4t 4 _ 2π 0 = √ 41 M _ 50π 2 + 64π 4 ¸ = π [50 + 64π 2 ] _ 50 + 128 3 π 2 _ 5.1. Campos vectoriales Sea − → F = (F 1 , F 2 , F 3 ) un campo vectorial en R 3 continuo definida sobre la trayectoria − → c : [a, b] →R 3 con − → c perteneciente a C 1 . 441 Definimos la integral de l´ınea de − → F a lo largo de − → c por _ −→ c − → F d − → s = _ b a − → F ( − → c (t)) − → c (t)dt Esta definici´on se puede generalizar para campos vectoriales en R n Figura 5.3: Campo vectorial radial Figura 5.4: Campo circular tangencial Otras observaciones. 1) _ −→ c − → F d − → s tambi´en se puede definir si − → F ( − → c (t)) − → c (t) es continua por tramos, expresando en este caso la integral como una suma de integrales del tipo anterior. 2) Si C es trayectoria tal que − → c (t) ,= − → 0 , podemos usar el vector tangente unitario ´ T para calcular la integral de l´ınea como sigue. 442 Figura 5.5: Flujo de un fluido en un tubo cil´ındrico largo ´ T = − → c (t) | − → c (t)| _ −→ c − → F d − → s = _ b a _ − → F ( − → c (t)) − → c (t) | − → c (t)| _ | − → c (t)| dt = _ b a _ F( − → c (t)) ´ T _ | − → c (t)| dt = _ −→ c − → F ´ Tds Por lo tanto _ −→ c − → F d − → s = _ −→ c − → F ´ Tds Si ϕ es el ´ angulo entre − → F y ´ T − → F ´ T = _ _ _ − → F _ _ _ _ _ _ − → T _ _ _ cos ϕ = _ _ _ − → F _ _ _ cos ϕ =⇒ _ −→ c − → F d − → s = _ −→ c _ _ _ − → F _ _ _ cos ϕds Si − → F es un campo de fuerza , entonces esta ´ ultima integral es precisa- mente la definici´on de trabajo realizado por el campo de fuerza sobre una part´ıcula de masa unitaria que se mueve desde el punto inicial hasta un punto terminal a lo largo de la curva C descrita por la trayectoria − → c (t). 3) Otra manera usual de escribir la integral de l´ınea es: _ −→ c − → F d − → s = _ −→ c F 1 dx + F 2 dy + F 3 dz donde − → F = (F 1 , F 2 , F 3 ) 443 As´ı: _ −→ c − → F d − → s = _ −→ c F 1 dx + F 2 dy + F 3 dz = _ b a _ F 1 dx dt + F 2 dy dt + F 3 dz dt _ dt = _ b a (F 1 , F 2 , F 3 ) ( dx dt , dy dt , dz dt )dt = _ b a − → F ( − → c (t)) − → c (t) dt Ejemplo 1: Evaluar I = _ c x 2 ydx + (x 2 −y 2 ) dy , si C es el arco de la par´ abola y = 3x 2 desde (0, 0) hasta (1, 3) . Figura 5.6: Gr´afico de la curva y = 3x 2 Soluci´on: Al parametrizar la trayectoria se tiene C : x = t, y = 3t 2 con 0 ≤ t ≤ 1. luego − → c (t) = (t, 3t 2 ) =⇒ − → c (t) = (1, 6t) Sustituyendo t´erminos en el integrando, obtenemos I = _ 1 0 _ (3t 4 , _ t 2 −9t 4 _ ) (1, 6t) ¸ dt = _ 1 0 _ 3t 4 + _ t 2 −9t 4 _ 6t ¸ dt = _ 3 5 t 5 + 6 4 t 4 − 54 6 t 6 _ 1 0 = − 69 10 444 Ejemplo 2 Evaluar _ c yzdx + xzdy + xydz donde C est´ a formada por los segmentos de rectas que unen A(1, 0, 0) a B(0, 1, 0) a C (0, 0, 1) Soluci´on. En este caso − → C = −→ AB + −−→ BC + −→ CA por lo cual _ −→ c yzdx + xzdy + xydz = _ −→ AB yzdx + xzdy + xydz + _ −−→ BC yzdx + xzdy + xydz + _ −→ CA yzdx + xzdy + xydz Pametrizando cada segmento de trayectoria, produce −→ AB : x = 1 −t =⇒dx = −dt y = t =⇒ dy = dt z = 0 =⇒dz = 0 _ −→ AB yzdx + xzdy + xydz = _ 1 0 0dt = 0 −−→ BC : x = 0 =⇒dx = 0 y = 1 −t =⇒ dy = −dt z = t =⇒dz = dt _ −−→ BC yzdx + xzdy + xydz = _ 1 0 0dt = 0 −→ CA : x = t =⇒dx = dt y = 0 =⇒ dy = 0 z = 1 −t =⇒dz = −dt _ −→ CA yzdx + xzdy + xydz = _ 1 0 0dt = 0 Por tanto _ −→ c yzdx + xzdy + xydz = 0 Ejemplo 3 Evaluar I = _ −→ c − → F d − → s , donde − → F (x, y) = (xy, x 2 y) y − → c : [−1, 1] −→IR 2 tal que − → c (t) = (t, [t[). − → c / ∈ C 1 . Soluci´ on 445 La curva C es continua por tramos. As´ı − → c 1 : [−1, 0] −→IR 2 , − → c 1 (t) = (t, −t) =⇒dx = dt, dy = −dt − → c 2 : [0, 1] −→IR 2 , − → c 2 (t) = (t, t) =⇒dx = dt, dy = dt De modo que: − → c = − → c 1 + − → c 2 _ −→ c − → F d − → s = _ −→ c 1 − → F d − → s + _ −→ c 2 − → F d − → s = _ −→ c 1 xydx + x 2 ydy + _ −→ c 2 xydx + x 2 ydy = _ 0 −1 (−t 2 + t 3 )dt + _ 1 0 (t 2 + t 3 )dt = 0 Por tanto _ −→ c − → F d − → s = 0 5.2. Cambio de parametrizaci´ on La pregunta crucial que surge al pensar en una curva es ¿cambiar´ a el valor de la integral si se cambia la parametrizaci´ on de la curva sobre la que se integra?. En primer lugar, ilustremos esto con el siguiente ejemplo. Ejemplo 1 Si C es la mitad inferior del c´ırculo unitario en el plano xy que une los puntos (−1, 0) con (1, 0) . Calcular _ C (1 + xy)ds parametrizando de dos maneras diferentes esta curva. Soluci´ on. Sea − → c (t) = (t, − √ 1 −t 2 ) =⇒ − → c (t) = _ 1, t √ 1 −t 2 _ =⇒ | − → c (t)| = 1 √ 1 −t 2 entonces _ −→ c (1 + xy)ds = _ 1 −1 (1 −t √ 1 −t 2 ) 1 √ 1 −t 2 dt = _ 1 −1 _ 1 √ 1 −t 2 −t _ = π 446 Por otro lado, si ponemos − → c (t) = (cos t, sin t) con π ≤ t ≤ 2π − → c (t) = (−sin t, cos t) =⇒ | − → c (t)| = 1 entonces, al reemplazar los t´erminos del integrando queda _ −→ c (1 + xy)ds = _ 1 −1 (1 + sin t cos t) dt = _ t + sin 2 t 2 _ 2π π = π Por lo tanto, hemos obtenido el mismo valor para la integral a lo largo de estas dos trayectorias diferentes que tienen la misma traza. En segundo lugar, enunciemos un resultado general con el siguiente teo- rema. Teorema 5.2.1. Si − → c (t) y − → p (t) son dos parametrizaciones distintas suaves o suaves por tramos, que conecta los puntos P 0 y P 1 , que tienen la misma traza y direcci´on, y si f(x, y, z) est´a definida y es continua sobre la traza, entonces _ −→ c f(x, y, z)ds = _ −→ p f(x, y, z)ds 5.2.1. Reparametrizaci´ on Sea h : I 1 →I una funci´ on de clase C 1 con valores reales que sea biyectiva entre I = [a, b] y I 1 = [a 1 , b 1 ]. Si − → c : [a, b] → IR 3 una trayectoria C 1 por tramos. Entonces a la composici´ on − → p = − → c ◦ h : [a 1 , b 1 ] →IR 3 La llamamos reparametrizaci´on de − → c El valor de la integral de l´ınea no var´ıa por cambios de parametrizaci´on de la curva − → c , excepto por el signo si hay cambios de orientaci´on, el teorema correspondiente es: Teorema 5.2.2. Sea − → F un campo vectorial continuo y − → c : [a, b] → IR 3 , trayectoria de clase C 1 , y sea − → p : [a 1 , b 1 ] → IR 3 una reparametrizaci´on de − → c . Entonces i) Si − → p conserva la orientaci´on: _ −→ c − → F d − → s = _ −→ p − → F d − → s ii) Si − → p invierte la orientaci´on _ −→ p − → F d − → s = − _ −→ c − → F d − → s 447 Figura 5.7: Ejemplo 1: Sea − → F (x, y, z) = (yz, xz, xy) campo vectorial y − → c : [−5, 10] → IR 3 trayectoria dada por − → c (t) = (t, t 2 , t 3 ) . Una trayectoria opuesta: − → c op : [−5, 10] →R 3 se puede construir: − → c op (t) = c(a + b −t) en este caso queda − → c op (t) = − → c (5 −t) = ((5 −t), (5 −t) 2 , (5 −t) 3 ) . Evaluar _ −→ c − → F d − → s y _ −→ c op − → F d − → s . Soluci´on: _ −→ c − → F d − → s = _ 10 −5 _ F 1 dx dt + F 2 dy dt + F 3 dz dt _ dt = _ 10 −5 yzdx + xzdy + xydz = _ 10 −5 _ t 5 + 2t 5 + 3t 5 ¸ dt = 984,375 por otro lado _ −→ c op − → F d − → s = _ 10 −5 yzdx + xzdy + xydz 448 = _ 10 −5 _ −(5 −t) 5 −2(5 −t) 5 −3(5 −t) 5 ¸ dt = _ 10 −5 −6(5 −t) 5 dt = _ (5 −t) 6 ¸ 10 −5 = −984,375 5.3. Independencia de trayectoria Cuando el valor de una integral _ −→ c − → F d − → s sobre una trayectoria contin- ua por tramos que conecta dos puntos de una regi´ on R depende solamente de los puntos extremos, se dice que la integral es independiente de la trayectoria. Ejemplo. Independencia de trayectoria en una integral de l´ınea Si − → F(x, y) = _ y 2 x 2 , − 2y x _ ; A = (1, 1); B = (4, −2) Calcular el trabajo realizado por el campo de fuerza − → F al llevar un objeto desde A hasta B, por: a) un camino compuesto de un tramo horizontal seguido de un tramo vertical; b) un camino compuesto de un tramo vertical seguido de un tramo hori- zontal. Soluci´on. a) Si llamamos C a la curva, la podemos subdividir en las curvas C 1 y C 2 Tendremos _ −→ c − → F d − → s = _ −→ c 1 − → F d − → s + _ −→ c 1 − → F d − → s Calculamos ambas integrales por separado C 1 = _ x = 1 + t y = 1 , 0 ≤ t ≤ 3 =⇒ _ −→ c 1 − → F d − → s = _ −→ c 1 y 2 x 2 dx − 2y x dy = _ 3 0 1 (1 + t) 2 dt = _ − 1 (1 + t) _ 3 0 = 3 4 449 C 2 = _ x = 4 y = 1 −t , 0 ≤ t ≤ 3 =⇒ _ −→ c 2 − → F d − → s = _ −→ c 2 y 2 x 2 dx − 2y x dy = _ 3 0 2(1 −t) 4 dt = _ 1 2 t − 1 4 t 2 _ 3 0 = 3 2 − 9 4 = − 3 4 Con lo que resulta _ −→ c y 2 x 2 dx − 2y x dy = 3 4 + _ − 3 4 _ = 0 b) Llamemos C * a este otro camino, tambi´en lo podemos separar en dos tramos C 3 y C 4 . Tendremos _ −→ c ∗ − → F d − → s = _ −→ c 3 − → F d − → s + _ −→ c 4 − → F d − → s Calculamos ambas integrales por separado haciendo parametrizaciones: C 3 = _ x = 1 y = 1 −t , 0 ≤ t ≤ 3 =⇒ _ −→ c 3 − → F d − → s = _ c 3 y 2 x 2 dx − 2y x dy = _ 3 0 − 2(1 −t) 1 (−1)dt = _ 2t −t 2 ¸ 3 0 = −3 C 4 = _ x = 1 + t y = −2 , 0 ≤ t ≤ 3 =⇒ _ −→ c 4 − → F d − → s = _ −→ c 4 y 2 x 2 dx − 2y x dy = _ 3 0 − (−2) 2 (1 + t) 2 dt = _ − 4 (1 + t) _ 3 0 = 3 Sumando se obtiene _ −→ c ∗ y 2 x 2 dx − 2y x dy = −3 + 3 = 0 Por ambos caminos dio el mismo resultado. Intente otro camino para ´esta integral que lleve desde A hasta B. 450 5.4. Campos Conservativos Ciertos campos de fuerza importantes en la f´ısica provienen de un poten- cial escalar. Si existe una funci´on φ definida en una regi´ on R y si al campo de fuerza − → F tiene la propiedad de que − → F=∇φ = gradφ entonces se dice que φ es un potencial de − → F. 5.4.1. Campo gradiente Un campo vectorial continuo el cual se obtiene como el gradiente de una funci´ on escalar se llamar´a campo gradiente y una funci´ on φ de la cual se obtiene, la funci´on potencial − → F = ∇φ Teorema 5.4.1. (Teorema fundamental) Sea − → F un campo gradiente con potencial φ definida en una regi´on R y sean P 0 y P 1 puntos cualesquiera de R, entonces: _ −→ c − → F d − → s = φ(P 2 ) −φ(P 1 ) donde − → c : [a, b] → IR 3 tal que − → c (a) = P 1 , y − → c (b) = P 2 , y − → c es una trayectoria de clase C 1 . Demostraci´ on _ −→ c − → F d − → s = _ −→ c ∂φ ∂x dx + ∂φ ∂y dy + ∂φ ∂z dz = _ b a _ ∂φ ∂x dx dt + ∂φ ∂y dy dt + ∂φ ∂z dz dt _ dt = _ b a ∂φ ∂t ( − → c (t)) dt = [φ( − → c (t))] b a = φ(P 2 ) −φ(P 1 ) Por tanto _ −→ c − → F d − → s = φ(P 2 ) −φ(P 1 ) 451 Adicionalmente podemos afirmar que Teorema 5.4.2. Si − → F es un campo gradiente en una regi´on R, entonces _ −→ c − → F d − → s es independiente de la trayectoria. Comentario: Si − → F es campo gradiente existe φ definida en R tal que − → F=∇φ, y el teorema fundamental garantiza la independencia de trayectoria. Observaci´ on: No es dific´ıl ver que el rec´ıproco de este teorema tambi´en es cierto. Teorema 5.4.3. Si − → F es un campo vectorial continuo sobre una region R y si _ −→ c − → F d − → s es independiente de la trayectoria, entonces − → F es un campo gradiente. Curva simple C, se define como la imagen de una aplicaci´on − → c : I →IR 3 de C 1 que sea uno a uno en el intervalo I. Una curva simple es aquella que no se intersecta a si misma, − → c (a) y − → c (b) se llaman punto inicial y punto final de la curva. Curva cerrada simple, se define como − → c : [a, b] →IR 3 de C 1 tal que 1) es uno a uno en [a, b] 2) − → c (a) = − → c (b) Observaci´ on: i) Si la curva satisface s´olo (2) es curva cerrada. ii) Las curvas simples cerradas tienen dos direcciones de movimiento posi- ble. Integrales de l´ınea sobre curvas simples orientadas y curvas cerradas sim- ples C. Sea C una curva simple orientada imagen de − → c : [a, b] → IR 3 entonces definimos: _ C − → F d − → s = _ −→ c − → F d − → s y _ C f ds = _ −→ c f ds De las curvas cerradas y la independencia de trayectoria se tiene el siguiente teorema. Teorema 5.4.4. _ C − → F d − → s es independiente de la trayectoria en R si y solo si _ C − → F d − → s = 0 para toda curva cerrada C contenida en R. Un dominio D es conexo si dos puntos cualesquiera de D se pueden unir con un segmento de recta y todos los puntos de la recta pertenecen a D Una regi´on es simplemente conexa y abierta si ella es un conjunto abierto y tal que toda curva simple cerrada en D encierra puntos que solo est´ an en el interior de D. Si un campo de fuerza tiene la propiedad de que el trabajo 452 realizado sobre una part´ıcula en movimiento conforme se mueve de un punto a otro es independiente de la trayectoria, se llama campo conservativo. De los teoremas anteriores se puede formular lo siguiente: ¨ Una condici´ on necesaria y suficiente para que un campo de fuerzas sea consevativo es que sea un campo gradiente” Teorema 5.4.5. Sean M(x, y) y N(x, y) funciones con derivadas parciales continuas en un conjunto abierto y conexo D. Entonces el campo vectorial − → F (x, y) = (M(x, y), N(x, y))es conservativo si y solo si ∂M ∂y = ∂N ∂x en D Ejemplo 1 Sea − → F (x, y) = (2xy, (x 2 −y)) a) Probar que es campo conservativo. b) Hallar funci´ on potencial. Soluci´on. a) ∂N ∂x = 2x ∂M ∂x = 2x _ ¸ ¸ ¸ _ ¸ ¸ ¸ _ =⇒Campo conservativo en IR 2 b) Existe Potencial φ(x, y) tal que ∂φ(x, y) ∂x = 2xy ⇒φ(x, y) = _ 2xydx + h 1 (y) ∂φ(x, y) ∂y = x 2 −y ⇒φ(x, y) = _ _ x 2 −y _ dx + h 2 (x) ⇒ φ(x, y) = x 2 y + h 1 (y) φ(x, y) = x 2 y − y 2 2 + h 2 (x) ⇒φ(x, y) = x 2 y − y 2 2 + C (Rotacional) Sea − → F (x, y, z) = (P(x, y, z), Q(x, y, z), R(x, y, z)) funci´on vectorial, se define el rotacional de − → F,denotado por rot − → F (x, y, z) o ∇ − → F (x, y, z) por: ∇ − → F (x, y, z). = ¸ ¸ ¸ ¸ ¸ ¸ ¸ ¸ ´ i ´ j ´ k ∂ ∂x ∂ ∂y ∂ ∂z P Q R ¸ ¸ ¸ ¸ ¸ ¸ ¸ ¸ 453 Teorema 5.4.6. Si − → F (x, y, z) = (P(x, y, z), Q(x, y, z), R(x, y, z)) funci´on vectorial donde P, Q, R son funciones con derivadas parciales continuas en un conjunto abierto y conexo D. Entonces −→ F es un campo conservativo ⇐⇒∇ − → F (x, y, z) = − → 0 en D es decir si y s´olo si ∂P ∂y = ∂Q ∂x , ∂P ∂z = ∂R ∂x , ∂Q ∂z = ∂R ∂y Ejemplo 2 Verifique que − → F (x, y, z) = (yz, xz, xy) es un campo conservativo en IR 3 Soluci´on. ∇ − → F (x, y, z). = ¸ ¸ ¸ ¸ ¸ ¸ ¸ ¸ ´ i ´ j ´ k ∂ ∂x ∂ ∂y ∂ ∂z yz xz xy ¸ ¸ ¸ ¸ ¸ ¸ ¸ ¸ = ((x −x) , −(y −y) , (z −z)) = (0, 0, 0) o sea ∇ − → F (x, y, z) = − → 0 en IR 3 Ejemplo 3 Dado el campo vectorial − → F (x, y, z) = (3y 2 z+ye x , 6xyz+e x , 3xy 2 ).Verifique que es campo conservativo y calcular potencial. Soluci´on: ∂R ∂y = ∂Q ∂z ⇒6yz + e x = 6yz + e x ∂R ∂x = ∂P ∂z ⇒3y 2 = 3y 2 ∂Q ∂x = ∂P ∂y ⇒6yz + e x = 6yz + e x Luego ∇ − → F (x, y, z) = − → 0 en IR 3 ,es entonces claramente un campo vectorial conservativo, entonces existe φ(x, y, z) tal que ∂φ(x, y, x) ∂x = 3y 2 z + ye x ⇒φ(x, y, z) = _ _ 3y 2 z + ye x _ dx + h 1 (y, z) = 3xy 2 z + ye x + h 1 (y, z) 454 ∂φ(x, y, x) ∂y = 6xyz + e x ⇒φ(x, y, z) = _ (6xyz + e x ) dx + h 1 (x, z) = 3xy 2 z + ye x + h 1 (x, z) ∂φ(x, y, x) ∂z = 3xy 2 ⇒φ(x, y, z) = _ 3xy 2 dx + h 1 (x, z) = 3xy 2 z + h 1 (x, y) De aqu´ı se deduce que ∴ φ(x, y, z) = 3xy 2 z + ye x Adem´ as, si se pide calcular I = _ C − → F d − → s y C es la curva descrita por − → c (t) = (cos t, sent, t), t ∈ [0, 2π] , entonces I = _ C − → F d − → s = _ 3xy 2 z + ye x ¸ 3 0 = φ(1, 0, 2π) −φ(1, 0, 0) = 0 −0 = 0 (Divergencia de un campo vectorial) Si − → F (x, y, z) = (P(x, y, z), Q(x, y, z), R(x, y, z)) tal que P, Q y R tienen derivadas parciales en alguna regi´ on D. La divergencia de − → F se denota por: div − → F = ∇ − → F = ∂P(x, y, z) ∂x + ∂Q(x, y, z) ∂y + ∂R(x, y, z) ∂z Si − → F es un campo vectorial, entonces la div − → F da informaci´on acerca del flujo en un punto P(x,y,z): i) Si div − → F < 0 en un punto P(x,y,z) , entonces el flujo del campo − → F se orienta hacia el punto y se dice que hay un sumidero en P. ii) Si div − → F > 0 en un punto P(x,y,z) , entonces el flujo del campo − → F se orienta desde el punto y se dice que hay una fuente en P. iii) Si div − → F = 0 en un punto P(x,y,z) , entonces el flujo del campo − → F es nulo. 455 Introducci´ on Teorema de Green El teorema de Green, que enunciaremos a continuaci´on, dice que bajo ciertas condiciones las integrales de l´ınea pueden expresarse y calcularse con integrales dobles. Comencemos por definir una regi´on compacta R descrita simultaneamente por las desigualdades φ 1 (x) ≤ y ≤ φ 2 (x), a ≤ x ≤ b denominada regi´ on orientada en direcci´ on del eje y ψ 1 (y) ≤ x ≤ ψ 2 (y), c ≤ y ≤ d denominada regi´ on orientada en direcci´ on del eje x donde las funciones φ 1 y φ 2 son continuas y seccionalmente suaves en [a, b] y las funciones ψ 1 y ψ 2 son continuas y seccionalmente suaves en [c, d] . En esta regi´ on cerrada R con una frontera continua seccionalmente suave consideramos definidas las funciones M y N continuamente diferenciables ( o de clase C 1 ) definidas en R. La curva C frontera de la regi´on R, se dice que esta orientada positivamente ,si al caminar sobre ella la regi´ on R estar´ a a su izquierda. En este marco de ideas y condiciones se plantea el teorema de Green. 5.4.2. Teorema de Green Sea R una regi´on cerrada y acotada con frontera C,orientada en sentido positivo. Si M, N, ∂M ∂y y ∂N ∂x son continuas en R. Entonces _ C Mdx + Ndy = _ _ R _ ∂N ∂x − ∂M ∂y _ dA 456 Figura 5.8: Regi´on cerrada de tipo a, frontera orientada positiva Demostraci´ on. Supongamos que R est´ a descrita por φ 1 (x) ≤ y ≤ φ 2 (x), a ≤ x ≤ b _ C Mdx = _ C 1 Mdx + _ C 2 Mdx = _ b a M(x, φ 1 (x))dx + _ a b M(x, φ 2 (x))dx =⇒ _ C Mdx = _ b a [M(x, φ 1 (x)) −M(x, φ 2 (x))] dx Por otra parte _ _ R ∂M ∂y dA = _ b a _ φ 2 (x) φ 1 (x) ∂M ∂y dA = _ b a [M(x, φ 2 (x)) −M(x, φ 1 (x))] dx Luego _ C Mdx = − _ _ R ∂M ∂y dA De manera simlar y considerando ahora ψ 1 (y) ≤ x ≤ ψ 2 (y), c ≤ y ≤ d Se establece que _ C Ndy = _ _ R ∂N ∂x dA 457 Figura 5.9: Regi´on cerrada de tipo b, frontera orientada positiva Por lo tanto _ C Mdx + Ndy = _ _ R _ ∂N ∂x − ∂M ∂y _ dA Ejemplo 1 Evaluar la integral I = _ C _ y 2 + senx 2 _ dx + _ cos y 2 −x _ dy donde C es la frontera de la regi´on cuadrada 0 ≤ x ≤ 1, 0 ≤ y ≤ 1 Figura 5.10: Frontera de la regi´on cuadrada 0 ≤ x ≤ 1, 0 ≤ y ≤ 1 458 Soluci´on. Usando el Teorema de Green podemos afirmar que M(x, y) = y 2 + senx 2 =⇒ ∂M ∂y = 2y N(x, y) = cos y 2 −x =⇒ ∂N ∂x = −1 Por lo tanto aplicando el teorema I = _ 1 0 _ 1 0 (−1 −2y)dxdy = _ 1 0 (−1 −2y)dy = _ −y −y 2 ¸ 1 0 = −2 Comentario. Sin usar el teorema de Green este ejercicio resulta muy complicado de resolver, vale la pena que lo intente. 459 Ejemplo 2 Calcular I = _ C e y dx + (xe y + 2y)dy con D = _ (x; y) ∈ IR 2 /x 2 + y 2 ≤ 1 _ Figura 5.11: D = _ (x; y) ∈ IR 2 /x 2 + y 2 ≤ 1 _ Soluci´on Por el Teorema de Green podemos afirmar que M(x, y) = e y =⇒ ∂M ∂y = e y N(x, y) = xe y + 2y =⇒ ∂N ∂x = xe y + 2y Por lo tanto aplicando el teorema I = _ _ D _ ∂N ∂x − ∂M ∂y _ dxdy = _ _ (e y −e y )dxdy = 0 Corolario 5.4.1. Aplicando el teorema de Green a M = −y y N = x se deduce que A(D) = 1 2 _ C xdy −ydx donde A(D) es el ´area de la regi´on D contenida en el plano XY , y C es el borde de D recorrrido en sentido positivo. Esta f´ormula nos permite calcular un ´area en t´erminos de una integral de l´ınea. 460 Figura 5.12: Regi´on x 2 a 2 + y 2 b 2 ≤ 1 Ejemplo 3 Consideremos la regi´ on x 2 a 2 + y 2 b 2 ≤ 1, cuyo borde es una elipse de semi-ejes dados por a y b.Calcular el ´ area de la region D Soluci´on Podemos parametrizar la elipse usando x(t) = acost , y(t) = bsent, t ∈ [0, 2π], y luego se obtiene − → r (t) = (acost, bsent) =⇒ − → r (t) = (−asent, b cos t) A(D) = 1 2 _ 2π 0 (xy −yx )dt = 1 2 _ 2π 0 (ab cos 2 t −(−absen 2 t))dt = πab 5.5. Aplicaciones de la integral de trayectoria A continuacion se resumen las formulas para el c´ alculo de la masa y el momento de resortes, varillas delgadas y alambres a lo largo de una curva suave C en el espacio. Masa: M = _ C δ (x, y, z) ds, donde δ = δ (x, y, z) es la densidad y ds un elemento diferencial de arco. Momentos con respecto a los planos coordenados: M yz = _ C x δ (x, y, z) ds; M xz = _ C y δ (x, y, z) ds; M xy = _ C z δ (x, y, z) ds Coordenadas del centro de Masa x = M yz M ; y = M xz M ; z = M xy M Momentos de inercia con respecto a los ejes y otras rectas I x = _ C (y 2 + z 2 ) δ (x, y, z) ds, I y = _ C (x 2 + z 2 ) δ (x, y, z) ds; I z = _ C (y 2 + z 2 ) δ (x, y, z) ds; 461 I L = _ C r 2 δ (x, y, z) ds donde r (x, y, z) es la distancia del punto (x, y, z) a la recta L. Radio de giro con respecto a una recta L. R L = _ I L M Problema 1 Dado el resorte de densidad constante δ = 1 con forma helicoidal − → r (t) = cos 4t ´ i + sen4t ´ j + t ´ k, 0 ≤ t ≤ 2π. Encontrar la masa del resorte y el momento de inercia con respecto al eje z Soluci´on La masa del resorte esta dada por M = _ C δ (x, y, z) ds = _ C (1)ds = _ 2π 0 [ − → r (t)[ dt Encontremos primero [ − → r (t)[ = _ (−4sen4t) 2 + (4 cos 4t) 2 + 1 = √ 17. Luego evaluemos la integral M = _ 2π 0 [ − → r (t)[ dt = _ 2π 0 √ 17dt M = 2π √ 17 El momento de inercia con respecto al eje z se define por: I z = _ C _ x 2 + y 2 _ δds = _ 2π 0 _ cos 2 4t + sen 2 4t _ (1) √ 17dt = _ 2π 0 √ 17dt = 2π √ 17 462 Problema 2 Sea un arco de metal con forma de semicircunferencia y 2 +z 2 = 1, z ≥ 0. Si la densidad en el punto (x, y, z) del arco es δ (x, y, z) = 2 −z , determinar las coordenadas del centro de masa del arco. Soluci´on. Por simetr´ıa sabemos que x = 0; y = 0; dado que el arco est´ a el el plano yz con la masa distribuida simetricamente con respecto al eje z. Para encontrar z , parametizamos el arco de semicircunferencia como: − → r (t) = cos t ´ j+sent ´ k, 0 ≤ t ≤ π =⇒ [ − → r (t)[ = _ (−sent) 2 + (cos t) 2 = 1 Por otra parte, δ (x, y, z) = 2 −z =⇒ δ ( − → r (t)) = (2 −sent) Ahora, determinemos la masa utilizando la definici´ on de integral de trayec- toria M = _ C δ (x, y, z) ds = _ π 0 δ ( − → r (t)) [ − → r (t)[ dt = _ π 0 (2 −sent) (1) dt = [2t + cos t] π 0 M = 2π −2 A continuaci´ on calculemos el momento de inercia M xy Luego, z = M xy M = 8 −π 2 (2π −2) ≈ 0, 57. Finalmente, la coordenada del centro de masa es (0; 0; 0, 57) . Problema 3 Un alambre tiene la forma de una semicircunferencia de radio a. La densi- dad lineal de masa en un punto P es directamente proporcional a la distancia de P a la recta que pasa por los extremos del alambre, ¿cu´ al es la masa total del alambre?. Soluci´on En primer lugar situar el alambre en el plano xy de sistema coordenado, como en la figura: Ahora, la densidad puede interpretarse como un campo escalar, ya que, en cada punto P = (x, y) de la curva que describe el alambre, la densidad es δ(x, y) = ky. Calcular la masa total equivale a sumar la densidad sobre todos los puntos 463 de la curva, y as´ı lo que queremos determinar es M = _ C δ (x, y, z) ds = _ b a δ ( − → r (t)) [ − → r (t)[ dt donde C es la curva que describe el alambre, la semicircunferencia de radio a. Una parametrizaci´ on para C es: − → r (t) = (a cos t, asent) , t ∈ [0, π] entonces − → r (t) = (−asent, a cos t) =⇒ [ − → r (t)[ = a Lo que implica que la masa del alambre es M = _ π 0 (kasent) (a) dt = ka 2 _ π 0 sentdt = ka 2 [−cos t] π 0 = 2ka 2 Problema 4 Un alambre tiene forma de circunferencia, x 2 + y 2 = a 2 . Determine su masa y su momento de inercia respecto de un di´ ametro si la densidad en un punto (x, y) del alambre est´ a dada por la funci´on δ(x, y) = [x[ +[y[ . Soluci´on La masa del alambre viene dada por la expresi´on: M = _ C δ (x, y, z) ds = _ b a δ ( − → r (t)) [ − → r (t)[ dt siendo C la curva cuya trayectoria representa la forma del alambre, en este caso una circunferencia que parametrizamos por: − → r (t) = (acost, asent); t ∈ [0, 2π] que es de clase C 1 . − → r (t) = (−asent, a cos t) =⇒ | − → r (t)| = 1 Por lo tanto la masa es igual a 464 M = _ 2π 0 δ ( − → r (t)) [ − → r (t)[ dt = _ 2π 0 ([a cos t[ +[asent[)adt = a 2 _ π/2 0 (cos t + sent) dt + a 2 _ π π/2 (−cos t + sent) dt + +a 2 _ 3π/2 π (−cos t −sent) dt + a 2 _ 2π 3π/2 (cos t −sent) dt = a 2 [sent −cost] π/2 0 + a 2 [−sent −cost] π π/2 + +a 2 [−sent + cost] 3π/2 π + a 2 [sent + cost] 2π 3π/ = 8a 2 Para calcular el momento de inercia respecto de un di´ ametro necesitamos la distancia de un punto cualquiera (x, y) a dicho di´ ametro. Para simplificar, tomaremos como eje el eje OX, por tanto, la funci´on que da la distancia de un punto al eje es r(x, y) = [y[. Teniendo en cuenta la definici´ on del momento de inercia respecto de un eje se tiene: I L = _ C r 2 δ (x, y, z) ds = _ C y 2 ([x[ +[y[) ds = a 4 _ 2π 0 sen 2 t ([sent[ +[cost[) dt = a 4 _ π/2 0 sen 2 t (sent + cost) dt + a 4 _ π π/2 sen 2 t (sent −cost) dt +a 4 _ 3π/2 π sen 2 t (−sent −cost) dt + a 4 _ 2π 3π/2 sen 2 t (−sent + cost) dt = 4a 4 5.5.1. ´ Area de una pared Problema 1 La base de una pared en el primer cuadrante es la trayectoria − → c : _ 0, π 2 _ → IR 2 ,dada por − → c (t) = (3 cos 3 t, 3sen 3 t) y su altura es en cada punto (x, y) es f (x, y) = 1 + y 3 .Calcular el ´ area de la pared. Soluci´on 465 El ´area de una pared esta dada por la integral de trayectoria _ C f (x, y) ds = _ b a f ( − → c (t)) [ − → c (t)[ d donde f (x, y) ≥ 0, ∀ (x, y) ∈ C En este caso tenemos que − → c (t) = (3 cos 3 t, 3sen 3 t) , t ∈ _ 0, π 2 _ − → c (t) = (−9 cos 2 tsent, 9sen 2 t cos t) =⇒ | − → c (t)| = 9sent cos t f ( − → c (t)) = 1 + sen 3 t As´ı, la integral es _ C _ 1 + y 3 _ ds = _ π/2 0 _ 1 + sen 3 t _ (9sent cos t) dt = 9 _ π/2 0 _ sent + sen 4 t _ cos tdt = 9 _ sen 2 t 2 + sen 5 t 5 _ π/2 0 = 63 10 Problema 2 La base de una pared en el primer cuadrante es la trayectoria − → c :[0, 1] → IR 2 ,dada por − → c (t) = (t, t 2 ) y su altura es en cada punto (x, y) es f (x, y) = √ 1 + 4y. a) Determinar la longitud de la base de la pared. b) Calcular el ´ area de la pared. Soluci´on Tenemos que − → c (t) = (t, t 2 ) =⇒ − → c (t) = (1, 2t) =⇒ | − → c (t)| = √ 1 + 4t 4 La longitud de la pared se determina por l = _ 1 0 | − → c (t)| dt = _ 1 0 √ 1 + 4t 2 dt = 1 2 _ t √ 1 + 4t 2 + 1 2 ln _ 2t + √ 1 + 4t 2 _ _ 1 0 = 1 2 _ √ 5 + 1 2 ln _ 2 + √ 5 _ _ 466 El ´area de una pared esta dada por la integral de trayectoria _ C f (x, y) ds = _ 1 0 f ( − → c (t)) | − → c (t)| dt = _ 1 0 _ √ 1 + 4t 4 __ √ 1 + 4t 4 _ dt = _ 1 0 _ 1 + 4t 4 _ dt = _ t + 4 5 t 5 _ 1 0 = 9 5 5.6. Aplicaciones de la integral de l´ınea Definici´ on . Sea C una curva simple y regular, y sea − → F :⊆ R 3 → R 3 un campo vectorial continuo. Definimos la integral de trabajo (o integral de l´ınea) de − → F sobre la curva C ⊆ D por W = _ C − → F d − → r = _ b a − → F ( − → r (t)) − → r (t) dt donde − → r : [a, b] →IR 3 es una parametrizaci´on regular de C Cuando la curva C es cerrada, entonces se puede escribir W = _ C − → F d − → r y esta integral recibe el nombre de circulaci´ on de − → F a lo largo de C. Si − → F representa el campo de velocidades de un fluido, la circulaci´ on es la integral de la componente tangencial de la velocidad a lo largo de la curva cerrada C, proporcionando la cantidad neta de giro del fluido alrededor de C. (Campo Conservativo) En general, se dice que un campo vectorial − → F : D ⊆ R 3 →R 3 es conservativo en D si existe un potencial φ : D ⊆ R 3 →R tal que − → F = ∇φ sobre D. En efecto, si C es una curva 467 regular parametrizada por − → r : [a, b] →IR 3 , entonces tenemos que _ C − → F d − → r = _ b a − → F ( − → r (t)) − → r (t) dt = _ b a ∇φ( − → r (t)) − → r (t) dt = _ b a d dt [∇φ( − → r (t))] dt = φ( − → r (b)) −φ( − → r (a)) Proposici´on 5.6.1. Sea − → F = − → F (x, y, z) un campo vectorial continuo sobre un abierto conexo de IR 3 . Entonces las propiedades sobre − → F siguientes, son equivalentes, para que sea campo vectorial conservativo: i) ∇ − → F = − → 0 ii) Para toda curva C ⊆ D cerrada y regular por pedazos se tiene _ C − → F d − → r = 0 iii) Para cualquier par de curvas regulares, C 1 ⊆ D y C 2 ⊆ D, con iguales puntos inicial y final, se tiene _ C 1 − → F d − → r = _ C 2 − → F d − → r Problema 1 Calcular el trabajo ejercido por el campo vectorial − → F (x, y, z) = (3x − 2y, y + 2z, −x 2 )sobre una part´ıcula que se mueve seg´ un la curva C dada por x = z 2 , z = y 2 desde el punto (0, 0, 0) hasta el (1, 1, 1). Soluci´ on En primer lugar parametricemos la curva C: − → r (t) = (t 4 , t, t 2 ), t ∈ [0, 1] De modo que − → r (t) = (4t 3 , 1, 2t) y − → F ( − → r (t)) = (3t 4 −2t, t + 2t 2 , −t 8 ) 468 Calculando directamente tenemos W = _ C − → F d − → r = _ b a − → F ( − → r (t)) − → r (t) dt = _ 1 0 (3t 4 −2t, t + 2t 2 , −t 8 ) (4t 3 , 1, 2t)dt = _ 1 0 (12t 7 −8t 4 + t + 2t 2 −2t 9 )dt = _ 3 4 t 8 − 8 5 t 5 + 1 2 t 2 + 2 3 t 3 − 1 5 t 10 _ 1 0 = 13 15 Problema 2 Un campo de fuerza gravitacional − → F (x, y, z) esta dado por − → F (x, y, z) = k | − → r | 3 − → r . Calcular el trabajo que realiza la fuerza gravitacional que act´ ua sobre una part´ıcula que se mueve a lo largo del eje x desde el P 0 (1, 0, 0) hasta P(2, 0, 0) . Soluci´on. Si parametrizamos la trayectoria tenemos: − → r (t) = (t, 0, 0) 1 ≤ t ≤ 2 Ahora, determinemos la funci´ on compuesta: − → F ( − → r (t)) = k t 3 (t, 0, 0). Aplicando la definici´on de trabajo tenemos W = _ C − → F d − → r = _ 2 1 − → F ( − → r (t)) − → r (t) dt = _ 2 1 k t 3 (t, 0, 0) (1, 0, 0) dt = _ 2 1 k t 2 dt = _ − k t _ 2 1 = k 2 Problema 3 Si una part´ıcula es atra´ıda hacia el origen por una fuerza cuya magnitud es proporcional a la distancia | − → r | de la part´ıcula al origen, ¿que trabajo se hace cuando se mueve la part´ıcula desde el punto (0, 1) hasta el (1, 2) a lo largo de y = 1 + x 2 suponiendo un coeficiente de roce µ entre la part´ıcula y la trayectoria?. Desprecie la fuerza de gravedad. 469 Soluci´on. Formulemos el problema usando el parametro longitud de arco s W = _ C − → F d − → r = _ s 2 s 1 − → F ( − → r (s)) − → r (s) ds = _ s 2 s 1 − → F ( − → r (s)) ´ t (s) ds = _ s 2 s 1 F T ( − → r (s)) ds La fuerza que trabaja es la componente tangencial a la curva. Entonces, determinemos esta componente a partir de la informacion. Sea θ el ´angulo que forma la tangente a la curva en un punto P (x, y) cualquiera con el eje x. Sea φ el ´angulo que forma el radio vector P con el eje y. Sea α el ´angulo entre la tangente y el radio vector en P. Al mover la part´ıcula a lo largo de la trayectoria, hay que realizar trabajo contra dos fuerzas que tienen componente tangencial, a saber, la componente tangencial de la fuerza central: F t = F cos α = kr cos α y la fuerza de rozamiento F r = µF n = µFsenα = µkrsenα. Donde F n es la componente de la fuerza central que es perpendicular a la trayectoria y que act´ ua para mantener la part´ıcula en su trayectoria. Por tanto la fuerza tangencial resultante es: F T = kr cos α + µkrsenα Aplicando, el teorema del ´angulo externo de la geometr´ıa plana se tiene: α = φ −θ Reemplazando en la expresi´ on anterior, queda F T = kr cos (φ −θ) + µkrsen(φ −θ) . Luego, el trabajo queda W = _ s 2 s 1 F T ( − → r (s)) ds = _ L 0 (kr cos (φ −θ) + µkrsen(φ −θ)) ds = k _ L 0 r cos (φ −θ) ds + kµ _ L 0 rsen(φ −θ) ds = k _ L 0 r [cos φcos θ + senφsenθ] ds + +kµ _ L 0 r [senφcos θds −senθ cos φ] ds 470 Ahora bien, el trabajo expresemos el trabajo en coordenadas cartesianas x = r cos φ dx = cos θds y = rsenφ dy = senθds Por lo tanto, sustituyendo ´estas en la ´ ultima expresi´ on para W, se tiene: W = k _ C xdx + ydy + µk _ C ydx −xdy Parametizando la curva C, queda: − → r (t) = (t, t 2 + 1), 0 ≤ t ≤ 1 =⇒ − → r (t) = (1, 2t), 0 ≤ t ≤ 1 Sustituyendo en la integral anterior, obtenemos W = k _ 1 0 (t, t 2 + 1) (1, 2t)dt + kµ _ 1 0 (t 2 + 1, −t) (1, 2t)dt = k _ 1 0 (2t 3 + 3t)dt + kµ _ 1 0 (1 −t 2 )dt = k _ 1 2 t 4 + 3 2 t 2 _ 1 0 + kµ _ t − 1 3 t 3 _ 1 0 W = 2k + 3 2 µk Es el trabajo total efectuado durante el movimiento. Problema 4 Sea − → F (x, y) el campo de fuerzas definido en IR 2 por − → F (x, y) = (2x + ycos(xy), xcos(xy)) Calcular el trabajo realizado por − → F sobre cualquier curva cerrada con- tenida en IR 2 Soluci´on Si el campo es conservativo, el trabajo realizado por el campo sobre cualquier curva cerrada ser´ a nulo. Por tanto, supongamos que existe φ : IR 2 → IR diferenciable tal que − → F = ∇φ. Entonces, se debe satisfacer ∂φ ∂x = 2x + ycos(xy) ∂φ ∂y = xcos(xy) _ (1) Integrando la primera ecuaci´on respecto de x obtenemos, φ(x, y) = x 2 + sen(xy) + h(y). 471 Si derivamos φ respecto de y , luego comparamos con la segunda ecuaci´ on de (1) obtenemos que xcos(xy) + h (y) = xcos(xy) lo que implica h (y) = 0 =⇒ h(y) = c Por lo tanto, la funci´ on φ(x, y) = x 2 + sen(xy) + c verifica que − → F = ∇φ Finalmente W = _ C − → F d − → r = 0 Problema 5 ¿Para qu´e valores de a∈ IR el campo vectorial − → F (x, y, z) = (axy −z 3 , (a −2)x 2 , (1 −a)xz 2 ) es conservativo? Para esos valores, calcular la funci´ on potencial. Soluci´on Para cualquier valor de a el campo − → F es de clase C 1 en R 3 y ser´a conservativo si su rotacional es cero ∀(x; y; z) ∈ R 3 . Calculemos el rotacional ∇ − → F = ¸ ¸ ¸ ¸ ¸ ¸ i j k ∂ ∂x ∂ ∂y ∂ ∂z axy −z 3 (a −2)x 2 (1 −a)xz 2 ¸ ¸ ¸ ¸ ¸ ¸ = (0, −3z 2 −(1 −a)z 2 , 2x(a −2) −ax) que se anula si se cumplen las ecuaciones: (1 −a)z 2 + 3z 2 = 0 2x(a −2) −ax = 0 Por tanto, para a = 4 el campo − → F es conservativo y ∃φ : IR 3 → IR tal que − → F = ∇φ = (4xy −z 3 , 2x 2 , −3xz 2 ),entonces 472 ∂φ ∂x = 4xy −z 3 ∂φ ∂y = 2x 2 ∂φ ∂z = −3xz 2 Integrando la primera ecuaci´on respecto de x, queda φ(x, y, z) = _ (4xy −z 3 )dx = 2x 2 y −xz 3 + h(y, z) Por tanto φ(x, y, z) = 2x 2 y −xz 3 + g(z) derivando la funci´ on φ con respecto a z y luego comparamos con la tercera ecuaci´ on tenemos ∂φ ∂z = −3xz 2 + g (z) = −3xz 2 =⇒ g (z) = 0 =⇒ g (z) = c En consecuencia, la funci´on potencial del campo − → F es φ(x, y, z) = 2x 2 y −xz 3 + c Problema 6 Una masa M en el origen en IR 3 ejerce una fuerza sobre una masa m lo- calizada en − → r = (x, y, z) con magnitud G mM r 2 y dirigida hacia el origen. Aqu´ı G es la constante gravitacional, que depende de las unidades de medi- ci´ on y r = [ − → r [ = _ x 2 + y 2 + z 2 ,si recordamos que − − → r r es un vector unitario dirigido hacia el origen, entonces podemos escribir el campo de fuerza como − → F = −G mM r 3 − → r a) Mostrar que el campo vectorial es conservativo. b) Hallar un potencial escalar para − → F . c) Hallar el trabajo realizado por − → F al trasladar la part´ıcula de masa m desde un punto P 1 hasta P 2 d) ¿Cual es el trabajo realizado por la fuerza − → F al mover la part´ıcula de masa m desde el infinito hasta una distancia r del origen?. 473 Soluci´on a) En primer lugar calculemos ∇ − → F = ∇ _ −G mM r 3 − → r _ = −GmM _ ∇ _ 1 r 3 _ − → r + 1 r 3 ∇ − → r _ Puesto que de las identidades b´asicas del an´alisis vectorial, tenemos i) ∇ _ φ − → F _ = ∇(φ) − → F + φ(∇ − → F ) ii)∇(r n ) = nr n−2− → r Entonces, queda: ∇ − → F = −GmM _ − 3 r − → r − → r + 1 r 3 ∇ − → r _ De modo que el primer t´ermino se anula pues − → r − → r = 0 Luego queda a´ un por calcular el segundo t´ermino ∇ − → r = ¸ ¸ ¸ ¸ ¸ ¸ i j k ∂ ∂x ∂ ∂y ∂ ∂z x y z ¸ ¸ ¸ ¸ ¸ ¸ = (0, 0, 0) Por tanto ∇ − → F = − → 0 para − → r ,= − → 0 b) Luego existe φ tal que − → F = −∇φ.(En f´ısica se define que el campo de fuerza proviene del valor negativo del gradiente de la funci´on escalar). Aprovechando la simetr´ıa radial del campo de fuerza expresemos el gradiente en coordenadas cil´ındricas ∂φ ∂r = G mM r 2 =⇒ φ( − → r ) = _ G mM r 2 dr φ( − → r ) = − GmM r + C Donde c es una constante de integraci´on c) Como el campo es conservativo, el trabajo depende de la posici´on de los puntos inicial y final y el trabajo es igual a _ C − → F d − → r = φ( − → r (2)) −φ( − → r (1)) = GmM _ 1 r 1 − 1 r 2 _ 474 donde r 1 y r 2 son las distancias radiales de los puntos P 1 y P 2 respectivamente al origen. d) Si r 1 →∞ y r 2 = r se tiene que W = − GmM r Problema 7 Dado el campo de fuerzas − → F (x, y) = (y 3 + 1, 3xy 2 + 1). a) ¿Es − → F conservativo? en caso que lo sea, hallar la funci´on potencial de − → F . b) Hallar el trabajo realizado al mover un objeto desde el punto (0, 0) al (2, 0),a lo largo de la semicircunferencia (x −1) 2 + y 2 = 1 con y ≥ 0. c) Hallar el trabajo realizado al mover el objeto a lo largo de la circun- ferencia completa. Soluci´on Examinemos el rotor de − → F ∇ − → F = ¸ ¸ ¸ ¸ ¸ ¸ i j k ∂ ∂x ∂ ∂y 0 y 3 + 1 3xy 2 + 1 0 ¸ ¸ ¸ ¸ ¸ ¸ = _ 0, 0, 3y 2 −3y 2 _ = (0, 0, 0) Por tanto, estamos en presencia de un campo conservativo, luego existe una funci´ on escalar φ(x, y) tal que ∂φ ∂x = y 3 + 1 ∂φ ∂y = 3xy 2 + 1 Integrando la primera ecuaci´on respecto de x, queda φ(x, y) = _ (y 3 + 1)dx = xy 3 + x + h(y) Para calcular la funci´onh(y),calculamos la derivada parcial de φ respecto de 475 y luego comparamos con la segunda ecuaci´on: ∂φ ∂y = 3xy 2 + h (y) = 3xy 2 + 1 =⇒ h (y) = 1 =⇒ h(y) = y + c En consecuencia, la funci´on potencial del campo − → F es φ(x, y, z) = xy 3 + x + y + c b) Como − → F es conservativo la integral es independiente del camino, ´ unicamente depende de los puntos inicial y final. El trabajo realizado al mover el objeto desde (0, 0) hasta (2, 0) ser´a: _ C − → F d − → r = φ(2, 0) −φ(0, 0) = (2 + c) −c = 2 c) A lo largo de la circunferencia tenemos una curva cerrada,lo que implica que el trabajo ser´ a nulo: W = _ C − → F d − → r = 0 Problemas propuestos para reflexionar 1.- Considere dos curvas que parten del punto P = (0, 2) y finalizan en Q = (π, 0); la braquist´ ocrona dada por la ecuaci´on: − → r (t) = (t + sen(t + π); 1 −cos(t + π)); t ∈ [0; π]; y el segmento rectitl´ıneo que va de P a Q. Pruebe que si se suelta una bola que se desliza por cada curva, bajo el efecto de la gravedad, entonces llega antes la bola de la braquist´ ocrona que la del segmento rectil´ıneo. T´engase en cuenta que el tiempo total, para una curva cualquiera, es la integral a lo largo de dicha curva del campo escalar φ(x, y) = 1 _ 2g (h 0 −y) donde g es la gravedad y h 0 la altura inicial de la bola. 2.- Calcular el trabajo realizado por una fuerza proporcional al vector dirigido hacia el origen, sobre el recorrido del primer cuadrante de la elipse x 2 a 2 + y 2 b 2 = 1, desde el punto (a, 0) hasta el (0, b). 476 Soluci´ on: k 2 (a 2 −b 2 ) con k > 0. 3.- Dado h > 0, sea C la curva que se encuentra sobre la superficie defini- da por x 2 +y 2 = z 2 h 2 , de forma tal que la altura z = z() satisface la ecuaci´ on diferencial dz d = z; z(0) = h donde z y representan las coordenadas cil´ındri- cas. (a) Parametrice la ecuaci´on de la curva C − → r (θ) .Calcule la curvatura κ y la torsi´ on τ (b) Considere el campo vectorial − → F (x, y, z) = _ 1 x , 1 y , − 1 z 2 _ Sea C 0 la restricci´ on de C a ∈ _ π 6 , π 3 _ .Calcule el trabajo realizado por el campo − → F al desplazar una part´ıcula a trav´es de C 0 . Soluci´ on a) − → r (θ) = _ e θ cos θ, e θ senθ, he θ _ , curvatura κ = √ 2 e θ (2 + h 2 ) ; torsi´onτ = h e θ (2 + h 2 ) b) _ C − → F d − → r = φ( − → r (θ))[ π/3 π/6 = π 3 + 1 h _ 1 e π/3 − 1 e π/6 _ 5.7. Problemas Resueltos Problemas 1 Calcular la integral de trayectoria _ −→ r x 3 y ds, donde − → r es la trayectoria y = x 2 2 entre los puntos (0, 0) y (2, 2) . Soluci´on Primero, determinemos la ecuaci´ on param´etrica de la trayectoria x = t y = t 2 2 _ _ _ t ∈ [0, 2] ⇐⇒ − → r (t) = _ t, t 2 2 _ Derivando la expresi´ on anterior, queda − → r (t) = (1, t) =⇒ | − → r (t)| = √ 1 + t 2 Adem´ as f(x, y) = x 3 y =⇒ f ( − → r (t)) = 2t A partir de la definici´on de integral de trayectoria tenemos 477 _ −→ r x 3 y ds = _ 2 0 2t √ 1 + t 2 dt = _ 2 3 _ 1 + t 2 _ 3/2 _ 2 0 = 2 3 _ √ 75 −1 _ Problema 2 Dada la funci´ on escalar f(x, y) = 2xy, calcular la integral de trayectoria a lo largo de la curva elipse x 2 9 + y 2 4 = 1 desde el punto (3, 0) hasta (0, 2). Soluci´on Observemos que − → r es el segmento de elipse que est´ a en el primer cuadrante.Entonces al parametrizar la curva queda x = 3 cos t y = 2sent _ t ∈ _ 0, π 2 ¸ ⇐⇒ − → r (t) = (3 cos t, 2sent) Derivando la trayectoria − → r (t) = (−3 sin t, 2 cos t) ⇐⇒ | − → r (t)| = √ 5sen 2 t + 4 Calculemos la funci´on escalar f sobre la trayectoria f(x, y) = 2xy =⇒ f(x (t) , y (t)) = 6 cos tsent Calculemos la integral _ −→ r 2xyds = _ π/2 0 12 cos tsent √ 5sen 2 t + 4dt = _ 4 5 _ 5sen 2 t + 4 _ 3/2 _ π/2 0 = 76 5 Problema 3 Calcular la integral de l´ınea _ −→ r xydx + x 2 dy, donde − → r es la trayectoria x 2 + 4y 2 = 4, x > 0. Soluci´on Primero, escribamos la ecuaci´ on param´etrica de la trayectoria orientada positivamente x = 2 cos t y = sent _ t ∈ _ − π 2 , π 2 ¸ =⇒ − → r (t) = (2 cos t, sent) 478 −→ F(x, y) = _ xy, x 2 _ =⇒ −→ F(x (t) , y (t)) = _ 2 cos tsent, (2 cos t) 2 _ Determinemos el vector − → r = (−2sent, cos t) Calculemos la integral _ −→ r xydx + x 2 dy = _ π/2 −π/2 _ 2 cos tsent, (2 cos t) 2 _ (−2sent, cos t) dt = _ π/2 −π/2 (−4sen 2 t cos t + 4 cos 3 t)dt = _ π/2 −π/2 (−8sen 2 t cos t + 4 cos t)dt = _ − 8 3 sen 3 t + 4sent _π 2 − π 2 = 8 3 Problema 4 Calcular la integral de l´ınea _ −→ r −y 2 dx + xdy, donde − → r es la trayectoria y 2 = 2x −x 2 , tal que x > 1, y > 0. Soluci´on Observemos que − → r es el segmento de circunferencia: y 2 = 2x −x 2 ⇐⇒ (x −1) 2 + y 2 = 1 tal que x > 1, y > 0. Entonces: x = 1 + cos t y = sent _ t ∈ _ 0, π 2 ¸ =⇒ − → r (t) = (1 + cos t, sent) =⇒ − → r (t) = (−sent, cos t) Calculemos el campo vectorial − → F sobre la trayectoria −→ F(x, y) = (−y 2 , x) =⇒ −→ F(x (t) , y (t)) = (−sen 2 t, 1 + cos t) 479 Calculemos la integral _ −→ r −y 2 dx + xdy = _ π/2 0 _ sen 3 t + cos 2 t + cos t _ dt = _ π/2 0 _ _ 1 −cos 2 t _ sent + ( 1 + cos 2t 2 ) + cos t _ dt = _ −cos t + cos 3 t 3 + t 2 + sen2t 4 + sent _ π/2 0 = 1 − 1 3 + π 4 + 1 = 5 3 + π 4 Problema 5 Calcular la integral de l´ınea _ −→ r (8x +z)dx +2xz 2 dy −4y 2 dz, siendo − → r la curva definida por las ecuaciones: z = 9 −2x 2 −4y 2 , z = 1. Soluci´ on Observemos, que la curva contenida en el plano z = 1, es la elipse 2x 2 + 4y 2 = 8,con semi ejes a = 2 y b = √ 2, que se parametriza mediante. x = 2 cos t y = √ 2sent z = 1 _ _ _ t ∈ [0, 2π] =⇒ − → r (t) = _ 2 cos t, √ 2sent, 1 _ t ∈ [0, 2π] Calculemos el campo vectorial − → F sobre la trayectoria −→ F(x, y, z) = (8x + z, 2xz 2 , −4y 2 ) =⇒ −→ F(x (t) , y (t)) = (16 cos t + 1, 4 cos 1t, 1) Evaluemos el vector − → r (t) = _ −2sent, √ 2 cos t, 0 _ luego, obtenemos −→ F(x (t) , y (t)) − → r (t) = (16 cos t + 1, 4 cos t, 1) _ −2sent, √ 2 cos t, 0 _ Entonces la integral de l´ınea es _ −→ r (8x+z)dx+2xz 2 dy−4y 2 dz = _ 2π 0 _ −32sent cos t −2sent + 4 √ 2 cos 2 t _ dt 480 = _ −16sen 2 t + 2 cos t ¸ 2π 0 + 4 √ 2 _ 2π 0 _ 1 + cos 2t 2 _ dt = 4 √ 2 _ t 2 + sen2t 4 _ 2π 0 = 4 √ 2π Problema 6 Calcular el trabajo producido por campo de fuerzas dado por − → F = (3x+ 4y, 2x+3y 2 ), a lo largo de la circunferencia C de radio 2 centrada en el origen y recorrida con orientaci´ on positiva. Soluci´on Definimos el trabajo mediante la integral de l´ınea _ −→ r − → F d − → r = _ b a − → F ( − → r (t)) − → r (t) dt Luego, parametrizando la trayectoria tenemos: − → r (t) = (2cost, 2sent), t ∈ [0, 2π]. =⇒ − → r (t) = (−2sent, 2 cos t) Reemplazando el integrando, queda W = _ (3x + 4y, 2x + 3y 2 , 0) (dx, dy, dz) = _ 2π 0 (6cost + 8sent, 4cost + 12sen 2 t, 0) (−2sent, 2cost, 0)dt = _ 2π 0 [−16sen2t + 8cos2t]dt = −16π + 8π = −8π. Trabajo negativo significa que el campo de fuerza disipa energ´ıa. 5.7.1. Campo conservativo Problemas 1 Sea el campo vectorial − → F : IR 3 →IR 3 dado por − → F (x, y, z) = (x, −y, z). Calcular la integral _ −→ r − → F d − → r 481 a) Si C es la circunferencia x 2 + y 2 = 4 recorrida en el sentido positivo. b) Si C es la recta que une P = (1, 0, 0) con Q = (1, 0, 4). c) Si C es la helicoide − → r (t) = (cos(4πt), sen(4πt), 4t), t ∈ [0, 1] que une P = (1, 0, 0) con Q = (1, 0, 4). Soluci´on a) Las ecuaci´ on param´etrica de la circunferencia de radio 2 centrada en el origen y recorrida en sentido positivo, es − → r (t) = (2 cos t, 2sent, 0) t ∈ [0, 2π] =⇒ − → r (t) = (−2sent, 2 cos t, 0) Entonces, la integral de linea queda _ −→ r − → F d − → r = _ −→ r (x, −y, z) (dx, dy, dz) = _ 2π 0 (2 cos t, −2sent, 0) (−2sent, 2 cot s, 0) dt = _ 2π 0 −8 cos tsentdt = 0 b) La ecuaci´ on param´etrica de la recta que une P = (1, 0, 0) con Q = (1, 0, 4) es: − → r (t) = − → P + ( − → Q − − → P )t = t(1, 0, 4) t ∈ [0, 1] =⇒ − → r (t) = (1, 0, 4) . Entonces _ −→ r − → F d − → r = _ 1 0 (t, 0, 4t) (1, 0, 4) dt = _ 1 0 16tdt = _ 8t 2 ¸ 1 0 = 8 c) A partir de la ecuaci´ on de la helicoide se obtiene − → r (t) = (−4πsen(4πt), 4π cos(4πt), 4) Sustituyendo t´erminos en el integrando, queda _ −→ r − → F d − → r = _ 1 0 (cos(4πt), −sen(4πt), 4t) (−4πsen(4πt), 4π cos(4πt), 4)dt = _ 1 0 (−8πsen(4πt) cos(4πt) + 4t)dt = _ 1 0 16tdt = _ 8t 2 ¸ 1 0 = 8 482 El valor de la integral de l´ınea es el mismo por ambas trayectorias. Problema 2 Calcular la integral _ −→ r 2x cos ydx − x 2 senydy, donde − → r : [1, 2] → IR 2 definida por − → r (t) = _ e t−1 , sen π t _ . Soluci´on. Determinemos si el campo vectorial es conservativo, de modo que calcu- lamos ∇ − → F = ¸ ¸ ¸ ¸ ¸ ¸ ¸ ¸ i j k ∂ ∂x ∂ ∂y ∂ ∂z 2x cos y −x 2 seny 0 ¸ ¸ ¸ ¸ ¸ ¸ ¸ ¸ = (0, 0, 0) Como hallamos que ∇ − → F = − → 0 , entonces − → F tiene una funci´ on potencial φ(x, y) tal que ∂ ∂x φ(x, y) = 2x cos y ∂ ∂y φ(x, y) = −x 2 seny _ ¸ _ ¸ _ Integrando la primera ecuaci´on parcialmente con respecto a x, se tiene φ(x, y) = x 2 cos y +h(y) =⇒ ∂ ∂y φ(x, y) = −x 2 seny +h (y) = −x 2 seny h (y) = 0 ⇐⇒ h(y) = c En consecuencia, la funci´ on potencial φ(x, y) para − → F (x, y) es φ(x, y) = x 2 cos y + c Entonces , podemos afirmar que _ −→ r 2x cos ydx −x 2 senydy = _ −→ r ∇φ d − → r = φ( − → r (2)) −φ( − → r (1)) donde φ( − → r (2)) = φ _ e, sen π 2 _ = e 2 + c φ( − → r (1)) = φ(1, senπ) = 1 + c Por tanto. obtenemos: _ −→ r 2x cos ydx −x 2 senydy = e 2 −1 483 Problema 3 Considere el campo vectorial − → F (x, y, z) en IR 3 definido por : − → F (x, y, z) = _ yz 1 + x 2 y 2 z 2 , xz 1 + x 2 y 2 z 2 , xy 1 + x 2 y 2 z 2 _ Evaluar _ −→ r yzdx + xzdy + xydx 1 + x 2 y 2 z 2 ,donde − → r es: a) el segmento rect´ılineo entre (0, 0, 0) y (1, 1, 1) . b) la intersecci´on de x 2 + y 2 + (z −1) 2 = 1, con x 2 + y 2 + z 2 = 1. Soluci´on Se tiene que las componentes del campo vectorial son continuas ∀ (x, y, z) ∈ IR 3 Primero, verifiquemos si el campo vectorial es conservativo o no. ∇ − → F = ¸ ¸ ¸ ¸ ¸ ¸ ¸ ¸ ¸ i j k ∂ ∂x ∂ ∂y ∂ ∂z yz 1 + x 2 y 2 z 2 xz 1 + x 2 y 2 z 2 xy 1 + x 2 y 2 z 2 ¸ ¸ ¸ ¸ ¸ ¸ ¸ ¸ ¸ = (0, 0, 0) Puesto que ∂ ∂y xy 1 + x 2 y 2 z 2 = ∂ ∂z xz 1 + x 2 y 2 z 2 , ∂ ∂z yz 1 + x 2 y 2 z 2 = ∂ ∂x xy 1 + x 2 y 2 z 2 , etc. = (0, 0, 0) Como hallamos que ∇ − → F = − → 0 , entonces − → F tiene una funci´ on potencial φ(x, y) tal que: ∂ ∂x φ(x, y) = yz 1 + x 2 y 2 z 2 ∂ ∂y φ(x, y) = xz 1 + x 2 y 2 z 2 ∂ ∂z φ(x, y) = xy 1 + x 2 y 2 z 2 _ ¸ ¸ ¸ ¸ ¸ _ ¸ ¸ ¸ ¸ ¸ _ Integrando la primera ecuaci´on parcialmente con respecto a x, se tiene φ(x, y) = arctg (xyx) + h(y, z) =⇒ ∂ ∂y φ(x, y) = xz 1 + x 2 y 2 z 2 + h (y, z) = xz 1 + x 2 y 2 z 2 h (y, z) = 0 ⇐⇒ h(y, z) = g (x) 484 En consecuencia, la funci´ on potencial φ(x, y) para − → F (x, y)es φ(x, y) = arctg (xyx) + g (z) =⇒ φ(x, y) = xy 1 + x 2 y 2 z 2 + g (x) = xy 1 + x 2 y 2 z 2 g (x) = 0 ⇐⇒ g (x) = c Entonces , podemos concluir que φ(x, y) = arctg (xyx) + c En este caso hallamos , que el valor de la integral _ −→ r yzdx + xzdy + xydx 1 + x 2 y 2 z 2 = _ −→ r ∇φ d − → r = φ(1, 1, 1) −φ(0, 0, 0) = arctg (1) −arctg (0) = π 4 Si − → r es la intersecci´on de dos esferas la curva resultante es cerrada, en consecuencia _ −→ r yzdx + xzdy + xydx 1 + x 2 y 2 z 2 = _ −→ r ∇φ d − → r = 0 5.7.2. Teorema de Green Problema 1 Verificar el teorema de Green para el campo vectorial − → F (x, y, z) = (2(x 2 + y 2 ), (x + y) 2 ) ,donde las curvas frontera de la regi´on D corresponden al contorno del tri´angulo con v´ertices en los puntos (1, 1) , (2, 2) ,y (1, 3) orientado positivamente. Soluci´on Como el campo vectorial − → F (x, y) es de clase C 1 , y la regi´ on D conexa, entonces el teorema de Green afima que: _ C Pdx + Qdy = _ _ D _ ∂ ∂x Q− ∂ ∂y P _ dxdy Identificando t´erminos, tenemos que P (x, y) = 2(x 2 + y 2 ) =⇒ ∂P ∂y = 4y 485 Q(x, y) = (x + y) 2 =⇒ ∂Q ∂x = 2(x + y) Entonces calculemos __ D _ ∂Q ∂x − ∂P ∂y _ dxdy = __ D 2 (x −y) dxdy donde D = ¦(x, y) ∈ IR 2 : 1 ≤ x ≤ 2, x ≤ y ≤ 4 −x¦ . Luego __ D 2 (x −y) dxdy = _ 2 1 _ 4−x x 2 (x −y) dydx = _ 2 1 _ xy − y 2 2 _ 4−x x dx = _ 2 1 _ 2x(4 −x) −(4 −x) 2 −2x 2 + x 2 _ dx = −4 _ 2 1 (x −2) 2 dx = −4 _ (x −2) 3 3 _ 2 1 = − 4 3 Calculemos directamente la integral de l´ınea, segmentando la frontera en tres curvas: _ C Pdx + Qdy = _ C 1 Pdx + Qdy + _ C 2 Pdx + Qdy + _ C 3 Pdx + Qdy Parametricemos los segmentos de curvas que unen los puntos (1, 1) y (2, 2) ; (2, 2) y (1, 3) ; (1, 3) y (1, 1) Ac´a debe ir gr´afico Sea C 1 la recta y = x, 1 ≤ x ≤ 2 =⇒ − → r (t) = (t, t) , t ∈ [1, 2] =⇒ − → r (t) = (1, 1) , t ∈ [1, 2] entonces: 486 _ C 1 2(x 2 + y 2 )dx + (x + y) 2 dy = _ 2 1 _ 2 _ t 2 + t 2 _ + (2t) 2 ¸ dt = _ 2 1 8t 2 dt = 8 _ t 3 3 _ 2 1 = 56 3 Sea C 2 la recta y = 4 −x, 1 ≤ x ≤ 2 =⇒ − → r (t) = (4 −t, t) , t ∈ [2, 3] =⇒ − → r (t) = (−1, 1) , t ∈ [2, 3] , entonces: _ C 2 2(x 2 + y 2 )dx + (x + y) 2 dy = _ 3 2 _ 2 _ (4 −t) 2 + t 2 _ (−1) + (4) 2 ¸ dt = _ 3 2 _ −2 _ 16 −8t + 2t 2 _ + 16 ¸ dt = −4 _ 3 2 _ t 2 −4t + 4 ¸ dt = −4 _ 3 2 [t −2] 2 dt = −4 _ (t −2) 3 3 _ 3 2 = − 4 3 Sea C 3 la recta x = 1, 1 ≤ y ≤ 3 =⇒ − → r (t) = (1, 3 −t) , t ∈ [0, 2] =⇒ − → r (t) = (0, −1) , t ∈ [0, 2] , entonces: _ C 3 2(x 2 + y 2 )dx + (x + y) 2 dy = _ 2 0 (4 −t) 2 (−1)dt = _ (4 −t) 3 3 _ 2 0 = 8 3 − 64 3 = − 56 3 Por lo tanto, al sumar los tres t´erminos tenemos: _ C 2(x 2 + y 2 )dx + (x + y) 2 dy = 56 3 − 4 3 − 56 3 = − 4 3 Lo que muestra la validez de la formula del teorema de Green. 487 Problema 2 Verificar el teorema de Green para _ C x 2 ydx + xy 2 dy, donde C es la frontera de la regi´ on R en el primer cuadrante, limitada por las gr´ aficas de y = x, y 3 = x 2 . Soluci´on. Primero, calculemos la integral de l´ınea considerando la orientaci´ on pos- itiva de la frontera, dividiendola en dos segmentos C 1 y = x y C 2 y 3 = x 2 . Determinemos los puntos que se intersectan ambas curvas: y = x y 3 = x 2 _ =⇒ x 3 = x 2 =⇒ x 2 (x −1) = 0 ⇐⇒ x = 0 y x = 1 Luego, ambas curvas se intersectan en los puntos (0, 0) y (1, 1) .En consecuencia la regi´on R queda delimitada por R = _ (x, y) ∈ IR 2 /0 ≤ x ≤ 1, x ≤ y ≤ x 2/3 _ Parametrizando el segmento de curva C 1 tenemos: C 1 : − → r 1 (t) = (t, t) , t ∈ [0, 1] =⇒ − → r 1 (t) = (1, 1) Calculemos el campo vectorial sobre la curva C 1 − → F ( − → r 1 (t)) = _ t 3 , t 3 _ =⇒ − → F ( − → r 1 (t)) − → r 1 (t) = _ t 3 , t 3 _ (1, 1) = _ t 3 + t 3 _ = 2t 3 Para C 2 encontramos C 2 : − → r 2 (t) = _ 1 −t, (1 −t) 2/3 _ , t ∈ [0, 1] =⇒ − → r 1 (t) = _ −1, − 2 3 (1 −t) −1/3 _ Luego, la funci´ on compuesta para el campo sobre C 2 es: − → F ( − → r 2 (t)) = _ (1 −t) 8/3 , (1 −t) 7/3 _ =⇒ − → F ( − → r 1 (t)) − → r 1 (t) = _ (1 −t) 8/3 , (1 −t) 7/3 _ _ −1, − 2 3 (1 −t) −1/3 _ = −(1 −t) 8/3 − 2 3 (1 −t) 2 Entonces la integral de l´ınea queda _ C x 2 ydx + xy 2 dy = _ 1 0 2t 3 dt − _ 1 0 ((1 −t) 8/3 + 2 3 (1 −t) 2 )dt = _ t 4 4 + 3(1 −t) 11/3 11 + 2(1 −t) 3 9 _ 11 0 = 1 4 − 3 11 − 2 9 = 1 198 488 Por otra parte el campo vectorial − → F (x, y) de clase C 1 , es decir campo con- tinuo con primera derivada continua,definido en la regi´on R conexa,acotado por una frontera cerrada, entonces podemos aplicar el teorema de Green que afima: _ C x 2 ydx + xy 2 dy = _ _ R _ ∂ ∂x (x 2 y) − ∂ ∂y (xy 2 ) _ dxdy donde R = _ (x, y) ∈ IR 2 /0 ≤ x ≤ 1, x ≤ y ≤ x 2/3 _ ,entonces: _ _ R _ ∂ ∂x (x 2 y) − ∂ ∂y (xy 2 ) _ dxdy = _ 1 0 _ x 2/3 x _ y 2 −x 2 ¸ dydx = _ 1 0 _ y 3 3 −x 2 y _ x 2/3 x dx = _ 1 0 _ x 2 3 −8x 8/3 + 2 3 x 3 _ dx = _ x 3 9 − 3 11 x 11/3 + x 4 6 _ 1 0 = 1 9 − 3 11 + 1 6 = 1 198 Con esto,verificamos el teorema de Green en este caso particular. Problema 3 Calcule la integral _ C −y 2x 2 + 3y 2 dx+ x 2x 2 + 3y 2 dy a lo largo de la curva C formada por los lados del cuadrado con v´ertices en (1, 1) , (−1, 1) , (−1, −1) , (1, −1) . Soluci´ on. Claramente vemos que el campo vectorial − → F (x, y) en la regi´ on acotada por C no es continuo, con primeras derivadas parciales continuas en el origen (0, 0) . Luego, vamos a excluir el origen de la regi´ on . Dado que la regi´on envuelta por la curva C, que excluye la singularidad, no es simplemente conexa, se tiene que: _ C Pdx + Qdy + _ C 1 Pdx + Qdy = __ D _ ∂Q ∂x − ∂P ∂y _ dxdy donde la curva C 1 es la elipse con ecuaci´on 2x 2 + 3y 2 = r 2 , orientada en el sentido horario, con normal apuntando hacia fuera de la regi´on D. 489 Por otra parte. ∂Q ∂x = 3y 2 −2x 2 (2x 2 + 3y 2 ) 2 ∂P ∂y = 3y 2 −2x 2 (2x 2 + 3y 2 ) 2 _ ¸ ¸ _ ¸ ¸ _ =⇒ _ ∂Q ∂x − ∂P ∂y _ = 0 Entonces, tenemos _ C Pdx + Qdy + _ C 1 Pdx + Qdy = __ D _ ∂Q ∂x − ∂P ∂y _ dxdy = 0 =⇒ _ C Pdx + Qdy = − _ C 1 Pdx + Qdy =⇒ _ C Pdx + Qdy = _ C − 1 Pdx + Qdy Parametrizando C − 1 como − → r (t) = _ r √ 2 cos t, r √ 3 sent _ con 0 ≤ t ≤ 2π Se obtiene _ C − 1 Pdx + Qdy = _ 2π 0 _ − rsent √ 3r 2 _ − r √ 2 sent _ + r √ 2r 2 cos t _ r √ 3 cos t __ dt = 1 √ 6 (2π) = _ 2 3 π Por lo tanto, la integral _ C Pdx + Qdy = _ 2 3 π Problema 4 Sea C una curva cerrada simple que encierra una regi´on D = _ (x, y) ∈ IR 2 / x 2 4 + y 2 5 = 1 _ Calcular el ´ area del interior de la elipse usando el teorema de Green. Soluci´on. 490 A partir del teorema de Green tenemos A(D) = 1 2 _ C xdy −ydx Parametricemos la ecuaci´on de la elipse , mediante − → r (t) = (2cos(t), √ 5sen(t)), 0 ≤ t ≤ 2π. Entonces x(t) = 2cos(t), y(t) = √ 5sen(t), luego dx = −2sen(t), dy(t) = √ 5 cos(t) Reemplazando t´erminos en el integrando 1 2 _ C xdy −ydx = 1 2 _ 2π 0 (2 √ 5cos 2 (t) + 2 √ 5sen 2 (t))dt = √ 5 _ 2π 0 dt = 2π √ 5 Problema 5 Considere la regi´ on R del plano x 2 +(y −a) 2 ≤ a 2 ; x 2 +y 2 ≥ 2a 2 y usando el teorema de Green, verifique que el ´ area de dicha regi´ on coincide con el ´ area de un cuadrado de lado a. Soluci´on La curva C 1 descrita por la ecuaci´ on x 2 + (y −a) 2 = a 2 ,corresponde a la circunferencia con centro en (0, a) y radio a y la curva C 2 es la ecuaci´ on x 2 + y 2 = 2a 2 de la circunferencia con centro en (0, 0) y radio a √ 2. Calculemos los puntos de intersecci´ on de ambas curvas, igualando ambas ecuaciones, produce 2a 2 −2ay = 0 =⇒ y = a Sustituyendo este resultado en la segunda ecuaci´on, obtenemos x 2 = a 2 ⇐⇒ x = ±a Por lo tanto, los puntos de intersecci´ on de ambas curvas son P 1 = (−a, a) y P 2 = (a, a) que tienen coordenadas polares (a √ 2, 3π/4) y (a √ 2, π/4) respectivamente. En consecuencia, la curva cerrada C que forma la frontera de R es la uni´ on 491 de la curva C 1 parametrizada por: x(t) = acost, y(t) = a + asent, donde t ∈ [0, π] y de la curva C 2 parametrizada por x(t) = a √ 2cost, y(t) = a √ 2sent, dondet ∈ [π/4, 3π/4]. El teorema de Green afirma que A(R) = 1 2 _ C xdy −ydx donde la orientaci´on de C es positiva. A(R) = 1 2 _ C 1 xdy −ydx + 1 2 _ C 2 xdy −ydx Las orientaciones de C 1 y C 2 , van en sentido opuesto a los punteros del reloj, para que C tenga orientaci´ on positiva. Entonces A(R) = 1 2 _ π 0 (a 2 sent + a 2 )dt + 1 2 _ π/4 3π/4 (2a 2 )dt = 1 2 _ a 2 [−cos t + t] π 0 + 2a 2 [t] π/4 3π/4 _ = 1 2 _ 2a 2 + πa 2 −πa 2 _ = a 2 Resultado que verifica que el ´ area de la regi´on R es igual a la de un cuadrado de lado a. 5.8. Problemas propuestos 5.8.1. Integral de trayectoria 1.- Calcular el ´ area de la superficie lateral cuya base est´ a sobre la curva − → r del plano XY y la altura en cada (x,y) es z = f (x, y) ,donde: a) f(x, y) = xy , − → r : x 2 + y 2 = 1 desde (1, 0) hasta (0, 1) . b) f(x, y) = h , − → r : y = 1 −x 2 desde (1, 0) hasta (0, 1) . c) f(x, y) = xy, − → r : y = 1 −x 2 desde (1, 0) hasta (0, 1) . Respuestas 492 a) A(S) = 1 2 , b)A(S) = h 4 _ 2 √ 5 + ln _ 2 + √ 5 _¸ , c)A(S) = 1 120 _ 25 √ 5 −11 ¸ 2.- Evaluar las siguientes integrales de trayectoria _ −→ r f (x, y, z) ds a) f(x, y, z) = e √ z , − → r : [0, 1] →IR 3 dada por − → r (t) = (1, 2, t 2 ) . b) f(x, y, z) = yz, − → r : [1, 3] →IR 3 dada por − → r (t) = (t, 3t, 2t) . c) f(x, y, z) = x + y y + z , − → r : [1, 2] →IR 3 dada por − → r (t) = _ t, 2 3 t 3/2 , t _ . d) f(x, y, z) = z, − → r : [0, 2π] →IR 3 dada por − → r (t) = (tcost, tsent, t) . Respuestas a) _ −→ r e √ z ds = 2, b) _ −→ r yzds = 52 √ 14, c) _ −→ r x + y y + z ds = 16 3 −2 √ 3 d) _ −→ r zds = 1 3 _ _ (2 + 4π 2 ) 3 −2 √ 2 _ 3.- Evaluar las siguientes integrales de trayectoria _ −→ r f (x, y) ds a) f(x, y) = 8y 2 −5x, la curva C dada por x 2/3 + y 2/3 = 1. b) f(x, y) = x + y, siendo C un tri´ angulo de v´ertices (0, 0), (1, 0) y (0, 1), recorrido en sentido antihorario. Respuestas a) _ C (8y 2 −5x)ds = 12 b) _ C (x + y)ds = 1 + √ 2 5.8.2. Integral de l´ınea 4.- Evaluar las siguientes integrales de l´ınea: a) _ −→ r xdx + ydy + zdz, donde − → r : [−1, 2] → IR 3 dada por − → r (t) = (t 2 , 3t, 2t 3 ) . b) _ −→ r 2xyzdx + x 2 zdy + x 2 ydz, donde − → r es una curva simple orientada que une (1, 1, 1) con (1, 2, 4). c) _ −→ r x 2 zdx + 6ydy + yz 2 dz, donde − → r : [1, 3] →IR 3 dada por − → r (t) = (t, t 2 , ln t) . d) _ C xydx +(x −y)dy, donde C esta formado por los segmentos de recta que van desde (0, 0) a (2, 0) y de (2, 0) a (3, 2) . e) _ C (x+2)dx+3zdy+y 2 dz,siendo C la curva intersecci´on de las superficies x 2 + y 2 + z 2 = 1; z = x −1 493 Respuestas. a) _ −→ r xdx + ydy + zdz = 147 b) _ −→ r 2xyzdx + x 2 zdy + x 2 ydz = 7 c) _ −→ r x 2 zdx + 6ydy + yz 2 dz = 249, 49 d) _ C xydx + (x −y)dy = 17 3 e) _ C (x + 2)dx + 3zdy + y 2 dz = 3 √ 2 4 π 5.- Evaluar la integral de l´ınea de − → F a lo largo de cada una de las siguientes trayectorias. a) − → F (x, y) = (x 2 −2xy, y 2 −2xy),a lo largo de la par´ abola y = x 2 desde (−1, 1) a (1, 1) b) − → F (x, y, z) = (x, y, xz −y) sobre el segmento de recta desde el punto (0, 0, 0) hasta el punto (1, 2, 4). c) − → F (x, y) = _ x+y x 2 +y 2 , x+y x 2 +y 2 _ a lo largo de la circunferencia x 2 + y 2 = a 2 recorrida en sentido positivo. d) − → F (x, y) = _ x √ 1+x 2 +y 2 , y √ 1+x 2 +y 2 _ sobre el cuarto de elipse x 2 a 2 + y 2 b 2 = 1 situado en el primer cuadrante, desde el punto (a, 0) hasta el punto (0, b). e) − → F (x, y, z) = (yz, xz, xy) a lo largo de la h´elice − → r (t) = (a cos t, asent, bt) entre los puntos (a, 0, 0) y (a, 0, 2bπ). Respuestas a) _ −→ C − → F d − → r = − 14 15 b) _ −→ C − → F d − → r = 23 6 c) _ −→ C − → F d − → r = 0 d) _ −→ C − → F d − → r = √ 1 + b 2 − √ 1 + a 2 e) _ −→ C − → F d − → r = 0 5.8.3. Campos conservativos 6.- Pruebe que la integral _ −→ C (6xy 2 − y 3 )dx + (6x 2 y − 3xy 2 )dy ; es inde- pendiente del camino que une los puntos (1, 2) con (3, 4). a) Calcule el valor de la integral parametrizando el segmento. b) Verifique el resultado anterior utilizando la funci´ on potencial. Respuestas a) _ 1 0 − → F (r (t)) . − → r (t) dt = 236 b) − → F = ∇φ(x, y, z) = 3x 2 y 2 −xy 3 + C 494 7.- ¿Para qu´e valores de a ∈ IR el campo vectorial − → F (x, y, z) = (axy − z 3 ; (a−2)x 2 ; (1−a)xz 2 ) es conservativo? Para esos valores, calcule la funci´ on potencial. Respuesta Si a=4 la funci´ on potencial del campo − → F es φ(x; y; z) = 2x 2 y −xz 3 +C. 8.- En los casos en los que − → F sea conservativo, halle la correspondiente funci´ on potencial. a) − → F (x, y, z) = (10xz 3 + 1, −6y 2 , 15x 2 z 2 ). b) − → F (x, y, z) = (2xy 3 , x 2 z 3 , 3x 2 yz 2 ) . Respuestas a) − → F es conservativo, funci´ on potencial: φ(x, y, z) = 5x 2 z 3 +x −2y 3 +C b) − → F no es conservativo. 9.- Sea − → F (x, y, z) = (z 3 + 2xy, x 2 , 3xz 2 ). a) Muestre que − → F es un campo conservativo. b) Calcule directamente la integral de − → F alrededor del contorno del cuadra- do conv´ertices (±1, ±1) . c) Determine la funci´ on potencial. 5.8.4. Teorema de Green 10.- Usar el teorema de Green para hallar el valor de la integral de l´ınea: a) _ −→ r xy 2 dx + x 2 ydy, donde − → r : [0, 2π] → IR 2 dada por − → r (t) = (4 cos t, 2sent) b) _ −→ r xydx + x 2 dy, donde − → r es la frontera que limita la regi´ on y ≥ x 2 , y ≤ x c) _ C (arctgx + y 2 )dx + (e y −x 2 )dy, donde C es la frontera que encierra la regi´ on 1 ≤ x 2 + y 2 ≤ 9, y ≥ 0 d) _ C 2xydx + (x 2 + 2x)dy, donde Ces la frontera que limita la regi´on interior a la elipse x 2 9 + y 2 4 = 1 y exterior a la circunferencia x 2 + y 2 = 1 e) _ C (x 2 + y 2 ) dx + (2x + y 2 ) dy, donde C es la frontera del cuadrado con v´ertices (0, 0) , (k, 0) , (0, k) , (k, k) . Respuestas a) _ −→ r xy 2 dx + x 2 ydy = 0 b) _ −→ r xydx + x 2 dy = 1 12 495 c) _ −→ r (arctgx + y 2 )dx + (e y −x 2 )dy = − 104 3 d) _ −→ r 2xydx + (x 2 + 2x)dy = 10π e) _ C (x 2 + y 2 ) dx + (2x + y 2 ) dy = k 2 (2 −k) 10.- Demuestre que _ C −y x 2 + y 2 dx+ x x 2 + y 2 dy = 2π, donde C es cualquier curva cerrada , simple , que incluya el origen 11.- Calcular el ´ area de una elipse con semi ejes a, b usando el teorema de Green. Respuesta A(R) = πab. 12.- Hallar el ´ area acotada por el arco de hipocicloide x = a (t −sent) , y = a (1 −cos t) ,donde a > 0, y 0 ≤ t ≤ 2π, y el eje x. Respuesta A(R) = 3πa 2 5.9. Autoevaluaciones Autoevaluaci´ on N o 1 Tiempo 2 horas Pregunta 1 Sea − → F (x, y) el campo de fuerzas definido por − → F (x, y) = (x 2 + 7y, −x + y sin y 2 ) .Calcular _ C − → F d − → r sobre la frontera del tri´angulo de v´ertices (0, 2) , (0, 0) y (1, 0) us- ando la definici´ on de integral de l´ınea. Pregunta 2 Evalue la integral _ C _ 1 + e √ x _ dx + (cos y 2 + x 2 ) dy, siendo C una curva cerrada, ubicada en el primer cuadrante, formada por los arcos de circunferencia de radios 1 y 2 respectivamente y por los segmentos rectos 1 ≤ x ≤ 2, 1 ≤ y ≤ 2. Pregunta 3 Calcular la integral I = _ C − → F d − → r ,si − → F = (x, y, z) es un campo vectorial definido en R 3 C es dada por y 2 + z 2 = 1, x = z entre A = (1, 0, 1) y B = (0, 1, 0) . Pauta de Correcci´ on 496 Pregunta 1 La circulaci´on a lo largo de la frontera del tri´ angulo es la suma de las circulaciones en cada uno de los lados _ C − → F d − → r = _ C 1 − → F d − → r + _ C 2 − → F d − → r + _ C 3 − → F d − → r Parametrizando cada uno de los lados del tri´ angulo se tiene C 1 : y = 0, − → r 1 (t) = (t, 0) , t ∈ [0, 1] =⇒ − → r 1 (t) = (1, 0) y − → F ( − → r 1 (t)) = _ t 2 , −t _ C 2 : y = 2 (1 −x) , − → r 2 (t) = (−t, 2 (1 + t)) , t ∈ [−1, 0] =⇒ − → r 2 (t) = (−1, 2) y − → F ( − → r 1 (t)) = _ t 2 + 14 (1 + t) , t + 2 (1 + t) sin (2 (1 + t)) 2 _ C 3 : x = 0, − → r 3 (t) = (0, −t) , t ∈ [−2, 0] =⇒ − → r 3 (t) = (0, −1) y − → F ( − → r 1 (t)) = _ −7t, −t sin (−t) 2 _ Sustituyendo t´erminos en los integrandos , obtenemos _ C − → F d − → r = _ 1 0 t 2 dt + _ 0 −1 _ −t 2 −14 (1 + t) + 2t + 4 (1 + t) sin (2 (1 + t)) 2 ¸ dt + _ 0 −2 t sin t 2 dt = _ t 3 3 _ 1 0 + _ − t 3 3 −14t −7t 2 + t 2 − cos (2 (1 + t)) 2 2 _ 0 −1 + _ − cos t 2 2 _ 0 −2 = −8 Pregunta 2 De acuerdo con el teorema de Green tenemos _ C _ 1 + e √ x _ dx + _ cos y 2 + x 2 _ dy = __ D _ ∂ ∂x _ cos y 2 + x 2 _ − ∂ ∂y _ 1 + e √ x _ _ dxdy = __ D 2xdxdy 497 Cambiando a coordenadas polares ,queda __ D 2xdxdy = 2 __ D ∗ (r cos θ) rdrdθ = 2 _ π/2 0 _ 2 1 _ r 2 cos θ _ drdθ = 2 _ π/2 0 _ r 3 3 _ 2 1 cos θdθ = 14 3 [sin θ] π/2 0 = 14 3 Pregunta 3 Como el rotacional del campo − → F : ∇ − → F = ¸ ¸ ¸ ¸ ¸ ¸ i j k ∂ ∂x ∂ ∂y ∂ ∂z x y z ¸ ¸ ¸ ¸ ¸ ¸ = (0, 0, 0) es nulo el campo es conservativo =⇒ existe φ(x, y, z) tal que − → F = ∇φ(x, y, z) Es decir ∂φ ∂x = x, ∂φ ∂y = y, ∂φ ∂z = z. Integrando parcialmente la primera componente con respecto a x, obtenemos φ(x, y, z) = x 2 2 + h(y, z) =⇒ ∂φ ∂y (x, y, z) = ∂h ∂y = y =⇒ h(y, z) = y 2 2 + g (z) De modo que φ(x, y, z) = x 2 2 + y 2 2 + g (z) =⇒ ∂φ ∂z (x, y, z) = g (z) = z Por tanto, el potencial es φ(x, y, z) = x 2 2 + y 2 2 + z 2 2 y la integral de l´ınea queda _ C − → F d − → r = _ C xdx + ydy + zdz = φ(B) −φ(A) = 1 2 −1 = − 1 2 Autoevaluaci´ on N o 2 Tiempo 2 horas Pregunta 1. 498 Calcular el trabajo realizado por el campo de fuerza − → F (x, y, z) = (x, y, z) al mover una part´ıcula a lo largo de la parab´ ola y = x 2 , z = x,desde x = 0 hasta x = 2. Pregunta 2. Verifique el teorema de Green para el campo − → F : IR 2 →IR 2 dado por − → F (x, y) = (−x 2 y, xy 2 ) sobre la regi´ on R x 2 + y 2 ≤ a 2 . Pregunta 3. Calcular _ C 2xydx + x 2 dy si C es la curva x 2 4 − y 2 9 = 1 entre los puntos A = _ 4, 3 √ 3 _ y B = (2, 0) . Pauta de Correcci´ on Pregunta 1. El trabajo realizado por el campo de fuerza − → F se define por la integral de trayectoria W = _ C − → F d − → r donde C es la trayectoria, que en este caso se describe por la ecuaci´ on param´etrica − → r (t) = (t, t 2 , t) t ∈ [0, 2] =⇒ − → r (t) = (1, 2t, 1). Adem´ as al evaluar el campo sobre la trayectoria queda − → F ( − → r (t)) = (t, t 2 , t) Entonces, el valor del trabajo es _ 2 0 − → F ( − → r (t)) − → r (t)dt = _ 2 0 _ t + 2t 3 + t _ dt = 12 Pregunta 2. La frontera de R orientada positivamente es la curva C, dada por la circunferencia x 2 + y 2 = a 2 , que puede parametrizar como: x = a cos t, y = a sin t, 0 ≤ t ≤ 2π Entonces, por una parte se tiene 499 _ C − → F d − → r = _ C −x 2 ydx + xy 2 dy = _ 2π 0 (a 4 cos 2 t sin 2 t + a 4 cos 2 t sin 2 t)dt = _ 2π 0 (2a 4 cos 2 t sin 2 t)dt = _ 2π 0 a 4 2 sin 2 2tdt = πa 4 2 Por otra parte _ _ R _ ∂(xy 2 ) ∂x − ∂ (−x 2 y) ∂y _ dxdy = _ _ R (y 2 + x 2 )dxdy = _ 2π 0 _ a 0 r 2 rdrdθ = _ 2π 0 _ r 4 4 _ a 0 dθ = πa 4 2 Pregunta 3 M´etodo 1 Como f (x, y) = 2xy y g (x, y) = x 2 verifican condiciones de continuidad y derivabilidad , y ∂g ∂x = 2x = ∂f ∂y , entonces existe potencial φ(x, y) = x 2 y+C tal que: _ C 2xydx + x 2 dy = φ(B) −φ(A) = (0 + C) − _ 48 √ 3 + C _ = −48 √ 3 M´etodo 2 Parametrizando la curva como x (t) = 2 cosh t, y (t) = 3 sinh t 500 Autoevaluaci´ on N o 3 Tiempo 2 horas Pregunta 1 Calcular _ C (2xy 3 + yz)dx + (3x 2 y 2 + xz)dy +xydz si C es determinada por a) − → r (t) = (t, t 2 , t 3 ) desde A = (1, 1, 1) hasta B = (2, 4, 8) . b) x 2 + y 2 + z 2 = 2, z = x 2 + y 2 . Pregunta 2 a) Probar que el ´area de una regi´on D limitada por curva regular ( o curvas regulares por tramos) es dada por A(D) = _ C xdy. b) Aplicar la f´ormula anterior para calcular el ´area de D limitada por las curvas y = x +2, y = x 2 Pregunta 3 Calcular _ C zdx +xdy +ydz si C es la curva resultante de la intersecci´ on de las superficies x 2 + y 2 + z 2 = 4, z = 1. Pauta de Autoevaluaci´ on Pregunta 1 Las componentes del campo vectorial − → F son: f (x, y, z) = 2xy 3 + yz; g (x, y, z) = 3x 2 y 2 + xz; h(x, y, z) = xy continuas con primeras derivadas continuas Como el rotacional del campo − → F , tiene derivadas parciales continuas y ∇ − → F = ¸ ¸ ¸ ¸ ¸ ¸ i j k ∂ ∂x ∂ ∂y ∂ ∂z 2xy 3 + yz 3x 2 y 2 + xz xy ¸ ¸ ¸ ¸ ¸ ¸ = (0, 0, 0) es nulo entonces es conservativo =⇒ existe φ(x, y, z) tal que − → F = ∇φ(x, y, z) Es decir ∂φ ∂x = 2xy 3 + yz, ∂φ ∂y = 3x 2 y 2 + xz, ∂φ ∂z = xz. De la primera de estas expresiones se obtiene que φ(x, y, z) = _ (2xy 3 + yz)dx + h(y, z) φ(x, y, z) = x 2 y 3 + xyz + h(y, z) =⇒ Derivando esta expresi´ on con respecto a y y usando las dos expresiones restantes para φ, tenemos ∂φ ∂y (x, y, z) = 3x 2 y 3 + xz + ∂h ∂y = 3x 2 y 3 + xz =⇒ h(y, z) = g (z) De modo que φ(x, y, z) = x 2 y 3 + xyz + g (z) =⇒ 501 Derivando esta expresi´ on con respecto a z ∂φ ∂z (x, y, z) = xy + g (z) = xy =⇒ g (z) = C Entonces una funci´on potencial para − → F es φ(x, y, z) = x 2 y 3 + xyz + C Finalmente la integral _ C (2xy 3 + yz)dx + (3x 2 y 2 + xz)dy + xydz = φ(B) −φ(A) = 318 b) En este caso _ C (2xy 3 + yz)dx + (3x 2 y 2 + xz)dy + xydz = 0 pues el campo es conservativo y la curva C es cerrada Pregunta 2 a) El teorema de Green asegura que _ C P (x, y) dx + Q(x, y) dy = __ D _ ∂Q(x, y) ∂x − ∂P (x, y) ∂y _ dxdy Sea P (x, y) = 0 y Q(x, y) = x en D ,usando la expresi´ on anterior se obtiene _ C xdy = __ D (1) dxdy = A(D) a) Calculemos la circulaci´ on , siendo C = C 1 ∪ C 2 la curva que limita a regi´ on D. Parametrizando las curvas se tiene C 1 : − → r 1 (t) = _ t, t 2 _ , −1 ≤ t ≤ 2 =⇒ y (t) = t 2 y y (t) = 2t C 2 : − → r 2 (t) = (t, t + 2) , −1 ≤ t ≤ 2 =⇒ y (t) = t + 2 y y (t) = 1 502 Aplicando la f´ ormula anterior A(D) = _ C xdy = _ 2 −1 t (2t) dt + _ 1 2 tdt = _ 2 3 t 3 _ 2 −1 + _ t 2 2 _ 1 2 = 9 2 Pregunta 3 Si C es la curva resultante de la intersecci´on de las superficies x 2 +y 2 +z 2 = 4, z = 1 =⇒ x 2 + y 2 = 3. Determinemos la ecuaci´on param´etrica de la trayectoria C : − → r (t) = _ √ 3 cos t, √ 3 sin t, 1 _ , 0 ≤ t ≤ 2π − → r (t) = _ − √ 3 sin t, √ 3 cos t, 0 _ Calculemos la integral de l´ınea _ C zdx + xdy + ydz = _ 2π 0 (− √ 3 sin t + 3 cos 2 t + 0)dt = 3 _ 2π 0 cos 2 tdt = 3π 503 Cap´ıtulo 6 Integrales de superficie La integral de superficie generaliza la integral doble, tal como la integral de l´ınea generaliza la integral de Riemann vista en el primer curso de C´ alculo. Para ver el significado de esta integral sigamos el siguiente razonamiento: S es una superficie definida por z = g(x, y) y R la regi´on proyecci´on de S en el plano xy. Supongamos que g, g x , g y son continuas sobre R. Sea P una partici´ on de R tal que cada subrect´ angulo R i generado por la partici´on es la proyecci´on un S i porci´on de la superficie S generada por la partici´ on P. Sea ´S i el ´ area de S i Sea adem´ as f : S −→IR una funci´ on continua. Definimos en t´erminos de las sumas intermedias de Riemann la integral de superficie de f sobre S como __ S f(x, y, z)dS = l´ım p→0 n i,j=1 f(x i , y i , z i )´S i Para el c´alculo de ´esta integral: Sea S superficie definida por z = g(x, y) y R regi´ on proyecci´on de S en el plano xy. Suponiendo g, g x , g y son continuas sobre R y f funci´ on continua sobre S, entonces __ S f(x, y, z)dS = __ R xy f(x, y, g(x, y)) _ 1 + [g x ] 2 + [g y ] 2 dA De manera similar. Si S se expresa de la forma x = g(y, z), (y, z) ∈ R yz entonces 504 __ S f(x, y, z)dS = __ R yz f(g(y, z), y, z) _ 1 + [g y ] 2 + [g z ] 2 dA Si S se expresa de la forma y = g(x, z), (x, z) ∈ R xz entonces __ S f(x, y, z)dS = __ R xz f(x, g(x, z), z) _ 1 + [g x ] 2 + [g z ] 2 dA Ejemplo 1: Eval´ ue __ S xydS S : z = 9 −x 2 , 0 ≤ x ≤ 2, 0 ≤ y ≤ x Figura 6.1: Soluci´on: __ S xydS = __ R xy xy √ 1 + 4x 2 dA = _ 2 0 _ x 0 xy √ 1 + 4x 2 dydx ¡No por aqu´ı! 505 __ S xydS = __ R xy xy √ 1 + 4x 2 dA = _ 2 0 _ 2 y xy √ 1 + 4x 2 dxdy = 391 √ 17 + 1 240 Observaci´ on. 1.- Si f(x, y, z) = 1 =⇒ ´ Area de S = __ S dS. = __ R _ 1 + [g x ] 2 + [g y ] 2 dA 2.- Si f(x, y, z) = ρ(x, y, z) densidad de una l´amina ,entonces Masa de l´ amina = __ S ρ(x, y, z)dS Ejemplo 2: Calcule el ´ area lateral del cono z 2 = x 2 + y 2 entre los planos z = 1 y z = 4 usando integral de superficie. Figura 6.2: Cono z 2 = x 2 + y 2 acotado por los planos z = 1 y z = 4 Soluci´on. En primer lugar calculemos el integrando _ 1 + [g x ] 2 + [g y ] 2 donde 1 + [g x ] 2 + [g y ] 2 = 1 + x 2 x 2 + y 2 + y 2 x 2 + y 2 = 2 entonces ´ Area de S = __ S dS = __ √ 2 R dA 506 Cambiando a coordenadas polares, se tiene: __ √ 2 R dA = √ 2 _ 2π 0 _ 4 1 rdrdθ = √ 2 _ 2π 0 _ r 2 2 _ 4 0 dθ = 8 √ 2 _ 2π 0 dθ = 16 √ 2π 6.1. Superficie orientada Decimos que una superficie es orientada si se puede definir un vector normal unitario − → N en todo punto de la superficie que no pertenezca a la frontera de forma tal que lo vectores normales var´ıan de forma continua sobre la superficie S. Figura 6.3: Superficie orientada Sea S una superficie descrita por z = g(x, y) . Hay dos vectores normales unitarios a S en (x o , y o , g(x o , y 0 )) a saber, si ponemos G(x, y, z) = z −g(x, y) entonces − → N = ∇G |∇G| o sea − → N = −g x ´ i −g y ´ j + ´ k _ 1 + [g x ] 2 + [g y ] 2 vector normal unitario Podemos orientar todas estas superficies tomando el lado positivo de S como el lado desde el cual apunta − → N. As´ı, el lado positivo de dicha superficie esta determinado por la normal unitaria − → N con componente positiva ´ k. Por otra parte, si ponemos G(x, y, z) = g(x, y) −z 507 − → N = ∇G |∇G| = g x ´ i + g y ´ j − ´ k _ 1 + [g x ] 2 + [g y ] 2 Corresponde, al lado negativo de la superficie con normal unitaria − → N con componente ´ k hacia abajo. 6.1.1. Integral de flujo. Si − → F es un campo vectorial perteneciente a C 1 definido sobre una super- ficie orientada S, con vector unitario ´ N, entonces la integral de superficie de − → F sobre S es. __ S − → F d − → S = __ S − → F ´ NdS La que se llama integral de flujo. El nombre es debido a su aplicaci´on a un problema f´ısico de un flu´ıdo a trav´es de una superficie. Supongamos que una superficie S est´ a inmersa en un fluido que tiene un campo de velocidades continuo F. Entonces Volumen de fluido a trav´es de S (por unidad de tiempo) = __ S − → F ´ NdS S es superficie orientada mediante ´ N, y − → F (x, y, z) = (P(x, y, z), Q(x, y, z), R(x, y, z)) campo vectorial tal que P, Q, R son funciones escalares con primeras derivadas parciales continuas en S. Adem´ as si ρ = ρ(x, y, z) representa la densidad del fluido en cada (x, y, z)de S,entonces Masa del flu´ıdo a trav´es de S (por unidad de tiempo) = __ ρ S − → F ´ NdS Ejemplo Sea S la parte del paraboloide z = 4 − (x 2 + y 2 ) Sobre el plano XY, orientado hacia arriba. Un flu´ıdo de densidad constante ρ = k pasa a trav´es de S seg´ un el campo de velocidades. − → F (x, y, z) = (x, y, z) 508 Hallar la raz´ on de flujo de masa a trav´es de S Soluci´on: G(x, y, z) = z −4 + x 2 + y 2 =⇒∇G = (2x, 2y, 1) =⇒ ´ N = 2x ´ i + 2y ´ j + ´ k _ 1 + 4x 2 + 4y 2 Masa del flu´ıdo a trav´es de S = __ S k − → F ´ NdS __ S k − → F ´ NdS = k __ R xy _ x, y, 4 − _ x 2 + y 2 __ (2x, 2y, 1) _ 1 + 4x 2 + 4y 2 _ 1 + 4x 2 + 4y 2 dA = k __ R xy _ 2x 2 + 2y 2 + 4 − _ x 2 + y 2 _¸ dA = k __ R xy _ x 2 + y 2 + 4 ¸ dxdy = k _ 2π 0 _ 2 y _ 4 + r 2 _ rdrdθ = k _ 2π 0 12dθ = 24kπ Por lo tanto Masa del fluido a trav´es de S (por unidad de tiempo) = 24kπ En base a lo observado en el desarrollo de este ejercicio se tiene el siguiente planteamiento: ”Si S es una superficie definida por z = g(x, y) y R xy denota la regi´on proyecci´on de S en el plano xy, entonces __ S − → F ´ NdS = __ R xy − → F (−g x ´ i −g y ´ j + ´ k)dA orientada hacia arriba, y __ S − → F ´ NdS = __ R xy − → F (g x ´ i + g y ´ j − ´ k)dA orientada hacia abajo. 509 6.1.2. Superficies Parametrizadas. En el tratamiento anterior de este tema hemos supuesto que una superficie viene definida por la ecuaci´ on z = g(x, y) , sin embargo se debe aclarar que hay superficies que no se pueden definir de esta forma, extenderemos la idea definiendo la forma param´etrica de dar una superficie. Una superficie parametrizada es una funci´ on − → Φ : D ⊆ IR 2 →IR 3 donde D es alguna dominio de IR 2 La superficie S que corresponde a − → Φ es su imagen , es decir S = − → Φ (D) Podemos escribir − → Φ (u, v) = (x (u, v) , y (u, v) , z (u, v)) , (u, v) ∈ D V´ease figura 6.4 Si cada una de las funciones componentes son diferenciables de clase C 1 , entonces llamamos a S una superficie diferenciable o de clase C 1 . Figura 6.4: Superficie parametrizada 6.1.3. Vector normal a S : Definimos en cada punto − → Φ (u o , v o ) − → T u = ∂x(u 0 , v 0) ∂u ´ i + ∂y(u 0 , v 0) ∂u ´ j + ∂z(u 0 , v 0) ∂u ´ k es vector tangente a la curva t −→ Φ(t, v 0 ) en Φ(u o , v o ) − → T v = ∂x(u 0 , v 0) ∂v ´ i + ∂y(u 0 , v 0) ∂v ´ j + ∂z(u 0 , v 0) ∂v ´ k es vector tangente a curva t →Φ(u o , t) en Φ(u o , v o ) − → T u − → T v es vector normal a la superficie S. 510 Ahora el concepto de superficie suave se da en la siguiente definici´ on. Decimos que la superficie S es suave en − → Φ (u o , v o ) si − → T u − → T v ,= 0 en (u o , v o ) ∈ D. Diremos que una superficie es suave si es suave en todos los puntos − → Φ (u o , v o ) ∈ S. Ejemplo 1 Sea S la superficie descrita por x 2 + y 2 + z 2 = r 2 , z ≥ 0 a) Parametrizar usando coordenadas esf´ericas. b) Determine el vector normal a la superficie S. Figura 6.5: semisfera de ecuaci´on x 2 + y 2 + z 2 = r 2 , z ≥ 0 Soluci´on: a) La superficie x 2 +y 2 +z 2 = r 2 , z ≥ 0 corresponde a una semiesfera con centro el el origen (0, 0, 0) radio r. Usando coordenadas esf´ericas tenemos: x = r sin ucos v y = r sin usin v z = r cos u _ _ _ Luego la superficie parametrizada es − → Φ (u, v) = (r sin ucos v, r sin usin v, r cos u) , (u, v) ∈ D donde D = _ (u, v) /0 ≤ u ≤ π 2 , 0 ≤ v ≤ 2π _ . b) Determinemos los vectores tangentes a la superficie S − → T u = − → Φ u = (r cos ucos v, r cos u sin v, −r sin u) − → T v = − → Φ v = (−r sin u sin v, r sin u cos v, 0) Calculemos el producto vectorial − → T u − → T v = ¸ ¸ ¸ ¸ ¸ ¸ i j k r cos ucos v r cos u sin v −r sin u −r sin u sin v r sin u cos v 0 ¸ ¸ ¸ ¸ ¸ ¸ 511 Por lo tanto, el vector normal a la esfera es − → T u − → T v = r 2 _ sin 2 u cos v, sin 2 u sin v, sin ucos v _ . Ejemplo 2 Sea S la superficie descrita por z 2 = x 2 + y 2 , z ≥ 0. a) Parametrizar usando coordenadas cil´ındricas. b)¿Es diferenciable ´esta superficie? c) Probar que no es suave en (0, 0, 0). Figura 6.6: cono con v´ertice en el origen Soluci´on: a) La superficie z 2 = x 2 + y 2 , z ≥ 0 corresponde a un cono su v´ertice en (0, 0, 0) . Usando coordenadas cil´ındricas tenemos: x = ucos v y = usin v z = u _ _ _ =⇒ z 2 = x 2 + y 2 = u 2 _ cos 2 u + sin 2 v _ ⇐⇒ z = u, u ≥ 0 Luego la superficie parametrizada queda − → Φ (u, v) = (ucos v, usin v, u) , u ≥ 0 b) Como cada una de las componentes es diferenciable como funci´ on de u, v entonces S es una superficie diferenciable. c) Veamos si es o no una superficie suave 512 − → T u = (cos v, sin v, 1) =⇒ − → T u (0, 0) = (cos 0, sin 0, 1) ∴ − → T u (0, 0) = (1, 0, 1) − → T v = (−usin v, ucos v, 0) =⇒ − → T v (0, 0) = (0 (−sin 0) , 0 cos 0, 0) ∴ − → T v = (0, 0, 0) = − → 0 As´ı −→ T u − → T v = − → 0 , de modo que la superficie no es suave en (0, 0, 0). Para efectos de c´alculo, consideraremos s´ olo superficies suaves por pedazos que sean uniones de im´agenes parametrizadas − → Φ i : D i ⊆ IR 2 → IR 3 ,para las cuales: i) D i es una regi´ on elemetal del plano. ii) − → Φ i es de clase C 1 y uno a uno, excepto quiz´as en la frontera D. iii) S i es la imagen de − → Φ i es suave , excepto quiz´as en un n´ umero finito de puntos. 6.1.4. ´ Area de una superficie parametrizada Definimos el ´ area de una superficie parametrizada S , denotada por A(S) ,mediante: A(S) = __ D _ _ _ −→ T u − → T v _ _ _ dudv donde _ _ _ −→ T u − → T v _ _ _ es la norma del vector normal − → T u − → T v . Propiedades 6.1.1. i) Si S es la uni´on de superficies parametrizadas S = ∪ S i entonces A(S) = i=1 A(S i ) ii) _ _ _ −→ T u − → T v _ _ _ = ¸ _ ∂ (x, y) ∂ (u, v) _ 2 + _ ∂ (y, z) ∂ (u, v) _ 2 + _ ∂ (z, x) ∂ (u, v) _ 2 513 Figura 6.7: ´ Area de una superficie parametrizada Figura 6.8: manto del paraboloide z = x 2 + y 2 , 0 ≤ z ≤ a 2 Ejemplo 1 Determine el ´ area del manto del paraboloide z = x 2 + y 2 , 0 ≤ z ≤ a 2 . Soluci´on: Usando coordenadas cil´ındricas tenemos − → Φ (u, v) = _ ucos v, u sin v, u 2 _ , (u, v) ∈ D donde D = ¦(u, v) /0 ≤ u ≤ a, 0 ≤ v ≤ 2π¦ Como el ´ area del manto est´ a dado por A(S) = __ D _ _ _ −→ T u − → T v _ _ _ dudv 514 Calculemos el integrando ∂ (x, y) ∂ (u, v) = ¸ ¸ ¸ ¸ cos v −usin v sin v ucos v ¸ ¸ ¸ ¸ = u, ∂ (y, z) ∂ (u, v) = ¸ ¸ ¸ ¸ sin v ucos v 2u 0 ¸ ¸ ¸ ¸ = −2u 2 cos v ∂ (z, x) ∂ (u, v) = ¸ ¸ ¸ ¸ 2u 0 cos v −usin v ¸ ¸ ¸ ¸ = −2u 2 sin v As´ı el integrando del ´area es _ _ _ −→ T u − → T v _ _ _ = √ u 2 + 4u 4 cos 2 v + 4u 4 sin 2 v = u √ 1 + 4u 2 Es evidente que _ _ _ −→ T u − → T v _ _ _ se anula para u = 0 y − → Φ (0, v) = (0, 0, 0) para cualquier v. Luego (0, 0, 0) es el ´ unico punto donde la superficie no es suave. Entonces __ D _ _ _ −→ T u − → T v _ _ _ dudv = _ 2π 0 _ a 0 u √ 1 + 4u 2 dudv = 1 12 _ 2π 0 _ _ 1 + 4u 2 _ 3/2 _ a 0 dv = π 6 _ _ 1 + 4a 2 _ 3/2 −1 _ ´ Area de una superficie en coordenadas cartesianas. Una susperficie S dada de la forma z = g (x, y) donde (x, y) ∈ D, admite la parametrizaci´ on − → Φ (x, y) = (x, y, g (x, y)) , (x, y) ∈ D Si f es de clase C 1 , esta parametrizaci´on es suave, entonces − → T x = _ 1, 0, ∂g ∂x _ y − → T y = _ 0, 1, ∂g ∂y _ =⇒ − → T x − → T y = _ − ∂g ∂x , − ∂g ∂y , 1 _ Entonces: _ _ _ − → T x − → T y _ _ _ = ¸ _ ∂g ∂x _ 2 + _ ∂g ∂y _ 2 + 1. De este modo la f´ ormula para el ´ area de la superficie se reduce a A(S) = __ D _ _ _ −→ T x − → T y _ _ _ dxdy = __ D ¸ _ ∂g ∂x _ 2 + _ ∂g ∂y _ 2 + 1 dxdy 515 Resultado que coincide con lo propuesto en la primera parte. 6.1.5. Integral de una funci´ on escalar sobre una su- perficie. Si f : D ⊂ IR 3 →IR,dada por f (x, y, z) es una funci´ on continua definida sobre una superficie parametrizada S, definimos, la integral de f sobre S como: _ _ S f dS = __ D f _ − → Φ (u, v) __ _ _ −→ T u − → T v _ _ _ dudv Si S es la uni´on de superficies parametrizadas S = ∪ S i entonces _ _ S f dS = i=1 _ _ S i f dS Ejemplo Evaluar _ _ S z dS, donde S es el hemisferio superior de radio r de la esfera z = _ r 2 −x 2 −y 2 . Soluci´on. Para este problema es conveniente representar el hemisferio parametricamente por: − → Φ (u, v) = (r sin ucos v, r sin usin v, r cos u) , (u, v) ∈ D sobre la regi´ on D del plano uv dada mediante D = _ (u, v) /0 ≤ u ≤ π 2 , 0 ≤ v ≤ 2π _ De las ecuaciones param´etricas obtenemos, los vectores tangentes a la superficie S. − → T u = − → Φ u = (r cos ucos v, r cos u sin v, −r sin u) − → T v = − → Φ v = (−r sin u sin v, r sin u cos v, 0) Calculemos el producto vectorial − → T u − → T v = ¸ ¸ ¸ ¸ ¸ ¸ i j k r cos ucos v r cos u sin v −r sin u −r sin u sin v r sin u cos v 0 ¸ ¸ ¸ ¸ ¸ ¸ Luego, el vector normal a la esfera es 516 − → T u − → T v = r 2 _ sin 2 u cos v, sin 2 u sin v, sin ucos v _ =⇒ _ _ _ − → T u − → T v _ _ _ = r 2 sin u De modo que _ _ S z dS = _ 2π 0 _ π/2 0 (r cos u) _ r 2 sin u _ dudv = r 3 _ 2π 0 _ π/2 0 cos usin ududv = r 3 _ 2π 0 _ sin 2 u 2 _ π/2 0 dv = πr 3 6.1.6. Integral de Superficie de campos vectoriales Sean S una superficie regular orientable, ´ N : S → IR 3 un campo de normales continuo sobre S, y − → F : Ω ⊆ IR 3 → IR 3 un campo vectorial continuo definido sobre un abierto que contiene a S. Definimos la integral de − → F sobre la superficie S orientada seg´ un ´ N mediante __ −→ Φ − → F ´ N dS = __ D − → F _ − → Φ u − → Φ v _ dudv donde − → Φ: D ⊆ IR 2 →IR 3 es una parametrizaci´ on regular de S compatible con la orientaci´on, esto es, tal que ´ N = − → Φ u −→ Φ v _ _ _ − → Φ u −→ Φ v _ _ _ Para interpretar correctamente el valor de la integral de flujo es necesario especificar el campo de normales ´ N. Por ejemplo, si orientamos un casquete esf´erico usando la normal exterior ´ N = ´ r, la integral de flujo corresponde al flujo neto que sale de la esfera a trav´es del casquete. Si este valor fuese negativo, significa que lo que el flujo entra a la esfera. 517 Ejemplo 1: Sea S es la esfera unitaria x 2 +y 2 +z 2 = 1 parametrizada en coordenadas esf´ericas por − → Φ (φ, θ) = (sin φcos θ, sin φsin θ, cos φ) , 0 ≤ φ ≤ π, 0 ≤ θ ≤ 2π. Sea − → F (x, y, z) = (x, y, z) radio vector del origen. Calcular __ −→ Φ − → F ´ N dS Soluci´ on: En este caso D es regi´on del plano φ θ definida por: 0 ≤ φ ≤ π, 0 ≤ θ ≤ 2π. − → T φ = (cos φcos θ, cos φsenθ, −sin φ) − → T θ = (−sin φsin θ, sin φcos θ, 0) − → T φ − → T θ = _ −cos φsin 2 θ _ ´ i − _ sin φsin 2 θ _ ´ j + (cos φsin θ) ´ k Notese que el vector normal apunta hacia el lado positivo de la superficie − → F _ − → T φ − → T θ _ = sin φ __ −→ Φ − → F ´ N dS = __ D sin φdφdθ = _ 2π 0 _ π 0 sin φdφdθ = 4π Observaci´ on Si S es una superficie regular orientada seg´ un un campo de normales ´ N y si S − es la misma superficie pero con la orientaci´ on opuesta − ´ N, entonces se tiene que __ S − − → F d − → S = − __ S − → F d − → S 518 6.1.7. Aplicaci´on al campo de la f´ısica: Calcular el flujo del campo el´ectrico producido por una carga Q en el origen, a trav´es del manto de la esfera S de radio R orientado seg´ un la normal exterior. El campo el´ectrico producido por la carga Q viene dado por − → E = Q 4πε o ´ r r 2 donde ε o una constante universal . De esta forma se obtiene el siguiente flujo el´ectrico __ S − → E d − → S = __ S Q 4πε o ´ r R 2 ´ rdS Parametrizando la superficie en coordenadas esf´ericas se tiene − → Φ (φ, θ) = (Rsin φcos θ, Rsin φsin θ, Rcos φ) , 0 ≤ φ ≤ π, 0 ≤ θ ≤ 2π =⇒ _ _ _ − → Φ φ − → Φ θ _ _ _ = R 2 sin Φ __ S Q 4πε o ´ r R 2 ´ rdS = _ 2π 0 _ π 0 Q 4πε o 1 R 2 R 2 sin φdφdθ = Q ε o 6.2. Teoremas de Gauss y de Stokes Ahora dos teoremas de gran importancia en el campo de las aplicaciones, particularmente en la f´ısica, el primero de ellos es el teorema de Gauss o teorema de la divergencia. 6.2.1. Divergencia En IR 3 ,sea − → F un campo vectorial,anteriormente hemos definimos div − → F = ∇ − → F = ∂P ∂x + ∂Q ∂y + ∂R ∂z 519 Ejemplo. Sea − → F (x, y, z) = (xy, y 2 +e xz 2 , sen(xy)), calcule la divergencia del campo vectorial divF = ∂xy ∂x + ∂(y 2 + e xz 2 ) ∂y + ∂sen(xy) ∂z = 3y El foco de atenci´ on lo ponemos ahora en regiones R s´ olidas simples ( por ejemplo cajas rect´ angulares, esferas, elipsoides etc). La frontera S es una superficie cerrada y convenimos que la orientaci´on positiva es hacia afuera, representada por el vector normal ´ N hacia afuera de la superficie. En este contexto enunciamos el teorema. 6.2.2. Teorema de la divergencia de Gauss. Sea R una regi´ on simple s´olida limitada por una superficie cerrada S orientada por un vector normal unitario dirigido al exterior de R. Si − → F es un campo vectorial cuyas funciones componentes P, Q y R tienen derivadas parciales continuas en la regi´ on R, entonces __ S − → F ´ NdS = ___ R div − → F dV Demostraci´ on. S´ olo daremos algunas ideas generales para indicar un procedimiento para la demostraci´ on Sea − → F (x, y, z) = (P(x, y, z), Q(x, y, z), R(x, y, z)) =⇒div − → F = ∇ − → F = ∂P ∂x + ∂Q ∂y + ∂R ∂z Integrando ___ R div − → F dV = ___ R ∂P ∂x dV + ___ ∂Q ∂y R dV + ___ R ∂R ∂z dV Por otro lado, si ´ N es el normal unitario de S hacia afuera la integral de superficie es __ S − → F d − → S = __ S − → F ´ NdS = __ S P ´ i ´ NdS + __ S Q ´ j ´ NdS + __ S R ´ k ´ NdS 520 Siguiendo el procedimiento de la demostraci´ on del teorema de Green basta probar las siguientes tres igualdades: ___ R ∂P ∂x dV = __ S P ´ i ´ NdS ___ ∂Q ∂y R dV = __ S Q ´ j ´ NdS ___ R ∂R ∂z dV = __ S R ´ k ´ NdS Detalles y comentarios los puede consultar en el texto C¸ ´ alculo de James Stewart, Tercera edici´ on (Thomson), p´agina 935” Ejemplo 1: Sea S esfera unitaria x 2 +y 2 +z 2 = 1. Demostrar que __ S (xy + yz + xz) dS = 0 Soluci´on: Claramente x ´ i + y ´ j + z ´ k es vector normal a la esfera en todo (x, y, z) . ´ N = x ´ i + y ´ j + z ´ k _ x 2 + y 2 + z 2 = x ´ i + y ´ j + z ´ k, ya que x 2 + y 2 + z 2 = 1 consideramos ahora el siguiente arreglo para identificar xy + yz + xz = (y, z, x) (x, y, z) = (y, z, x) − → N =⇒ − → F = (y, z, x) ∇ − → F = ∂P ∂x + ∂Q ∂y + ∂R ∂z = 0 + 0 + 0 = 0 Aplicando Teorema de Divergencia 521 __ S (xy + yz + xz) dS = ___ R ∇ − → F dV = ___ R 0dV = 0 Ejemplo 2 Verificar el teorema de la divergencia __ S − → F ´ NdS = ___ R div − → F dV para − → F (x, y, z) = (2x − y) ´ i − (2y − z) ´ j + z ´ k S: limitado por los planos coordenados y x + 2y + z = 6 Figura 6.9: S limitado por los planos coordenados y x + 2y + z = 6 Soluci´on. En este caso S = S 1 + S 2 + S 3 +S 4 donde S 1 ser´ a el lado de la figura en el plano x + 2y + z = 6, S 2 el lado de la figura en el plano coordenado yz, S 3 el lado de la figura en el plano coordenado xz, S 4 el lado correspondiente al plano coordenado xy. Tenemos __ S − → F ´ NdS = __ S 1 − → F ´ N 1 dS + __ S 2 − → F ´ N 2 dS + __ S 3 − → F ´ N 3 dS ++ __ S 4 − → F ´ N 4 dS 522 __ S 1 − → F ´ N 1 dS = __ R xy − → F (x, y, z) (−g x (x, y), −g y (x, y), 1)dA = __ R xy − → F (x, y, z) (1, 2, 1)dA = __ R xy (2x −y, −(2y −z), z) (1, 2, 1)dA = __ R xy ((2x −y) −2(2y −z) + z)dA = __ R xy (2x −5y + 3(6 −x −2y)dxdy = _ 6 0 _ − 1 2 x+3 0 (−x −11y + 18)dydx = _ 6 0 (− 7 8 x 2 + 9 2 x + 9 2 )dx = 45 __ S 2 − → F ´ N 2 dS = __ S 2 − → F (− ´ i)dS = __ S 2 (y −2x)dS = __ R yz (y −2(6 −2y −z)dydz = _ 3 0 _ 6−2y 0 (5y + 2z −12) dzdy = −27 __ S 3 − → F ´ NdS = __ S 3 − → F (− ´ j)dS = __ S 3 (2y −z)dS = __ R xz (2( 6 −x −z 2 ) −z)dxdz = _ 6 0 _ 6−x 0 (6 −x −2z) dzdx = 0 __ S 4 − → F ´ N 4 dS = __ S 4 − → F (− ´ k)dS = __ R xy 0dxdy = 0 De estos c´ alculos se tiene que __ S − → F ´ NdS = 45 −27 + 0 + 0 = 18 523 Por el otro lado div − → F = 2 −2 + 1 = 1, la integral triple es ___ R div − → F dV = ___ R dV = _ 6 0 _ − x 2 +3 0 _ 6−x−2y 0 dzdydx = _ 6 0 _ − x 2 +3 0 (6 −x −2y)dydx = _ 6 0 _ x 2 4 −3x + 9 _ dx = 18 6.2.3. Teorema de Stokes. Este teorema da la relaci´on entre una integral de superficie sobre una superficie orientada S y una integral de l´ınea a lo largo e una curva cerrada siempre seccionalmente suave C con orientaci´ on positiva que acota a S. Teorema 6.2.1. Sea S una superficie limitada por una curva C ,orienta- da positivamente, seccionalmente suave. Si − → F es un campo vectorial cuyas componentes tienen derivadas parciales continuas en una regi´on abierta que contiene a S y C, entonces. _ C − → F d − → s = __ S _ rot − → F _ d − → S = __ S _ ∇ − → F _ d − → S Recuerde que la integral de l´ınea se puede expresar: _ C Pdx + Qdy + Rdz o _ C − → F ´ Tds Ejemplo 1 Use el teorema de Stokes para evaluar _ C − → F d − → s C , donde − → F (x, y, z) = (3z, 5x, −2y) y C es la intersecci´ on del plano z = y + 3 con el cilindro x 2 + y 2 = 1. Oriente la elipse en sentido contrario al movimiento de las agujas del reloj, vista desde arriba. Soluci´on. La intersecci´on del plano con el cilindro permite visualizar que S en este caso es la porci´on del plano que queda al interior del cilindro y que C es la elipse intersecci´ on 524 _ C − → F d − → s = __ S _ ∇ − → F _ ´ NdS ∇ − → F = ¸ ¸ ¸ ¸ ¸ ¸ ´ i ´ j ´ k ∂ ∂x ∂ ∂y ∂ ∂z 3z 5x −2y ¸ ¸ ¸ ¸ ¸ ¸ = (−2 ´ i + 3 ´ i + 5 ´ k) z −y + 3 = 0 =⇒ ´ N = 1 √ 2 (− ´ j + k) Calculando _ ∇ − → F _ ´ N = (−2 ´ i + 3 ´ i + 5 ´ k) 1 √ 2 (− ´ j + k) = 2 √ 2 = √ 2 y entonces la integral ser´ a _ C − → F d − → s = __ S √ 2dS = √ 2 __ S dS = √ 2 Area(S) = √ 2 (1 √ 2π) = 2π S es el interior de una elipse de semi ejes 1 y √ 2. 6.3. Problemas Resueltos Problema 1 Sea la funci´on − → r : D → IR 2 , definida por − → r (r, θ) = (r cos θ, r senv, r) donde D = ¦(r, θ) /0 ≤ r ≤ 1, 0 ≤ θ ≤ 2π¦ una parametrizaci´ on de un cono S. Hallar su ´ area de superficie. Soluci´on El ´ area de una superficie parametrizada se define A(S) = _ _ D | − → r u − → r v | dudv Calculemos el integrando de ´ area usando la propiedad: | − → r r − → r θ | = ¸ _ ∂ (x, y) ∂ (r, θ) _ 2 + _ ∂ (y, z) ∂ (r, θ) _ 2 + _ ∂ (z, x) ∂ (r, θ) _ 2 siendo los Jacobianos ∂ (x, y) ∂ (r, θ) = ¸ ¸ ¸ ¸ cos θ −rsenθ senθ r cos θ ¸ ¸ ¸ ¸ = r 525 Figura 6.10: Representaci´on param´etrica de un cono ∂ (y, z) ∂ (r, θ) = ¸ ¸ ¸ ¸ senθ r cos θ 1 0 ¸ ¸ ¸ ¸ = −r cos θ ∂ (z, x) ∂ (r, θ) = ¸ ¸ ¸ ¸ 1 0 cos θ −rsenθ ¸ ¸ ¸ ¸ = rsenθ As´ı que el integrando de ´ area es | − → r r − → r θ | = √ r 2 + r 2 cos 2 θ + r 2 sen 2 θ = r √ 2 Por tanto,el ´area de la superficie S es A(S) = _ _ D | − → r r − → r θ| dudv = _ 2π 0 _ 1 0 √ 2rdrdθ = _ 2π 0 √ 2 _ r 2 2 _ 1 0 dθ = √ 2 2 _ 2π 0 dθ = √ 2π Observaci´on, | − → r r − → r θ|se anula para r = 0,pero − → r (0, θ) = (0, 0, 0) ∀θ. As´ı, (0, 0, 0) es el ´ unico punto donde la superficie no es suave. Problema 2 Calcular el ´ area S de la regi´ on del cono definido por x 2 = y 2 +z 2 , interior al cilindro x 2 + y 2 = a 2 , acotado en el octante x ≥ 0, y ≥ 0, z ≥ 0. Soluci´on Parametrizando la ecuaci´on del cono, usando coordenadas cartesianas (y, z),se tiene que 526 Figura 6.11: regi´on del cono definido por x 2 = y 2 +z 2 interior al cilindro x 2 +y 2 = a 2 , acotado en el octante el primer octante − → r (y, z) = _ _ y 2 + z 2 , y, z _ , (y, z) ∈ D La superficie S est´a al interior del cilindro , por lo que x 2 + y 2 = 2y 2 + z 2 ≤ a 2 . Entonces, la regi´on D esta definida sobre el plano yz por D = ¦(y, z) ∈ IR 2 /2y 2 + z 2 ≤ a 2 , y ≥ 0, z ≥ 0¦ . El ´area de la superficie es: A(S) = _ _ D | − → r y − → r z | dydz luego ,si − → r y = _ y _ y 2 + z 2 , 1, 0 _ , − → r z = _ z _ y 2 + z 2 , 0, 1 _ =⇒ − → r y − → r z = (1, − y _ y 2 + z 2 , − z _ y 2 + z 2 ) =⇒ | − → r y − → r z | = √ 2 A(S) = _ _ D √ 2dydz Observemos que: 2y 2 + z 2 ≤ a 2 , y ≥ 0, z ≥ 0 ⇐⇒ 527 y 2 _ a √ 2 _ 2 + z 2 a 2 ≤ 1, y ≥ 0, z ≥ 0 es el ´area de la cuarta parte de la elipse Usando cambio de variables tenemos: x = a √ 2 r cos θ y = arsenθ =⇒ ¸ ¸ ¸ ∂(x,y) ∂(r,θ) ¸ ¸ ¸ = ¸ ¸ ¸ ¸ ¸ a √ 2 cos θ − a √ 2 rsenθ asenθ ar cos θ ¸ ¸ ¸ ¸ ¸ = a 2 √ 2 r y 2 _ a √ 2 _ 2 + z 2 a 2 ≤ 1 ⇐⇒ 0 ≤ r 2 ≤ 1 =⇒ 0 ≤ r ≤ 1 y 0 ≤ θ ≤ π Entonces la regi´on D ∗ = ¦(r, θ) /0 ≤ r ≤ 1 y 0 ≤ θ ≤ π¦ .Luego, si A(S) = _ _ D √ 2dydz = √ 2 _ _ D ∗ a 2 √ 2 rdrdθ = a 2 _ π/2 0 _ 1 0 rdrdθ = a 2 _ π/2 0 r 2 2 ¸ ¸ ¸ ¸ 1 0 dθ = πa 2 4 Problema 3 Calcular el ´ area S de la regi´ on del manto del cilindro x 2 + y 2 = 2y comprendida entre y + z = 2 y z = 0. Figura 6.12: Regi´on del manto del cilindro x 2 + y 2 = 2y comprendida entre y + z = 2 y z = 0. D, parametrizaci´on de esta regi´on Soluci´on El ´area de la superficie es: A(S) = _ _ D | − → r u − → r v | dudv 528 Parametrizando la superficie del cilindro, x 2 +y 2 = 2y ⇐⇒ x 2 +(y−1) 2 = 1 Usando coordenadas cilindricas (u, v),se tiene que − → r (u, v) = (cos u, 1 + senu, v) , (u, v) ∈ D donde D = ¦(u, v) /0 ≤ u ≤ 2π, 0 ≤ v ≤ 2 −(1 + senu)¦ Determinemos el integrando de ´ area − → r u − → r v = ¸ ¸ ¸ ¸ ¸ ¸ i j k −senu cos u 0 0 0 1 ¸ ¸ ¸ ¸ ¸ ¸ = (cos u, −senu, 0) =⇒ | − → r u − → r v | = 1 Por tanto,el ´area de la superficie S es A(S) = _ _ D dudv = _ 2π 0 _ 1−senu 0 dvdu = _ 2π 0 (1 −senu) du = 2π 6.3.1. Integrales de superficie Problema 1 Sea S el octante positivo de la superficie x 2 + y 2 + z 2 = 1.Calcular la integral de superficie _ _ S 1 _ x 2 + y 2 + (z −1) 2 dS Soluci´on Parametricemos la Superficie S usando coordenadas esf´ericas, es decir − → r (u, v) = (senu cos v, senu senv, cos u) onde D ∗ = ¦(u, v) /0 ≤ u ≤ π/2, 0 ≤ v ≤ π/2¦ Calculemos el valor del integrando sobre S 529 1 _ x 2 + y 2 + (z −1) 2 = 1 _ sen 2 u cos 2 v + sen 2 u sen 2 v + (cos 2 u −1) 2 = 1 √ sen 2 u + cos 2 u + 1 −2 cos u = 1 √ 2 −2 cos u Calculemos el producto vectorial − → r u − → r v = ¸ ¸ ¸ ¸ ¸ ¸ i j k cos ucos v cos u senv −senu −senu senv senu cos v 0 ¸ ¸ ¸ ¸ ¸ ¸ = _ sen 2 u cos v, sen 2 u senv, senucos v _ Luego , obtenemos la norma del producto : | − → r u − → r v | = (sen 4 u + sen 2 u cos 2 u) 1/2 = [senu[ = senu porque 0 ≤ u ≤ π/2 Finalmente, apliquemos la definici´ on de integral de superficie y calculemos _ _ S 1 _ x 2 + y 2 + (z −1) 2 dS = _ π/2 0 _ π/2 0 | − → r u − → r v | √ 2 −2 cos u dudv = _ π/2 0 _ π/2 0 senu √ 2 −2 cos u dudv = 1 √ 2 _ π/2 0 _ 2 √ 1 −cos u ¸ π/2 0 dv = 2 √ 2 _ π/2 0 dv = π √ 2 Problema 2 Evaluar __ S z 2 ds, donde S es el paraboloide z = x 2 + y 2 , comprendido entre z = 1 y z = 4. Soluci´on Para este problema tenemos dos opciones para representar parametrica- mente la corona del paraboloide. 530 Figura 6.13: Usando coordenadas cartesianas rect´ angulares − → r (x, y) = (x, y, x 2 + y 2 ) , donde D = ¦(x, y) ∈ IR 2 /1 ≤ x 2 + y 2 ≤ 4¦ Calculemos el vector normal a la superficie S, dado por − → r x − → r y = ¸ ¸ ¸ ¸ ¸ ¸ i j k 1 0 2x 0 1 2y ¸ ¸ ¸ ¸ ¸ ¸ = (2x, 2y, 1) Luego , obtenemos la norma del producto : | − → r x − → r y | = _ 4x 2 + 4y 2 + 1 Aplicando la definici´on de integral de superficie __ S z 2 ds = _ _ D z 2 | − → r y − → r z | dxdy = _ _ D (x 2 + y 2 ) _ 4x 2 + 4y 2 + 1dxdy Usando coordenadas polares para evaluar la integral doble, 531 _ _ D (x 2 + y 2 ) _ 4x 2 + 4y 2 + 1dxdy = _ 2π 0 _ 2 1 r 2 √ 4r 2 + 1rdrdθ = _ 2π 0 _ r 2 1 12 _ 4r 2 + 1 _ 3/2 ¸ ¸ ¸ ¸ 2 1 − 2 12 _ 2 1 r _ 4r 2 + 1 _ 3/2 dr _ dθ = _ 2π 0 _ r 2 1 12 _ 4r 2 + 1 _ 3/2 ¸ ¸ ¸ ¸ 2 1 − 1 120 _ 4r 2 + 1 _ 5/2 ¸ ¸ ¸ ¸ 2 1 _ dθ = π 6 _ 17 3/2 −5 3/2 − 1 10 17 5/2 + 1 10 5 3/2 _ 6.3.2. Integral de Flujo de un campo vectorial Problema 1 Calcular el flujo para − → F (x, y, z) = (3x, 3y, z) en la superficie z = 9 − x 2 −y 2 tal que z ≥ 0 Figura 6.14: Soluci´on Usando coordenadas cartesianas rectangulares, la parametrizaci´ on de la superficie S es − → r (x, y) = (x, y, 9 −x 2 −y 2 ) , donde D = ¦(x, y) ∈ IR 2 /x 2 + y 2 ≤ 9, z ≥ 0¦ Calculemos el vector normal a la superficie S, dado por − → r x − → r y = ¸ ¸ ¸ ¸ ¸ ¸ i j k 1 0 −2x 0 1 −2y ¸ ¸ ¸ ¸ ¸ ¸ = (2x, 2y, 1) Aplicando la definici´ on de integral de superficie al flujo a trav´es de S, se obtiene 532 __ S − → F ´ nds = __ D − → F ( − → r (x, y)) ( − → r x − → r y ) dxdy __ D (3x, 3y, z) (2x, 2y, 1)dxdy = __ D _ 6x 2 + 6y 2 + z _ dxdy = __ D _ 6x 2 + 6y 2 + 9 −x 2 −y 2 _ dxdy = __ D _ 5x 2 + 5y 2 + 9 _ dxdy Cambiando a coordenadas polares __ D _ 5x 2 + 5y 2 + 9 _ dxdy = _ 2π 0 _ 3 0 _ 5r 2 + 9 _ rdrdθ = 567π 2 Alternativamente, podemos parametrizar directamente la superficie S en coordenadas cil´ındricas. − → r (r, θ) = _ r cos θ, rsenθ, 9 −r 2 _ , 0 ≤ r ≤ 3, 0 ≤ θ ≤ 2π Calculemos el vector normal a la superficie y el campo vectorial sobre la superficie: − → n = − → r r − → r θ = ¸ ¸ ¸ ¸ ¸ ¸ i j k cos θ senθ −2r −rsenθ r cos θ 0 ¸ ¸ ¸ ¸ ¸ ¸ = = _ 2r 2 cos θ, 2r 2 senθ, r _ Aplicando la definici´ on de integral de superficie al flujo sobre S, se obtiene 533 __ S − → F ´ nds = __ D − → F ( − → r (r, θ)) ( − → r r − → r θ ) drdθ = __ D _ 3r cos θ, 3 rsenθ, 9 −r 2 _ _ 2r 2 cos θ, 2r 2 senθ, r _ drdθ = __ D _ 6r 3 (cos 2 θ + sen 2 θ) + 9r −r 3 _ drdθ = __ D _ 5r 3 + 9r _ drdθ = _ 2π 0 _ 3 0 _ 5r 3 + 9r _ drdθ = 567π 2 Problema 2 Sea − → F el campo vectorial − → F (x, y, z) = (y, x, xz) . Calcular la integral de flujo __ S (∇ − → F ) ´ ndS, donde ´ n el campo normal exterior al paraboloide, y S el sector de la superficie del paraboloide z = x 2 +(y −1) 2 , interior al cilindro x 2 + (y −2) 2 = 3. Figura 6.15: S, sector de la superficie del paraboloide z = x 2 + (y − 1) 2 , interior al cilindro x 2 + (y −2) 2 = 3 Soluci´on Tenemos que calcular la integral de flujo __ S (∇ − → F ) ´ ndS = __ D (∇ − → F ) − → n dS 534 En primer lugar, calculamos el rotor del campo − → F , ∇ − → F = ¸ ¸ ¸ ¸ ¸ ¸ ¸ ¸ i j k ∂ ∂x ∂ ∂y ∂ ∂z y x xz ¸ ¸ ¸ ¸ ¸ ¸ ¸ ¸ = (0, −z, 0), Parametricemos el sector de la superficie del paraboloide z = x 2 +(y−1) 2 interior al cilindro x 2 +(y −2) 2 = 3 usando coodenadas cilindricas con centro en el punto (0,2,0) y radio √ 3.Entonces − → Φ(u, v) = (ucosv, 2 + usenv, u 2 + 2usenv + 1) , 0 ≤ u ≤ √ 3, 0 ≤ v ≤ 2π. Determinemos los vectores tangentes a la superficie del paraboloide − → Φ u (u, v) = (cosv, senv, 2u + 2senv) − → Φ v (u, v) = (−usenv, ucosv, 2ucosv) Dado que en el v´ertice (0, 1, 0) del paraboloide el vector normal apunta hacia el exterior , tenemos que la normal exterior tiene componentes (0,0,-1), entonces − → n = − → Φ v (u, v) − → Φ u (u, v) = ¸ ¸ ¸ ¸ ¸ ¸ i j k −usenv ucosv 2ucosv cosv senv 2u + 2senv ¸ ¸ ¸ ¸ ¸ ¸ = _ 2u 2 cosv, 2u 2 senv + 2u, −u _ En consecuencia, la integral queda __ D ∇ − → F − → n dS = _ _ D −(0, u 2 + 2usenv + 1, 0) (2u 2 cosv, 2u 2 senv + 2u, −u)dudv = − _ √ 3 0 _ 2π 0 (2u 4 senv + 4u 3 sen 2 v + 6u 2 senv + 2u 3 + 2u)dvdu = − _ √ 3 0 _ 4u 3 _ v 2 − sen2v 4 _ + (2u 3 + 2u)v _ 2π 0 du = − _ √ 3 0 _ 8πu 3 + 4πu _ du = _ 2πu 4 + 2πu 2 ¸ √ 3 0 = −24π Problema 3 Calcular el flujo de salida del campo vectorial − → F (x, y, z) = (2y, zy, 3z) a trav´es de la superficie cil´ındrica x 2 +y 2 = 4 acotada en el primer octante por los planos x = 0, y = 0, z = 0, z = 5. 535 Figura 6.16: Superficie cilindrica x 2 + y 2 = 4, acotada en el primer octante por los planos coordenados Soluci´on Tenemos que calcular __ S − → F ´ ndS = __ D − → F _ − → Φ (θ, z) _ _ − → Φ θ − → Φ z _ dθdz Parametricemos la superficie cil´ındrica − → Φ(θ, z) = (2 cos θ, 2 sin θ, z) donde D = ¦(θ, z) /0 ≤ θ ≤ π/2, 0 ≤ z ≤ 5¦. Determinemos los vectores tangentes a la superficie − → Φ θ (θ, z) = (−2 sin θ, 2 cos θ, 0) − → Φ z (θ, z) = (0, 0, 1) El vector normal que apunta al exterior de la superficie es − → Φ θ (θ, z) − → Φ z (θ, z) = ¸ ¸ ¸ ¸ ¸ ¸ i j k −2senθ 2 cos θ 0 0 0 1 ¸ ¸ ¸ ¸ ¸ ¸ = (2cosθ, 2senθ, 0) 536 Entonces, el flujo de − → F a trav´es de esta superficie es __ D − → F _ − → Φ (θ, z) _ _ − → Φ θ − → Φ z _ drdz = _ _ D (4 sin θ, 2z sin θ, 3z) (2 cos θ, 2 sin θ, 0) dθdz = _ π/2 0 _ 5 0 4 _ 5 sin 2θ + z sin 2 θ _ dzdθ = 4 _ π/2 0 _ sin 2θ + 25 2 _ 1 −cos 2θ 2 __ dθ = 20 _ −cos 2θ 2 _ π/2 0 + 50 _ θ 2 − sin 2θ 4 _ π/2 0 = 20 + 25 2 π 6.3.3. Teorema de la divergencia de Gauss Problema 1 Calcular el flujo __ S − → F ´ ndS del campo vectorial − → F (x, y, z) = (x 2 + senz, xy + cos z, e y ) a trav´es de la frontera S limitada por la superficie cil´ındrica x 2 + y 2 = 2y y los planos z = 0, y + z = 2, Figura 6.17: S limitada por la superficie cil´ındrica x 2 +y 2 = 2y y los planos z = 0, y + z = 2, Soluci´on 537 Puesto que el campo vectorial − → F es continuo, con primeras derivadas parciales continuas en IR 3 y la regi´ on V⊆ IR 3 encerrada es conexa y S es una superficie cerrada suave, entonces es aplicable el teorema de la divergencia de Gauss. __ S − → F ´ ndS = ___ V ∇ − → F dV Para resolver la ultima integral necesitamos describir el s´ olido V. Comple- tando el cuadrado, la ecuaci´on de la superficie cil´ındrica dada puede escribirse como: x 2 + y 2 = 2y ⇐⇒ x 2 + (y −1) 2 = 2y. Luego, la proyecci´on del s´ olido sobre el plano XY es la circunsferencia de centro (0, 1) y radio 1. Por otra parte z esta acotado entre los dos planos dados 0 ≤ z ≤ 2 −y. Por tanto, el s´ olido esta dado por V = _ (x, y, z) ∈ IR 3 / − _ 2y −y 2 ≤ x ≤ _ 2y −y 2 , 0 ≤ y ≤ 2, 0 ≤ z ≤ 2 −y _ Calculemos la divergencia del campo vectorial: ∇ − → F = ∂ ∂x (x 2 + senz) + ∂ ∂y (xy + cos z) + ∂ ∂z (e y ) = 3x Entonces: ___ V 3xdxdydz = _ 2 0 _ √ 2y−y 2 − √ 2y−y 2 _ 2−y 0 3xdzdxdy = _ 2 0 _ √ 2y−y 2 − √ 2y−y 2 3x (2 −y) dxdy = _ 2 0 (2 −y) _ 3 2 x 2 _¸ ¸ ¸ ¸ √ 2y−y 2 − √ 2y−y 2 dy = 0 Problema 2 Verificar el teorema de la divergencia para el campo vectorial − → F (x, y, z) = (x, y, z) sobre la esfera x 2 + y 2 + z 2 = a 2 . Soluci´on El teorema de la divergencia afirma que. __ S − → F ´ ndS = ___ V ∇ − → F dV La divergencia de − → F es 538 ∇ − → F = ∂ ∂x (x) + ∂ ∂y (y) + ∂ ∂z (z) = 3. de modo que. ___ V ∇ − → F dV = ___ V 3 dV = 3 _ 4 3 πa 3 _ = 4πa 3 Calculemos ahora el Flujo sobre la superficie. __ S − → F ´ ndS = __ D − → F ( − → r (u, v)) ( − → r u − → r v ) dudv Parametricemos la superficie usando coordenadas esf´ericas. − → r (u, v) = a (sin u cos v, sin usin v, cos u) , con D = ¦(u, v) /0 ≤ u ≤ π, 0 ≤ v ≤ 2π¦ Calculemos el producto vectorial − → r u − → r v = a 2 ¸ ¸ ¸ ¸ ¸ ¸ i j k cos ucos v cos u sin v −sin u −sin u sin v sin u cos v 0 ¸ ¸ ¸ ¸ ¸ ¸ = a 2 _ sin 2 u cos v, sin 2 u sin v, sin ucos u _ Por consiguiente − → F ( − → r (u, v)) ( − → r u − → r v ) = a 3 sin u entonces la integral de flujo queda __ S − → F ´ ndS = __ D a 3 sin ududv = _ 2π 0 _ π 0 a 3 sin ududv = a 3 _ 2π 0 [−cos u] π 0 dv = 2a 3 _ 2π 0 dv = 4πa 3 Por tanto, comparando estos resultados hemos verificado el teorema de Gauss. Problema 3 539 Figura 6.18: Regi´on acotada por la frontera x 2 + y 2 = 4, z = 0, y z = 3 Verificar el teorema de la divergencia para − → F (x, y, z) = (x 3 , y 3 , z 3 ) en R, la regi´ on acotada por la frontera x 2 + y 2 = 4, z = 0,y z = 3. Soluci´on El teorema de la divergencia afirma que __ S − → F ´ nds = ___ R ∇ − → F dV Calculemos el flujo que produce el campo sobre la frontera: __ S − → F ´ nds = __ S 1 − → F ´ nds + __ S 2 − → F ´ nds + __ S 3 − → F ´ nds Si definimos parametrizacion de las superficies inferior y superior que limitan el cilindro tenemos: S 1 : − → r (u, v) = (u, −v, 0) D ∗ = ¦(u, v) /0 ≤ u ≤ 2, 0 ≤ v ≤ 2π, 0¦ El vector normal con orientacion positiva es − → r u − → r v = ¸ ¸ ¸ ¸ ¸ ¸ i j k 1 0 0 0 −1 0 ¸ ¸ ¸ ¸ ¸ ¸ = (0, 0, −1) As´ı , obtenemos: __ S 1 − → F ´ nds = __ D ∗ − → F ( − → r (u, v)) ( − → r u − → r v ) dudv = __ D ∗ _ u 3 , −v 3 , 0 _ (0, 0, −1) dudv = 0 Analogamente para: 540 S 3 : − → r (u, v) = (u, v, 0) donde D ∗ = ¦(u, v) /0 ≤ u ≤ 2, 0 ≤ v ≤ 2π¦ − → r u − → r v = ¸ ¸ ¸ ¸ ¸ ¸ i j k 1 0 0 0 1 0 ¸ ¸ ¸ ¸ ¸ ¸ = (0, 0, 1) Luego: __ S 3 − → F ´ nds = __ D ∗ − → F ( − → r (u, v)) ( − → r u − → r v ) dudv = __ D ∗ _ u 3 , v 3 , 3 3 _ (0, 0, 1) dudv = _ 2 0 _ 2π 0 3 3 dudv = 108π Si definimos la parametrizaci´ on para el manto del cil´ındro se tiene: S 2 : − → r (u, v) = (2 cos u, 2senu , v) donde D ∗ = ¦(u, v) /0 ≤ u ≤ 2π, 0 ≤ v ≤ 3¦ El vector normal con orientacion positiva es, − → r u − → r v = ¸ ¸ ¸ ¸ ¸ ¸ i j k −2senu 2 cos u 0 0 0 1 ¸ ¸ ¸ ¸ ¸ ¸ = (2 cos u, 2senu, 0) En consecuencia, __ S 2 − → F ´ nds = __ D ∗ − → F ( − → r (u, v)) ( − → r u − → r v ) dudv = __ D ∗ _ (2 cos u) 3 , (2senu) 3 , v 3 _ (2 cos u, 2senu, 0) dudv = 16 _ 3 0 _ 2π 0 _ cos 4 u + sen 4 u _ dudv = 16 _ 3 0 _ 2π 0 1 8 (6 + 2 cos 4u) dudv = 16 _ 3 0 1 8 _ 6u + 2 4 sen4u _ 2π 0 dv = 2 (12π) _ 3 0 dv = 72π 541 Por lo tanto, sumando las integrales de flujo se tiene: __ S − → F ´ nds = 102π + 72π = 180π Por otra parte, como ∇ − → F = ∂x 3 ∂x + ∂y 3 ∂y + ∂z 3 ∂z = 3 (x 2 + y 2 + z 2 ) , entonces ___ R ∇ − → F dV = ___ R 3 _ x 2 + y 2 + z 2 _ dV Usando coordenadas cil´ındricas para evaluar la integral triple, tenemos x = r cos θ y = rsenθ z = z _ _ _ =⇒ 0 ≤ x 2 + y 2 ≤ 4 ⇐⇒ 0 ≤ r 2 ≤ 4 ⇐⇒ 0 ≤ r ≤ 2 Entonces la regi´on R ∗ = ¦(r, θ, z) /0 ≤ r ≤ 2 , 0 ≤ θ ≤ 2π, 0 ≤ z ≤ 3¦ ___ R 3 _ x 2 + y 2 + z 2 _ dV = 3 _ 2π 0 _ 2 0 _ 3 0 _ r 2 + z 2 _ rdzdrdθ = 3 _ 2π 0 _ 2 0 _ r 2 z + r 3 3 _ 3 0 rdrdθ = 3 _ 2π 0 _ 2 0 _ 3r 2 + 9 ¸ rdrdθ = 3 _ 2π 0 _ 3 4 r 4 + 9 2 r 2 _ 2 0 dθ = 3 _ 2π 0 30dθ = 90θ[ 2π 0 = 180π Problema 4 Calcular el flujo del campo − → F (x, y, z) = (x + y 3 , 2y −e z , −3z −1) en la R la regi´on acotada por la frontera S x 2 + y 2 + 3z 2 = 1. Soluci´on: Usemos el teorema de la divergencia de Gauss para calcular el flujo, que afirma que, __ S − → F ´ nds = ___ R ∇ − → F dV Calculemos la divergencia del campo vectorial en la regi´ on R. 542 ∇ − → F = ∂ ∂x (x + y 3 ) + ∂ ∂y (2y −e z ) + ∂ ∂z (−3z −1) = 1 + 2 −3 = 0 Entonces el flujo del campo vectorial − → F a trav´es de S verifica __ S − → F ´ nds = ___ R 0 dV = 0 6.3.4. Teorema de Stokes Problema 1 Verificar el teorema de Stokes para − → F (x, y, z) = (−y 3 , x 3 , z 3 ) donde S es la porci´ on del plano x + y + z = 1 al interior del cilindro x 2 + y 2 = 1. Figura 6.19: S es la porci´on del plano x + y + z = 1 al interior del cilindro x 2 + y 2 = 1 Soluci´on. El teorema de Stokes afirma que _ C − → F d − → r = __ S ∇ − → F ´ ndS Calculemos ∇ − → F = ¸ ¸ ¸ ¸ ¸ ¸ ¸ ¸ i j k ∂ ∂x ∂ ∂y ∂ ∂z −y 3 x 3 z 3 ¸ ¸ ¸ ¸ ¸ ¸ ¸ ¸ = (0, 0, 3x 2 + 3y 2 ), y − → N = ∇(x + y + z −1) = (1, 1, 1) 543 __ S ∇ − → F ´ ndS = __ D ∇ − → F − → N dxdy = __ D 3(x 2 + y 2 ) dxdy donde D = ¦x, y) ∈ IR 2 /x 2 + y 2 ≤ 1¦ . Usando coordenadas polares : x = r cos θ y = rsenθ =⇒ [∂ (x, y)∂ (r, θ)[ = ¸ ¸ ¸ ¸ cos θ −rsenθ senθ r cos θ ¸ ¸ ¸ ¸ = r x 2 + y 2 ≤ 1 ⇐⇒ 0 ≤ r 2 ≤ 1 =⇒ 0 ≤ r ≤ 1 y 0 ≤ θ ≤ 2π Entonces la regi´on D ∗ = ¦(r, θ) /0 ≤ r ≤ 1 y 0 ≤ θ ≤ 2π¦ .Luego, si __ D 3(x 2 + y 2 ) dxdy = 3 _ 1 0 _ 2π 0 r 3 dθdr = 6π _ 1 0 r 3 dr = 3π 2 Problema 2 Verificar el teorema de Stokes para evaluar la integral de l´ınea _ C xdx + yz 2 dy + xzdz donde C es la intersecci´on de la semiesfera x 2 + y 2 + z 2 = 1, z ≥ 0 y el cilindro x 2 + y 2 = y. Soluci´on El teorema de Stokes afirma que _ C − → F d − → r = __ S ∇ − → F ´ ndS donde S es la regi´ on de la superficie de la semiesfera cuya frontera es la curva C. Calculemos directamente la integral de l´ınea, parametrizando la curva C mediante coordenadas cil´ındricas x = r cost, y = r sent, z = z. 544 Figura 6.20: Al sustituir en la ecuaciones de las superficies que definen la curva C, tenemos que z = √ 1 −r 2 , r = sent donde 0 ≤ t ≤ π. Luego, la ecuaci´ on de la curva es − → r (t) = _ sin t cos t, sin 2 t, _ 1 −sin 2 t _ , 0 ≤ t ≤ π Usando identidades trigonom´etricas sin 2 t = 1 2 (1−cos 2t), cos 2 t = 1 2 (1+ cos 2t) se tiene: − → r (t) = _ 1 2 sin 2t, 1 2 (1 −cos 2t), cos t _ , 0 ≤ t ≤ π − → r (t) = (cos 2t, −sin 2t, −sin t) , 0 ≤ t ≤ π Evaluemos la integral de linea: _ C xdx + yz 2 dy + xzdz = _ π 0 _ 1 2 sin 2t cos 2t − 1 2 (1 −cos 2t) cos 2 tsen2t − 1 2 sin 2t cos t sin t _ dt = _ π 0 _ 1 2 sin 2t cos 2t − 1 4 (1 −cos 2 2t) sin 2t − 1 4 sin 2 2t _ dt = _ π 0 _ 1 2 sin 2t cos 2t − 1 4 (1 −cos 2 2t) sin 2t − 1 4 _ 1 −cos 4t 2 __ dt = _ 1 4 sin 2 2t + 1 8 cos 2t − 1 24 cos 3 2t − 1 8 t + sin 4t 32 _ π 0 = − π 8 545 Por otra parte, debemos evaluar __ S ∇ − → F ´ ndS = __ D ∇ − → F − → NdA Como S es la regi´on de la superficie de la semiesfera cuya frontera es la curva C, una parametrizacion de S viene dada por − → T (r, θ) = _ r cos θ, rsenθ, √ 1 −r 2 _ , 0 ≤ r ≤ senθ, 0 ≤ θ ≤ π A continuaci´ on calculemos el vector normal a la superficie: − → N = − → T r − → T θ = ¸ ¸ ¸ ¸ ¸ ¸ ¸ i j k cos θ sin θ − r √ 1 −r 2 −r sin θ r cos θ 0 ¸ ¸ ¸ ¸ ¸ ¸ ¸ = _ r 2 cos θ √ 1 −r 2 , r 2 sin θ √ 1 −r 2 , r _ La orientaci´on del vector normal a la superficie es compatible con la orientaci´ on de su frontera C. Determinemos el rotor del campo vectorial ∇ − → F = ¸ ¸ ¸ ¸ ¸ ¸ ¸ ¸ i j k ∂ ∂x ∂ ∂y ∂ ∂z x yz xz ¸ ¸ ¸ ¸ ¸ ¸ ¸ ¸ = (−2yz, −z, 0), Por lo que la funci´ on compuesta queda ∇ − → F _ − → T (r, θ) _ = _ −2r sin θ √ 1 −r 2 , − √ 1 −r 2 , 0 _ Entonces, estamos en condiciones de calcular la integral de superficie, __ D ∇ − → F − → NdA = _ π 0 _ sin θ 0 _ −2r 3 sin θ cos θ −r 2 sin θ _ drdθ = − _ π 0 _ r 4 2 sin θ cos θ + r 3 3 sin θ _ sin θ 0 dθ = − _ π 0 _ sin 5 θ 2 cos θ + sen 4 θ 3 _ dθ = − _ sin 6 θ 12 _ π 0 − 1 3 _ π 0 sin 4 θdθ = − 1 3 _ π 0 sin 4 θdθ 546 Puesto que el integrando queda sin 4 θ = _ 1 −cos 2θ 2 _ 2 = 1 4 _ 1 −2 cos 2θ + cos 2 2θ _ = 1 4 _ 1 −2 cos 2θ + 1 + cos 4θ 2 _ = 1 4 _ 3 2 −2 cos 2θ + cos 4θ 2 _ Podemos calcular − 1 3 _ π 0 sin 4 θdθ = − 1 3 _ π 0 1 4 _ 3 2 −2 cos 2θ + cos 4θ 2 _ dθ = − 1 3 _ 3θ 2 −sin 2θ + sin 4θ 2 _ π 0 = − π 8 Problema 3 Sea − → F (x, y, z) = (x, y, z), calcular la integral de l´ınea _ C − → F dr, siendo C la curva intersecci´on de las superficies S 1 dada por ecuaci´on z = x 2 + 2y 2 y S 2 por la ecuaci´ on z = 4 −x 2 . Figura 6.21: C la curva intersecci´on de las superficies S 1 dada por ecuaci´on z = x 2 + 2y 2 y S 2 por la ecuaci´on z = 4 −x 2 Soluci´on El teorema de Stokes afirma que _ C − → F d − → r = __ S ∇ − → F ´ ndS 547 donde la curva C es la frontera de la superficie S dada por S = ¦(x, y, z) ∈ IR3 : x 2 + y 2 ≤ 2, z = 4 −x 2 ¦ Calculemos ∇ − → F = ¸ ¸ ¸ ¸ ¸ ¸ ¸ ¸ i j k ∂ ∂x ∂ ∂y ∂ ∂z x y z ¸ ¸ ¸ ¸ ¸ ¸ ¸ ¸ = (0, 0, 0). Por lo tanto __ S ∇ − → F ´ ndS = __ S (0, 0, 0) ´ ndS Lo que indica que estamos frente a un campo conservativo. Problema 4 Calcular __ S ∇ − → F ´ ndS, siendo − → F = (xz + yz 2 + x, xyz 3 + y, x 2 z 4 ) y S = S 1 ∪ S 2 donde S 1 es la superficie cil´ındrica x 2 +y 2 = 1, 0 ≤ z ≤ 1 y S 2 es la semiesfera x 2 + y 2 + z 2 = 1, z ≥ 0. Figura 6.22: Superficie S = S 1 ∪S 2 donde S 1 es la superficie cil´ındrica x 2 +y 2 = 1, 0 ≤ z ≤ 1 y S 2 es la semiesfera x 2 + y 2 + z 2 = 1, z ≥ 0 Soluci´on. El teorema de Stokes afirma que __ S ∇ − → F ´ ndS = _ C − → F d − → r donde la curva C es la frontera de la superficie S dada por x 2 + y 2 = 1 Parametizando la circunferencia, queda: − → r (t) = (cos t, sin t, 0) , t ∈ [0, 2π] − → r (t) = (−sin t, cos t, 0) , t ∈ [0, 2π] . 548 − → F ( − → r (t)) = (cos t, sin t, 0) Calculemos la integral _ C − → F d − → r = _ 2π 0 (cos t, sent, 0) (−sin t, cos t, 0) dt = 0 Problema 5 Determine la circulaci´on del campo − → F = (x 2 −y, 4z, x 2 ) alrededor de la curva C dada por la intesecci´ on del plano z = 4 y el cono z = _ x 2 + y 2 ,recorrida en sentido positivo. Figura 6.23: Plano z = 4 y cono z = _ x 2 + y 2 Soluci´on El teorema de Stokes afirma que _ C − → F d − → r = __ S ∇ − → F ´ ndS donde la curva C es la frontera de la superficie S dada por S = ¦(x, y, z) ∈ IR3 : x 2 + y 2 ≤ z 2 , z ≤ 4¦ Parametrizando el cono se tiene − → T (r, θ) = (r cos θ, r sin θ, r) , donde D = ¦(r, θ) /0 ≤ r ≤ 2, 0 ≤ θ ≤ 2π¦ A continuaci´ on calculemos el vector normal a la superficie: − → N = − → T r − → T θ = ¸ ¸ ¸ ¸ ¸ ¸ i j k cos θ sin θ 1 −r sin θ r cos θ 0 ¸ ¸ ¸ ¸ ¸ ¸ = (−r cos θ, −r sin θ, r) 549 La orientaci´ on del vector normal a la superficie es hacia el exterior del cono Adem´ as ∇ − → F = ¸ ¸ ¸ ¸ ¸ ¸ ¸ ¸ i j k ∂ ∂x ∂ ∂y ∂ ∂z x 2 −y 4z x 2 ¸ ¸ ¸ ¸ ¸ ¸ ¸ ¸ = (−4, −2x, 1). Luego la funci´ on compuesta ∇ − → F _ − → T (r, θ) _ es ∇ − → F _ − → T (r, θ) _ = (−4, −2r cos θ, 1) Por lo tanto __ S ∇ − → F ´ ndS = __ D ∇ − → F − → N drdθ = _ _ D (−4, −2r cos θ, 1) (−r cos θ, −r sin θ, r) drdθ = _ 2 0 _ 2π 0 (4r cos θ + r sin θ + r) dθdr = 4π 6.4. Ejercicios Propuestos 6.4.1. ´ Area de una superficie 1. Problema Calcule el ´ area de la superficie de x 2 + y 2 + z 2 = a 2 , a > 0, z ≥ 0. Respuesta A(R) = 2πa 2 2. Problema Calcule el ´ area de la superficie del sector del plano ax +by +cz +d = 0 que queda dentro del cilindro x 2 + y 2 = r 2 (c ,= 0, r > 0) . Respuesta A(R) = πr 2 √ a 2 + b 2 + c 2 c 550 Figura 6.24: Superficie del sector del plano ax + by + cz + d = 0 que queda dentro del cilindro x 2 + y 2 = r 2 (c ,= 0, r > 0) Figura 6.25: 3. Problema Calcule el ´ area del helicoide − → r (u, v) = (ucos v, usin v, v) para 0 ≤ u ≤ 1, 0 ≤ v ≤ 2π Respuesta A(R) = π(ln( √ 2 + 1) + √ 2 4. Problema Hallar el ´ area de la superficie S descrita por la parametrizaci´ on: − → r : D ⊂ R 2 →R 3 ; − → r (u; v) = (ucos v; usin v; u 2 ) ; donde D =¦(u, v) ∈ IR 2 /0 ≤ u ≤ 4, 0 ≤ v ≤ 2π¦ . Respuesta I = 1 12 _ (65) 3/2 −1 _ 551 Figura 6.26: 5. Problema Obtener el ´ area de la superficie S , frontera de = ¦(x, y, z) ∈ IR 3 : (z −1) 2 ≤ x 2 + y 2 y x 2 + y 2 + z 2 ≤ 1¦ Figura 6.27: Respuesta ´ Area de la superficie S = S esf ∪ S cono = 2π + π √ 2 6. Problema Calcule el ´ area de la porci´on de superficie c´ onica z 2 = x 2 + y 2 situada por encima del plano z = 0 y limitada por la esfera x 2 + y 2 + z 2 = 2ax Respuesta I = √ 2 4 πa 2 552 Figura 6.28: 7. Problema Calcule el ´ area de la superficie S de una pelota de rugby formada por la rotaci´ on de la curva x = cos z, y = 0, − π 2 ≤ z ≤ π 2 alrededor del eje z. Respuesta Ecuacion param´etrica de la superficie − → r (u, v) = (cos ucos v, cos usin v, u) A = 2π _√ 2 + ln _ 1 + √ 2 _¸ 6.4.2. Integrales de funciones escalares sobre superficie 1. Problema Calcule la integral _ S x 2 zdS ; siendo S la superficie externa de x 2 + y 2 = a 2 comprendida entre z = −2 y z = 2. Figura 6.1: Respuesta 553 _ S x 2 zdS = 0 2. Problema Calcule la integral de superficie _ S (x 4 − y 4 + y 2 z 2 − x 2 z 2 + 1)dS . Si la superficie S es la hoja superior del cono z 2 = x 2 + y 2 que est´ a en el interior del cilindro (x −1) 2 + y 2 = 1. Figura 6.2: Superficie S, hoja superior del cono z 2 = x 2 +y 2 que est´a en el interior del cilindro (x −1) 2 + y 2 = 1 Respuesta A = √ 2π 554 3. Problema Calcule la integral de superficie I = _ S zdS, donde S es la superficie parametrizada por − → r (x, y) = (x, y, 2x + 3y + 2a), (x, y) ∈ D tal que D = ¦(x, y) ∈ IR 2 /x 2 + y 2 ≤ a 2 ¦ . Figura 6.3: Aca voy Respuesta I = 2 √ 14πa 3 4. Problema Calcule la integral de superficie I = _ S _ 1 −x 2 −y 2 dS, donde S es una pelota de rugby formada por la rotaci´ on de la curva x = cos z, y = 0, − π 2 ≤ z ≤ π 2 alrededor del eje z. Respuesta Superficie parametrizada − → r (u, v) = (cos ucos v, cos usin v, u) , D = _ (u, v) / − π 2 ≤ u ≤ π 2 , 0 ≤ v ≤ 2π _ I= 4π 3 _ 2 √ 2 −1 _ 6.4.3. Integral de Flujo 1. Problema Calcule la integral del campo vectorial − → F (x, y, z) = (x, y, z) a trav´es de la superficie lateral del paraboloide x 2 + y 2 = 2az , con 0 ≤ z ≤ 2a. Respuesta I = −4πa 3 2. Problema Calcule la integral del campo vectorial − → F (x, y, z) = (yz, x, z 2 ) a trav´es de la superficie del cilindro parab´ olico y = x 2 , con 0 ≤ x ≤ 1, 0 ≤ z ≤ 4,donde la primera componente de − → n es positiva. 555 (a) Campo vectorial (b) paraboloide Figura 6.1: (a) Campo vectorial (b) sector de cilindro parab´olico Figura 6.2: Respuesta Se tiene que − → n = (2x, −1, 0) I = 2 3. Problema Calcule la integral del campo vectorial − → F (x, y, z) = (0, yz, z 2 ) a trav´es de la superficie S cortada del cilindro y 2 + z 2 = 1, por los planos x = 0; ≤ x = 1. Respuesta Se tiene que − → n = (0, y, z) I = 2 556 (a) Campo vectorial (b) Cilindro cortado por dos planos Figura 6.3: 6.4.4. Teorema de la divergencia de Gauss 1. Problema Sea − → F (x, y, z) = (xz, yz, −z 2 ). Calcule __ S − → F ´ nds ; siendo S la cara externa del paraboloide x 2 + y 2 = 3z; entre z = 0 y z = 1. Figura 6.1: Paraboloide x 2 + y 2 = 3z limitado por los planos z = 0 y z = 1 a) Directamente. b) Aplicando el teorema de Gauss. Respuesta. I = −3π 2. Problema Calcular el flujo __ S − → F ´ nds, del campo vectorial − → F = (x, y, z) a trav´es de la superficie S del elipsoide ax 2 + by 2 + cz 2 = 1 557 Respuesta. __ S − → F ´ nds = 2π √ abc 3. Problema Calcular el flujo __ S − → F ´ nds,del campo vectorial − → F = (2xy 2 , z 3 , −x 2 y) a trav´es la superficie S formada por el hemisferio superior de la esfera x 2 + y 2 + z 2 = a 2 y el cilindro x 2 + y 2 = a 2 (a) Campo vectorial (b) Superficie formada por el hemisfe- rio superior de la esfera y el cilindro Figura 6.2: Respuesta. __ S − → F ´ nds = 4 15 πa 5 558 4. Problema Calcular el flujo __ S − → F ´ nds, del campo vectorial − → F = _ 0, tan z + e sin xz , y 2 _ a trav´es de la superficie S del semi-elipsoide 2x 2 +3y 2 +z 2 = 6, z ≥ 0, con la normal apuntando hacia el exterior. (a) Campo vectorial (b) Semi-elipsoide Figura 6.3: Respuesta. __ S − → F ´ nds = _ 3 2 π 559 5. Problema Calcular el flujo __ S − → F ´ nds, del campo vectorial − → F = (x, y, z) (x 2 + y 2 + z 2 ) 3/2 a trav´es de la superficie S dada por el paraboloide z = 2−x 2 −y 2 ,tales z ≥ 0, con la normal apuntando hacia el exterior. Pendiente Respuesta. __ S − → F ´ nds = π _ 2 − √ 2 _ 6. Problema Sea S la superficie del paraboloide z = x 2 +(y −1) 2 , interior al cilindro x 2 + (y − 2) 2 = 3. Sea − → F el campo vectorial − → F (x, y, z) = (y, x, xz) . Calcular la integral __ S ∇ − → F ´ nds,utilizando el teorema de Gauss, donde ´ n es la normal exterior al paraboloide. (a) Campo vectorial (b) Paraboloide interior al cilindro Figura 6.4: Respuesta Colocando una tapa T a la superficie S, de modo que S ∗ = S T sea una superficie cerrada, podemos aplicar el teorema de Gauss __ S ∇ − → F ´ nds = − __ T ∇ − → F ´ nds = −2π 7. Problema Sea S la porci´ on del paraboloide z + 1 = x 2 + y 2 , situada debajo del plano z = 1, y sea − → F (x, y, z) = (0, x −2yz, x 2 ). Hallar __ S ∇ − → F ´ nds, 560 donde ´ n es la normal exterior al paraboloide, utilizando el teorema de Gauss. (a) Campo vectorial (b) Paraboloide situada de- bajo del plano Figura 6.5: Respuesta Colocando una tapa T a la superficie S, de modo que S ∗ = S T sea una superficie cerrada, podemos aplicar el teorema de Gauss __ S ∇ − → F ´ nds = − __ T ∇ − → F ´ nds = −2π 6.4.5. Teorema de Stokes 1. Problema Sea S el sector del paraboloide z = x 2 + y 2 , situado en el primer octante , limitado por el plano z = 1 y los planos principales, y sea − → F = (y −z, z −x, x −y) . Calcular la integral de l´ınea _ C − → F d − → r . Respuesta _ C − → F d − → r . = 8 3 − π 2 2. Problema Para el campo vectorial − → F = (x 2 + ye z , y 2 + ze x , z 2 + xe y ) definida so- bre S : x 2 +y 2 +z 2 = a 2 con z ≥ 0.Calcular el flujo del rotacional de − → F a trav´es de S. Respuesta _ C − → F d − → r = __ S ∇ − → F ´ nds = −πa 2 561 (a) Campo vectorial (b) Sector de paraboloide, limita- do por planos Figura 6.1: (a) Campo vectorial (b) hemisferio superior de la esfera Figura 6.2: 3. Problema Dado el campo vectorial − → F (x, y, z) = (y +sin x, z 2 +cos y, x 3 ) y la curva − → r (t) = (sin t, cos t, sin 2t); t ∈ [0; 2π]; halle _ C − → F d − → r utilizando el teorema de Stokes. Respuesta I = −π 2 562 (a) Campo vectorial (b) paraboloide hiperb´olico Figura 6.3: 4. Problema Calcule el flujo del campo − → F (x, y, z) = (x, y, z) a trav´es de la superficie del s´olido limitado por x 2 + y 2 = 9, z = 0, y z = 0. a) Directamente. b) Aplicando el Teorema de Stokes Respuesta I = 81π 5. Problema Calcule la integral _ C y 2 dx+xydy +xzdz, siendo C la curva intersecci´ on del cilindro x 2 + y 2 = 2y y el plano y = z. a) Directamente. b) Aplicando el teorema de Stokes. Figura 6.4: Respuesta 563 I = 0 6. Problema Calcule la integral I = _ C (y −1)dx +z 2 dy +ydz, aplicando el teorema de Stokes, siendo C la curva intersecci´on de las superficies x 2 +y 2 = z 2 2 , z = y + 1 Figura 6.5: Respuesta I = − √ 2π 6.5. Aplicaciones En esta secci´on presentamos algunas aplicaciones simples que ilustran la utilizaci´ on de las, integrales de lineas, integral de flujo y de los teoremas de Gauss , Stokes y Green . 6.5.1. Aplicaciones Integral de Flujo Problema 1 Calcular el flujo del campo el´ectrico producido por una carga Q en el origen, a trav´es del manto de la esfera S de radio R orientado seg´ un la normal exterior. Soluci´on El campo el´ectrico producido por la carga Q viene dado por 564 − → E = Q 4πε o ´ r r 2 donde ε o una constante universal . De esta forma se obtiene el siguiente flujo el´ectrico __ S − → E d − → S = __ S Q 4πε o ´ r R 2 ´ rdS Parametrizando la superficie en coordenadas esf´ericas se tiene − → Φ (φ, θ) = (Rsin φcos θ, Rsin φsin θ, Rcos φ) , 0 ≤ φ ≤ π, 0 ≤ θ ≤ 2π =⇒ _ _ _ − → Φ φ − → Φ θ _ _ _ = R 2 sin Φ __ S Q 4πε o ´ r R 2 ´ rdS = _ 2π 0 _ π 0 Q 4πε o 1 R 2 R 2 sin φdφdθ = Q ε o Por tanto __ S − → E d − → S = Q ε o Corresponde a la formulaci´ on integral de la Ley de Gauss, que se aplica tambi´en a distribuciones de carga m´ as generales. Problema 2 Calcular el flujo del campo el´ectrico generado por una carga Q en el origen, sobre el plano infinito x = 1. Soluci´on En este caso la normal es constante ´ N = ´ i . El campo el´ectrico producido por una carga Q en coordenadas cartesianas, esta dado por − → E = Q 4πε o ´ r r 2 = Q 4πε o (x, y, z) [x 2 + y 2 + z 2 ] 3/2 El flujo del campo viene dado por __ S − → E ´ idS = __ S Q 4πε o (1, y, z) [x 2 + y 2 + z 2 ] 3/2 (1, 0, 0) dS = Q 4πε o _ ∞ −∞ _ ∞ −∞ dxdy [1 + y 2 + z 2 ] 3/2 565 Utilizando el cambio de variables a coordenadas polares, tenemos y = rcosθ, z = rsenθ =⇒ y 2 + z 2 = r 2 y el jacobiano de transformaci´ on es ¸ ¸ ¸ ¸ ∂ (y, z) ∂ (r, θ) ¸ ¸ ¸ ¸ = r Reemplazando el integrando, obtenemos Q 4πε o _ ∞ 0 _ 2π 0 rdθdr [1 + r 2 ] 3/2 = Q 2ε o _ ∞ 0 rdr [1 + r 2 ] 3/2 = Q ε o _ − _ 1 + r 2 _ −1/2 _ ∞ 0 = Q ε o Problema 3 Si − → E(x, y, z) = (2x, 2y, 2z) es un campo el´ectrico, encontrar el flujo de − → E que sale a trav´es de la superficie cerrada S que consta de la semiesfera x 2 + y 2 + z 2 = 1, z ≥ 0, y su base. Soluci´on Como los puntos de la circunferencia x 2 + y 2 = 1, z = 0, son singulares, debemos descomponer la superficie en dos partes, de modo que llamaremos S 1 a la semiesfera y S 2 al c´ırculo que forma la tapa inferior. Una parametrizaci´ on de S 1 viene dada por la funci´on − → r 1 (u, v) = ( sin ucos v, sin usin v, cos u) donde D 1 = _ (u, v) ∈ IR 3 /0 ≤ u ≤ π 2 , 0 ≤ v ≤ 2π _ El vector normal exterior a la superficie es − → T u − → T v = (sin 2 ucos v, sin 2 usin v, cos usin u) De este modo, el flujo a trav´es de S 1 es __ S 1 − → E ´ ndS = __ D 2(cos 2 v sin 3 u + sin 2 v sin 3 u + sin ucos 2 u)dudv = _ 2π 0 _ π/2 0 2 sin ududv = 4π An´ alogamente, si parametrizamos la superficie S 2 por − → r 2 (u, v) = (ucosv, u sin v, 0) 566 donde D 2 = ¦(u, v) ∈ IR 2 /0 ≤ u ≤ 1, 0 ≤ v ≤ 2π¦ − → r u − → r v = ¸ ¸ ¸ ¸ ¸ ¸ i j k cos v sin v 0 −usin v u cos v 0 ¸ ¸ ¸ ¸ ¸ ¸ = (0, 0, u) El flujo a trav´es de la superficie S 2 es __ S 2 − → E ´ ndS = __ D 2(ucos v, u sin v, 0) (0, 0, u) dudv = __ D 0 dudv = 0 Por lo tanto, la suma de ambos flujos da como resultado __ S − → E ´ ndS = 4π Problema 4 Si − → F (x, y, z) = (0, √ y, 0) representa la velocidad de un fluido. Calcular la raz´on del flujo de dicho fluido que atraviesa la superficie S = ¦(x, y, z) : x 2 + z 2 = y, 0 ≤ y ≤ 1¦. Figura 6.6: Superficie S = ¦(x, y, z) : x 2 + z 2 = y, 0 ≤ y ≤ 1¦ Soluci´on 567 En primer lugar, consideremos la siguiente parametrizaci´on de la superficie: r(u, v) = (ucos v, u 2 , usin v), 0 ≤ u ≤ 1, 0 ≤ v ≤ 2π. El vector normal a la superficie viene dado por: − → T u − → T v = ¸ ¸ ¸ ¸ ¸ ¸ i j k cos v 2u sin v −usin v 0 ucos v ¸ ¸ ¸ ¸ ¸ ¸ = = (2u 2 cos v, −u, 2u 2 sin v) Entonces el flujo esta dado por, _ _ S − → F ´ ndS = _ 2π 0 _ 1 0 (0, u, 0) (2u 2 cos v, −u, 2u 2 sin v)dudv = _ 2π 0 _ 1 0 (−u 2 )dudv = _ 2π 0 _ − u 3 3 _ 1 0 dv = − 2π 3 El signo negativo del resultado es debido a que el vector normal tiene sentido opuesto al del movimiento del fluido (la componente ´ j de − → F es positiva pero la de ´ n es negativa). As´ı pues, la raz´on de flujo que entra a la superficie es de −2π 3 . 6.5.2. Aplicaci´on del teorema de Gauss Problema 1 Suponga que la temperatura de un punto de una esfera S es proporcional al cuadrado de la distancia de dicho punto al centro de la esfera. Se sabe que el campo estacionario de temperaturas − → J asociado a este potencial queda definido por la expresi´ on − → J = −∇T, donde k > 0 representa la conductivi- dad t´ermica del medio y T la temperatura. Calcular directamente la raz´on total del flujo de calor que atraviesa la esfera, suponiendo que tiene centro el origen y radio a. Soluci´on Debemos calcular _ _ S − → J ´ ndS = _ _ D − → J ( − → r (u, v)) _ − → T u − → T v _ dudv Consideremos la parametrizaci´on de S dada por: 568 − → r (u, v) = (a sin ucos v, a sin usin v, a cos u) donde D = ¦(u, v) ∈ IR 3 /0 ≤ u ≤ π, 0 ≤ v ≤ 2π¦ Determinemos el flujo de calor. Como T(x, y, z) = C(x 2 +y 2 +z 2 ), entonces − → J (x, y, z) = −k∇T(x, y, z) = −kC(2x, 2y, 2z) El vector normal exterior a la superficie es − → T u − → T v = (a 2 sin 2 ucos v, a 2 sin 2 usin v, a cos usin u) y la raz´ on de flujo a trav´es de la esfera viene dada por la integral _ _ S − → J ´ ndS = − _ 2π 0 _ π 0 kC∇T _ − → T u − → T v _ dudv = − _ 2π 0 _ π 0 2kCa 3 sin ududv = −8kCπa 3 Problema 2 Suponga que la temperatura en una regi´ on de IR 3 est´ a dada por T(x, y, z) = C(x 2 + y 2 + z 2 ). Se sabe que el campo estacionario de temperaturas − → J asociado a este potencial queda definido por la expresi´on − → J = −∇T.Suponga que la regi´on contiene a la esfera S centrada en el origen y de radio a. Calcular el flujo de calor que sale a trav´es del casquete esf´erico S,usando el teorema de Gauss , dada la conductividad t´ermica > 0 del material. Soluci´on Sea Ω un abierto de IR 3 de frontera regular S orientada seg´ un la normal exterior ´ N El flujo de calor producido por el campo estacionario de temperaturas es __ S − → J ´ NdS = ___ V ∇ − → J dV = −κ ___ V ∇ (∇T) dV = −κ ___ V ∇ 2 T dV 569 Calculemos el Laplaciano de temperaturas ∇ 2 T = ∂ 2 x 2 ∂x 2 + ∂ 2 y 2 ∂y 2 + ∂ 2 z 2 ∂z 2 = C(2 + 2 + 2) = 6C Reemplazando el integrando, queda __ S − → J d − → S = −6κC ___ Ω dV = −6κC _ 4 3 πR 3 _ = −8κCπR 3 Problema 3 El hecho de que la temperatura decrece hacia el origen hace que el gra- diente apunte en esa direcci´ on de modo que el flujo neto de calor que entra a la esfera es negativo. El filtro de una m´aquina de lavar ropa tiene la forma c´onica descrita por el conjunto Ω = _ (x, y, z) ∈ IR 3 / _ x 2 + y 2 ≤ z ≤ 3 _ , por el circula una corriente de agua, cuyo campo de velocidades esta dado por la expresi´ on − → F (x, y, z) = (2yz cos (y 2 ) , 2x cos (x 2 ) , 1) . a) Muestre que la cantidad de agua en el interior del filtro se mantiene constante, suponiendo que la densidad el agua es ρ = 1 gr/cm 3 . b) Usando el teorema de Stokes, calcule el flujo total de agua trav´es de las paredes del filtro Soluci´ on Por el teorema de la divergencia de Gauss, se tiene que el flujo total de agua a trav´es de las paredes del filtro es __ S − → F ´ nds = ___ Ω ∇ − → F dV Calculemos la divergencia del campo de velocidades ∇ − → F = _ ∂ ∂x _ 2yz cos _ y 2 __ + ∂ ∂x _ 2x cos _ x 2 __ + ∂ ∂x (1) _ =⇒ ∇ − → F = 0 570 Figura 6.7: Ω = _ (x, y, z) ∈ IR 3 / _ x 2 + y 2 ≤ z ≤ 3 _ Como ∇ − → F = 0,el flujo total de agua a trav´es de las paredes del filtro es nulo. Por lo tanto la cantidad de agua que entra al filtro es igual a la que sale, entonces la cantidad de agua en el interior del filtro se mantiene constante. b) Sea S la pared del manto de Ω .Como − → F es un campo de divergencia nula en IR 3 , entonces − → F es un campo rotacional, es decir, existe un campo vectorial − → R tal que − → F = ∇ − → R. En consecuencia, se busca − → R que satisfaga la condicion anterior. Sea − → R = (R 1 , R 2 , R 2 ) tal que ∂R 3 ∂y − ∂R 2 ∂z = 2yz cos _ y 2 _ ∂R 1 ∂z − ∂R 3 ∂x = 2xz cos _ x 2 _ ∂R 2 ∂x − ∂R 1 ∂y = 1 Este problema tiene muchas soluciones.Formulemos un modelo en que R 1 = 0, entonces ∂R 3 ∂y − ∂R 2 ∂z = 2yz cos _ y 2 _ − ∂R 3 ∂x = 2xz cos _ x 2 _ ∂R 2 ∂x − = 1 Integrando parcialmente las dos ´ ultimas ecuaciones se obtiene 571 ∂R 3 ∂y − ∂R 2 ∂z = 2yz cos _ y 2 _ R 3 = −zsen _ x 2 _ + f (x, y) R 2 = x + g (y, z) Se tiene m´ as de una soluci´ on, consideremos g (x, y) = 0, luego queda ∂R 3 ∂y = 2yz cos _ y 2 _ R 3 = −zsen _ x 2 _ + f (x, y) R 2 = x Derivando la segunda ecuaci´on parcialmente con respecto a y y se iguala con la primera ∂ ∂y f (x, y) = 2yz cos _ y 2 _ Integrando parcialmente con respecto a y esta ´ ultima expresi´on produce f (x, y) = zsen _ y 2 _ + h(z) Para simplificar consideremos h(z) = 0,por consiguiente R 3 = −zsen _ x 2 _ + zsen _ y 2 _ Finalmente, el campo − → R queda: − → R = _ 0, x, −zsen _ x 2 _ + zsen _ y 2 __ El teorema de Stokes afirma que __ S ∇ − → R ´ ndS = _ C − → R d − → r El flujo que produce el campo − → R a trav´es del manto es igual a la integral de l´ınea sobre la frontera C que limita la supercicie S. C = ¦(x, y, z) ∈ IR 3 / x 2 + y 2 = 9, z = 3¦ Parametrizando la curva se tiene − → r (t) = (3 cos t, 3set, 3) , t ∈ [0, 2π] _ C − → R d − → r = _ 2π 0 − → R ( − → r (t)) − → r (t) dt 572 = _ 2π 0 _ 0, 3 cos t, 27sen 3 t −27sent cos 2 t _ (−3sent, 3 cos t, 0) dt = _ 2π 0 9 cos 2 tdt = 9 _ 2π 0 _ 1 + cos 2t 2 _ dt = 9 2 _ sen(2t) 2 + t _ 2π 0 = 9π 6.5.3. Aplicaci´on teorema de Stokes Problema 1 Calcular el trabajo producido por la fuerza − → F (x, y, z) = (−y 3 , x 3 , −z 3 ) ,sobre la trayectoria recorrida en el sentido positivo, dada por la intersecci´ on de las superficies x + y + z = a, x 2 + y 2 = a 2 . Figura 6.8: Soluci´on Como el campo es continuo y la trayectoria es cerrada, podemos calcular el trabajo aplicando el teorema de Stokes. W = _ C − → F d − → r = __ S _ ∇ − → F _ ´ n dS = __ D _ ∇ − → F _ − → n | − → n | | − → n | dA Al parametrizar en coordenadas cartesianas el plano que contiene a la curva obtenemos − → φ (x, y) = (x, y, a −x −y), donde (x, y) ∈ D tal que x 2 + y 2 ≤ a 2 . 573 Calculemos el integrando del lado derecho de la formula anterior ∇ − → F = ¸ ¸ ¸ ¸ ¸ ¸ ¸ ¸ i j k ∂ ∂x ∂ ∂y ∂ ∂z −y 3 x 3 −z 3 ¸ ¸ ¸ ¸ ¸ ¸ ¸ ¸ = (0, 0, 3x 2 + 3y 2 ) El vector normal al plano esta dado por − → n = ∇φ = (1, 1, 1) , donde φ = x + y + z −a = 0 En consecuencia, al sustituir t´erminos en al ´ ultima integral queda __ D _ ∇ − → F _ − → n dxdy = __ D _ 0, 0, 3x 2 + 3y 2 _ (1, 1, 1) dxdy = 3 __ D (x 2 + y 2 )dxdy Si hacemos cambio a coordenadas polares obtenemos 3 _ 2π 0 _ a 0 _ ρ 2 _ (ρ) dρdθ = 3 _ 2π 0 _ ρ 4 4 _ a 0 dθ = 3 a 4 4 _ 2π 0 dθ = 6πa 4 4 Corresponde al trabajo total realizado por el campo de fuerzas. Problema 2 Calcular la circulaci´ on del campo de velocidades de un flu´ıdo dado por − → F (x, y, z) = (arctan(x 2 ), 3x, e 3z tan (z)) a lo largo de la intersecci´ on de la esfera x 2 + y 2 + z 2 = 4, con el cilindro x 2 + y 2 = 1, con z > 0. Soluci´on. La circulaci´on de un campo vectorial es su integral de l´ınea a lo largo de una curva cerrada. Vemos que el campo vectorial es bastante complejo, por lo que calcular directamente la integral de linea puede resultar engorroso. Se espera que al ocupar el teorema de Stokes se simplifique el calculo. Entonces _ C − → F d − → r = __ S _ ∇ − → F _ ´ n dS 574 Figura 6.9: Calculemos el integrando del lado derecho de la formula anterior ∇ − → F = ¸ ¸ ¸ ¸ ¸ ¸ ¸ ¸ i j k ∂ ∂x ∂ ∂y ∂ ∂z arctan(x 2 ) 3x e 3z tan(z) ¸ ¸ ¸ ¸ ¸ ¸ ¸ ¸ = (0, 0, 3) Luego, queda _ C − → F d − → r = __ D (0, 0, 3) − → n dA Para determinar el vector normal parametricemos la superficie en coor- denadas cil´ındricas x = ρ cos θ y = ρ sin θ z = _ 4 −ρ 2 _ _ _ =⇒ − → r (ρ, θ) = _ ρ cos θ, ρ sin θ, _ 4 −ρ 2 _ donde (ρ, θ) ∈ D tal que 0≤ ρ ≤ 1, 0 ≤ θ ≤ 2π Luego − → n = ( − → r ρ − → r θ ) = ¸ ¸ ¸ ¸ ¸ ¸ ¸ ¸ i j k cos θ sin θ − ρ _ 4 −ρ 4 −ρ sin θ ρ cos θ 0 ¸ ¸ ¸ ¸ ¸ ¸ ¸ ¸ = _ ρ 2 cos θ _ 4 −ρ 4 , ρ 2 sin θ _ 4 −ρ 4 , ρ _ Reemplazando t´erminos en el integrando se tiene 575 _ C − → F d − → r = __ D (0, 0, 3) _ ρ 2 cos θ _ 4 −ρ 4 , ρ 2 senθ _ 4 −ρ 4 , ρ _ dρdθ = _ 2π 0 _ 1 0 3ρdρdθ = _ 2π 0 3 _ ρ 2 2 _ 1 0 dθ = 3 2 _ 2π 0 dθ = 3π 6.5.4. Aplicacion teorema de Green Problema 1 Usando el teorema de Green determinar el momento de inercia de una arandela homog´enea de densidad superficial σ,que tiene masa M, radio inter- no a y radio externo b; respecto de uno de sus di´ ametros. Figura 6.10: regi´on R = _ (x, y) ∈ IR 2 / a 2 ≤ x 2 + y 2 ≤ b 2 _ Soluci´on. Determinaremos el momento de inercia con respecto al di´ametro que es colineal con el eje x. El momento de inercia con respecto a ´este eje esta dado por I x = __ R σy 2 dA donde σ es la densidad superficial. 576 La regi´on R = ¦(x, y) ∈ IR 2 / a 2 ≤ x 2 + y 2 ≤ b 2 ¦ , no es simplemente conexa, sin embargo, se puede extender el teorema de Green, sumando inte- grales de l´ıneas sobre fronteras separadas, de un n´ umero finito de subregiones (simplemente conexas), para construir una integral sobre una sola frontera. __ R _ ∂Q ∂x − ∂P ∂y _ dxdy = _ C 1 Pdx + Qdy − _ C 2 Pdx + Qdy = _ C 1 Pdx + Qdy + _ −C 2 Pdx + Qdy Buscamos P y Q, tales que ∂Q ∂x − ∂P ∂y = y 2 Consideremos en este caso Q = 0 =⇒ − ∂P ∂y = y 2 =⇒ P = − y 3 3 Asi, el momento de inercia queda: I x = __ R σy 2 dA = σ __ C 1 − y 3 3 dx − _ C 2 − y 3 3 dx _ = σ __ C 1 − y 3 3 dx + _ C 2 y 3 3 dx _ Parametrizamos ambas curvas: C 1 : _ x = b cos t y = b sin t =⇒ _ dx = −b sin tdt dy = b cos tdt con 0 ≤ t ≤ 2π C 2 : _ x = a cos t y = a sin t =⇒ _ dx = −a sin tdt dy = a cos tdt con 0 ≤ t ≤ 2π Reemplazando t´erminos en la ´ ultima integral, queda: I x = σ __ 2π 0 − b 3 sin 3 t 3 (−b sin t)dt + _ 2π 0 a 3 sin 3 t 3 (−a sin t)dt _ = σ (b 4 −a 4 ) 3 _ 2π 0 sin 4 tdt = σ (b 4 −a 4 ) 3 _ 2π 0 sin 2 t _ 1 −cos 2 t _ dt Aplicando las identidades trigonom´etricas: cos 2 t = (1 + cos 2t) 2 , sen 2 t = (1 −cos 2t) 2 , obtenemos 577 I x = σ (b 4 −a 4 ) 3 _ 2π 0 1 4 [(1 −cos 2t)(1 −cos 2t)] dt = σ (b 4 −a 4 ) 3 _ 3t 8 − sen2t 4 + sen4t 32 _ 2π 0 = σ (b 4 −a 4 ) 3 _ 6π 8 _ = πσ(b 2 −a 2 ) (b 2 + a 2 ) 4 = M(b 2 + a 2 ) 4 Problema 2 Un fluido de densidad constante gira alrededor del eje z con velocidad − → v = w(−y ´ i+x ´ j) donde w es una constante positiva llamada rapidez angular, muestre que la circulaci´ on del campo de velocidades es _ c v d − → r = 2πR 2 w Soluci´on La circulaci´on de un campo vectorial es su integral de l´ınea a lo largo de una curva cerrada. Por el teorema de Stokes, la circulaci´ on de − → v alrededor de la circunferencia C de radio R que acota a un disco S en el plano xy , esta dado por Calculemos ∇ − → v = ¸ ¸ ¸ ¸ ¸ ¸ ¸ ¸ i j k ∂ ∂x ∂ ∂y ∂ ∂z −wy wx 0 ¸ ¸ ¸ ¸ ¸ ¸ ¸ ¸ = (0, 0, (w −(−w)) = (0, 0, 2w) Luego, (∇ − → v ) ´ kdS = 2w ´ k ´ kdS = 2wdxdy.Entonces _ C − → v d − → r = __ S (∇ − → v ) ´ k dS = __ S 2wdxdy = 2w _ πR 2 _ Problema 3 Sea el campo de fuerzas − → F (x, y) = (2xy 2 +y, 2x 2 y + x 2 2 +x), demostrar que en cualquier camino cerrado sim´etrico con respecto al eje y, la circulaci´on 578 es cero. Soluci´on Tenemos que −→ F (x, y) = (2xy 2 +y, 2x 2 y + x 2 2 +x) es un campo vectorial continuo con derivadas parciales ∂Q ∂x = 4xy + x + 1 ∂P ∂y = 4xy + 1 _ ¸ _ ¸ _ , continuas definidas en la regi´ on de integraci´on. Luego, aplicando el teorema de Green se tiene _ C − → F d − → r = _ (Pdx + Qdy) = __ R _ ∂Q ∂x − ∂P ∂y _ dxdy = __ R xdxdy donde R = ¦(x, y) ∈ IR 2 / −a ≤ x ≤ a, Ψ 1 (x) ≤ y ≤ Ψ 2 (x)¦ Particionemos la Regi´ on R en dos subregiones sim´etricas con respecto al eje y, tal que: R = R 1 ∪ R 2 y R 1 ∩ R 2 = φ.Entonces __ R xdxdy = __ R 1 xdxdy + __ R 2 xdxdy Figura 6.11: Regi´on R = _ (x, y) ∈ IR 2 / −a ≤ x ≤ a, Ψ 1 (x) ≤ y ≤ Ψ 2 (x) _ sub- dividida en dos regiones R 1 y R 2 Se tiene que x es una funci´ on impar definida en un intervalo sim´etrico, entonces _ 0 −a xdx + _ a 0 xdx = 0 579 Por lo tanto __ R 1 xdxdy + __ R 2 xdxdy = 0 En consecuencia _ C − → F d − → r = 0 6.5.5. Aplicaciones al electromagnetismo Problema 1 Sean − → E (t, x, y, z)y − → B (t, x, y, z) los campos el´ectrico y magn´etico respec- tivamente que dependen de la posici´ on y del tiempo, en el espacio. Sea S una superficie con frontera C. Definimos Voltaje alrededor de C = _ C − → E d − → r Flujo magnetico a trav´es de S = __ S − → B ´ ndS La ley de Faraday afirma que el voltaje alrededor de C es igual a la raz´on de cambio negativo del flujo magn´etico a trav´es de S, es decir _ C − → E d − → r = − 1 c ∂ ∂t __ S − → B ´ ndS (1,1) Utilice el teorema de Stokes para deducir la Ley de Faraday a partir de la ecuaci´on diferencial de Maxwell ∇ − → E = − 1 c ∂ − → B ∂t Soluci´on Supongamos que se cumple ∇ − → E = − 1 c ∂ − → B ∂t Sea C una curva cerrada, estacionaria en las coordenadas x, y, z, y S una superficie limitada por C . Si − → B (t, x, y, z) es el campo magn´etico medido en x, y, z en el instante t, entonces tomando la integral de superficie sobre S se tiene. __ S (∇ − → E) ´ ndS = − 1 c __ S ∂ ∂t − → B ´ ndS Transformando el lado izquierdo por medio del teorema de Stokes: _ C − → E d − → r = − 1 c __ S ∂ ∂t − → B ´ ndS 580 Aplicando linealidad al lado derecho obtenemos _ C − → E d − → r = − 1 c ∂ ∂t __ S − → B ´ ndS que es la ley de Faraday. Problema 2 La ley de Amp´ere afirma que si la densidad de corriente el´ectrica est´ a de- scrita por un campo vectorial − → J ,y el campo magnetico es − → B, entonces la circulacion − → B alrededor de la frontera C que limita una susperficie S es igual a la integral de − → J sobre S es decir _ C − → B d − → r = 4π c __ S − → J ´ ndS Pruebe que la ley de Amp´ere se deduce a partir de la ecuaci´on de Maxwell del estado estacionario ∇ − → B = 4π c − → J Soluci´on Supongamos que se cumple ∇ − → B = 4π c − → J Tomando la integral de superficie sobre S se tiene. __ S (∇ −→ B) ´ ndS = 4π c __ S − → J ´ ndS Aplicando el teorema de Stokes al lado izquierdo de la ecuaci´ on _ C − → B d − → r = 4π c __ S − → J ´ ndS que es la ley de Ampere. Esta Ley tambi´en se puede expresar como _ C − → B d − → r = 4π c I donde I = __ S − → J ´ ndS es la intensidad de la corriente el´ectrica en una superficie S limitada por una curva cerrada C. 581 Problema 3 Hallar el campo magn´etico − → B producido por un alambre recto infinito, a una distancia r del alambre que lleva una corriente estacionaria I. Soluci´on Considere un alambre recto extendido sobre el eje z desde -∞ hasta +∞.Como hay simetr´ıa cilindrica, el alambre coincide con el eje axial del cilindro,se escoge una trayectoria circular con con un punto del eje z como centro, con radio r Por la simetr´ıa, el vector − → B no es solamente constante azimutal sino tambien tienen la disma direcci´on que d − → r y su magnitud _ _ _ − → B _ _ _es constante alrededor de la trayectoria C. Por consiguiente _ C − → B d − → r = 4π c I _ C _ _ _ − → B _ _ _ |d − → r | cos 0 = 4π c I _ _ _ − → B _ _ _ _ C |d − → r | = 4π c I _ _ _ − → B _ _ _ (2πr) = 4π c I Finalmente queda − → B = 2 cr I´ e θ Problema 4 Sean − → E (t, x, y, z)y − → B (t, x, y, z) los campos el´ectrico y magn´etico respec- tivamente que dependen de la posici´ on y del tiempo, en el espacio. que sat- isfacen la ecuaciones de Maxwell ∇ − → E = − 1 c ∂ − → B ∂t ∇ − → B = 1 c ∂ − → E ∂t + 4π c − → J ∇ − → E = 4πρ ∇ − → B = 0 582 que est´ an escritas para los campos en el vac´ıo, en presencia de una densidad de carga el´ectrica ρy de corriente,es decir,cargas en movimiento de densidad − → J a) Deducir la forma integral de la ecuaci´ on de Maxwell _ C − → B d − → r = 1 c ∂ ∂t __ S − → E ´ ndS + 4π c __ S − → J ´ ndS donde C representa una espira de alambre por el que fluye corriente en el sentido contrario al de los punteros del reloj con respecto al vector unitario normal a la superficie ´ n. La integral de superficie del primer t´ermino del lado derecho de la ecuaci´ on es llamada flujo el´ectrico y el segundo corresponde a la intensidad de corriente I. Si S cualquier superficie orientada con frontera C. b) Suponga que −→ B(t, x, y, z) es un campo solenoidal, es decir, ∇ (∇ − → B) = 0, que satisface ecuaci´ on (2) de Maxwell. Demostrar que la densidad de corriente satisface la ecuaci´on de continuidad. 4π c _ ∇ J + ∂ρ ∂t _ = 0 Soluci´on b) Consideremos la ecuaci´ on de Maxwell. ∇ − → B = 1 c ∂ − → E ∂t + 4π c − → J Tomando la integral de superficie sobre S se tiene. __ S (∇ −→ B) ´ ndS = 1 c __ S ∂ − → E ∂t ´ ndS + 4π c __ S − → J ´ ndS Aplicando el teorema de Stokes al lado izquierdo de la ecuaci´ on _ C − → B d − → r = 1 c __ S ∂ − → E ∂t ´ ndS + 4π c __ S − → J ´ ndS Aplicando linealidad al primer t´ermino del lado derecho, obtenemos _ C − → B d − → r = 1 c ∂ ∂t __ S − → E ´ ndS + 4π c __ S − → J ´ ndS que es la forma integral de la ecuaci´ on de Maxwell. c) Consideremos la ecuaci´ on 583 ∇ − → B = 1 c ∂ − → E ∂t + 4π c − → J Tomemos la divergencia en los dos lados . ∇ (∇ − → B) = 1 c ∂ ∂t ∇ − → E + 4π c ∇ − → J Aplicando la condici´on de solenoidal y reemplazando ∇ − → E por (3) 0 = 1 c ∂ ∂t (4πρ) + 4π c ∇ − → J Por lo tanto, se tiene 4π c _ ∇ − → J + ∂ ∂t ρ _ = 0 Problema 5 Sean − → E y − → B dos vectores con derivadas parciales continuas de segundo orden con respecto a la posici´ on y el tiempo . Supongase ´ ademas que − → E y − → B satisfacen las ecuaciones de Maxwell en el vac´ıo, en ausencia de densidad de cargas ρ y densidad de corriente − → J : ∇ − → E = 0 (6.5.1) ∇ − → B = 0 (6.5.2) ∇ − → E = − 1 c ∂ − → B ∂t (6.5.3) ∇ − → B = 1 c ∂ − → E ∂t (6.5.4) donde c es una constante que corresponde a la velocidad de la luz en el vac´ıo a) Demostrar que − → E y − → B satisfacen la ecuaci´ on de onda ∇ 2 − → Ψ = 1 c 2 ∂ 2 − → Ψ ∂t 2 b) Demostrar que − → E y − → B satisfacen la ecuaci´on : ∂ ∂t _ 1 2 ( − → E 2 + − → B 2 ) _ + c∇ ( − → E − → B) = 0 584 c) Integre la expresi´ on anterior sobre un volumen Ω encerrado dentro de una superficie Σ, y demostrar que ∂ ∂t ___ Ω _ 1 2 ( − → E 2 + − → B 2 ) _ dV + c __ S ( − → E − → B) ´ ndS = 0 Soluci´on a) Si tomamos el rotacional de la ecuaci´ on 6.5.3 tenemos: ∇(∇ − → E) = − 1 c ∂ ∂t (∇ − → B) Eliminamos a ∇ − → B por medio de la ecuaci´on 6.5.4, entonces ∇(∇ − → E) = − 1 c ∂ ∂t ( 1 c ∂ − → E ∂t ) ∇(∇ − → E) = − 1 c 2 ∂ 2 − → E ∂t 2 Aplicando al rotacional la identidad: ∇(∇ − → E) = ∇(∇ − → E) −∇ 2 − → E se tiene: ∇(∇ − → E) −∇ 2 − → E = − 1 c 2 ∂ 2 − → E ∂t 2 Por tanto, aplicando la ecuaci´ on 6.5.1 al primer t´ermino se tiene el campo el´ectrico − → E satisface la ecuaci´ on de onda ∇ 2 − → E = 1 c 2 ∂ 2 − → E ∂t 2 Por otra parte, si tomamos el rotacional de la ecuaci´on 6.5.4 tenemos: ∇(∇ − → B) = 1 c ∂ ∂t (∇ − → E) Eliminamos a ∇ − → E por medio de la ecuaci´on 6.5.3, entonces ∇(∇ − → B) = 1 c ∂ ∂t (− 1 c ∂ − → B ∂t ) ∇(∇ − → B) = − 1 c 2 ∂ 2 − → B ∂t 2 Aplicando al rotacional la identidad: 585 ∇(∇ − → B) = ∇(∇ − → B) −∇ 2 − → B se tiene: ∇(∇ − → B) −∇ 2 − → B = − 1 c 2 ∂ 2 − → B ∂t 2 En consecuencia, aplicando la ecuaci´on 6.5.2, al primer t´ermino se tiene que el campo el´ectrico − → B tambi´en satisface la ecuaci´ on de onda ∇ 2 − → B = 1 c 2 ∂ 2 − → B ∂t 2 b) Si hacemos en la ecuaci´ on 6.5.3 el producto interno por − → H queda − → B (∇ − → E) = − 1 c − → B ∂ ∂t − → B Analogamente , si hacemos en la ecuaci´on 6.5.4 el producto interno por − → E se tiene − → E (∇ − → B) = 1 c − → E ∂ − → E ∂t Restamos la segunda ecuaci´ on de la primera, entonces obtenemos − → B (∇ − → E) − − → E (∇ − → B) = − 1 c − → B ∂ ∂t − → B − 1 c − → E ∂ − → E ∂t − → B (∇ − → E) − − → E (∇ − → B) = − 1 c _ − → B ∂ ∂t − → B + − → E ∂ − → E ∂t _ Ahora, reemplazando por las identidades: − → B (∇ − → E) − − → E (∇ − → B) = ∇ ( − → E − → B) − → B ∂ ∂t − → B + − → E ∂ − → E ∂t = ∂ ∂t _ 1 2 ( − → B 2 + − → E 2 ) _ La ecuaci´on anterior se convierte en ∇ ( − → E − → B) = − 1 c ∂ ∂t _ 1 2 ( − → B 2 + − → E 2 ) _ por lo tanto, multiplicando por la constante c, obtenemos: ∂ ∂t _ 1 2 ( − → E 2 + − → B 2 ) _ + c∇ ( − → E − → B) = 0 586 c) Si integramos la ´ ultima expresi´ on sobre el volumen Ω dentro de una superficie cerrada S, obtenemos ___ Ω _ ∂ ∂t _ 1 2 ( − → E 2 + − → B 2 ) _ + c∇ ( − → E − → B) _ dV = 0 Aplicando linealidad queda: ∂ ∂t ___ Ω _ 1 2 ( − → E 2 + − → B 2 ) _ dV + c ___ Ω ∇ ( − → E − → B)dV = 0 Usando el teorema de la divergencia en el segundo t´ermino de la ecuaci´on queda: ∂ ∂t ___ Ω _ 1 2 ( − → E 2 + − → B 2 ) _ dV + c __ S ( − → E − → B) ´ ndS = 0 6.6. Auto evaluaciones Autoevaluaci´ on N o 1 Tiempo 2 horas Problema 1 Dado el campo vectorial − → F (x, y, z) = (0, 0, z) . a) calcular __ S − → F ´ ndS, siendo S la superficie descrita por la ecuaci´ on param´etrica: − → r (u, v) = (sin ucos v, sin usin v, cos usin u) , donde D = ¦(u, v) /0 ≤ u ≤ π, 0 ≤ v ≤ 2π¦ b) determine el volumen de la regi´ on Ω acotada por la superficie S. Problema 2 Sea − → r = (x, y, z) en una superficie cerrada regular S. Obtener __ S − → r ´ ndS en t´erminos del volumen V que encierra S. Problema 3. Calcular __ S − → F ´ ndS, siendo el campo vectorial − → F (x, y, z) = _ x 2 + y 3 + z 4 , x 2 + y 3 + z 4 , x 3 + y 4 + z 4 _ y S la superficie formada por las seis caras de paralelep´ıpedo Ω = _ (x, y, z) ∈ IR 3 /3 ≤ x ≤ 5, 1 ≤ y ≤ 2, 0 ≤ z ≤ 1 _ 587 Pauta de Correcci´ on Problema 1 a) Calculemos directamente el flujo del campo − → F sobre la superficie S. __ S − → F ´ ndS = __ D − → F ( − → r (u, v)) − → n dudv Se tiene que − → F ( − → r (u, v)) = (0, 0, cos usin u) y el vector normal exterior a la superficie es − → n = − → r u − → r v = ¸ ¸ ¸ ¸ ¸ ¸ i j k cos ucos v cos usin v −sin 2 u −sin usin v sin ucos v 0 ¸ ¸ ¸ ¸ ¸ ¸ = _ sin 3 ucos v, sin 3 usin v, cos usin u _ − → F ( − → r (u, v)) − → n = (0, 0, cos usin u) _ sin 3 ucos v, sin 3 usin v, cos usin u _ Reemplazando en el integrando obtenemos: _ 2π 0 _ π 0 cos 2 usin 2 ududv = 1 4 _ 2π 0 _ π 0 sin 2 2ududv = 1 4 _ 2π 0 _ π 0 _ 1 −cos 4u 2 _ dudv = 1 4 _ 2π 0 _ 2u −sin 4u 4 _ π 0 dv = 1 4 _ 2π 0 π 2 dv = π 2 4 el flujo del campo − → F sobre la superficie S. b) Sea Ω la regi´on cuyo volumen esta acotado por la superficie S. De acuerdo con el teorema de la divergencia de Gauss, se tiene __ S − → F ´ ndS = ___ Ω ∇ − → F dV Como al calcular ∇ − → F = 0 + 0 + 1 = 1,obtenemos 588 __ S − → F ´ ndS = ___ Ω dV = V (Ω) Por tanto, el volumen pedido es V (Ω) = π 2 4 Problema 2. De acuerdo con el teorema de la divergencia de Gauss se tiene __ S − → r ´ ndS = ___ Ω ∇ − → r dV = ___ Ω 3dV = 3V donde V es el volumen encerrado por S Problema 3 Sea la regi´on cuyo volumen esta acotado por la superficie S. De acuerdo con el teorema de la divergencia de Gauss, se tiene __ S − → F ´ ndS = ___ Ω ∇ − → F dV Como al calcular ∇ − → F = 2x + 3y 2 + 4z 3 ,obtenemos 589 ___ Ω ∇ − → F dV = ___ Ω _ 2x + 3y 2 + 4z 3 _ dxdydz = _ 5 3 _ 2 1 _ 1 0 _ 2x + 3y 2 + 4z 3 _ dzdydx = _ 5 3 _ 2 1 _ 2xz + 3y 2 z + z 4 ¸ 1 0 dydx = _ 5 3 _ 2 1 _ 2x + 3y 2 + 1 _ dydx = _ 5 3 __ 2xy + y 3 + y _¸ 2 1 dx = _ 5 3 (2x + 8)dx = _ x 2 + 8x ¸ = 32 Autoevaluaci´ on N o 2 Tiempo 2 horas Problema 1. Calcular la integral __ S − → F ´ ndS, en donde − → F (x, y, z) = (xy 2 , x 2 y, y) , y S es la superficie del cilindro x 2 + y 2 = 1 acotado por los planos: z = 1 y z = −1. Problema 2. Sea − → F (x, y, z) = _ 0, √ y, 0 _ el campo de velocidades de un fluido. Calcular el flujo __ S − → F ´ ndS, que atraviesa la superficie S = ¦(x, y, z) ∈ IR 3 /x 2 + y 2 = y, 0 ≤ y ≤ 1¦ . Problema 3. Verificar el teorema de Stokes para − → F (x, y, z) = (3y, −xz, yz 2 ) , siendo S la superficie del paraboloide 2z = x 2 + y 2 , limitada por z = 2.Considere que ´ n es la normal exterior a la superficie S. Pauta de Autocorreci´on Problema 1 De acuerdo con el teorema de la divergencia de Gauss,se tiene __ S − → F ´ ndS = ___ Ω ∇ − → F dV 590 siendo Ω la regi´ on limitada por la superficie S, es decir el cilindro macizo dado por la ecuaci´ on Ω = ¦(x, y, z) ∈ IR 3 /x 2 + y 2 ≤ 1, −1 ≤ z ≤ 1¦ . Teniendo en cuenta que el integrando en cordenadas cartesianas ∇ − → F = y 2 + x 2 + 0,queda: ___ Ω ∇ − → F dV = ___ Ω _ x 2 + y 2 _ dxdydz Por la simetria del integrando y del dominio conviene aplicar el cambio de coordenadas cil´ındricas x = ρ cos θ, y = ρ sin θ , cuyo jacobiano es J (ρ, θ, z) = ρ. Entonces la regi´on Ω transforma a Ω ∗ = ¦(ρ, θ, z) /ρ 2 ≤ 1, −1 ≤ z ≤ 1¦ y el integrando produce ∇ − → F = ρ 2 ,luego. ___ Ω _ x 2 + y 2 _ dxdydz = ___ Ω ∗ _ ρ 2 _ ρdρdθdz = _ 1 −1 _ 2π 0 _ 1 0 ρ 3 dρdθdz = _ 1 −1 _ 2π 0 _ ρ 4 4 _ 1 0 dρdθdz = π . Problema 2. En primer lugar consideremos la siguiente parametrizacion de la Superfi- cie − → r (u, v) = (ucos v, u 2 , usin v) , donde Ω = ¦(u, v) , 0 ≤ u ≤ 1, 0 ≤ v ≤ 2π¦ Determinemos el vector normal a la superficie − → r u (u, v) = (cos v, 2u, sin v) − → r v (u, v) = (−usin v, 0, ucos v) − → r u − → r v = ¸ ¸ ¸ ¸ ¸ ¸ i j k cos v 2u sin v −usin v 0 ucos v ¸ ¸ ¸ ¸ ¸ ¸ = (2u 2 cos v, −u, 2u 2 sin v) Entonces 591 __ S − → F ´ ndS = __ Ω − → F ( − → r (u, v)) ( − → r u − → r v ) dudv = _ 2π 0 _ 1 0 (0, u, 0) _ 2u 2 cos v, −u, 2u 2 sin v _ dudv = _ 2π 0 _ 1 0 −u 2 dudv = − 2π 3 Problema 3. De acuerdo con el teorema de Stokes, debemos verificar que _ C − → F d − → r = __ S ∇ − → F ´ ndS En primer lugar, calculemos _ C − → F d − → r Determinemos la curva C que limita la Superficie abierta del paraboloide: si z = 2 =⇒ x 2 + y 2 = 4 Parametricemos la curva C, con una orientaci´ on positiva, se tiene − → r (t) = (2 cos t, 2 sin t, 2) t ∈ [2π, 0] y evaluemos el campo sobre la curva − → F ( − → r (t)) = (6 sin t, −4 cos t, 8 sin t) , luego _ C − → F d − → r = − _ 0 2π _ 12 sin 2 t + 8 cos 2 t _ dt = _ 2π 0 _ 8 + 4 sin 2 t _ dt = _ 2π 0 _ 8 + 4 _ 1 −cos 2t 2 __ dt = 16π + 2 _ t − sin 2t 2 _ 2π 0 = 20π 592 Calculemos, ahora __ S ∇ − → F ´ ndS = __ R ∇ − → F ( − → r y − → r x ) dxdy Las ecuaci´ on param´etrica de la superficie S en coordenadas cartesianas es − → r (x, y) = _ x, y, x 2 + y 2 2 _ ,y el vector normal exterior es − → n = − → r y − → r x = ¸ ¸ ¸ ¸ ¸ ¸ i j k 0 1 y 1 0 x ¸ ¸ ¸ ¸ ¸ ¸ = (x, y, −1) Adem´ as ∇ − → F = ¸ ¸ ¸ ¸ ¸ ¸ i j k ∂/∂x ∂/∂y ∂/∂z 3y −xz yz 2 ¸ ¸ ¸ ¸ ¸ ¸ = (z + x, 0. −(z + 3)) Finalmente R es la proyecci´ on sobre el plano xy, R = ¦(x, y) ∈ IR 2 /x 2 + y 2 = 4¦ ,entonces __ R _ _ x 2 + y 2 2 _ 2 + x, 0, − x 2 + y 2 2 −3 _ (x, y, −1) dxdy = __ R _ x _ x 2 + y 2 2 _ 2 + x 2 + x 2 + y 2 2 + 3 _ dxdy Haciendo un cambio a coordenadas polares x = ρ cos θ, y = ρ sin θ _ 2π 0 _ 2 0 _ 1 4 ρ 5 cos θ + ρ 2 cos θ + 1 2 ρ 2 + 3 _ ρdρdθ = 128 28 _ 2π 0 cos θdθ + 4 _ 2π 0 cos 2 θdθ + 8 _ 2π 0 dθ = 0 + 4 _ 2π 0 1 + cos 2θ 2 dθ + 16π = 4π + 16π = 20π Autoevaluaci´ on N o 3 Tiempo 2 horas Pregunta 1 Usar el teorema de Stokes para resolver, __ S ∇ − → F ´ ndS , si la superficie S esta dada por z = 1 −x 2 −y 2 con z ≥ 0 y − → F = (y, z, x) . 593 Pregunta 2 Determinar el ´ area de la superficie x 2 + y 2 = 4, limitada por el plano z = 0 y por la superficie z = 2x 2 + y 2 Pauta Correcciones Pregunta 1 El teorema de Stokes asegura que __ S ∇ − → F ´ ndS = _ C − → F d − → r donde C es la curva cerrada que limita S. Sea C: 1 − x 2 − y 2 = 0; z = 0 =⇒ x 2 + y 2 = 1 corresponde a una circunferencia en el plano xy centrada en el origen de radio 1. Al parametrizar la curva considerando una orientaci´ on positiva, produce − → r (t) = (cos t, sin t, 0) t ∈ [2π, 0] =⇒ − → r (t) = (−sin t, cos t, 0) t ∈ [2π, 0] =⇒ − → F ( − → r (t)) = (sin t, 0, cos t) Calculemos entonces la integral de l´ınea − → F de sobre la curva C _ C − → F d − → r = _ 2π 0 [sin t (−sin t) + 0 + 0] dt = − _ 2π 0 sin 2 tdt = − _ 2π 0 _ 1 −cos 2t 2 _ dt = − _ t 2 − sin 2t 4 _ 2π 0 = −π Pregunta 2 M´etodo 1 En primer lugar, consideremos la Superficie cil´ındrica parametrizada por − → r (u, v) = (2 cos u, 2 sin u, v) , donde Ω = ¦(u, v) , 0 ≤ u ≤ 2π, 0 ≤ v ≤ 4 cos 2 u + 4¦ Determinemos el vector normal a la superficie − → r u (u, v) = (−2 sin u, 2 cos u, 0) − → r v (u, v) = (0, 0, 1) − → r u − → r v = ¸ ¸ ¸ ¸ ¸ ¸ i j k −2 sin u 2 cos u 0 0 0 1 ¸ ¸ ¸ ¸ ¸ ¸ = (2 cos u, 2 sin u, 0) 594 [ − → r u − → r v [ = 2 El ´area de la superficie es entonces A(S) = _ _ Ω [ − → r u − → r v [ dudv = _ 2π 0 _ 4 cos 2 u+4 0 2dvdu = 2 _ 2π 0 _ 4 cos 2 u + 4 _ du = 24π M´etodo 2 El ´area de la superficie se puede determinar con la integral curvil´ınea A(S) = _ C zds Sea C: x 2 + y 2 = 4 es a una circunferencia en el plano xy centrada en el origen de radio 2. Al parametrizar la curva considerando una orientaci´ on positiva, produce − → r (t) = (2 cos t, 2 sin t) , t ∈ [2π, 0] =⇒ − → r (t) = (−2 sin t, 2 cos t) , t ∈ [2π, 0] =⇒ z ( − → r (t)) = _ 4 cos 2 u + 4 _ Luego, el ´ area de la superficie es A(S) = _ 2π 0 _ 4 cos 2 u + 4 _ 2dt = 24π 595 Bibliograf´ıa [1] Tom M. Ap´ ostol, Calculus, Vol I, 2 a edici´ on, Ed. Revert´e, 1982 [2] George Arfken, Mathematical Methods for Physicists, Academic Press, 1985 [3] R.V. Churchill, Fourier Series and Boundary Value Problems, 2 o ed. Nueva York, Mac Graw Hill,1963 [4] Erwing Kreyszing, Matem´ aticas Avanzadas para Ingenier´ıa, Vol I y II, Limusa, 1995 [5] R. Larson ,R.Hostetler, B. Edwars, C´alculo con Geometr´ıa Anal´ıtica, Vol 2, Mc Graw Hill, 1999. [6] J.E. Marsden, A.J. Tromba, C´ alculo Vectorial, Addison Wesley Long- man, 1998 [7] Peter V. O’Neil, Matem´aticas Avanzadas para Ingenier´ıa, Thomson, 2005 [8] Claudio Pita Ruiz, C´alculo Vectorial, Prentice-Hall Hispanoam´erica, SA, 1995 [9] George B. Thomas Jr., C´alculo varias variables, Addison-Wesley, Pear- son 2000S 596
Copyright © 2024 DOKUMEN.SITE Inc.